Você está na página 1de 458

N.

PISKOUNOV

CÁLCULO DIFERENCIAL
E INTEGRAL
VOLUME II

T R A D U Ç A O DE:

ANTÓNIO EDUARDO PEREIRA T E IX EIR A


Licenciado em Economia (U. P.)
Contabilista diplomado (I. C. P.)

MARIA JO S É PEREIRA T E IX EIR A


Contabilista diplomada (I. C. P.)

11.'’ EDIÇÃO EM LÍNGUA PORTUGUESA

E D I Ç Õ E S L O P E S DA S I L V A - P O R T O - 1 9 9 7
Título da 4." edição do original

H. C. nHCKVHOB

flHdxDEPEHI^HAJIbHOE

HHTErPAJIbHOE

HCHHCJIEHHfl

TOM I

H3flATEJIbCTBO «HAVKA»
MOCKBA

Todos os direitos de adaptação e de reprodução por todos os processos, reservados


para todos os paises de expressão Portuguesa, de acordo com as leis em vigor.
© LIVRARIA LOPES DA SILVA —EDITORA
Dedicamos este nosso trabalho aos nossos que­
ridos pais e irmãs e duma maneira geral aos
familiares que mais de perto nos acompanham,
O tradutor dedica ainda esta tradução, em espe­
cial, aos queridos colegas Maria Luísa e José Alves
Baptista que sempre o acompanharam nos seus
estudos superiores, nas matérias versadas na pre­
sente obra e ainda a todos os queridos alunos,
que directa ou indirectamente, contribuiram para a
prossecução desta tradução.

OS TRADUTORES

A tradução desta segunda parte dedico-te.


especialmente a ti, ROMAC.

O TRADUTOR
Composto e Impresso nas Oficinas Gráficos Reunidos, Lda.
R. Alvares Cabral, 22 ■32 - Telef. 2000608 - Fax 2007184
4050 PORTO - 3 000 ex. - OUT. / 97 - Dep. Legal 56 868 / 92
í n d i c e

Prefácio 11

CAPITULO XIII
Equações diferenciais

§ 1. Posição do problema. Equação do movimento do corpo para um


meio em que a resistência é proporcional à velocidade. Equação
da catenária
Definições
Equações diferenciais de primeira ordem (noções gerais)
Equações com variáveis separadas e separáveis. Problema da desin­
tegração do rádio
5. Equações homogéneas de primeira ordem .
6 . Çquações redutíveis a equações homogéneas .
7. Equações lineares de primeira ordem .
8 . Equação de Bernoulli . . . .
9. Equações de diferenciais totais . . . .
§ 10. Factor integrante
§ 11. Envoltória duma família de curvas
§ 12. Soluções singulares das equações diferenciais c i
§ 13. Equação de Clairaut . . . .
§ 14. Equação de Lagrange . . . .
§ 15. Trajectórias ortogonais e isogonais
§ 16. Equações diferenciais de ordem superior a um (noções gerais)
§ 17. Equação da forma = f ( x ) ....................................................
§ 18. Alguns tipos de equações diferenciais de .segunda ordem que se
reduzem a equações de primeira o r d e m .......................... ...... 65
Í N D I C E

§ 19. Integração gráfica das equações diferenciais de segunda ordem 73


§ 20. Equações lineares homogéneas. Definições e propriedades gerais 75
§ 21. Equações lineares homogéneas de segunda ordem de coeficientes
c o n sta n te s..................................................................................... 82
§ 22. Equações diferenciais lineares homogéneas de ordem n de coeficientes
co n sta n te s..................................................................................... 86
§ 23. Equações lineares não homogéneas de segunda ordem . . . . 89
§ 24. Equações lineares não homogéneas de segunda ordem de coefi­
cientes constantes . * ..................................................................................... 93
§ 25. Equações lineares não homogéneas de ordem n ...........99
§ 26. Equação diferencial dasoscilações m ecânicas......................................... 103
§ 27. Oscilações livres . ........................................................................ 105
§ 28. Oscilações forçadas. . . . ................................... 107
§ 29. Sistemas de equações diferenciais..............................................112
§ 30. Sistemas de equações diferenciais lineares de coeficientes constantes 118
§ 31. Noção sobre a teoria da estabilidade de Liapounov . . . . 125
§ 32. Solução aproximada dasequaçõesdiferenciais de primeira ordem
pelo método Euler .................................................................................... 131
§ 33. Solução aproximada das equações diferenciais pelo método dos
diferenciais finitos baseados na aplicação da fórmula de Taylor.
Método de A d a m s ....................................................................................134
§ 34. Método aproximado de integração dos sistemas de equações di­
ferenciais dc primeira o rd e m .................................................... 141
E x ercício s....................................................................... ..........................146

CAPITULO XIV
Integrais múltiplos

§ 1. Integral duplo . 160


§ 2. Cálculo dos integrais d u p lo s ....................................................................... 162
§ 3. Cálculos dos integrais duplos (cont.) .......................................................... 168
§ 4. Aplicação dos integrais duplos ao cálculo de áreas evolumes . 174
§ 5. Integrais duplos em coordenadas p o la r e s ............................................. 177
§ 6. Mudança de variáveis num integral duplo (caso geral) . 185
§ 7. Cálculo das áreas de superfícies . . . * . ......................................... 190
Í N D I C E

§ 8. Densidade dedistribuição de matéria e integral duplo 194


§ 9. Momento de inércia duma figurap l a n a ...............................................195
,§ 10. Coordenadas do centro de gravidadeduma figura plana . 200
§ 11. Integrais t r i p l o s ......................................................................................... 202
§ 12. Cálculo dos integrais trip lo s ......................................................................203
§ 13. Mudança de variáveis num integrald u p lo ...............................................208
§ 14. Momento de inérci^ e coordenadas do centro degravidade dum corpo 212
§ 15. Integrais que dependem dump a râ m e tro ............................. 215
Exercícios . . . .......................................................... . 216

CAPÍTULO XV
Integrais curvilíneos e integrais de superfície

§ 1. Integral curvilíneo.......................................................................................... 223


§ 2. Cálculo do integral cu rv ilín e o ................................................................ 226
§ 3. Fórmula de G reen .......................................................................................... 232
Condições para que um integral curvilíneo não dependa do caminho
de integração.................................................... 235
§ 5. Integrais de s u p e r f íc ie ................................. . . 240
§ 6. Cálculo dos integrais de superfície . 242
§ 7. Fórmula de Stokes....................................... . . 245
§ 8. Fórmula de Ostrogradsky................................ . . 250
§ 9. Operador hamiltoniano e algumas aplicações . . 253
E x ercício s.......................................................... . . 257

CAPITULO XVI
Séries

§ 1. Soma duma s é r ie ................................................................................264


§ 2. Condição necessária de convergência de uma série . . . . . 267
§ 3. Comparação das séries comtermospositiv o s........................................... 270
§ 4. Regra de A le m b e rt..........................................................................272
§ 5. Regra de C a u c h y ..........................................................................................276
§ 6. Comparação com um integral........................................................................278
§ 7. Séries alternadas. Teorema de L e ib n iz .......................................................281
§ 8. Séries de termos de jinais quaisquer. Convergência absoluta e semi-
-convergência . . . • .......... . . . . 283
Í N D I C E

§ 9. Séries de fu n çõ es......................................... ............................... 287


§ 10. Séries m ajoráveis................................................................................288
§ 11. Continuidade da soma duma s é r ie ................................................ 290
§ 12. Integração e derivação de sé rie s...................................................... 293
§ 13. Séries inteiras ou séries de potências.Intervalo deconvergência 296
§ 14. Derivação de séries in te ira s .............................................................301
§ 15. Séries de potências áe x — a .............................................................303
§ 16. Séries de Taylor e de M ac la u rin .....................................................304
§ 17. Exemplos de desenvolvimento defunções emséries. . . . 306
§ 18. Fórmulas de Euler . . 308
§ 19. Fórmula geral do b in ó m io .............................................................309
§ 20. Desenvolvimento da função Log (1 + jf) emsérie inteira. Cálculo de
lo g aritm o s............................................................................................ 312
§ 21. Aplicação das sériesao cálculo dos integraisdefinidos . . . . 314
§ 22. Aplicação das sériesà integração de equações diferenciais . . . 316
§ 23. Equação de B e s s e l ..........................................................................319
E x ercício s............................................................................................ 325

CAPÍTULO XVII
Séries de Fourier

§ 1. Definição. Posição do p ro b lem a................................................................ 334


§ 2. Exemplos de desenvolvimento de funções em séries de Fourier . 339
§ 3. Uma nota sobre o desenvolvimento das funções periódicas em
série de F o u r i e r .......................................................................................... 344
§ 4. Séries de Fourier de funções pares c ímpares . . . . . 347
§ 5. Séries de Fourier das funções deperíodo 2 / ......................................... 348
§ 6. Desenvolvimento em série de Fourier duma função não periódica 350
§ 7. Aproximação, em média, duma função dada por meio de poli­
nómios trignométricos..............................................................................352
§ 8. Integral de D iric h le t....................................................................................358
§ 9. Convergência duma série de Fourier num dado ponto . . . . 361
§ 10. Algumas condições, suficientes para a convergência duma série
de F o u rie r................................................................................................. 363
§ 11. Análise harmónica numérica ....................................................................... 366
§ 12. O integral de F o u r i e r ............................................................................. 368
§ 13. Forma complexa do integral deF o u rie r...................................................... 372
E x ercício s....................................................................................................... 374
Í N D I C E

CAPÍTULO XVIII
Equações da física matemática

§ 1. Principais tipos de equações dafísica m atem ática...................................377


§ 2. Estabelecimentoda equação para cordasvibrantes. Formulação do
problema aos limites. Estabelecimento da equação para oscilações
eléctricàs nos f i o s .......................................................................................... 378
§ 3. Resolução da equação das cordas vibrantes pelo método de separação
das variáveis (método de F o u rie r)..........................................................382
§ 4. Equação da propagação do calor numa barra. Enunciado do problema
aos lim ites....................................................................... . . . . 386
§ 5. Propagação do calor no e sp a ç o ..................................................................389
§ 6. Resolução do primeiro problema dos limites para a equação do
calor pelo método dasdiferenças fin ita s......................................................393
§ 7. Propagação do calor numa barra infinita....................................................395
§ 8. Problemas que conduzem ao estudo das soluções das equações de
Laplace. Enunciado dos problemasde lim ite s ......................................... 401
§ 9. Equação de Laplace em coordenadas cilíndricas. Resolução do
problema de Dirichlet para um arco com valores constantes da
função procurada sobre os círculos interior exterior . . . . 406
§ 10. Resolução do problema de Dirichlet para o círculo . . . . 409
§ 11. Solução do problema de Dirichlet pelo método das diferenças finitas 413
E x ercício s....................................................................................................... 416

CAPÍTULO XIX
Cálculo operacional e aplicações

§ 1. Original e im agem............................................................................. 420


§ 2. Imagem das funções ao(/), sen/, c o s / .... 422
§ 3. Imagens das funções com escala modificada da variável indepen
dente. Imagem das funções sen a/, c o s a / ................................. 424
§ 4. Propriedade de linearidadeda im ag em .......................................... 425
§ 5. Teorema do deslocamento................................................................ 426
§ 6. Imagem das funções sen hat, cos hat, sen a/, cos at 426
§ 7. Derivação da imagem ........................................................................ 428
§ 8. Imagem das derivadas....................................................................... 430
§ 9. Dicionário de imagens....................................................................... 431
10 ÍN D IC E

§ 10. Equação auxiliar duma equação diferencial dada . . . . 433


§ 11. Teorema da d e c o m p o sição .......................................................... 437
§ 12. Exemplos de resolução das equações diferenciais e dos sistemas
de equações diferenciais pelo método do cálculo operacional . 439
§ 13. Teorema do enrolamento (convolution)....................................... 441
§ 14. Equações diferenciais das oscilações mecânicas. Equações diferenciais
da teoria dos circuitos eléctricos.................................................... 443
§ 15. Resolução da equação diferencial das oscilações . . . . 445
§ 16. Estudos das oscilações liv re s .......................................................... 447
§ 17. Estudo das oscilações harmónicas amortecidas no caso duma força
exterior p erió d ica.............................................................................. 448
§ 18. Solução da equação das oscilações no caso da ressonância . 450
§ 19. Teorema do retardam ento................................................................. 451
E x ercício s........................................................................................... 453
PREFACIO

A 3.* edição em língua francesa conserva como essencial o


conteúdo da 2.*' edição. Certos capítulos foram profundamente revistos
e completados, em especial aqueles que tratam de certos ramos das
matemáticas modernas, cujo conhecimento é nos nossos dias indis­
pensável a todo o engenheiro. Na parte «Exercícios» aumentou-se o
número de problemas, insistindo sobre aqueles que, mais difíceis, exi­
gem mais reflexão. O material desta nova edição é apresentado em
dois volumes.
No primeiro volume, os capítulos iniciais «Número, variável, função»
e «Limite e continuidade das funções» foram resumidos na medida
do possível. Certas questões, habitualmente tratadas nestes capítulos,
foram conscientemente reportadas aos capítulos seguintes. Isto permitiu
abordar mais ràpidamente a derivada, noção fundamental do cálculo
diferencial; esta necessidade foi-nos ditada pelas exigências das outras
disciplinas do ensino técnico superior. O bom fundamento duma tal
disposição foi felizmente confirmado pela experiência de vários anos.
No f m do primeiro volume inseriu-se os anexos I e II expondo
problemas muito importantes para o engenheiro: «Estabelecimento duma
dependência funcional a partir de dados experimentais pelo método
dos mínimos quadrados» e «Fórmula de interpolação de Newton.
Derivação numérica».
No segundo volume, para assegurar aos estudantes uma prepa­
ração matemática que lhes permita abordar as disciplinas ligadas à
aufDmação e aos métodos de cálculo automático, que são hoje ensi­
nadas nos estabelecimentos de ensino técnico superior, vários desen­
volvimentos, tratando em detalhe destas questões, foram inseridos:
«Integração numérica das equações diferenciais e sistemas de equações
diferenciais» (♦), «Integração de sistemas diferenciais lineares», «Noção
sobre a teoria da estabilidade de Liapounov», «Operador hamiltoniano»,
«Integral de Fourier», etc.

(*) Os métodos de cálculo numérico h£Ú3Ítüalmente tratados nos cursos


de análise são igualmente expostos neste manual.
Esta edição foi também completada por dois novos capítulos
«Equações da física matemática» (capítulo XVIII) e «Cálculo opera­
cional e aplicações» (capítulo XIX).
O capítulo XVIII passa em revista as equações fundamentais
da física matemática. Tem-se dado uma importância particular à
análise da natureza dos fenómenos físicos que conduzem às equações de
diferentes tipos e aos problemas de limites correspondentes. Uma grande
importância foi igualmente concedida aos métodos numéricos de reso­
lução das equações diferenciais às derivadas parciais.
No capítulo XIX expôs-se as noções fundamentais do cálculo
operacional e o método operacional de resolução das equações dife­
renciais. Elas são indispensáveis para o estudo de numerosas disciplinas
aplicadas, em especial as ligadas à electrotécnica.
Um grande número de problemas e de exercícios, que esclarecem
a maior parte dos vínculos que existem entre as matemáticas e
as outras disciplinas, foram incluídos neste manual. Os problemas e
os exercícios foram especialmente escolhidos para cada capítulo do
curso a fim de contribuir para a assimilação da parte teórica. Alguns
foram resolvidos e comentados a título de exemplos. Isto torna o
uso deste manual particularmente precioso para o estudo auto-
-didáctico.
Devo exprimir a minha profunda gratidão às Edições Mir que
aceitaram a tradução e a publicação desta obra.

O autor

NOTA SOBRE A PRESENTE EDIÇÃO

Esta edição, a 4.*^ em francês, reproduz a 3.^ que se esgotou


ràpidamente.
Procedemos, no entanto, às correcções que o autor julgara neces­
sárias para esta nova edição, a fim de apresentar aos leitores uma
obra ainda mais digna da sua confiança.
O EDITOR
Capitulo XIII

EQUAÇÕES DIFERENCIAIS

§ 1. Posição do problema. Equação do movimento do corpo para


um meio em que a resistência é proporcional à velocidade.
Equação da catenâria

Suponhamos que a função y = f(x) exprime um fenómeno do


ponto de vista quantitativo. Examinando este fenómeno, é muitas vezes
impossível estabelecer directamente o carácter da dependência entre
y e X, mas pode-se estabelecer uma dependência entre as quantidades
X, y e derivadas de y em relação a j c : / , . . . . y(n)^ isto é. que
se pode escrever uma equação diferencial
Deduzir da relação entre x, y e as derivadas a relação directa
entre y e jc , isto é, encontrar y = f ( jc ) , é ainda o que se chama integrar
uma equação diferencial
Consideremos dois exemplos.
Exemplo — 1. Deixe-se cair um corpo de massa m duma certa altura.
Pede-se para estabelecer a lei de variação da velocidade da queda v, se o corpo
experimentar uma rcsislência de travagem da parte do ar proporcional à velo­
cidade (sendo o coeficiente de proporcionalidade k), isto é, encontrar v = / (/).
Resolução — Em virtude da segunda lei de Newton
dv

em que -4 ^ ^ a aceleração do corpo em movimento (a derivada da velocidade


at
em relação ao tempo) e F, a força que age sobre o corpo no sentido do
mov"hnento. Esta força é constituída por duas forças: pela força de gravidade mg
e pela resistôncia do ar = kv (toma-se o sinal menos porque esta força é oposta
à velocidade). Assim
dv
m —r—t=tmg—kv, ( 1)
at
dv
Temos uma relação entre a função desconhecida v e a sua derivada ,
dt
isto é, uma equação diferencial sobre a função desconhecida v. (Ê a equação
do movimento de certos tipos de paraquedas). Resolver esta equação diferencial,
é procurar uma função v = / (/), que a verifica idênticamente. Existe uma
infinidade de tais soluções. O leitor verificará fàcilmente que toda a função
da forma
-A ,
I mg
Ce ( 2)
k
14 CÁLCULO DIFERENCIAL E INTEGRAL

verifica a equaçSo (1) qualquer que seja a constante C. Mas qual destas fun­
ções dá a re la to procurada entre v e r ? Para a encontrar, imponhamos uma
condição suplementar: uma velocidade inicial v« (que, em especial, pode ser
nula) foi comunicada ao corpo na partida; suporemos que esta velocidade inicial
é conhecida, mas, então, a função procurada v = / (/) deve ser tal que se tenha
para r = 0 (no começo do movimento) v = w Substituindo / = 0, v = v# na
fórmula (2), tem-se:
vo = C-\- - j p ,

donde
mg
C — Vq—

Assim, a constante C é determinada. A dependência entre v e / , exprime-se,


pois, por:

(2 ' )

Resulta desta fórmula que para / sufícientemente grande a velocidade v


depende pouco de Vo.
Notemos que se á = 0 (isto é, se a resistência do ar for nula ou des­
prezável) encontra-se o resultado conhecido em física (*):
V = VQ-\-gt. (2')
Esta função satisfaz a equação diferen-
ciai (1) e à condição inicial: v = Vo para / = 0.
Exemplo — 2. Um fio flexível homogéneo
está suspenso pelas suas duas extremidades. Achar
a equação da curva de equilíbrio do fio subme­
tido ao seu próprio peso (tal é a posição que
tomam os cabos suspensos, os fios, as correntes).
Resolução — Sejam Mo(0, h) o ponto mais
baixo sobre o fio, M um ponto arbitrário sobre
este fio (fig. 244). Consideremos a porção de
fio M«Af. Esta porção está em equilíbrio sob a
acção de três forças:
1) \ tensão 7, que age tangencialmente no ponto M e formando com
o eixo Ox o ângulo 9 ;
2) A tensão H no ponto que age horizontalmente;
31 O peso ys dirigido verticalmente para baixo, em que 5 é o compri­
mento do arco AftAÍ, y o peso específico do fio.

(*) Pode-se deduzir a fórmula (2") para passagem ao limite

- ^ ~ \ = v o + gt.
+■
* J
EQUAÇÕES DIFERENCIAIS 15

Decompondo a tensão T nas suas componentes horizontal e vertical,


obtém-se as equações de equilíbrio:
T COS ^ =

rscn<p = Y*-
Obtém-se, dividindo membro a membro, estas duas igualdades

(3)
Suponhamos agora que se pode escrever a equação da curva procurada
sob a forma Y = / U)* Aqui, / (jc) é uma função desconhecida que é preciso
procurar. Notemos que

tg < f= r
Por conseguinte:
dy 1
dx ~ a
H (4)
em que se fez — = a.
Derivemos os dois membros da igualdade (4) em relação a x:
d^y 1 ds
dx'^ ~~ a dx
Mas sabe-sc que (ver § 1, Cap. VI)

(5)
Substituindo esta expressão na equação (5), obtém-se a equação dife­
rencial da curva procurada sob a forma:

(6)

Ela liga as derivadas primeira e segunda da função desconhecida y.


Sem nos preocuparmos com os métodos de resolução das equações, indi­
quemos que toda a função da forma

(7)
satisfaz à equação (6) quaisquer que sejam as constantes Ci e Cs. Ê fácil de
provar substituindo as derivadas primeira e segunda da função indicada na
equação (6). Indiquemos ainda, sem o demonstrar, que se tem aí todas as
soluções (para diversos Ci e Cs) da equação (6). Isso será demonstrado no § 18.
Os gráficos das funções assim obtidas são chamadas das catenárias.
Vejamos agora como convém escolher as constantes Ci e Cs para obter
precisamente a catenária cujo ponto inferior tem por coordenadas (0, b). Dado
que para jc = 0 se tem o ponto mais baixo da catenária, a tangente é« hori­
zontal naquele ponto, isto é, - ^ = 0. Além dissq, por hipótese, a ordenada é
dx
igual a h nesse ponto, isto é, > = b.
16 CAIXJÜLO DIFERENCIAL. E INTEGRAL.

Deduz-se da equaçfio (7)

Substituindo nesta última x 0, obtém-se Logo Cl == 0,


se ò é ordenada de Aio, tem-se, então, y = A para x = 0. Deduz-se da equação (7),
pondo j : = 0 e Oj 0, 6 = -.^ ( i - f i) + C2, donde € 2 =^0 — a.
Encontra-se, por fim:
* X

P = -2 (« “ + e “ )+ * » —“•
A equação (7) simplifica-se muito se se fizer a ordenada do ponto Aio
igual SL a. A equação da catenáría toma-se, então, em:

§ 2. Definições
Definição— 1. Chama-se equação diferencial a uma equação que
estabelece uma relação entre a variável independente x, a função
desconhecida y = / (jc) e suas derivadas / , y".......
Pode-se escrever simbòlicamente uma equação diferencial como
se segue: , ,
Fi x, y, y, y , y‘">) = 0
ou

{x y ^ f . . . . - ^ 1 = 0.
\ ’ ’ dx' da^' ’ di'*/
Sq y = f (x) é função de uma só variável independente, a equação
diferencial diz-se ordinária. Começaremos pelo estudo das equações
diferenciais ordinárias (*).
(t) Ao mesmo tempo que as equações diferenciai^ ordinárias, estuda-se
igual mente em análise matemática equações com derivadas parciais. Chamam-se
•Equações com derivadas parciais^ a uma relação entre a função desconhecida z,
qUe depende de duas Cu várias variáveis x, y, ..., estas próprias variáveis e
. . . • • j d z d z d ^ z
as derivadas parciais de z : , -r— , -rs-, etc.
dx dy d^x
Tem-se como exemplo de equação de derivadas parciais de função des­
conhecida z(x, y) a equação
dz dz
dx ^ dy
É fácil de veriticar que a função z = x^y^ (bem como muitas outras
funções) verifica esta equação.
No decorrer deste curso as equações de derivadas parciais são estudadas
no capítulo XVIII (vol. 2).
BQCIACÕES DIFERENCIAIS 17

Definição — 2. Chama-se ordem duma equação diferencial à or­


dem da derivada mais elevada contida nessa equação.
Assim,

y ' — 2xy^ + 5 = 0

é uma equação de primeira ordem.


A equação
y " + k y ' — by — s e n a := 0

é uma equação de segunda ordem, etc.


A equação considerada no exemplo 1 do parágrafo precedente
é uma equação de 1.® ordem e a do exemplo 2 de segunda ordem.

Definição — 3. Chama-se solução ou integral duma equação dife­


rencial a toda a função y = f(x) que verifica idênticamente essa equação.

Exámplo — 1. Seja a equação diferencial

d^y
+ í/ = 0.
dx^

As funções y = sen j:, y = 2 cos jr, y = 3 sen x — cos x e, mais geralmente,


toda a função da forma y = Ci sen x, y = Cz cos x ou
s e n x + C 2 cos a:

é solução da equaçãlo dada quaisquer que sejam as constantes Ci e Cs; é


fácil de verificar, substituindo estas funções na equação.

Exemplo — 2. Consideremos a equação


x2 —y = 0.
As suas soluções são funções da forma
y = x2 + Cx,

em que C á uma constante arbitrária. Com efeito, encontra-se, derivando a


função y=zx^-{-Cx:
i/' = 2a:+C.

Substituindo as expressões de y e y" na equação dada, obtém-se a


identidade
(2x + C) X — x2 — x2 — Cx = 0,
Cada uma das equações tratadas nos exemplos 1 e 2 possui uma infinidade
de soluções.
18 CALCULO DIFERENCIAL E INTEGRAL [Ch. X III

§ 3. B2quáções diferenciais de prim eira ordem (noções gerais)


1. Uma equação diferencial de 7.® ordem é da forma

y. «') = 0. (1)
Quando esta equação é resolúvel em / , pode-se pô-la sob a forma 5

y = f { x . y). ( 1 ')

Diz-se, então, que a equação diferencial é resolúvel em relação


à derivada. Tem-se para uma tal equação o teorema seguinte sobre a
unicidade da solução.
Teorema — Se na equação
y' = f(^y y)
a função f(x, y) e a sua derivada parcial ^ em relação a y forem
contínuas num certo domínio D do plano Oxy e se (Xo, yo) for um
ponto deste domínio, existe uma solução única y = <p(x) que satisfaz à
condição y — yo quando x — Xq.
Geomètricamente, este teorema significa que existe uma função
y ^ (p ( x \ e uma só, cuja curva representativa passa pelo ponto (xo, yo).
Resulta deste teorema que a equação (1') possui uma infinidade
de soluções diferentes (por exemplo, a solução que passa pelo ponto
(xo, yo)\ a solução que passa pelo ponto (;co, >^i); a que passa pelo
ponto (xo, >^2), etc., uma vez que estes pontos se encontram no
domínio D),
À condição para que a função y deva tomar o valor dado yo
quando x = Xo chama-se condição inicial. Muitas vezes escreve-se-la
sob a forma 1
y\x=XQ — £/o«
Definição— 1. Chama-se solução geral duma equação de 1.® or­
dem a uma função
y = i^{x, C), ( 2')
que depende duma constante arbitrária C e que satisfaz às seguintes
condições:
fl) ^tisfaz à equação ’ diferencial qualquer que seja o valor
concreto da constante C;
b) qualquer que seja a condição inicial y = yo quando x = Xo
isto é, {y)x=x^ = yo^ pode-se determinar um valor C = Co tal que a
função y = <p(x, Co) verifica a condição inicial dada. Supõe-se, então.
§ 3] EQUAÇÕES DIFERENCIAIS 19
que os valores Xo e yo pertencem ao domínio de variação das variáveis
jc e no qual são observadas as condições do teorema de existência e
da unicidade da solução.

2. Procurando a solução geral duma equação diferencial, somos


conduzidos muitas vezes a uma relação da forma
0)(x, y, C) = 0, (2')
não resolvida em y. Obtém-se a solução geral resolvendo esta relação
em relação a y. Todavia, não é sempre possível exprimir y a partir
de (2') por meio de funções elementares; conserva-se a solução geral
sob a forma implícita. Uma igualdade da forma O (x, y, C) = 0\ que
dá implicitamente a solução geral, chama-se integral geral da equação
diferencial.

Definição — 2. Chama-se solução particular a toda a função


y — [x, Co) deduzida da solução geral y = ^ {x, C), pondo nesta
última C = Co. A relação O (x, y, Co) = 0 diz-se, então, um integral
particular da equação.
Exemplo — 1. A equação de primeira ordem
dy ^ y
dx X
Q
tem como solução geral y = — \ pode-se verificá-la por uma simples substituição
na equação.
Procuremos a solução particular que satisfaz às condições iniciais:
Q
yo = 1 cuando ;co = 2. Substituindo estes valores na fórmula í/ —
obtém-se 1 = — , ou seja, C = 2. A solução particular procurada é, pois, a
9 ^
função ^
X

Sob o ponto de vista geométrico, o integral geral representa uma


família curvas planas que dependem dum parâmetro C. Estas curvas
chamam-se curvas integrais da curva diferencial dada. Um integral par­
ticular é Tepresentado por uma curva desta família que passa por um
dado ponto do plano.
Assim, no exemplo considerado, o integral geral é representado
geomètricamente pela família de hipérboles !/ = ^ ^ o integral parti­
cular, definido pela condição inicial dada. pela hipérbole que passa pelo
ponto Mo (2. 1). Representou-se na figura 245 as curvas da família cor-
respondentes aos diversos valores C = y , C = 1 , C = 2, C = — 1, etc.
20 CALCULO DIFERENCIAL E INTEGRAL

Para facilitar os raciocínios, chamaremos no seguimento solução


da equação não sòmente à função y = 9 (ar, Co) que satisfaz à equação
proposta, mas ainda à curva integral correspondente. Sendo assim, falar-
-se-á, por exemplo, da solução que passa pelo ponto (xo, yo).

Nota — A equação 4 ^ = — não admite solução que passe pelo


^ ax X
ponto do eixo Oy (fig. 245). Tal deve-se ao facto de ò segundo membro

c^-yi \c-Vz

___ 0

IV=-/
c->/z
F i | g. 245.

da equação ser indeterminado para x = 0 e, por conseguinte, não ser


contínua.

Resolver ou integrar uma equação diferencial consiste em:


d) procurar uma solução geral ou o seu integral geral (se as
condições iniciais não forem dadas) ou
b) procurar uma solução particular que satisfaça às condições
iniciais (se as houver).

3. Demos a interpretação geométrica das equações diferenciais


de primeira ordem.
Seja dada uma equação diferencial resolvida em relação à derivada:
| = / ( x , y) <l')
EQUAÇÕES DIFERENCIAIS 21

e seja y = (p{x, C) a sua solução geral. Esta solução geral define a


família das turvas integrais no plano Oxy,
A equação (1') determina para todo o ponto M, de coordenadas
;c e um valor da derivada ^ , isto é, o coeficiente angular da tan­
gente à curva integral que passa por esse ponto.
Por conseguinte, a equação diferencial (1') define um conjunto
de direcções ou, como se disse, um campo de direcções no plano Oxy,

Do ponto de vista geométrico, a integração duma equação dife­


rencial consiste em encontrar as curvas cuja tangente em cada ponto
se confunde com a direcção do campo nesse ponto.
Representou-se na figura 246 o campo de direcção definido pela
equação diferencial
^ = _ L
dx X
4. Consideremos, em seguida, o seguinte problema.
Seja dada uma família de curvas que dependem dum parâmetro C:
y = (f{x, C) (2)
tal que para todo o ponto do plano (ou dum domínio no plano)
apenas passe uma curva desta família.
Pergunta-se: qual a equação diferencial que admite esta família
de funções para integral geral?
22 CALCULO DIFERENCIAL E INTEGRAL [Ch . X III

Acha-se derivando a relação (2) em relação a x:

o. (3)
Uma vez que apenas passa uma só curva da família para qual­
quer ponto do plano, cada par de valores z, y define um único valor C
na equação (2). Substituindo este valor C na relação (3) encontra-se ^
como função de jc e Obtém-se, assim, uma equação diferencial que
é verificada para todas as funções da família (2).

Daqui resulta que para estabelecer a ligação entre jc e y e ^


dx
isto é, »para escrever a equação diferencial que admite para integral
geral a fórmula (2), é preciso eliminar C nas expressões (2) e (3).
Exemplo — 2. Encontrar a equação diferencial da família de parábolas
y= (fig. 247).
Acha-se derivando em relação a jc a equação da família
dy
= 2Cx.
dx
Substituindo C = ~ definida peía equação da família, obtém-se a equação
diferencial dada:
dy _ 2y
dx X '
Esta equação tem um sentido quando isto é, em todo o domínio
que não corte o eixo Oy.
4] EQUAÇÕES DIFERENCIAIS 23

§ 4. Equações com variáveis separadas e separáveis.


Problema da desintegração do rádio

Consideremos uma equação diferencial da forma

(1 )

em que o segundo membro é o produto duma função que depende


somente de x por uma função que depende somente de y. Transformemo-la
como se segue (supondo /2 (y) 0):

1
dy = /i (x) dx. ( 1 ')
/2Íy)
Supondo que a função y úe x é
conhecida, pode-se considerar (1'), como a
igualdade dé dois diferenciais, e as suas
primitivas distinguir-se-ão duma constante.
Integrando o primeiro membro em relação
a y e o segundo em relação a jc. obtém-se:

j — [ / i ( x ) í i r + C. Fig. 248
Obtivemos uma relação entre a solu­
ção y, a variável independente ;c e a constante independente ac e a>
constante arbitrária C, isto é. que se tem o integral geral da equação (1).
1. A equação diferencial (!')
M {x) dx-\- N (y) dy = 0 (2)
chama-se equação com variáveis separadas. Como se acaba de demons-.
trar, o seu integral geral é
í M ( x ) ã x + \ N ( y ) d y = C.
Exemplo — 1. Seja a equação de variáveis separadas
^ xdx-{-ydy — 0,
O seu integral geral, é « ,,2

o primeiro membro, não sendo negativo, implica o mesmo para o


segundo. Designando 2Ci, por C*, ter-se-á:
a:2 + £/2 = C'2.
É a equação de uma família de circunferências -concêntricas (fig. 248J
com centro na origem das coordenadas e de raio C.
24 CALCULO DIFERENCIAL E INTEGRAL

2. Uma equação da forma


il/i (x) N , (y) dx + {x) (y) dy = 0 (3)
chama-se equação de variáveis separáveis. Pode-se reduzir a uma equa­
ção de variáveis separadas (*(*) dividindo os dois membros pela expres­
são N i (y) M ^ i x ) :
M i{x )N j{y ) M ,{x)N ^(y)
N i(y )M ,{ x ) ~ ^N i(y )M A x ) ^
ou
M Ax) NAy) dy = 0.
dx
M Ax) * NAy)
que é uma equa^o do tipo (2).
Exemplo — 2. Seja a equação
dy y
dx
Separemos as variáveis;
dy dx
y X

Encontra-se, por integração:

í f = - í ^ + ^ ’
logo,

Log I y 1= — Log I X 1+ Log I C 1(•*) ou Log | j/1 = Log


Q
donde se óeúuz a solução geral y = — .
Exemplo — 3. Seja a equação
(1 + x) y dx + ( í — y ) x d y = 0 .
Separemos as variáveis:
1+ ^ d x + - ^ - ^ d y = 0 ; j á x + ( y — 1 j íij/= 0

Integrando, obtém-sé:
h o g \x \ + x + L o g \y \ — y = C ou Log | x i/1 + x — i/ = C
que é o integral geral da equação proposta.

(♦) Estas transformações são legítimas sòmente num domínio em que,


nem (y) nem M- (x) se anulem.
(**) Tendo em consideração as transformações ulteriores, designamos a
constante arbitrária por log | C l , o que é legítimo, porque log | C | (quando
C ^ O ) pode tomar qualquer valor de — oo a -H oo .
EQUAÇÕES DIFERENCIAIS 25

Exemplo — 4. A velocidade de desintegração do rádio é directafnente


proporcional à sua massa no instante considerado. Determinar a lei de variação
da massa do rádio em função do tempo, sabendo que no instante / = 0 a
massa era mo.
Determina-se a velocidade de desintegração como se segue. Seja m a
massa no instante / e m -H Am a massa no instante / -b A/. A massa desintegrada

no tempo At é Am. A relação é a velocidade média de desintegração.

O limite desta relação quando A/ —> 0


Am dm
dT
é a velocidade de desintegração no instante t.
Segundo as condições do problema
àm ,
-dT =
em que k é um coeficiente de proporcionalidade (k > 0). Introduzimos o sinal
menos uma vez que a massa decresce quando o tempo cresce e que, por
•X
conseguinte, dm
------<C 0
d^ • o
A equação (4) é uma equação de variáveis separáveis. Separemos as
variáveis:
^ Ê = -k d t.
m
Integrando, obtém-se:
Log m = — kt — Log C,

m = Ce~~^i, (5)

Dado que a massa do rádio era mo no instante r = 0, C deve satisfazer


à relação
= C,
Substituindo o valor de C na igualdade <5), obtém-se a expressão pro­
curada (ver fig. 249) da massa em função do tempo:
m = m^e (6)
26 CALCULO DIFERENCIAL E INTEGRAL

Dsduz-ss O caeficiente k das observações que se seguem. Seja a % a


fracção da massa inicial desintegrada no tempo /o . Tem-se, pois, a relação
- Mq

donde
— A:ío=Log ^1-—
100/

( ^ “ íõõ) •
Desta maneira estabeleceu-se que para a rádio k = 0,000436 (sendo a
unidade do tempo o ano).
Substituindo este valor de k na fórmula (6), obtém-se
^ = ^0^-0.000436/^

Encontramos o período de desintegração do rádio, isto é, o lapso de


tempo durante o qual se desintegra metade da massa inicial do rádio. Substi­
tuindo nesta última fórmula em vez de m, obtém-se a equação que define
o período T procurado:

mo
= moe - 0 . 0 0 0 4 3 6 T
donde
—0,000436r= — Log2
ou
Log2
T= :1590
0,000436
Notemos que outros problemas da física c da química igualmente con­
duzem à equação da fórmula (4).
Nota — A equação diferencial de variáveis separadas mais sim­
ples é:

^ = / (x) ou dy = f (x) dx.

O seu integral geral, escreve-se


y = U i ^ ) d x + (^-
Ocupamo-nos deste tipo de equação no capítulo X.

§ 6. Equações homogéneas de primeira ordem


Definição— 1. Diz-se que a função f{x, y) é uma função homo­
génea de grau n em relação às variáveis at e y se se tiver para todo o X
f(Xx, Xy) = r f { x , y).
EQUAÇÕES DIFERENCIAIS 27

Exemplo— 1. A função / (x, = + é homogénea c de grau 1,


porque
f(kx, Xy) = f (Xx)3 + (Xy)» = X f x3 + y3= X/ (x, y).

Exemplo — 2. f {x, y) = xy-~y^ 6 uma função homogénea do segundo


grau, porque (Xx) (Xy) — (Xy)^ = [xy — y*].

Exemplo — 3. / (x, y) = é uma função homogénea de grau zero.


xy
porque — íí? , isto é , / (Xx, Xy) = / (x, y)oa /(X x, Xy) =
(Xx) (Xy) xy ’
= X®/(x, y).
Definição — 2. A equação de primeira ordem

!= /(-.» ) (í)

diz-se homogénea em relação a x e 3^ se a funçáo / (x, j) for uma função


homogénea de grau zero em relação a x e y.
Resolução da equação homogénea — Tem-se. por hipótese, /(Xx,
^y) = / yy Fazeijdo nesta identidade X = — , obtém-se:

isto é, que uma função homogénea de grau zero depende sòmente da


relação ^ .
A equação (1) escreve-se. então, neste caso. sob a forma

Façamos a substituição:
f)-
u = i'. isto é, y = ux.
X

Tem-se. então:
dy , du
dx dx^*

Substituindo esta expressão da derivada na equação (1'). obtém-se


u + x - ^ = / ( l , u).
28 CALCULO DIFERENCIAL E ÍNTEGRAL

É uma equação de variáveis separáveis:


du , ^ du dx
x — = f{\,u)—u ou — ------ --------= — .
dx t{\,u)— u X
Por integração encontra-se

f =f-^+c.
ò f {\, u) — U ò X
Substituindo após integração obtém-se o integral da
equação (1').
Exemplo — 4. Seja a equação
dy _ xy
dx ~~ x^ — y^
Tem-se no segundo membro uma equação homogénea de grau zero, pois
a equação proposta é homogénea. Façamos a mudança de variáveis 1- = u,
X
Então:
dy , du

du u du
u-\- X
d x ~ í — u^ ’ dx 1 — 1/2 *
Separando as variáveis, tem-se:
(1—U2) du
X \ u^ u ) X
e por integração:
1 1
—Log| u | = L o g |x | + L o g |C l ou — — hog\uxC\,
2u^
Substituindo i / = i , obtém-se o integral geral da equação inicial:

É impossível exprimir aqui y em função de x por meio das funções ele­


mentares. Mas exprime-se fàcilmente x em função de y:
x = y y —2 L o g |6 'i/|.
Nota — A equação
M{ x , y) dx + N{x, y)dy = 0
aoenas será homogénea se M (x, y) e N (x, y) forem funções homo­
géneas do mesmo grau. Daqui resulta que a relação de duas funções
homogéneas dum único e mesmo grau é uma função homogénea de
grau zero.
EQUAÇÕES DIFERENCIAIS 29

Exemplo — 5. As equações

(2x + 3y) dx + (x— 2y) dy = 0,


. {x^+y^)dx— 2 x y d y ^ 0
sao homogéneas.

§ 6. Equações redutíveis a equações homogéneas


Reduzem-se a equações homogéneas as equações da forma
dy _ a x + by - f c
dx UiX + bty + Cl
Se Cl = c = 0, a equação (1) é, evidentemente, homogénea. Su­
ponhamos agora que c e Ci (ou um deles) não são nulos. Façamos a
mudança de variáveis:
x = xi + h, y = yi -f k.
Então,
dVi
dx (2)
dxi
Substituindo na equação (2) as expressões das qaantldades x, y,
obtém-se:
dyi axi byi-\- ah-\- hk-{- c
(3)
dxi aiXi 6ij/i -t- a j i -f- bik Ci

Escolhamos h t k à» maneira que verifiquem as equações

= 0.
a h - \- b k -\-c = 0, 1
(4)
cL\h -f- h\h -(“ Cl = 0. /
isto é, definamos h e k de modo que seja solução do sistenaa de
equações (4).
A equação (3) torna-se, então, homogénea:
d y i ^ axi - f byt
dxi a^Xi -(- biyt
Resolvendo esta equação e voltando às antigas variáveis x e y
segundo as fórmulas (2), obtém-se a solução da equação (1).
O sistema (4) não tem solução quando
a b
= 0.
bt
30 CALC?ULO D IFEREN CIA L E INTEGRAL

isto é. quando abi — a^b. Mas, então, ^ = y = A. ou «i = Xa, = Xô,


e a equação (1) pode ser posta sob a forma
dy {ax + b y ) + c
(5)
dx X {ax + by) +
A substituição
z = ax by (6)
conduz, então, a equação dada a uma equação de variáveis separáveis.
Com efeito.
dz , ^dy
dx ' cte
donde
a_ "'
dx b dx b ' ^ '

Substituindo as expressões (6) e (7) na equação (5), obtém-se


i dz a _ z-{- c
b dx b Xz-\- Cl
que é uma equação de variáveis separáveis.
O processo utilizado para integrar a equação (1) apIica-se igual­
mente à integração da equação

— ^ \
dx Vai^' 4"
+ biy + q
Cl / ’
em que / é uma função arbitrária contínua.
Exemplo — 1. Seja a equação
dy + 3
dx X — y — 1

Para a reduzir a uma equação homogénea façamos a substituição x = Xi + h;


y = yi + k. Então,
diji Ji + t/i + X+ fe—3
dxi — i/i + X— k — 1
Resolvendo o sistema de duas equações

h-\-k — 3 = 0 ; h — k — 1 = 0,
vem.
A= 2, k = \.
Obtém-se assim a equação homogénea
àyj__ xi + yi
dXi 1/i
EQUAÇÕES DUFERENCIAIS 31

que se resolve fazendo a substituição

I L = u:
tem-se:
àyi1 du
dx^
, du 1 4 -m

e obtém-se uma equação de variáveis separáveis:


du l-|-u2
dxi 1
Separemos as variáveis:
1“ U , dx4
-------- du = ------
1+
Integrando, tem-se:

arc tg u —Y Log (1 + u^) = Log xj + Log C,

arc tg u = Log {Cx^ ~ \/T + ü ^


ou
Cx^
V\
Substituindo nesta última igualdade----
Xi em vez de ii, obtém-se:
j--------- arctg —
c y ífT ír? = e **.
Por tim, passando às variáveis jc e 3^, obtém-se:

C l / ( í - 2 ) 2 + (j,_ l)2 = * “ * -2 .
Exemplo — 2. Não se pode fazer a substituição x= ^xi-\-h, y = y| + /p
na equação
2 i + y —1
4 r+ 2 y + 5 ’
porque o sistema de equação que serve para definir h c k é incompatívd (sendo
2 1
o determinante ^ 2 coeficientes das variáveis nulo).
Pode reduzir-se e§ta equação a uma equação de variáveis separáveis fazendo
a substituição ;
2x + p = 2.
Tem-se, então, y' = z' — 2 e a equação toma-se
z —1
Z '-2 :
2z + 5

5z + 9
^2z + 5
32 CALCULO DIFEREN CIA L E INTEGRAL

Deduz-Be
| - z + - ^ L o g |5 2 + 9 | = x + C.

Como z = 2x + y obtém-se, finalmente, a solução da equação dada sob


a forma
( 2 x + j / ) + - ^ Log 110x + 5v + 91 = x + C

OU
ÍO i/ —5x -|- 7 Log I lOx -f 5i/ + 9 I= C 1 ,
isto é, sob a forma implícita.

§ 7. Equações IKreares de primeira ordem


Definição — Chama-se equação linear de primeira ordem a uma
equação linear em relação à função desconhecida e à sua derivada.
Escreve-se
^ + P{ x) y = Q (x), ( 1)

em que C(x) e Q U) são funções contínuas de x dadas (ou constantes).


Resolução da equação linear (1) — Vamos procurar a solução da
equação (1) sob a forma de produto de duas funções de ;c:
y = u{x)v (x) ( 2)

Poder-se-á tomar arbitràriamente uma destas funções; a outra


será, então, definida por (1).
Derivando os dois membros da igualdade (2) encontra-se:
dy_
dx ^ d x ^ ^dx'
Substituindo a expressão da derivada ^ obtida na equação (l),
dx
ter-se-á
dv . du

ou

(3)

Escolhamos a função v 3e modo que se tenha


p4-P v~0. (i)
dx
EQUAÇÕES D IFEREN CIA IS 33

Separando as variáveis nesta equação diferencial em v, tem-se:

- = -P dx.
V

Int^rando. obtém-se
— Log Cl - f Log v = — I P d x
ou

Como nos basta ter uma solução qualquer não nula da equa­
ção (4), tomaremos para função v (jc):

v{x) = e ^ ,
(5)
em que \ P dx é uma primitiva qualquer. É evidente que v (a:) ^ 0.
Substituindo o valor encontrado de v (jc) na equação (3). obtém-se
tendo em atenção que dv i .
dx ~ /

ou d u __ Q{x)
dx v{x)

donde Q{x)
dx - f C,
(x)

Substituindo na fórmula (2), obtém-se, finalmente:

<?(x)
r= y (x ) dx-f-Ci;(x). ( 6)
(x)

'Nota — É evidente que a expressão (6) não muda se se tomar


em vez da função v (x) definida por (5) uma função qualquer Vi (x) =
= Cv{x)
3
34 CALCULO D IFEREN CIA L E INTEGRAL

Com efeito, obtém-se, substituindo Vi (jc) em vez de v (x):

yz=Cv{x) f 9 dx -f- CCv (3:).


J Cv{x)
C desaparece do primeiro termo do segundo membro; o pro­
duto CC do segundo termo é uma constante arbitrária que se pode
designar simplesmente por C, e voltamos a encontrar a expressão (6).
Se se fizer j dx = (p (x), a equação (6) toma a forma
y=.v(x)(p(x) + Ci;(x). (6')
É evidente que se está no integral geral, porque se pode escolher C
de maneira que seja satisfeita a condição inicial:
y = Uo quando x = Xq.
C é determinado pela equação
U o = ^ (^0) (^0) + C u {xq).
Exemplo — Resolver a equaçSo

Resolução — Façamos y = uv; tem-se

du dv , du
-^= U -y - + ^-V .
dx dx dx

Substituindo a expresão 4 ^ na equação dada, obtém-se:


dx
dv , du
u —— \- —— V - ■UV={x+i)9,
dx dx x-\-í
( dv 2 \ du
(7)
Para a determinação de v, obtém-se a equação

dv 2
i;= 0,
dx x+1
isto é,
dv 2dx
V x+ í ’
donde
Log y = 2 Log (0:4-1)
ou
y= (r+ 1)2.
EQUAÇÕES DIFEJRENCIAIS 35

Substituindo a expressão da função v na equação (7), obtém-se pela deter­


minação de II, a equação

donde
„ = (£ ^ + c .

Por conseguinte, o integral geral da equação dada escreve-se


(x + 1)"- + C(X + 1)2.
y= -
A família obtida é a solução geral. Qualquer que seja a condição inicial
(xo, y o ) em que jco^ — 1, pode-se escolher sempre C de modo que a solução
particular correspondente satisfaça à condição inicial dada. Assim, a solução
particular que satisfaz à condição yo = 3 para Xo = 0 é definida como se segue:

3 _ ( 0 + l ) ‘ _j_C(0+l)i;

Por conseguinte, a solução particular procurada é


(^ + 1 )" ■ 5
y- 2 -+ y (x + D * .
Todavia, se se toma a condição inicial (jco, yo) de modo que Xo = — 1,
não se pode deduzir uma solução particular que satisfaça a esta condição.
2
Tal deve-se ao facto da função P ( x ) = -----------ser descontínua no ponto
X -j-1
jCo = — 1 e as condições do teorema da existência da solução não serem observadas.

§ 8. Eqaação de Bernoalli
Consideremos uma equação da forma (♦)
dy
- - \ - P { x ) y ^ Q { x ) y ’^,
( 1)

em que P{x) q Q (x) são funções contínuas de x (ou constantes) e


n =7^ 0. (caso contrário não se teria uma equação linear).

(*) Nesta equação se conduz o problema sobre o movimento do corpo


se a resistência do meio F depender da velocidade: F =

A equação do movimento será, então,


du . dv
-
dt m m

3*
36 CALCULO D IFEREN CIA L E INTEGRAL

Esta equação que se chama equação de Bernoulli, reduz-se a uma


equação pela seguinte transformação.
Dividindo todos os termos da equação por obtém-se:

( 2)
Façamos, em seguida, a substituição:
z = z/“ " + \
Então,
dz , , -ndy
= ( ^ n + í)y
dx dx
Substituindo na equação (2), obtém-se:

ÉL ^ n + l) P z = { - n + i ) Q .
dx
É uma equação linear.
Calculando o seu integral geral e substituindo em e a sua expres­
são obtém-se o integral geral da equação de Bernoulli.

Exemplo — Resolver a equação

^ - \ - x y = x^y^. (3)
dx
Resolução — Dividindo todos os termos por y’, obtém-se:
y~^y' + xy~^ = x^. (4)
Introduzamos a nova função z= y~^.
Tem-se, então,

Substituindo na equação (4), obtém-se:


dz
— 2xz = —2x3. (5)
dx
É uma equação linear.
Achemos o seu integral geral:
dz dv . du
z = uv\ —j — = u - 5 -------\- - 3 —
V.
dx dx ^ dx
dz
Substituámos na equação (5) as expressões de z e de
dx
dü du
u —— 1--3— v— 2xuv= —2x3
dx dx
ou
EQUAÇÕES D IFEREN CIA IS 37

Anulemos a expressão entre parêntesis:

-=----- 2xv = 0 ; — = 2x d x ;
dx V
Log u = x^\ u=
Obtém-se, para definir u, a equação

-2 l3 .
dx
Separemos as variáveis:
d u = —2e“ * V dx, w = —2 ^ e“ **x® d * + C .
Integrando por partes, vem
u= + C;
z = ui; = -|-1+ Ce^^.
Tem-se pois o integral geral da equação dada:
1
y-^ = x^-{-í-]-Ce^2 ou y
K i a + l + Ce**

Nota — Tal como para as equações lineares, demonstra-se que


se pode encontrar a solução da equação de Bernoulli sob a forma de
produto de duas funções:
y = u { x ) v (x),
onde v(x) é uma função arbitrária não nula, que satisfaz à equação
v' Pv = 0.

§ 9. Equações de diferenciais totais

Definição — A equação
M {x, y) dx + N (x, y) dy = 0 ( 1)
chama-se equação de diferençais totais se, M {x, y) e N (x, y) forem
funções continuas deriváveis tais que
dM dN
(2 )
dx

e as derivadas parciais e ^ sejam contínuas num certo domínio.

Integração de diferenciais totais — Mostremos que se o primeiro


membro da equação (1) for um diferencial total, a condição (2) é
38 CALCULO D IFERENCIA L E INTEGRAL

observada, e inversamenle, se a condição (2) for observada, o primeiro


membro da equação (1) é o diferencial total duma certa função u (jc.
isto é. que a equação (1) é da forma

du {x, y) = 0 (3)
cujo integral geral é da forma
U (x, y) = C.
Em primeiro lugar suponhamos que o primeiro membro da
equação (1) é o diferencial total duma certa função u (j :, y), isto é,

M{ x, y)dx + N ( x , y) dy = du = ^ d x + ^ d y ;
então.
du du
M = ^; N = ^ . (4)
dx dy
Derivando a primeira relação em ordem a 3^ e a segunda em
ordem a x, obtém-se:
dM d\ dN d^u
dy dxdy dx dy dx
Supondo que as derivadas segundas são contínuas, tem-se
dM dN
dy dx
isto é, que a igualdade (2) é uma condição necessária para que o
primeiro membro da equação (1) seja o diferencial total duma certa
função u (jc. >^). Mostremos que esta condição é também suficiente,
isto é. que se as igualdade (2) tiverem lugar, o primeiro membro
da equação ( 1 ) é o diferencial total duma certa função u ( jc , y).
Da relação
g = « ( x , V)

deduz-se:
u = l M { x , y) dx + Cp (i/),
3C0
em que jco é a abcissa dum ponto arbitrário no domínio de existência
da solução.
Integrando em relação a jc , suponhamos y constante, e, por con­
seguinte, a constante de integração é substituída aqui por uma função
(piy) de modo que seja observada a segunda relação de (4).
EQUAÇÕES D IFERENCIA IS 39

Para este efeito, derivemos (**) os dois membros d i última igual­


dade em relação a y e igualemos o resultado a N (x, y):

= N{x, y).
dy
J ày
Xo
dM dN
Mas c o m o ^ ^ = ^r— , pode-se escrever:
dy dx
X

f - dx + cp' {y) = N, isto é, ^ v) l*o "t" ^ (y) — ^ í/O


J dx
Xo

ou
N (x, y) - N {xo. y) + 9 ' (í/) = ^ ^ I/)*
Por conseguinte,
cp' {y) = ^ (^ 0 . y)
ou
cp {y) = \ ^ (^0, y) <^y +

Consequentemente, a função u (x, y) será da forma


y
u = j M (x, y ) d x + I N {xq, y) dy - f Cj.

P(jCo, }^o) representa aqui um ponto na vizinhança do qual existe a


solução da equação diferencial (1).
Igualando esta expressão a uma constante arbitrária C, obtém-se
o integral geral da equação (1):

\ M {x, y) d x - \ - l N (xo, y) dy = C. (5)


^0 Ih

(*) O integral ^ M (x, y) dx depende de y. Para encontrar a derivada


*0
deste integral em relação a y, é preciso derivar em relação a y a função sob
o sinal soma: ^
^ f ^ dM
~ ^ ] M ( x , y)dx=--^ - ^ d x .
xo Xo
Isto resulta do teorema de Leibnitz sobre a derivação dum integral
definido em relação a um parâmetro (ver § 10, capítulo XI).
40 CALCULO D IFEREN CIA L E INTEGRAL

Exemplo — Seja a equação


*2àX —
—^ (ix+ ^— -— dy = 0.

certifiquemo-nos de que se trata de um diferencial total.


Designemos
2x
M=
y*
então, 6x d_N 6x
ày ~~ dx
A condição (2) é observada para O primeiro membro da equação
dada é, pois, o diferencial total duma certa função u (x, y). Procuremos essa
função.
_ du 2x
Como — = —5-, tem-se
dx
« = ^ -|F < i* + 'P (y )= -^ + <P(y).

em que <p (y) é uma função de y que é preciso determinar.


Derivemos esta relação em ordem a y e tenhamos em consideração que
du __ _y^ —3x^
dy y*
Tem-se

por conseguinte.
1 1
<P'(í')=-rr> =

Logo, o integral geral da equação proposta é

4 - ^ = c .

§ 10. Factor integrante


Suponhamos que o primeiro membro da equação
M{x, y)dx + N{x, y)dy = 0 (1)
não é um diferencial total. Por vezes é possível escolher uma função
/I (x, y) tal, que se se multiplicar o primeiro membro da equação
proposta por esta função, este primeiro membro se transforma num
diferencial total. A solução geral da equação assim obtida coincide
com a solução geral da equação proposta; a função /i (x, y) diz-se um
factor integrante da equação (l).
EQUAÇÕES DIPETRENCIAIS 41

Para encontrar um factor integrante /x, procede-se como se segue;


multipliquemos os dois membros da equação dada pelo factor integrante,
ainda desconhecido, fx:
liM dx + (xiV dy = 0.

última equação seja uma equação


e suficiente que se tenha:
d(\iM ) d{]iN)
»
dy dx
isto é.
dM dN
= u -------f iv ,
ày dy dx dx
ou ainda
. dy, ( dN dM \
= U -------
dy dx V dx dy ) '
Obtém-se, calculando o quociente dos dois membros desta última
equação por /i:

y ^Log|Lt^ ^ 7 V dM
( 2)
dy dx dx dy
É evidente que qualquer /x {x, y) que satisfaça a esta última
equação é um factor integrante da equação (1). A equação (2) é uma
equação de derivadas parciais da função desconhecida ju, que depende
de duas variáveis x q y. Demonstra-se que, nas condições determinadas,
ela possui uma infinidade de soluções e resulta que a equação (1)
tem um factor integrante. Mas no caso geral, é mais difícil de deter­
minar >^) em (2) do que integrar a equação proposta (1). É sòmente
nestes dois casos particulares que se chega a determinar a função /x {x, y).
Suponhamos, por exemplo, que a equação (1) admite um factor
integrante dependente sòmente de y. Então,
^Log|Ll
:0
dx
e obtém-se para /x uma equação diferencial ordinária:
dN dM
d Log jui dx dy
dy M
42 CALCULO D IFEREN CIA L E INTEGRAL

donde se determina (para uma quadratura) Log/x e portanto /x. É claro


que se não pode proceder assim se a expressão
ÔN dM
dx dy
M

não depender de jc.


dN dM
Duma maneira análoga, se a expressão não depender
de y mas somente de x encontra-se fàcilmente o factor integrante que
depende somente de x.
Exemplo — Resolver a equação
{y + xy2) dx— x d y = 0.
Resolução — Aqui M = y + xy^; N = - - x ;
ÕM, , ^ dN . dM dN
— _ l + 2 iy ,
Daí resulta que o primeiro membro da equação não é um diferencial
total. Vejamos se esta equação admite um factor integrante, dependente sòmente
de y. Terido em atenção que
dN dM
dx dy _ —1 — 1 — 2xy _ ^
M “ y + xy"^ ~ y
conclui-se que de facto assim é. Achemo-lo:
dLog í i _____^
dy
donde

L o g p = —2Logí/, soitp, = - ^ .
Obtém-se, após multiplicação de todos os termos da equação proposta
pelo factor integrante n, a equação

de diferenciais totais = = ---- .Resolvendo esta equação, encontra-se


V dy dx í/2 )
o seu integral geral:
x x^
- + - ^ + 0= 0
ou
2x
P= x^-\-2C
EQUAÇÕES D IFEREN CIA IS 43

§ 11. Envoltório duma família de curvas

Seja uma equação da forma


o (x, y, C) = 0, (1)
em que x q y são as coordenadas cartesianas variáveis e C um parâ­
metro susceptível de tomar diversos valores fixos.
Para cada valor dado do parâmetro C, a equação (1) define uma
certa curva no plano Oxy. Dando a C todos os valores possíveis, obte-

F i g . 250. F i g. 251.

mos uma família de curvas dependentes dum parâmetro. Por conseguinte,


a equação (1) é a equação duma família de curvas dependente dum
parâmetro (ela contém somente um parâmetro arbitrário).
Definição — Chama-se envoltório L duma família de curvas com
um parâmetro, a uma curva tangente em cada um dos seus pontos
a uma curva da família (fig. 250).
Exemplo — 1. Consideremos a família de curvas
{x-C)^ + y^ = R^,
em qu3 R é uma constante e C um parâmetro.
É a equação duma família de círculos de raio R centrados sobre o
eixo Ox. É evidente que esta família admite como envoltório as rectas y = R,
y = -R (fig. 251).
Equação do envoltório duma família de curvas. Seja a família
de curvas
0) (x, y, 0 = 0 (1)

dependente dum parâmetro C.


Suponhamos que esta família tem um envoltório cuja equação pode
ser posta sob a forma y = (p (jc), sendo ^ (jc) uma função contínua
44 CALCULO D IFERENCIA L E INTEGRAL

derivável. Consideremos um ponto Af (jc, y) do envoltório. Este ponto


pertence também a uma certa curva da família (1). Corresponde a
esta curva um determinado valor do parâmetro C que, para (x, y)
dado, é definido pela equação (1) C = C ( jc , y). Por conseguinte, tem-se
para todos os pontos do envoltório a igualdade
(^{x, y, C{x, í/)) = 0. (2)
Suponhamos que C ( jc , >^) é uma função derivável não constante
em nenhum intervalo dos valores de jc , y considerados. Calculemos a
partir da equação (2) do envoltório o coeficiente angular da tangente
ao envoltório no ponto M {x, y). Derivemos a igualdade (2) em relação
a JC, considerando y como função de j c :

dO dOdC dO dO dC
í/' = 0
dx dC dx dy dC dy
ou
dC dC .
-j- ^ y y “V = 0. (3)
L dx dy
Deduz-se, em seguida, o coeficiente angular da tangente no ponto
M{x, y) à curva da família (1) da igualdade
(4)
(C é constante na curva dada).
^ Suporemos Oy 0, senão tomaríamos jc como função e y como
variável. Dado que o coeficiente angular k do envoltório é igual ao
da curva da família, deduz-se de (3) e (4):

Mas como para o envoltório C {x, y) ^ constante,


dC , d c

e tem-se, pois, para os pontos desta última


a)c(:r, y, C) = ü. (5)
Por conseguinte, determtna-se o envoltório pelas dtms equações
seguintes:
O c,(x y, C) = 0, I (6 )
<I>c {xc, y, C) = 0. J
EQUAÇÕES D IFEREN CIA IS 45

Inversamente, se, eliminando C destas equações, se obtém y = (p (x),


em que ^ (x) é uma função derivável t constante sobre esta curva,
então y = (p(x) é a. equação do envoltório.
N ota— 1. Se uma certa função y = (p{x) representa o lugar
geométrico dos pontos singulares da família (1), isto é, pontos tais que
O i = 0 e (l>y = 0, as coordenadas destes pontos verificam igualmente
as equações (6).
Com efeito, pode-se exprimir as coordenadas dos pontos singu­
lares em função do parâmetro C que entra na equação 0)*
x = ^(C ), y = ix(C). (7)
Se se substituir ests expressões na equação (1), obtem-se uma
identidade em C:
(t>lX(C), ii(C), C1=0.

Derivando esta identidade em relação a C, obtém-se:

como se tem, qualquer que seja o ponto singular, as igualdades =


<I)y = 0, resulta que se tem também para estes pontos (1)^ = 0.
Acabamos, pois, de demonstrar que as coordenadas dos pontos
singulares verificam as equações (6).
Assim, as equações (6) definem quer o envoltório quer o lugar
geométrico dos pontos singulares das curvas da família (1), quer uma
combinação duma e doutra. Por conseguinte, tendo obtida uma curva
que satisfaça às equações (6), importa fazer um estudo especial para
determinar se a curva obtida é o envoltório, ou melhor, um lugar de
pontos singulares.
Exemplo — 2. Encontrar o envoltório da família dos círculos dependentes
dum parâmetro C
( x _ Q 2 + y2 —i}2 = 0. ' ‘ iVc
Resolução — Derivando obtém-se a equação da família em relação a C:
2 (x —C) = 0. <
Eliminando C nestas duas equações, obtém-se:
y 2 — = 0 ou y = zh/?.
Resulta de considerações geométricas que o par de rectas obtidas é bem
o envoltório (e não um lugar de pontos singulares, dado que os círculos da
família não têm pontos singulares).
Exemplo — 3. Achar o envoltório da família de rectas:
x c o s a + y sen a — p = 0, (a)
em que a é o parâmetro.
46 CALCULO D IFEREN CIA L E INTEGRAL

Resolução — Encontra-se derivando em relação a a, a equação da família


— a; s e n a + y COS a = 0. (b)

Para eliminar o parâmetro a das equações (a) e (b), multipliquemos a


P"imeira por cos a, a segunda por sen a e juntemo-las membro a membro.
Obtém-se:
x = p co sa.
Encontra-se, substituindo esta expressão na igualdade (b):
i/ = psén a .
ÊleVfemos ao quadrado os dois membros das equações precedentes e
juntemo-los. Tem-se:
x 2 - j- z / 2 = p 2 .

É a equação de um círculo. Este círculo é o envoltório da família de


rectas (e não um lugar de pontos singulares, porque as rectas não têm sin-
gularidadê) ffig. 252X

Exemplo — 4. Achar o envoltório das trajectórias dos projécteis lançados


por um canhão à velocidade v„ sob ângulos diferentes. Supõe-se que os projécteis
são lançados da origem das coordenadas c que as suas trajectórias se encontram
no plano Oxy (despreza-áe a resistência do ar).
Resolução — Achemos em primeiro lugar a equação da trajectória dum
projéctil lançado sob um ângulo a no sentido positivo do eixo Ox. O movimento
do projéctil é a sobreposição de dois movimentos: dum movimento uniforme de
velocidade * Vo na direcção do lançamento e dum movimento de queda sob
a acção da gravidade. A posição do projéctil M será, pois, definida eàn cada
instante t pelas igualdades (fig. 253):
x = vqí cos a,
gt^
yQ = VQt a ---- Y ' •
São as equações paramétricas da trajectória (o parâmetro é o tempo).
Eliminando t, encontra-se a equação da trajectória sob a forma:

j, _ I tg a — 2 , a ’
EQUAÇÕES D IFEREN CIA IS 47

Por fim, introduzindo a notação i g a = k, 75^ = ^» obtém-se:

y = kx — (i k"^). ( 8)

É a equação das parábolas que passam pela origem, de eixos verticais


e de ramos virados para baixo. Oblém-se diferentes trajectórias fazendo variar k.
rx equação (8) é pois a equação duma família de parábolas com um parâmetro,
que são as trajectórias dos projécteis lançados sob diferentes ângulos a e
com ^a velocidade inicial dada (fig. 254).

Procuremos o envoltório desta família de parábolas.


Derivando os dois membros da equação (8) em relação a k, obtém-se:
X— 2akx^ — 0.
(9)
Eliminemos k nas equações (8) e (9). Obtém-se:

É a equação duma parábola de vértice no ponto e eixo é Oy.


Não é um lugar de pontos singulares (as parábolas (8) não tem pontos
singulares). Assim, a parábola
1 2

é o envoltório da família de trajectórias. Chama-se parábola de segurança, porque


a região que se encontra em redor desta parábola está fora do alcance dos
projécteis lançados com a velocidade inicial Vo.
Exemplo — 5. Encontrar o envoltório da família de parábolas semi-
-cúbicas
í/3 — { x — C)2 = 0.

Resolução — Derivemos a equação dada em relação ao parâmetro C:


2(x —C) = 0.
Obtém-se eliminando o parâmetro C das duas equações
í/ = 0.
48 CALCULO D IFEREN CIA L E INTEGRAL

O eixo Ox é o lugar geométrico dos pontos singulares (dos pontos de


reversão de primeira espécie) (fig. 255). Com efeito, procuremos os pontos
singulares da curva
y3__(a:--C)2 = 0,
sendo C fixo. Encontra-se derivando em relação a x e y ;
/ ’i = - 2 ( x - C ) = 0;
Fy = 3í/2 =; 0.
Resolvendo as três últimas equações, encontram-se as coordenadas do
ponto singular: x = C, y = 0, por conseguinte, cada curva da família tem um
ponto singular sobre o eixo Ox.

Os pontos singulares descrevem o eixo Ox completamente quando o


parâmetro C varia duma maneira contínua.
Exemplo — 6. Achar o envoltório e o lugar geométrico dos pontos
singulares da família
( y - C ) ^ - j ( x - C ) 3 = 0. ( 10)

Resolução — Derivando em ordem a C os dois membros da equação (10),


tem-se:
-2 (y -C )+ f3 (x -C )2 = 0

ou
C— (X —C )2= 0. ( 11)

Eliminemos agora o parâmetro C da igualdade obtida (11) e da equa­


ção (10) da família. Substituindo a expressão
y—C = ( i—C)2
na equação da família, obtém-se;
(i_ C )i_ _ (x _ C )3 = 0
-ou

( x - C ) 3 [ ( x - C ) - |] = 0 ;
EQUAÇÕES D IFEREN CIA IS 49

obtém-se assim dois valores de C aos quais correspondem duas soluções do


problema proposto.

Primeira resolução: Segunda resolução:


C= x; rC = x - 32- ;

deduz-sc, pois, da igualdade (11): deduz-se, pois, da igualdade (11):

y —X—(x—x)2 = 0
ou
y = x. y= x—
9 •

Obtivemos duas rectas y = x e y = x — g-. A primeira recta é o lugar


dos pontos singulares e a segunda o envoltório (fig. 256).

Nota — 2. Demonstramos, no § 7, cap. VI, que as normais a


uma curva eram ao mesmo tempo as tangentes à evoluta. Vp-se, pois.

que a evoluta duma curva é o envoltório da família das normais a


esta curva (fig. 257).
Esta nota permite ainda indicar um método para a procura da
evoluta: encontra-se a equação da evoluta duma curva definindo prèvia-
mente a família das normais a essa curva, procurando depois o envol­
tório desta família.
50 CALCULO D IFEREN CIA L E INTEGRAL

§ 12. Soluções singulares das equações diferenciais


de primeira ordem

Suponhamos que a equação diferencial

f { . . y. | ) = 0 ( 1)

tem por integral geral


(S>{x, y, C) = 0. ( 2)

Suponhamos que a família de curvas integrais da equação (2)


tem um envoltório. Mostremos que este envoltório é igualmente uma
curva integral da equação diferencial (1).
Com efeito, o envoltório é tangente em cada um dos seus pontos
a uma certa curva da família, isto é, que ela tem nesse ponto uma
tangente comum com a curva. Por conseguinte, em cada ponto do
envoltório, as quantidades jc, y, / 's ã o as mesmas para o envoltório e
para a curva da família.
Ora para a curva da família as quantidades x, y, / verificam a
equação 0)- Resulta daí que a abeissa, a ordenada e o coeficiente
angular de cada ponto do envoltório verificam t a m b ^ esta mesma
equação, o que significa que o envoltório é uma curva integral e que
a sua equação é uma solução da equação diferencial dada.
Mas o envoltório não sendo em regra uma curva da família,
a sua equação não pode ser deduzida do integral geral (2), particula­
rizando C. É uma solução singular da equação diferencial.
Suponhamos conhecido o integral geral
O (o;, y, C) = 0;

eliminando C desta equação e o da equação (jc, y» C) = 0, obtém-se


uma equação ^ (x, y) = 0. Se esta função verifica a equação diferencial
(mas não pertence à família (2))„ é um integral singular.
Notemos que passam pelo menos duas curvas integrais por cada
ponto do integral singular, isto é, que a unicidade da solução é violada
em cada ponto dum integral singular.

Exemplo — Achar a salução singular da equação


y2 ( l+ y '2 ) = i?2. (♦)
Resolução — Calculemos o seu integral geral. Resolvamos a equação em
relação a y'; ______
dy V fí^-y ^
=± -
dx
EQUAÇÕES D IFERENCIA IS 51

Separando as variáveis, tem-se


yày
-= dx.

Ddduz-se o integral geral por integração:


(x — + =
Vê-se que a família de curvas integrais é a família de círculos de raio R
com centros no eixo das abcissas. O envoltório desta família de círculos é
dada pelas duas rectas y = R.
As funções y = ± R verificam a equação diferencial (*). Elas representam
os integrais singulares.

§ 13. E quação de O a ira u t


Consideremos a seguinte equação

chamada equação de Clairaut, Integra-se introduzindo um parâmetro


auxiliar. Façamos, com efeito, = p ; a equação (1) toma a forma
dx
y = xp + '^{p). (!')
Derivemos todos os termos desta última equação em relação a x,
tendo em vista que p = ^ é uma função de x:
dx

ou

[x + ij) '(p ) ] ^ = 0.

Anulemos, separadamente, cada factor. Obtém-se:


dp
^ = 0 (2)
dx'

X + '^'{p) = 0. (3)
1. A integração de (Z) dá p = C (C = cpnst). Substituindo este
valor de p na equação (1), encontra-se o seu integral geral
y = xC+Vp{C) (4)
que representa, do ponto de vista geométrico, uma janúlia de rectas.
4*
52 CAl/TULO D IFEREN CIA L E INTEGRAL

2. , Tiremos p da equação (3) de modo que seja função de x


e substituamos na equação (10: tem-se
y = x/>(x)-f t[jo(x)] (1'0
que. como vamos ver. é uma solução da equação (1).
Com efeito, encontra-se em virtude de (3):

% = P + [^ + ^ ' ( P ) ] % = P -

Por conseguinte, obtém-se uma identidade quando se substitui a


função (1'0 na equação (1):
xp -I- iK p ) = - f t|)(p).
A solução (1'0 não pode ser obtida a partir do integral geral (4)
na equação (1), paiticularizando C. É uma solução singular; obtém-se
eliminando p das ^quações

y^xp
a: + il)'
K(P) = 0 J
OU, o que equivale o mesmo, eliminando C nas equações

y = xC + ^ (C) ;
X -j- t|?c (C) = 0.
Vê-se, pois. que a solução singular da equação de Clairaut é o
envoltório da família de rectas definidas pelo integral geral (4).

Exemplo — Achar os integrais geral c singular da equação


dy
dy , dx

dy
Resolução — Obtém-se o integral geral substituindo por C:
dx
aC
y — xC +
y i + C2
Para obter a solução singular, derivemos esta última equação em relação
a C:
=0.
(l+,Ga)»/2
EQUAÇÕES D IFEREN CIA IS 53

Obtém-se a solução singular (o envoltório) sob forma paramétrica (sendo


C um parâmetro):

( i+ c y /* ’
oC»
y=

Eliminando o parâmetro C acha-se a dependência entre jc e 3^. Elevando


cada uma destas equações separadamente à potência % e juntando membro
a membro, obtém-se a solução singular sob a forma
*Vs + yVs^flVs.
Ê um astroide. Todavia, o envoltório da família de rectas (e portanto
a solução singular) não está representada pelo asteroide completamente, mas
pela metade esquerda (porque as equações
paramétricas do envoltório mostram que x < 0
(fig. 258).

§ 14. Equação de Lagrange

Assim se chama uma equação da


forma
y = x<p (y ')+ i|) (y'), (1)
Solução
4 particuUtr dy
em que ^ e ^ são fuâções dadas de ~ .
tC
Esta equação é linear em relação
a X e y, A equação de Clairaut, exa­
minada no parágrafo anterior, é um
caso particular da equação de Lagrange
quando ^ ( / ) = / . Tal como a equação de Clairaut, a equação de
Lagrange integra-se introduzindo um parâmetro auxiliar p, Façamos
y'= p \
a equação proposta toma, então, a forma
y = x(p(p) + ip ( j> ) . (1 ')

Derivando em relação a x, obtém-se

P = <P(P) + [«P' (P) + (P)] ^

OU
p - <í>(p) = (p) + f (p)] % ■ ( 1")
54 CALCULO D IFEREN CIA L E INTEGRAL

Encontra-se, logo à primeira vista, certas soluções desta equação


porque se torna numa identidade para qualquer valor constante p = Po
que verifique a condição
Po — <P(Po) = 0.

Com efeito quando p é constante, tem-se = 0 e os dois mem-


dx
bros da equação (1") anulam-se.
A solução correspondente a cada valor de p = po. isto é, ^ = Po.
é uma função linear de x (dado que a derivada ^ é somente constante
para as funções- lineares). Para encontrar esta função, basta substituir
na igualdade (1') o valor p = p„\
I/ = X(í)(po) + ll5(po).
Se esta solução não puder ser -deduzida da solução geral parti­
cularizando a constante arbitrária, é uma solução singular.
Encontremos neste momento a solução geral. Para tal efeito, escre­
vamos a equação (1") sob a forma
dx ^ <p'(p) _ •>I^'(P)
dp p — <P(P) P — <P(P)
e consideremos x como função de p. A equação obtida é, então, uma
equação diferencial linear em relação à função x(p).
Encontra-se, resolvendo-a.
x = \o(p, C). ( 2)

Eliminando o parâmetro p das equações (10 e (2), obtém-se o


integral geral da equação (1) sob a forma
0 ( x , y, 0 = 0.
Exemplo — Seja a equação

y = xy'^ + y'2.
( 1)
Façamos y'= p, obtém-se:
y = xp^ + p^.
(!')
Derivemos em ordem a x. Tem-se:
(10
p = pí + [2ip + 2 p ] - ^

Achemos as soluções singulares. Dado que p — p' quando po = 0 c Pi = 1,


ter-se-á como solução as funções (ver (!')):
y =z=o:.02 + 02, isto é, i/ = 0
TOUAÇOES D IFEREN CIA IS 55

y = x + í.

Saber-se-á se estas funções são soluções particulares ou singulares após


ter encontrado o integral geral. Para achar o integral geral escrevamos a
a equação (T") sob a forma
dx 2p______ 2
~dp *P— 1— P
e consideremos x como função da variável independente p. Integrando * a
equação linear (relativamente a x) obtida, tem-se

x = — 1- (II)
(P -D * *
Eliminando p nas equações (F) e (II), obtém-se o integral geral:
y = (c + y í+ i)® .
A equação proposta tem por integral singular
v=0,
dado que esta solução não resulta da solução geral particularizando C.
Quanto à função y = a: + l, não é uma solução singular, mas uma
solução particular; deduz da solução geral fazendo C = 0.

§ 15. Trajectórias ortogonais e isogonais


Consideremos uma família de curvas com um parâmetro
O {x y, C) = 0.
Chamam-se trajectórias isogonais às curvas que cortam todas as
curvas da família dada (1) sob um ângulo constante. Se se tiver um
ângulo recto, estas curvas são, então, trajectórias ortogonais,
Trajectórias ortogonais — Procuremos a equação das trajectórias
ortogonais. Escrevamos a equação diferencial da família de curvas
dada. eliminando o parâmetro C das equações

dL(x, y, C ) = 0

dd) d<l> dy
= 0.
dx dy dx
Seja

d")
essa equação diferencial.
56 CAL.CULO DIFERENCIAL. E INTEGRAL

é aqui o declive da tangente no ponto M(x, y) na curva cor­


respondente da família. Dado que a trajectória ortogonal que passa
pelo ponto M (x, y) é perpendicular à
curva correspondente, o seu declive ^
está ligado a ^dx pela relação (fig. 259)

( 2)
dx dyx
dx

Substituindo esta expressão na


equação (1') e omitindo o índice T,
obtém-se uma relação entre as coorde­
nadas dum ponto arbitrário (x, y) e o
declive da trajectória ortogonal deste
ponto, isto é, a equação diferencial das trajectórias ortogonais

F l X, y, (3)

O integral geral desta equação


(Di(x, y , C ) = 0

representa a família das trajectórias ortogonais.


As trajectórias òrtogonais encontram-se, por exemplo, quando
se estuda o derramamento plano dum fluído.
Consideremos o movimento plano dum fluído tal que o vector
velocidade v (x, y) da corrente seja definido em cada ponto do plano
Oxy. Se este vector apenas depende da posição do ponto e não do
tempo, diz-se que o movimento é estacionário. Vamos considerar um
tal movimento. Além disso, suporemos que existe um potencial de
velocidades, isto é, uma função u(x, y) tal que as projecções do
vector V (x , y) sobre os eixos de coordenadas y* (x, y) e v„ (x , y)
sejam derivadas parciais desta função em relação a x e
du du
dx = = (4)
EQUAÇÕES D IFEREN CIA IS 57

As curvas da família
u (x, y) = C (5)
chamam-se linhas de nível ou linhas equipotenciais.
As curvas cuja tangente em cada ponto se confunde com o
vector V (x, y) chamam-se linhas de corrente; elas materializam as
trajectórias das partículas em movimento.
Mostremos que as linhas de corrente são as trajectórias ortogonais
da família de linhas equipotenciais (fig. 260).
Seja o ângulo formado pelo vector velocidade v t o eixo Ox,
Tem-se, em virtude das relações (4),
du{x, y) du{x, y) _ \ V sen (p;
== V I COS (p;
dx ày
donde se deduz o declive da tangente à linha
de corrente
du{x, y)
ày
tgq):
du{x, y)
( 6) dx
F i g. 260.
Obtém-se o declive da tangente à linha equipofencial derivando
a relação (5) em relação a jc:

du dudy_

donde
du
d^ dx
dx ' dü' (7)
dy

Por conseguinte, o declive da tangente à linha equipotencial é o


inverso mudado de sinal do declive da tangente à linha de corrente.
Daí resulta que as linhas equipotenciais e as linhas de corrente
são ortogonais.
No caso dum campo eléctrico ou dum campo magnético, as
trajectórias ortogonais da família das curvas equipotenciais são as linhas
de força do campo.
CALCULO D IFEREN CIA L E INTEGRAL

Exemplo — 1. Achar as trajectórias ortogonais da família de parábolas


y = Cx\
Resolução — Escrevamos a equação diferencial da família,

y' = 2Cx,
Obtém-se eliminando C:

y ^
Substituindo nesta igualdade y' por — ^ , obtém-se a equação diferencial

Fig. 261
da família das trajectórias ortogonais
1 ^ 2
yy' ~~ ^
ou
X dx
ydy =
2 •
O seu integral geral é

-J- + \ - = C2.

Por conseguinte, as trajectórias ortogonais da família de parábolas dada


formam uma família de elipses de semi-eixos a = 2C, b = C'{/2 (fig- 261).
EQUAÇÕES D IFEREN CIA IS 59

Trajectórias isogonais — Suponhamos que as trajectórias cortam


as curvas duma dada família sob o ângulo a. Façamos tga = k.
O declive ^ = tga (fig. 262) da tangente à curva da famãia
dVT
e 0 declive = tg ^ da tangente à trajectória isogonal estão ligadas
pela relação
tgil5 — t g a
t g < P = = t g ( i | ) — a )^
1 + tg a tg il)’
isto é.
—k
dy dx (Z)
dx
dx

Substituindo esta expressão ná equação ( r ) e omitindo o índiee T,


obtém-se a equação diferencial das ^sajeetórias isogonais.

Exemplo — 2. Achar as trajectórias isogonais da família de raotas


y = Cx. (8)
Supõe-se que estas rectas são cortadas sob o ângulo a e fár-se-á iga = k.

Resolução — Escrevamos a equação diferencial da família de rectas. Deri­


vemos a equação (8) em relação a x:
dy
dx
Por outra via, deduz-se da mesma equação

c=l..

A equação da família de rectas é, pois,


dy
dx
Obtém-se a equação diferencial das trajectórias isogonais servindo-nos da
relação (20
dyr
dx

dx
60 CALCULO D IFEREN CIA L E INTEGRAL

Tem-se, pois, omitindo o índice T:

dy
dx l-J t-L

Obtém-se o integral geral integrando esta equaçáo homogénea:

Log + =X ** ■f'
que é a família das trajectórias. Para ver quais são as curvas desta família.

Fig. 263
passemos a coordenadas polares:

X = tg<p; V^^ + y^ = P*

Obtém-se, substituindo estas expressões em (9)


1
L ogp=-jf (p+LogC

p = C e '‘ .
Vê-se que a família das trajectórias isogonais é composta de espirais
logarítmicas (fig. 263).

§ 16. Equações diferenciais de ordem superior a um


(noções gerais)
Como indicámos mais acima (ver § 2). pode-se escrever simbò-
licamente uma equação diferencial de ordem n sob a forma

y, y , y = o (1)
EQUAÇÕES D IFEREN CIA IS 61

OU ainda, se se resolver em relação à derivada de ordem /i,


= y, y, y", . . . . y^"~% (1')
No presente capítulo, apenas consideraremos equações resolúveis
em relação à derivada de ordem mais elevada. Tem-se para estas
equações um teorema de existência e de unicidade da solução análogo
ao das equações de primeira ordem.
Teorema — Se na equação
y<'‘>= /(x . y , y .........
a função f(x, y, / , .... e as suas derivadas parciais em relação
a y, y . .... forem cortíínuas num certo domínio que contém os
valores x = Xo, y= yo, v'= ]/'<,■>• • •’ yo^'"~^l, existe uma solução e
só uma y = y(.x) da equação que verifica as condições

Vx=xo = yo.
Ux^ xq = yot
(2)

que se chamam as condições iniciais. A demonstração deste teorema


esta fora do alcance deste livro.
Se se considerar uma equação de segunda òrdem y" = f {x, y, y ),
as condições iniciais da solução para x = Xo, serão
y = yo. y = y i
em que Xo» yo* yo' são números dados. O sentido geométrico destas
condições é o seguinte: passa pelo ponto dado do plano {xo, yo) uma
única curva cujo declive da tangente neste ponto é yo'. Daí resulta que
se se der diferentes valores a /o sendo fixo o ponto Xo. yo. obtém-se
tantas curvas integrais de declives diferentes quantas passem pelo
ponto dado.
Introduzamos agora a noção de solução geral de uma equação
de ordem n.
Definição — Chama-se solução geral de uma equação de ordem n
a uma função
y — (p (Xy Cl, C2, . . . » Cyi)»

dependente de n constantes arbitrárias Cu C 2 * ... tal que:


a) verifica a equação quaisquer que sejam os valores das cons­
tantes Cl, C2. .... ;
62 CALCULO D IFEREN CIA L E INTEGRAL

b) sendo dadas as condições iniciais:

y^=xo ]/oi
yx=XQ = i/o»

— j/o ,
, 4» t
se possa escolher as constantes Cu C2. .... Cn de modo que a função
y = (p {x, Cl, C a v . Cn) verifique as condições (supõe-se que os valo­
res iniciais Xq, yo, y ', pertencem ao domínio de existência
da solução).
Uma relação da forma O {x, y, Ci, Cg. .. .. Cn) = 0, que define
a solução geral implicitamente, chama-se integral geral da equação
diferencial proposta.
Qualquer função que se deduza da solução geral, que concretiza
os valores Cu C2, ..., Cn, é uma solução particular. A curva represen­
tativa duma solução particular é uma curva integral da equação dife­
rencial dada.
Resolver (integrar) uma equação diferencial de ordem n, é:
1) achar a solução geral (se as condições iniciais não forem dadas)
2) encontrar a solução particular da equação que satisfaz às con­
dições iniciais (se as houver).
Damos, nos parágrafos seguintes, métodos de resolução de dife­
rentes equaçõçs de ordem n.

§ 17. Equação da forma = f (^)


A equação mais simples de ordem /i é da forma
(1)
Achemos o seu integral geral.
Integremos em relação a jc os dois membros da equação. Obtém-se,
tendo em consideração que y^^^ = :

f ( x ) d x + Cu
Xo

em que jco é um valor arbitrário fixo de jc e Ci uma constiante de


integração.
Integremos uma vez mais:
X X

y(n- 2 ) _ j y (a;) 0 x)d x + Cl (x — Xo) + C,.


*0 *0
EQUAÇÕES D IFEREN CIA IS = / (x) 63

Continuando assim, obtém-se (após n integrações) a expressão do


integral

Xo
••í Xo
f{ x ) d x . . . dx

(X — Xq) 7 1 - 2
C i( x -X o )" ” ‘
{n -í)l
+

+
{n-2)\

Para encontrar a solução particular que verifica as condições iniciais


V x — Xq Í/oÍ V x = X o — Uoi • • •; V x ^ x ] —

basta fazer
— í/o» C y i- i — ^ 0 » • • =
Exemplo — 1. Encontrar o integral geral da equaçSo
y"* = sen {kx)
e a solução particular que satisfaça às condições iniciais

í/x=0 = ^» í/i=0 = ^-
Resolução.

COS k x — 1
y '= ^ kx dx-\-C
0

y -~ l Cidjr+Cz

ou
senfe® X I ^ ^
y = ----- p - + - j g - + C,X+ C2.

Tal é O integral geral. Para encontrar a solução particular que satisfaça


às condições iniciais dadas, basta determinar os valores correspondentes de
Cl e C2.
Deduz-se da condição 1/35—0 = 0 C2 = 0.
Deduz-se da condição í/x = 0 = 1 C4 = l.
Por conseguinte, a solução particular procurada é
sen kx
y= ■ k^ ( t +<) •
Encontram-se equações diferenciais deste género na teoria da flexão
de vigas.
Exemplo — Consideremos uma viga prismática elástica que flexiona sob
a acção de forças exteriores, tão bem repartidas como concentradas. Levemos
o eixo Ox horizontalmente, por forma a confundir-se com o eixo da viga
antes da sua deformação e Oy verticalmente para baixo (fig. 264).
64 CALCULO D IFEREN CIA L E INTEGRAL

Qualquer força que aja sobre a viga (por exemplo, a carga, a reac^o
dos apoios) tem um momento em relação a uma secião transversal da viga
que é igual ao produto da força pela distância entre o ponto de aplicação
da força e a secção considerada. A soma M (jc) dos momentos de todas as
forças aplicadas dum mesmo lado da secção da abcissa x chama-se momento
flexionador da viga em relação à secção dada. Demonstra-se nos cursos de
resistência de materiais que o momento flexionador duma viga é
EJ
R '
onde E é o módulo de elasticidade, que depende do material, J o momento
de inércia da secção transversal da viga em relação ao eixo horizontal que
passa pelo centro de gravidade desta secção, R o raio de curvatura do eixo
curvo da viga, cuja expressão é dada pela
fórmula (§ 6, Cap. VI)

Por conseguinte, a equação diferencial


do eixo curvo da viga escreve-se
M(x)
( 2)
(l + í/'2) 3/2 EJ
Fig. 264 Se se admitir que as deformações são
pequenas e que os ângulos entre as tangentes
ao eixo da viga e o eixo Ox são pequenos, poder-se-á desprezar a quantidade
y'* que é o quadrado da pequena quantidade y' e fazer

R= - ^ .

A equação diferencial da viga flectida torna-se, então,


M(x)
y = - ( 2 ')
EJ
É unu equação da forma (1).
Exemplo — 3. Uma viga está embutida pela sua extremidade O e uma
força P age verticalmentc na extremidade L à distância / a partir da secção
de encaixe (fig. 264). Desprezar-se-á o peso da viga.
Consideremos a secção no ponto N (x), O momento de flexão em relação
à secção V é no caso dado
M(x) = (l — x) P.

A equação diferencial (2') transforma-se em

Condições iniciais: a deflexão v é nula quando j: = 0 e a tangente ao


eixo da viga curvada confunde-se com o eixo Ox, isto é.
EQUAÇÕES D IFEREN CIA IS 65

Integrando, vem

(3)
Em especial, a fórmula (3) define a flecha h na extremidade L:
pi»
* = »*=» = 2EJ •

§ 18. Alguns tipos de equações diferenciais de segunda ordem


que se rôduzmn a equa4;ões de primeira oídem
I — E quações da form a

â='(- ) 1
que não contém , explicitam ente, a função desconhecida y.
(1)

Resolução — Designemos a derivada ^ por p : ^ = p - Ter-se-á


dx ^

dx^ dx *
Substituindo estas expressões das derivadas na equação (1),
obtém-se uma equação de primeira ordem

p).

em que p é a função desconhecida de jc. Por integração, obtém-se o


integral geral
p = p (z, Cl),

depois, deduz-se da relação ^ = p o integral geral da equação (1):


dx-
y = l p ( x , C i)dx + C 2.
Exemplo — 1. Consideremos a equação diferencial da catenária (ver § 1)

Façamos
dy
dx = p ;
66 CALCULO D IFEREN CIA L E INTEGRAL

tem-se ^ dp
dx^ dx

e obtém-se uma equação diferencial de primeira ordem em relação à função


auxiliar p{x):

Separemos as variáveis
dp dx
"^1 + a •’

então.
L o g (p + V l+ /> * )= -J + Ci,

P -i

Mas como p = , esta última relação é uma equação diferencial que


contém a função desconhecida y. Obtém-se integrando a equação da catenária
(ver § 1)

Achemos a solução particular que satisfaz às condições iniciais seguintes:

í'i=o = 0-
A primeira condição conduz C2 = 0, a segunda, Ci = 0.
Obtém-se, finalmente:

Nota — Integra-se duma maneira análoga a equação

Fazendo = p, obtém-se para determinar p uma equação de


primeira ordem

| = / ( » . p).

Determinando p <m função de x, deduz-se y da relação = p


(ver § 17).
EQUAÇÕES D IFEREN CIA IS 67

II — Equações da forma

( 2)

que não contêm, explícitamente, a variável independente x. Façamos


de novo
dy
(3)
dx
mas consideremos agora que p é função de y (e não de x, como ante­
riormente). Ter-se-á
d^y dp dpdy dp
daf^ dx dydx dy^'

Substituindo na equação (2) as expressões ^ e ^ , obtém-se


dx dx*
uma equação de primeira ordem apoiando-se sobre a função auxiliar p:

P). W)

Integrándo, obtém-se, p como função de 3; e duma constante


arbitrária Cii
p = P (y, Cl).

Substituindo esta expressão na relação (3). encontra-se uma equação


diferencial de primeira ordem relativamente à função y de x:

= p { y , Cl).
dx
Separando as variáveis, tem-se:
dy
' dx.
p (y , Cl)
A integração desta equação fornece o integral geral da equação
proposta:
O (x, y, Cu C 2 ) = 0.
Exemplo — 2. Encontrar o integral geral da equação
_5

Resolução — Façamos p — c consideremos p como função de y.


do
Tem-se, então, = e obtém-se uma equação de primeira ordem em
68 CALCULO D IFEREN CIA L E INTEGRAL

que a função desconhecida é p:

Integrando esta equação, tem>se


2
p^ = C i— y ^ ou p = ± K C i — !/-*/» .
Mas p = ; obtém-se, pois, para y a equação
squaçãi
dx
dy
-=dx ou = dx.
Y c ^ —y -y ^ ± V C iy ^ '^ —\
e obtém-se

• + c ,- ± f

Para calcular este integral, façamos a substituição


l = í2.
Então, tem-se,
JL A
yV3 = (í2+ l )2

dy = 3t +

Por conseguinte,
r y^ !> d y ___ f 3 t(t» + l) ,, 3 /'f» ,
' Y C iy'^ '^ — i J ‘ Cf V 3 j

= - ^ K c y / a - l (Ciy*^^+2).
Finalmente, obtem-se
x + C2= ± - ^ y Cj1,'/3_1(C,í,*/3+2).
Exemplo — 3. Suponhamos que um ponto material descreve a recta Ox
sob a acção de uma força que depende sòmente da posição do ponto.
A equação diferencial do movimento é

dx
Seja Xo e = v© para / = 0.
at
dx
Multipliquemos os dois membros da equação por dt e integremos
dt
de 0 a r. Obtém-se:
1 / d x \2 1 (•
T I "áT) - y "“'S = J ^ (3^) dx.
EQUAÇÕES DIFERESNCIAIS 69

/ • ( x ) á x ] = 4 -'«^? = const.
XO

o primeiro termo da última igualdade representa a energia cinética do


ponto material e o segundo, a sua energia potencial. Resulta da igualdade
obtida que a soma das energias potencial e cinética
é constante durante o movimento.
Problema do pêndalo matemático, — Consi­
deremos um ponto material de massa m que se
move sob a acção do seu próprio peso sobre uma
circunferência L num plano vertical. Achemos a
equação do movimento abstraindo das forças de
resistência (fricção, resistência do ar, etc.).
Tomemos a origem das coordenadas no ponto
mais baixo da circunferência e dirijamos o eixo Ox
tangencialmente a esta última (fig. 265).
Seja / o raio da circunferência, s o com­
primento da porção de arco de origem O no ponto
variável M onde se encontra a massa m, sendo
este arco tomado com o sinal (5 > 0 se o ponto M
está à direita de O; s < 0 se o ponto M está
ã esquerda de O).
Propõe-se encontrar a dependência entre 5
e o tempo /.
Decomponhamos a força de gravidade mg
nas suas componentes tangencial e normal. A pri­
meira que é igual a — mg sen ç>, implica o movi­
mento, a segunda é compensada pela reacção da circunferência descrita pela
massa m.
A equação do movimento escreve-se, pois,
d^s
m - ^ = —mg sen 9.
s
Como se tem para a circunferência q) = - p , obtém-se a equação

d^s s
-= -g se n -.

É uma equação diferencial do tipo II (porque não contém esplidtamente


a variável independente /).
Integremo-la como foi acima indicado:
ds d^s __ dp
~dt dt^ ds
Por conseguinte.
dp

p dp = — gscn — ds.
70 CALCULO D IFEREN CIA L E INTEGRAL

logo
p^=^2gl COSy + Ci.

Designemos por so a distância máxima do ponto M. A velocidade do


ponto é nula quando s = a>:
ds
= 0.
dt 8=80 = P S = 8 0
Isto permite determinar C :
0 = 2gl COS-^ + C l,

donde
C i= —2gZcos-^ .

Por conseguinte,

^ ' = ( - í - ) ' = 2g í ( c o s ^ - c o s ^ )
OU, aplicando a esta última a fórmula relativa à diferença de cossenos:

( | l ) “= 4 ,,s e n * - ^ s e n f ç f , (5)
ou (♦)
I f — 2 VTi > / s « i sen í ç l , ( 6)

Ê uma equação de variáveis separáveis. Separemos as variáveis:


ds
:=2ygldt. (7)
j/s e n ^ -p s e n ^ -^

Suporemos por momentos que s ^ S e de modo que o denominador da


fracção não seja nulo. Se se suposer que .r = 0 quando t = 0 ter-se-á a
igualdade (7) ^

f ^ . _= 2 V f l t. (8)
/ s e n i l f sen-
21
Esta igualdade dá a dependência entre s e t, O integral à esquerda não
se exprime por meio de função elementar. O mesmo se diga de s como
função de /. Consideremos o problema posto aproximadamente. Suporemos
que os ângulos e são pequenos. Os ângulos — — não serão
L •' 21 21

(*) Tomamos o sinal mais antes do radical. Decorrerá da nota feita


no fim deste problema que não há lugar a examinar o caso do sinal menos.
EQUAÇÕES D IFEREN CIA IS 71

superiores a , Substituámos na equaçSo (6) os senos pelos ângulos


^ _ 9 i/TT i / ^+ ^0 ^0—^
dt y 21 21
ou (6 ')
dt I v*F ^.
Separemos as variáveis. Obtem-se (supondo provisòríamente que s =^sò)
ds
-f-d í. (V)

Suporemos de novo que s = 0 quando / = 0. Integrando esta última equa­


ção, tem-se:
Ii rio / a
( 8 ')
w y sõ -s-
ou

donde

(9)

Nota — Suposemos até agora que s=^So, Mas pode-se verificar, substi­
tuindo directamente, que a função (9) é solução da equação (60 qualquer
que seja r.
Notemos que a solução (9) é uma solução aproximada da equação (5),
dado que substituímos a equação (6) pela equação aproximada (60.
A igualdade (9) mostra que o p^nto Af (que se pode considerar como
a extremidade do pêndulo) executa oscilações harmónicas de período

■= 2. / - gf .
Este período não depende da amplitude da oscilação So.
Exemplo — 4. Problema da segunda velocidade cósmica.
Determinar a velocidade a que é preciso lançar um corpo verticalmente
para cima para que escape^ à atracção terrestre. Desprezar-se-á a resistência do ar.
Resolução — Designemos as massas da terra e do corpo respectivamente
por M t m. Em virtude da lei da atracção de Newton a força que solicita
o corpo m •€
. . M•m

em que r é a distância entre o centro da terra e o centro de gravidade do


corpo lançado, e A: a constante da gravitação universal.
A equação diferencial do movimento do corpo de massa m é
d^r M»m
dt^
ou
d^r
( 10)
* r2 •
72 CALCULO DIFEREN CIA L E INTEGRAL

Tomamos o sinal menos porque a aceleiUção é aqui negativa. A equa­


ção (10) é uma equação da forma (2). Resolvê-la-emos tomando por condições
iniciais:
para / = 0 r = R , ^ =
at

R é aqui o raio da Terra e vo, a velocidade de lançamento. Introduzamos


as notações
dr d^r dv dv dr dv
lü dr dt ^ dr
sendo v a velocidade do movimento. Obtém-se, substituindo na equação (10):
dv . M dr
V -r- = —k — . V d v = — km —r- .
dr r* r2
Integrando-se esta equação, vem

(11)

Determinemos Ci no caso de v = Vo na superfície da Terra (r = R)

4 = + fc j» f-i-+ c ,
ou
^ k M , vl
= +"2
Substituámos o valor encontrado C> na igualdade (11);

— = + k „M 1- - —
kM + - ^

OU

( 12)

Ora, a velocidade do corpo deve ser constantemente positiva (ela nSo


se anula), pois ^ 0. Como a quantidade — se torna arbitràriamente
pequena quando r cresce indefinidamente, a condição terá lugar para
A
todo o r sòmente se

2 (13)
ou

vo> y
Teni-se, pois, para a velocidade mínima
2kM
1^0=
/ R ’ (U )
EQUAÇÕES DIFEREN CIA IS 73

em que cm8
/c= 6,66.10-8 g s 2 ’
ií = 63*10'^ cm.

Na superfície da Terra, r = R, a aceleração da força de gravidade

Sendo assim, deduz-se da igualdade (10)


I. ^

ou
M = gR^
Substituindo este valor de M na fórmula (14), obtém-se:

i>o= V ^ = V 2 - 9 8 1 - 6 3 107=» 11,2-10»— =11,2 — .


s s

§ 19. Integrarão gráfica das equações diferraciais


de segunda ord«n
Vejamos qual é a interpretação geométrica duma equação dife­
rencial de segunda ordem. Seja a equação
y" = f {x, y, y')- (l)
Designemos por <p o ângulo formado pela tangente à curva com
o eixo positivo Ox\ tem-se

dx
= t g (p. (2)

Para explicitar o sentido geométrico da derivada segunda, lem­


bremo-nos que a fórmula do raio de curvatura duma curva num dado

f l.í i + i í í ,
y"

Donde
(1 +
R

(*) Suposemos até agora que o raio de curvatura cra um número


essencíalmente positivo, mas suporemos neste parágrafo que o aio de curvatura
poderá tomar os valores tanto positivos como negativos; se a curva é con­
vexa (y" < 0), suporemos o raio de curvatura negativo {R < 0); supô-lo-emos
positivo (R > 0) se a curva é côncava (y" > 0).
74 CALCULO D IFEREN CIA L E INTEGRAL

Ora
j/' = tg(p; 1 + / * = ! + tg*<jp = sec*(jp;

( r + y V ^ = |sec*«p| = — V -
I COS (P)

Por conseguinte.
y = (3)
R I COS®(p I

Substituindo na equação (1) as expressões obtidas para y e y \


ter-se-á

= í {x, y, tg c p ),
R I COS <p I

ou

R= ( 4)
|c o s ® c p |- /( a :, y, tg c p )

o que mostra
Fig. 266
rencial de segunda ordem determina a
grandeza do raio de curvatura da curva
integral, uma vez dadas as coordenadas do ponto e a direcção da
tangente neste ponto.
Daqui resulta um método de construção aproximada duma curva
integral que admite uma tangente contínua (*) em cada ponto; a curva
é constituída por arcos de círculos.
Assim, suponhamos que se deve traçar a curva integral da equa­
ção (1) que satisfaça às condições iniciais:

Ux= xq = j/ q; yx=XQ = yo.

Tracemos pelo ponto Mo (xo. yo) um raio MoTo de declive y =


= tg ^0 = y'o (fig. 266). Deduz-se da equação (4) R = Ro. Conside­
remos um segmento MoCo de comprimento Ro sobre a perpendicular
à direcção MoTo e tracemos do ponto C tomado para centro um pequeno
arco de círculo M qM^ de raio Ro^ Notemos que será preciso consi­
derar o segmento MoCo do lado conveniente para que 9 arco de círculo
seja convexo para cima quando /?o < 0 e que seja convexa para
baixo quando R o > ^ (ver nota pág. 73).

(*) Isto é, que o declive é uma função contínua do arco 5.


EQUAÇÕES D IFEREN CIA IS 75

Seja, em seguida, um ponto M i (jci, yO sobre o arco da curva


construído, suficientemente vizinho de Mo e seja tg^i o declive da tan­
gente MiTi à curva no ponto Mi. Deduzamos da equação (4) o valor
R = R i correspondente a Mi. Tracemos o segmento MjCi. igual a Ru
perpendicularmente a MiTi e, de Ci como centro, tracemos um arco
ií/iM g Tomemos em seguida sobre este arco um ponto
M2 (JC2, >^2). vizinho de Mu e continuemos a nossa construção até que
se obtenha uma porção de curva suficientemente grande formada de
arcos de círculos. Resulta do anterior que esta curva é aproximada­
mente uma curva integral que passa pelo ponto Mo. É evidente que a
curva constante será tanto mais aproximada da curva integral quanto
mais pequenos forem os arcos M qM i , M í M^, . . .

§ 20. Equações Lineares homogéneas.


Definições e propriedades gerais
Definição— 1. Uma equação diferencial de ordem n diz-se linear
se é do primeiro grau em relação à função desconhecida y e ks suas
derivadas / , .... isto é. se ela é da forma

«oí/ +diy + •■ + anU = f (x), ( 1)


em que ao» » ^ n f M são funções de x dadas ou constantes
e ã o ^ O qualquer que seja x no domínio de definição da equação (1).
Suporemos, no seguimento, que as funções « 0» «i, c f(x) são
contínuas para todos os valores de jc e que ã o ^ l (senão bastará dividir
todos os termos por ao), A função / ( jc ) chama-se o segundo membro
da equação.
Se / ( jc ) 0, a equação diz-se não homogénea ou ainda, com
segundo membro. Se / ( jc ) = 0, a equação escreve-se

+ ••• = o (2)
e diz-se homogénea ou sem segundo membro (o primeiro membro desta
equação é uma função homogénea do primeiro grau em relação a y, / ,

Estabeleçamos algumas propriedades fundamentais das equações


lineares homogéneas, cingindo-nos às demonstrações das equações de
segunda ordem.
Teorema— 1. Se yi e 3^2 forem duas soluções particulares da
equação linear homogénea de segunda ordem
y" + aiy' + a^y = 0, (3)
>*1 + y>2 também é solução desta equação.
76 CALCULO D IFEREN CIA L E INTEGRAL

Demonstração — Dado que 3^1 e ^2 são soluções da equaçãó pro­


posta, tem-se
y 'i + aiyí + 02y 1=
i = o0,, I
(4)
y'í + «1^2 + <*2^2! = 0 . /
Substituindo a soma + >^2 na equação (3) e tomando as iden­
tidades (4) em consideração, ter-se-á
(^1 + Vi)” + oi (yi + í/2)' + «2 (í/i + Vi) =
= iyi + flií/í + 02í/i) + {y'2 + «1^2 + <hyè = 0 + 0=0,
o que demonstra que + >^2 é solução da equação.
Teorema — 2. Se yi for solução da equação (3) e se C for uma
constante, Cyi é também uma solução desta equação.
Demonstração — Substituindo na equação (3) a expressão Cyi,
obtém-se:
{C y^" + 0 ,1 {Cy^' + <h {Cy,) = C {yi + a,y[ + a^y^ = C - 0 = 0;
e o teorema fica demonstrado.
Definição — 2. Duas soluções y^ e y 2 de (3) dizem-se linearmente
independentes sobre o segmento [a, 6] se a sua relação não for cons­
tante sobre este segmento, isto é, se

— constante.
í/2
Senão, as soluções dizem-se linearmente dependentes. Por outras
palavras, duas soluções 3^1 e 3^2 dizem-se linearmente dependentes, sobre
o segmento [a, b], se existir uma constante \ tal que ^ ^ P^^ra
G < jc < 6. Tem-se, então, 3^1 = kyz.
Exemplo — Seja a equação y'' — y = 0. Verifica-se fàcilmente que as
funçQes 5e~® são soluções desta equação. As funções e* e
são linearmente independentes em todo o segmento, dado que a relação - ^ = ^2*
e *
não permanece constante quando x varia.
3gX
As funções e* e 3e^, essas, são linearmente dependentes, p o is -^ = 3 = c o n s t.
e*
Definição — 3. Sendo 3^1 e ya função de x, o determinante

W{y„y^)=^^ ^ = y ,y '^ - y [ y ^
y'i y'2
cbama-se determinante de Wronski ou wromkien das funções dadas.
EQUAÇÕES D IFEREN CIA IS 77

Teorema — 3. Se as funções yi e y 2 forem linearmente depen­


dentes sobre o segmento [a, 6], o seu wronskien é idênticamente nulo
nesse segmento,
Con efeito, se 1/2 = onde \ = const, y' = e

yt Vz yi Vi Vi
W{yu yi) = = X = 0.
y 'i y'2 y 'i y\ y\

Teorema — 4. Se o determinante de Wronski W (yi, >^2) das solu­


ções yi e y 2 da equação (3) não for nulo no ponto x =X q do segmento
[a, 6] onde os coeficientes da equação são contínuos, ele não se anula
em qualquer parte daquele segmento.
Demonstração — Sendo yx e y 2 duas soluções da equação (3),
tem-se
Vi + ^ií/2 + ^ i /2 = 0» í/i H“ 4” (bVi = 0-

Multiplicando os termos da primeira igualdade por 3^1, os da


segunda por — >^2 e juntando, obtém-se:
{viVÍ — y'íyò + «1 (yií/2 — yiy^) = o. ( 5)

O coeficiente de ai em (5) é o wronskien W (yu y^ò e precisamente


^ {yu y ^ = — yiy^)- o primeiro termo é a derivada do
wronskien:
(^1. yi) = {yiy '2 — y ’iyi)' = — j/ í 'í/2.
Por conseguinte, a igualdade (5) escreve-se
W' + axW=0. (6)
Achemos a solução da primeira equação que satisfaça à condição
inicial W I x=Xq = PFo* Encontremos, em primeiro lugar, a solução geral
da equação (6) supondo W ^ 0. Separando as variáveis na equação (6),
dW
obtém-se ^ = —a^ da:. Integrando, vem
W

Log W = — J a^dx Log C


Xo
ou
X

Log = —^ dx,
C
Xo
78 CALCULO D IFEREN CIA L E INTEGRAL

donde
- j a, d*
W = Ce (7)

Há que notar que a função (7) se pode escrever e que verifica


a equação (6), do que se pode fàcümente comprovar pela substituição
directa desta função na equação (6). A hipótese fV=^0, já não é
indispensável. A fórmula (7) chama-se fórmula de Liouville.
Determinemos C de modo que seja verificada a condição inicial.
Fazendo x = Xo no primeiro e segundo membros da igualdade (7),
achamos
Wo = C.

Por conseguinte, a solução que verifica as condições iniciais será


da forma
—J a, dx
(W)=Woe (7')

Para a hipótese W o ^ O , Resulta, então, da igualdade (7) que


W qualquer que seja jc porque a exponencial não se pode anular
para valores finitos da variável. O teorema está demonstrado.
N ota— 1. Se o wronskien for nulo para um certo valor x = Xo
é, então, nulo para qualquer valor x do segmento considerado. Tal
resulta directamente da fórmula (7): se = 0 para x = jCo, então.

Por conseguinte, W = 0 qualquer que seja o valor do limite


superior x na fórmulaí (7).
Teorema — 5. Se as soluções yi e y 2 da equação (3) forem linear­
mente independentes entre o segmento [a, ó], o determinante de Wronski
formado com estas soluções não se anula em nenhum ponto deste
segmento.
Indiquemos a ideia da demonstração deste teorema sem a dar
completamente.
Suponhamos que íF = 0 num certo ponto do segmento; em vir­
tude do teorema 3. o wronskien será nulo em todos os pontos do
segmento [a, b\:
W = 0
ou
í/lí/2 — í/íí/2 = 0.
EQUAÇÕES D IFEREN CIA IS 79

Consideremos, em primeiro lugar, o§ intervalos contidos no


segmento [a, b] nos quais y i ^ O , Então,

y iy 2 z z M i = o
yl
ou

Por conseguinte, a relação ~ é constante em cada um dos inter­


valos mencionados
U2 = A,= const.
Vi
Reportando-nos ao teorema de existência e da unicidade, pode-se
demonstrar que y 2 = para todos os pontos do segmento [a, 6],
compreendendo aqueles em que = 0, o que é impossível, porque,
por hipótese, e y-z, são linearmente independentes. Por conseguinte,
o wronskien não se anula em nenhum ponto do segmento [a, è].
Teorema — 6. Se yi e y 2 forem duas soluções linearmente inde-
pendentes da equação (3), entãOy
y = C ii/i + ^2^2* ( 8)
em que Ci e Cg são constantes arbitrárias, é a solução geral desta
equação.
Demonstração — Resulta do teorema 1 e 2 que a função"
C\yi + C^y^
é a solução da equação (3), quaisquer que sejam as constantes Ci e Cg.
Mostremos agora que, quaisquer que sejam as condições iniciais
yx=^o = yo-» y'x=^o = y'o^ é possível escolher valores das constantes
Cl e Cg de modo que a solução particular correspondente Ciyi + Cg^^g
satisfaça às condições iniciais.
Substituindo as condições iniciais na igualdade (8), tem-se

yo-- + ^*2^20 20 » 1
(9)
yo = Ciy^o -)- C2^20
20» J
em que se fez
{yi)x=XQ — ^loí {y<^x=XQ — í/20*»
{y-òx=XQ = z/io; {y^x=XQ = y^o-
80 CALCULO D IFEREN CIA L E INTEGRAL

Pode-se tirar Ci e C2 do sistema (9), porque o determinante deste


sistema
VlQ V20
— í/loí/20 í/loí/20
Uio í/20

é o determinante de Wronski para jc = jCo e não é, pois, nulo (dado


que as soluções yi e >^2 são linearmente independentes). A solução par­
ticular deduzida da família (8) substituindo Ci e C2 pelos valores
encontrados satisfaz às condições iniciais dadas. O teorema está
demonstrado.
Exemplo — 2. A equação

1 1
cujos coeficientes «1 = — e ü2 = -----õ- são contínuos em todo o segmento que
X X“
não contenha o ponto jc = 0, admite as soluções particulares
1
í^i — í/2= —
(é fácil de verificar substituindo na equação). A solução geral é pois

y — —
X

Nota — 2. Não existe método geral que permita encontrar sob


forma finita a solução geral de uma equação diferencial linear de
coeficientes variáveis. Todavia, existe um tal método para as equações
de coeficientes constantes. Será objecto do parágrafo seguinte. No que
respeita às equações cujos coeficientes são variáveis, indicar-se-á no
capítulo XVI «Séries^», vários processos que permitem encontrar solu­
ções aproximadas que satisfaçam às condições iniciais.
Vamos demonstrar agora um teorema que permite encontrar a
solução geral de uma equação diferencial de segunda ordem de coefi­
cientes variáveis conhecido que seja uma solução particular.
Como sucede por vezes encontrar ou advinhar directamente uma
solução particular, este teorema pode ser muito útil em muitos casos.
Teorema — 7. Se se conhecer uma solução particular duma equa-
ção diferencial linear homogénea de segunda ordem, a procura da
solução geral reduz-se às quadraturas.
Demonstração — Seja uma solução particular conhecida *da
equação
y'' + = 0.
EQUAÇÕES D IFEREN CIA IS 81

Achemos uma outra solução particular da equação proposta tal


que yi e y 2 sejam linearmente independentes. A solução geral escrever-
-se-á, então, y = C iji + 02^2. em que Ci e C2 são constantes arbi­
trárias. Pode-se escrever, em virtude da fórmula (7) (ver a demonstração
do teorema 4):
dx
y2yi — y z y i= C e *

Por conseguinte, tem-se para a determinação de 3^2 uma equação


linear de primeira ordem. Integremo-la como se segue. Dividamos todos
os termos por y\:
y?.yi — VíHl ^ ^
ú y\
y
ou
]= ± C e-
dx\yt7 í/i

donde rc r S "
1 d x + r.
í/i ) y\

Como procuramos uma solução particular, ter-se-á, fazendo C = 0,


C = 1:
- I a, dx
y2 = y . \ dx. ( 10 )
y\

Ê evidente que yi e 3^2 são soluções línearmente independentes,


y,
porque — ^ c o n s t
A solução geral da equação proposta escreve-se, pois.

f r ^
y = Ciyt + Ciyi J — ^— X. ( 11)

Exemplo — 3.Achar a solução geral da equação


( l - x 2 ) y ''-2 x y '-{ -2 y = 0.
Resolução — Verifica-se^ directamente que esta equação tem por solução
particular = x. Achemos uma segunda solução particular ya tal que yi
e >2 sejam linearmente independentes.
6
82 CALCULO D IFEREN CIA L E INTEGRAL

—2x
Notando que fli = , obtém-se, em virtude da fórmula (10):
1—
? 22xi
x dx
1- x2 .-L o g ( l- x 2 )
dx = x f __ ^
*J x> (l- I*)
j)d x = x [ - l + i . L o g | ^ | ]
(■**■+ 2 (1—a:) + ^ + a : ) ;
A solução geral é pois
y=C'iX + C'2 ( “2 ’

§ 21. Equações lineares homogéneas de segunda ordem


de coeficientes constantes
Seja a equação linear homogénea de segunda ordem
y" + p y ' + Qy = o> (i)
em que p Q q são constantes reais. Para encontrar o integral geral
desta equação, basta, como demonstramos mais acima, encontrar duas
soluções particulares linearmente independentes.
Procuremos as soluções particulares sob a forma
y = ehx OU /c = const; ( 2)
então,
y ' = ke>‘=
‘-, y" = k V \
Substituamos estas expressões das derivadas na equação (1):
e’‘=
‘ { k ^ + p k + q) = 0.
Como 0, deve-se ter
-{ -p k -\- q— 0. (3)
Por conseguinte, se k é raiz da equação (3), a função será
solução da equação (1). A equação (3) chama-se equação característica
da equação ( 1).
A equação característica é uma equação de segunda ordem de
que designaremos as raízes por e kz. Tem-se:

*2 = — Y - l / y - í-

Os três casos seguintes se podem apresentar:


I — kl t k 2 são números reais distintos (ki ^ kn)\
II — Â:, e k 2 são números complexos;
III — kl Q k 2 são números reais iguais (ki = k-^.
Examinemos cada caso separadamente.
EQUAÇÕES D IFEREN CIA IS 83

I — As raízes da equação característica são reais e distintas:


kl ^ *2.
Ter-se-á, então, para soluções particulares
j/2 =
Estas soluções são linearmente independentes porque
Vz :e('‘*"'“) * # c o n s t.
«kl*
Ui «
O integral geral escreve-se, por conseguinte,
y = Cie*‘* + C2e'‘**.
Exemplo — 1. Ssja a equação
y'* + y ' — 2y = 0.
A equação característica escreve-se
A:2+ A;—2 = 0.
Achemos as raízes desta equação

*1.2= - Y ± | / ^ ^ + 2 :
A:i = l, /;£ = —2.
O integral geral é
y = Cie^+C2e-^^,
II — As raízes da equação característica são complexas.
Dado que as raízes complexas são conjugadas, façamos

Aíi = a + ip ; *2 == 05 — ip,
em que

Pode-se pôr as soluções particulares sob a forma


(4)
São funções complexas duma variável real que verifica a equação
diferencial (1) (ver § 4, cap. VII).
É evidente que se uma função complexa de variável real
y = u (x) iu {x) (5)
verifica a equação (1), esta equação é verificada separadamente pelas
funções u(x) e v(jc).
84 CAL.CUL.O D IFEREN CIA L E INTEGRAL.

G)m efeito, substituamos a expressão (S) na equação (1):


[u (i) -|- iv (x)]' + P [“ (^) + (^)]" + 9 i^) + W] = 0
OU

(u" + p u + 9“ ) + + P ^ ’ + 9^) — 0-
Mas uma função complexa apenas é nula se, e sòmente se, as
partes real e imaginária o forem separadamente, isto é,
u ' + pu + gu = 0,
v ' + p v -|- gi; = 0.
Acabamos de demonstrar que u (x) e v (x) são soluções da equa­
ção proposta.
Recopiemos as soluções complexas (4) sob a forma de soma
das partes real e imaginária;
Pi = «“* COS px -)- ie“* sen px,
Pa = c“* COS px — fe®* px.
De acordo com o que se acaba de demonstrar, as funções reais
seguintes serão soluções particulares da equação ( 1):
= e“* COS px, (6')
y, = e““ senPa:. (6')
As funções pj e Pa são linearmente independentes, porque

f f i _ e®*cosPx
= c o tg P x :? tc o n s t.
P2 e“* sen Px

Por conseguinte, a solução geral da equação (1), no case-em


que as raízes da equação característica forem complexas, toma a forma

p= + B i/2 = cosPx - f fie®*sen px


ou
p = {A COS Px + 5 sen Px), (7)

em que A t B são constantes arbitrárias.


Exemplo — 2. Seja a equação
y "+ 2i/' + 5y = 0.
Achar o integral geral c a solução particular que satisfaz às condições
iniciais yx=o = 0, i/i=o=r= 1. Construir a curva integral correspondente.
EQUAÇÕES DIFEREN CIA IS 85

Resoluções:
1. Escrevamos a equação característica
A:2+ 2fc+ 5 = 0
e determinemos as suas raízes:
—l + 2i, /c 2 = —1 — 2i.
O integral geral é, pois,
y= (i4 c o s2 a r+ 5 sen 2x).
2. Achemos a solução particular que satisfaça às condições iniciais dadas;
determinemos para esse efeito os correspondentes valores de A e B.

Deduz>se da primeira condição:


0 = e”®(/I COS 2*0 + B sen 2*0), donde A = 0.
Tendo em conta que
y ' — c"* 2B COS 2x— sen 2x,
deduz-se da segunda condição:
1 = 2B, ou seja B = y .

A solução particular procurada é, pois,

y = -i-e"*sen2z.
A curva está representada na figura 267.
III — A equação característica admite uma raiz real dupla.
Tem-se, então, = ko.
Obtém-se uma solução particular yi = em virtude dos racio­
cínios anteriores. É preciso encontrar uma segunda solução particular
linearmente independente da primeira (a função é idênticamente
86 CALCULO D IFEREN CIA L E INTEGRAL

igual a e não pode ser considerada como uma segunda solução


particular).
Procuraremos a segunda solução particular sob a forma
y^ = u (z) e***,
em que u (;c) é uma função desconhecida que se deve determinar.
Derivemos:
^2 = {u + k{u),
y ; = + /Ciue‘ * * = + 2 k ju ' + Afw).

Obtém-se, substituindo as expressões das derivadas na equação (1):


e*** [u ' + (2*1 + p ) u + (*J + p*i + q)u] = 0.
Como kl é uma raiz dupla da equação característica, tem-se
kl -\-pki -f- g = 0 .

Além disso. *1 = *2 = — ou 2*i = —p , 2ki + p = 0.


Por conseguinte, para encontrar u (x), torna-se necessário resolver
a equação = 0 ou w " = 0. Acha-se, integrando u = A x + B.
Pode-se fazer A = l, B = 0; tem-se, então, u = x. Pode-se, pois, tomar
para segunda solução a função
y^ = xe’‘'^.
Esta solução é linearmente independente da primeira, dado que
^ = a; ^ const. Tomar-se-á, pois, para integral geral, a função
Vi
y = C ie‘ ‘* + = e*'* {C ^ + C ^ ).

Exemplo — 3. Seja a equação


4y' + 4y==0.
A equação característica A:2—4A: + 4 = 0 tem por raízes k^ = k 2 = 2 . 0 inte­
gral geral escreve-se:
y = Cie^^ + C2xe2x,

§ 22. Equações diferenciais lineares homogéneas de ordein


n de coeficientes constantes
Consideremos uma equação diferencial linear homogénea de
ordem n:
,(n )
y + ^^ + • • • + ®7lí/ = 0. (1)
Suporemos que üu CI2 ........ Om são constantes. Antes de indicar
um método de resolução da equação (1), daremos duas definições que
nos serão úteis no seguimento.
EQUAÇOEJS D IFERENCIA IS 87

Definição— 1. v Se àt tiver para todo o jc do segmento [a, 6], a


igualdade
(x) = Ai(pi (x) 2^2 (^) “f" • • • "h l9n—1(^)*-

em que A u Ao.......são constantes, não todas nulas, diz-se que (^)


é uma combinação linear das funções <pi (x)^ ipg W» • • •» -i (^)*
Definição — Dizem-se linearmente independentes n funções <Pi (^)»
(P2 (x), . . (x), (p^ (x)se nenhuma delas puder ser representada
como combinação linear das outras.
N ota— 1. Resulta destas definições qüe se as funções (pi (x),
^2 (^)í • • M <Pn (^) forem linearmente dependentes, existem, então,
constantes Ci, Cz, não todas nulas, e tais que se tem, quaisquer
que seja x sobre o segmento [a, b],
C'i<Pl (ar) + ^ 2^2 W + ••• + (^) = 0.

Exemplos:
1. As funções = = í/3 = 3e*são linearmente dependentes, por-
1
que se tem para ^i = l» ^2 = ^» ^ 3 = — ^ a identidade

+ CgSe* = 0,
2. As funções í/i = 1, Vz = x, = linearmente independentes, porque
nâo se pode anular idênticamente a expressão
6*1•1 -f- C2X-f~
com Cj, 6*2» não todos nulos.
,, _phnx
3. As funções í/i = e^^^,í/2 = ^^**t •y yn —c » ,comAf,,A;2,...
arbitrários, são linearmente independentes (não demonstraremos esta proposição).

Passemos agora à solução da equação (1). Para esta equação


tem-se o seguinte teorema.
Teorema — Se as funções yi, yz, ..., yn forem soluções linearmente
indepnedeníes da equação (1), a sua solução geral é da forma
í/= + ^ 2Í/2 + ••• ( 2)
em que Cu Cz, Cn são constantes arbitrárias.
Se os coeficientes da equação (1) forem constantes, acha-se a
solução geral tal como para a equação de segunda ordem.
1 — Forma-se a equação característica

F + a i F “ ‘ + a2F “ 2+ . . . + f ln = 0.
88 CALCUIjO d if e r e n c ia l b in t e g r a l

2 — Acham-se as raízes da equação característica


• • *1
3 — De acordo com o carácter das raízes escrevem-se as soluções
particulares, linearmente independentes, partindo do que se segue:
a) corresponde a toda a raiz real simples k, uma solução par­
ticular e'‘*;
b) corresponde a todo o par de raízes complexas conjugadas
= a -|- ip e A**’ = a — iP duas soluções particulares e“* cos px
e e’ * sen Px *
c) corresponde a toda a raiz real k de ordem de multiplicidade
r tantas soluções lineares independentes
phx kx hx.

d) cx)rresponde a todo o par de raizes complexas conjugadas


= a + iP, = a — íp, de ordem de multiplicidade /i, 2/ül solu­
ções particulares

* COS px, COS Px, . . . , COS Px,


e®* sen Px, xe®* sen Px, . . . , x**” ^e®*sen Px.

O número destas soluções é igual ao gràu da equação caracte­


rística (que também é a ordem da equação diferencial proposta).
Demonstra-se que estas soluções são linearmente independentes.
4 — Tendo encontrado n soluções linearmente independentes 3^1,
3^2. .... yn. escreve-se a solução geral da equação diferencial proposta
sob a forma
y = CiVi - f + . . . + CnUnf

onde Cu C 2, ... Cn são constantes arbitrárias.


Exemplo — 4. Encontrar a solução geral da equação

Resolução — Formemos a equação característica

*4— 1 = 0.
As raízes desta equação são
^l = lt * 2 = — 1| *3 = 1, * 4 = — í.
O integral geral, é póis,
y ^ C i € ^ + C ^ - ^ - \ - A c o sx -f B senx,
cm que Ci, C2, A, B são constantes arbitrãrías.
EQUAÇÕES D IF E R ia^C IA IS 89

Nota — 2. Resulta do que precede que toda a difícuTdade da


resolução duma equação diferencial linear homogénea de coeficientes
constantes reside na resolução da equação característica correspondente.

§ 23. Equações lineares não homogéneas de segunda ordem

Seja uma equação linear não homogénea de segunda ordem


y" + a^y' + a^y = f (x). (1)

A estrutura da solução geral da equação (1) é dada pelo teorema


seguinte.

Teorema— 1. Á solução geral da equação não homogénea (1)


é a soma durna solução particular qualquer y* desta equação e da
solução geral y da equação homogénea correspondente

y" + fflií/' + <h.y = ^- ( 2>

Demonstração — Deve-se demonstrar que a soma

y = v + y* (3>
é a solução geral da equação (1). Demonstremos, em primeiro lugar,
que a função (3) é uma solução da equação (1).
Substituamos a soma ^ + i/* na equação (1) em vez de y,
Ter-se^á:

{«/ + y*)" + ffli (y + y*)' + «2 {y + y* ) = / (x)


ou
(y" + aiV' + (hy) + iy*" + ^ty*' + <hy*) = / (a^)- (4 >

Sendo y solução da equação (2), a expressão do primeiro parên­


tesis é idênticamente nula. Sendo uma solução da equaçção (1),
a expressão do segundo parêntesis é igual a /(jc). A igualdade (4) é
pois uma identidade. A primeira parte do teorema está assim demonstrada.
Mostremos agora que a expressão (3). é a solução geral da
equação (1), isto é, que se pode escolher as constantes arbitrárias que
ela contenha, de maneira a que sejam satisfeitas as condições iniciais:
U x — X q ----- í/o» 1
(5)
y'x^xo = y i í
90 CALCULO D IFEREN CIA L E INTEGRAL

quaisquer que sejam Xo,*'yo e (desde que jco seja tomado no domínio
de continuidade das funções «i, e / ( jc)).
Tendo em atenção que se pode por y sob a forma
y = Ciyi + C^V2,
em que yi e 3^2 são duas soluções linearmente independentes da equa­
ção (2) e Cl e C2 constantes arbitrárias, pode-se recopiar a igualdade (3)
sob a forma
y ^ C ^ y i + C^Vi + y*. (3')

Resulta das condições (5) que(*)


+ ^ 2Í/20 4" í/o = í/o»

^lí/lO + ^ 2^20 + í/o = í/o-


É-nos preciso deduzir Ci e C2 deste sistema, Recopiemo-lo sob
a forma
= U
^lí/io + ^ 21/20 — y oo - y l I
( 6)
^lUio + C2Í/20 = yo — y t - J
Note-se que o determinante dos coeficientes das incógnitas Ci e C 2
é o wronskien das funções yt e >^2 calculado no ponto x = Xn. Dado
que estas funções são linearmente independentes, por hipótese, o
wronskien não é nulo; o sistema (6) possui, pois, uma solução bem
determinada Ci e C2, isto é, que existem constantes Ci e C2 tais que
a fórmula (3) define a solução da equação (1) que satisfaz às condições
iniciais dadas. O teorema está completamente demonstrado.
Por conseguinte, se se conhecer a solução geral y da equação
sem seçundo membro (2) o problema reside em encontrar uma solução
particular qualquer y* da equação com segundo membro (1).
Indiquemos uiti método geral que permita encontrar soluções
particulares duma equação com segundo membro.
Método da variação das constantes — Escrevamos a solução geral
da equação homogénea (2):
y = Ciyi + Ciy^. (7)
Vamos procurar uma solução oarticular da equação não homo­
génea (1) sob a forma (7), tendo em consideração que Ci e C2 são
funções de x que é preciso determinar.

(♦) Aqui yio, ^20» y?» Í^ío» í/20» y*' sâo valores que tomam as funções
para x = xq.
y u y i , y * t y [ , yí^ y * '
EQUAÇÕES D IFEREN CIA IS 91

Derivemos a igualdade (7):


y = C{yi + CiVi + CiVi + ^^2^2*
Escolhamos as funções Ci e C2 de maneira que seja satisfeita a
igualdade
('iVi + C2Í/2 = 0. (3)
Sendo assim, a derivada primeira / torna-se
y = C i y i + ^'22/2*
Derivando agora esta expressão, acha-se
y ' = Ciyi + C21/2 + Ciyi + í ^2í/2-
Substituamos y, y \ / ' na equação (1). Obtém-se:

H” ^2Í/2 + -j- C2I/2 + + í^2Í/2) +


+ ^ + C22/2) = / W
ou
^1 (í/l + ^lí/l + ^ y Ò + ^2 (i/2 + ^li/2 + ^2/2) +
+ Cil/i + C2^2 = / (^)*

As expressões contidas nos dois primeiros parêntesis anulam-se


pelo facto de yi e y 2 serem soluções da equação homogénea. Por
conseguinte, esta última igualdade, toma a forma
C^ií/í + Í^22/2 = / (^)* (9)

Assim, a função (7) é uma solução da equação com segundo


membro (1) visto que as funções Ci e C2 satisfazem às equações (8)
e (9), isto é, se se tiver

+ C2Í/2 = 0, Cií/i 4" ('2y2 = / W*


Ora o determinante deste sistema é o wronskien das fuiíções
linearmente independentes yi e y^. pois que não é nulo; calcifla-se
C[ e C' como funções de x resolvendo o sistema anterior:
Cí=(pi(x), C2=(P2(^).
Integrando, calcula-se:
Cl = J (^) “h Ci; C2 = J 92 (^) + C2,
em que Ci e Cg são constantes de integração. \
92 CALCULO D IFEREN CIA L E INTEGRAL

Substituindo as expressões de Ci e C2 na igualdade (7), acha-se


um integral dependente de duas constantes arbitrárias Ci e isto é,
a solução geral da equação com segundo membro (*).
Exemplo — Determinar a solução geral da equação

Resolução — Determinemos a solução geral da equação homogénea

Tem-se:
Logy'=U>gx+LogC-, y '= C x ;
y *
assim,
y=Cix^+C2.
Para que esta expressão seja solução da equação proposta, é preciso
determinar Ci e C2 como funções de x do sistema
C[x'^ -j- C2 • 1 = 0, 2C[x -f- í/2 •0 = X.
Resolvendo este sistema, vem

Cí = l .

e por integração:
C i= - |- + C i, Cz----- -Ç -+ C 2.

Substituindo as funções encontradas na fórmula y = Cix2 + C2, obtém-se a


solução geral da equação com segundo membro:
— — x3
y = Cix2+ C2+-2------6"
_ — X» - . —
ou y = Cix2 + C2 + ~g“ , em que Cj e C2 são constantes arbitrárias.

O teorema seguinte pode ser útil para a procura de soluções


particulares.
Teorema — 2. Seja uma equação não homogénea
í/" + (hV + 0.2V= fi (^) + Í2 (^) (1 0 )
cujo segundo membro é a soma de duas funções /i (jc) e /z (x). Se
for uma solução particular da equação
/ + «2Í/ = / lW (1 1 )
(♦) Se se fizer Ci = C2 = 0, obtém-se uma solução particular da
equação (1).
EQUAÇÕES D IFEREN CIA IS 93

e y 2 uma solução particular da equação


y" + aiy' + (x), (12)
+ 3^2 é uma solução particular (*) da equação (10).
Demonstração — Substituindo a expressão + >^2 na equação (10),
obtém-se:
(y i 4" Hiò'* 4” ^1 (^1 4" 4“ ^2 (i/i 4" = / i (^) 4~ A (^)
ou
(í/i 4" n ii/í + fl2Í/i) 4" (í/2 4“ ^ ií /2 4 “ ^2Í/2) = / i (^) 4" /2 (^)* (13)
Resulta das igualdades (11) e (12) que (13) é uma identidade.
O teorema está demonstrado.

§ 24. EquaçSes lineares não homogéneas de segunda ordem


de coeficientes constantes
Seja a equação diferencial
y'" + p y ' + qy = í (^)* ( 1)
em que p t q sao numeros reais.
Indicou-se no parágrafo anterior um método geral de procura
das soluções das equações não homogéneas. Quando a equação é de
coeficientes constantes, por vezes é mais simples encontrar uma solução
particular sem integração. Consideremos tipos de equações (1) às quais
se aplica esta nota.
I — Suponhamos que o segundo membro da equação (1) é o
produto dum exponencial por um polinómio:
/(x ) = P „ (x )e “*. (2)
em que P» (x) é Um polinómio do grau n. Os casos seguintes podem-se
apresentar:
a) O número a não é uma rcáz da equação característica
A* + pA: + ç = 0.
É preciso, então, procurar a solução particular sob a forma
y. = (ylox" + ^ , x " - ‘ + . . . + ^ „) (x) (3>
Com efeito, substituindo y* na equação (1) e simplificando por
ter-se-á
Q'n {^) + (2a + p) Q'n (a:) + (a^ + pct + g) (x) = P„ (x). (4>
(*) Ê evidente que este teorema subsiste para um número arbitrário de
termos no segundo membro. /
94 CALCULO D IFERENCIA L E INTEGRAL

Qn (x) é um polinómio de grau n, Qn (x) e Qn (x) são, respec-


tívamente, polinómios de graus n — 1 e n — 2. Tem-se, pois, dum
lado e doutro da igualdade polinómios de grau n. Igualando oscoe
cientes das mesmas potências de x (o número de. coeficientes des­
conhecidos é igual a « + 1), obtém-se um sistema de « + 1 equaç
para a determinação dos coeficientes A q, A i , A z....... An-
b) a é uma raiz simples da equação característica.
Se se procurasse, então, uma solução particular sob a forma (3),
obter-se-ia no primeiro membro da igualdade (4) um polinómio de
grau n — 1, dado que o coeficiente de Qn(x), quer -h pa-j-q, é
nulo e que Qn (^) © Qn (x) são polinómios de graus inferiores a n.
Por conseguinte, a igualdade (4) não poderia ser uma identidade
qualquer»que seja a escolha das constantes Ao, Ai, Az....... An-
Então, no caso considerado, procurar-se-á a solução particular
sob a forma de polinómio de grau n + 1 privado do seu termo cons­
tante (porque este último desaparece após a derivação) (♦):

y* = ^Qn (^)
c) a é uma raiz dupla da equação característica. O grau do
polinómio baixa, então, de duas unidades quando se substitui a função
Qn (^) equação diferencial. Com efeito, sendo a uma raiz da
equação característica, + «p + ^ = 0; além disso, sendo a raiz dupla,
tem-se 2a — p (sabe-se, efectivamente, que a soma das raízes da
equação do segundo grau escrita acima é igual ao coeficiente do
termo do primeiro grau tomado com o sinal menos). Assim. 2a p = 0.
Resta, pois. no primeiro membro da igualdade (4) Ç" (x), isto é,
uih polinómio de grau n — 2. Para que o resultado da substituição
seja um polinómio de grau n, torna-se necessário procurar uma solução
particular sob a forma de produto de por um polinómio de
de grau m H- 2. A constante e o termo do primeiro grau deste poli­
nómio desaparecem então, após derivação e poder-se-á omitir na
solução particular.
Assim, quando a é uma raiz dupla da equação característica,
procurar-se-á uma solução particular sob a forma

Exemplo — 1. Achar a solução geral da equação


y" -\-3y = x.
Resolução — A solução geral da equação homogénea correspondente é

y^C ie-^ + Cze-^^-


(*) Notemos que todos os resultados acima são válidos quando a é
um número complexo (isto resulta das regras de derivação da função
dado que m é um número complexo arbitrário; ver § 4, Cap. VII).
EQUAÇÕES D IFERENCIA IS 95

Como o segundo membro da equação não homogénea é da forma


(isto é, da torma (x) e®*) e não sendo 0 raiz da equação característica
jfca _j_ 4^ _j_ 3 = 0, procuraremos uma solução particular sob a forma p* = Çi (ít) e®*,
isto é, que faremos

Substituamos esta expressão na equação proposta. Tem>se:


44o + 3 (i4oa: H-i4i) = X.
Deduz, igualando os coeficientes das mesmas potências de x dum e
doutro lado da igualdade:
3í4q— 11 44q 3i4j = 0^
donde
1 4
Aq= •

Por conseguinte.
* 1 4
9-
A solução geral y ^ y - { - y * será

I, = Cie-* + Cje-s* + -g-1- - .


Exemplo — 2. Achar a solução geral da equação
y ''-f9 y = (x 2 + l)^3x^
Resolução — Encontra-se fàcilmente a solução geral da equação
y -—C^ COS 3 x ~f“ ^2 SCO 3 x .
O segundo membro da equação dada e3x ^ ja forma
P 2 (^) c3*.
Como o coeficiente 3 no expoente não é uma raiz da equação caracte­
rística, procuramos uma solução particular sob a forma
y* — ou y* = (.4x2 + + 0 c®*.
Substituamos esta expressão na equação diferencial:
[9 (Axi+ fix + C) + 6 (2^x+ B) -|- 2.4 + 9 {Ax^+ B *+ C)] = (i* + 1) «sx.
Simplificando por e igualando os coeficientes das mesmas potências
de X, obtém-se:
18x4-1, 12/l + 18i? = 0, 2.4 - f 6i?+18C = l,
1 1 5
donde, A = j ^ ; P = — ^ \ C = ~ . A solução particular é, pois,
lo 2 .1 ol

J/*= X*__ L i- ! e3x


" U8 27 ' SI J
e a solução geral
y = Ci cos3x + C 2sen3x+ +
96 CALCULO D IFEREN CIA L E INTEGRAL

Exemplo — 3. Resolver a equação


íT —7y' + 6y = (x—2)tf*.
Resoluçõo — Aqui o segundo membro é da forma em que
o 1 do expoente é uma raiz simples do polinómio característico. Procuraremos,
pois, uma solução particular sob a forma
y* = xQi (x) e* ou y* = x { A x+B ) e* ;
Substituindo esta expresão na equação, tem-se:
[(^x2 + Bx) + ( ^ x + 2B) + 2 A - 1 (Ax^+ Bx) —
—7 (2Ax + B) + 6 (Ax^ + Bx)] e* = (x—2) e*,
ou ainda
(— lO^x — 5B + 2/1) e* = (X—2) c*.
Igualando os coeficientes das mesmas potências de x, vem
— 10^ = 1, —5B + 2 ^ = - 2 .
1 9
donde Tem-se, pois, para solução particular

e a solução geral escreve-se

^=Cie«*+C2e*+* ( e*.

n — Suponhamos o segundo membro da forma


f{x) = P (x) e“* COS Px + Ç (x) e“* sen Px, (5)
onde P (x ) e Q(x) são polinómios.
Pode-se examinar este caso como o anterior passando as funções
trigonométricas a exponenciais. Substituamos cosjSx e stnfix pdas suas
expressões exponenciais dadas pelas fórmulas de Euler (ver -§ S.
cap. V n ) . Obtém-se:
—e
/ (X) = i> (X) ----- + Q (X)
2i
ou
^(a+ip)x
f(x ) = ií.( x ) + ^ 9 ( x ) ]

( a —ip)x
+ ( 6)

Tem-se nos parêntesis rectos polinómios cujo grau é igual ao


grau mais elevado de P (x) ou de Q (x). Vê-se que o segundo membro
foi posto sob a forma do caso I.
EQUAÇOBS D IFEREN CIA IS 97

Mostra-se (não o demonstraremos) que se podem encontrar solu­


ções particulares que não contenham quantidades complexas. Por
conseguinte, quando o segundo membro da equação (1) é da forma
f(x, = P (x) COS ^ x + Q {x) sen (7)

sendo P (x ) e Q(x) polinómios, determina-se como se segue, a forma


da solução particular:
a) se a + ip não é raiz da equação característica, é preciso
procurar uma solução particular da equação (1) sob a forma

y*z=U {x) COS px + 7 (x) sen px, (8)


sendo U (x) e V (x) polinómios cuja grau é igual ao grau mais elevado
de P(x) ou de Qix)-.
b) se a + é raiz da equação característica, tomar-se-á uma
solução particular sob a forma
y = x[ ü (x) e“* C O S ^x-\-V(x) sen Pa:]. (9)
Para evitar possivei's erros, notemos que as formas indicadas das
soluções particulares (8) e (9) são evidentemente conservadas também
no caso em que o segundo membro da equação (1), um dos polinó­
mios P(x) e Q (x) é um polinómio idênticamente nulo, isto é, quando
o segundo membro é da forma
P (x) e“* COS Pa: ou Q (x) e“* sén Pa:.
Consideremos, em seguida, um caso particular importante. Supo­
nhamos que o segundo membro duma equação linear de segunda
ordem é ^ forma
f (x) = M COS Px + iV sen Px, (7')
em que M e N são constantes.
a) Se pi não é raiz da equação característica, procurar-se-á uma
solução particular sob a forma
y* = A COS px + 5 sen Px. (8')
b) Se pi é raiz da equação característica, procurar-se-á uma
solução particular sob a forma
y* = X (A COS Px - f 5 sen Px). (9')
Notemos que a função (70 é um caso particular da função (7)
(P(x) = M, Q (x) = N, a = 0); as funções (80 e (90 são casos par­
ticulares das funções (8) e (9).
CALCULO D IFEREN CIA L E INTEGRAL

Exemplo — 4. l>.termiiiar o integral geral da equação linear não


homogénea
+ + = 2 COS X.

Resolução — A equação característica 2A; + 5 = 0 tem por raízes


= — l + 2i, At2 = —1— 2í. O integral geral da equação homogénea correspon­
dente escreve-se pois
y = e-*(Ci cos2x + C2Scn 2x).
Procuremos uma solução particular da equação com segundo membro
sob ia forma
y* = >lcosx + B sen x,
sendo A e B constantes a determinar.
Substituamos y* na equação proposta. Tem-se:
—^^4 COS X—Bsén x + 2 (—^ sen x-{-B cos x) + 5 (A cos x + Bsen x) = 2 cos x.
Igualando os coeficientes de cosx e de senx, obtém-se duas equações para
determinar A t B\
— ^ + 2B + 5yl = 2 ; —B—2/l + 5B = 0,

donde ^ = 5 B = 4~ o •
A solução geral da equação proposta y = y + y*, é
2 1
y = e~^ {Cl cos 2x + C^2 2x) + cos x + sen x.
u 0
Exemplo — 5. Resolver a equação
y''-\-^y = cos 2x.
Resolução — As raízes da equação característica são ki = 2 i ^ k 2 = —2i ;
a solução geral da equação homogénea é, pois,
y = Ci COS 2x-f-C2 s e n 2x.
Procuremos uma solução particular da equação com segundo membro
sob a forma y* = x {A cos 2x + B sen 2x).

Tem-se: í/*' = 2x (— ^ sen 2x + B cos 2x) + ( A cos 2x + B sen 2x),


y * " = — ^ x { — A cos2x— B sen2x) + 4 ( — ^ sen2x + Bcos2x).

Substituamos estas expressões na equação proposta e igualemos os coefi­


cientes de cos 2x e sen 2x; obtém-se um sistema de equações para a determinação
áe A t B:
1
4B = 1 ; — 4 ^ = 0, donde .4 = 0, B = -^ •
O integral geral da equação dada é, pois,
4
y = Ci cos 2x + C2 sen2x + -^ X sen 2x. sen
Exemplo — 6. Resolver a equação
y*—y = cos X.
EQUAÇÕES D IFEREN CIA IS 99

ResoJução — O segundo membro da equação é da forma


/ (x) = (M COS x + s e n x)
com Aíc=3, ÍV= 0. A equação característica —1 = 0 tem como raízes =
t k 2 = —l.A solução geral da equação homogénea é
y = Cie* + C2e-*.
Como a + < P = 2 + i . l , não é raiz da equação característica, procurar-se-á
uma solução particular sob a torma
y* = e2x COS x-\-B sen x).
Substituindo esta expressão na equação, obtém-se, após redução dos termos
semelhantes,
(2A + 4B) COS X+ (— 4/1 + 2B) e** sen x = 3e2* cos x.
Igualando os coeficientes de cos x e sen x, vem
2A f 4 ^ = 3, — AA -h 2B = 0,
3 3
donde ^ = — , 5 = — . A solução particular é, pois,

3 3 \
(^ c o s a i+ y s c n x j ,
e a solução geral
3 3 \
(^ c o s z + -^ s e n ij .

§ 25. Equações lineares não homogéneas de ordem n


Seja a equação
+ *’ + . . . + a „ y = f{x). ( 1)
em que ai, ü2 ......., / (jc) são funções contínuas de x (ou constantes).
Suponhamos que se conhece a solução geral
1/ = C il/i + C 2 Í/2 + ••• + ^ n í/7 i (2)
da equação sem segundo membro
-f- • • • "f“ = 0. (3)
Tal como para a equação de segunda ordem, tem-se o teorema
seguinte.
Teorema — Se y é a solução geral da equação homogénea (3)
e uma solução particular da equação não homogénea (1),
Y = y -\-y*
é a solução geral da equação completa.
100 CALCULO D IFEREN CIA L E INTEGRAL

Por conseguinte, tal como para a equação de segunda ordem,


a integração da equação (1) resume-se na procura duma solução par­
ticular da equação com segundo membro.
Tal como para a equação de segunda ordem, pode-se achar uma
solução particular da equação (1) pelo método da variação das constantes
supondo que em (2) Ci, Cn sejam funções de x.
Formemos o sistema de equações (comparar § 23):
^1^1 “1“ ^'2^/2 + . • • + — 0,
^lí/l + ^2Í/2 + • • • + CnVn = 0»
(4)
2)

+ ...+ c 'jr - ^ ^ = í { x ) . j
Este sistema de equações de incógnitas C', .... Cn, tem uma
solução bem determinada. (O determinante dos coeficientes c ;, c ;, ....
Ç n é O determinante de Wronski das soluções particulares y ^ , y 2 , yn
da equação homogénea, que se supõem linearmente independentes;
não é, pois, nulo.)
O sistema (4) pode, pois, ser resolvido em relação às funções
C[, Cg, . . ., Cn.Integremo-las, uma vez encontradas:
Cl = J Cj dx -|- Ci‘, C2 = J C2 dx -f- C2; . . .;
C n = ^ Cndx
onde Cl, C2, . . ., Cn são constantes de integração.
Mostremos que a expressão
y* = Ciyi C2^2 + • • • + Cnyn (5)
é a solução geral da equação completa (1).
Derivemos a expressão (5) n vezes tendo em conta, de cada vez,
as igualdades (4); ter-se-á, então:
y* = Ciyi + C 2 Í/2 + C 3 ^ 3 Cj^yn,
y* = Ciyi + ^'2^/2 + ^3^3 + . . . + Cní/n,

+ C2Í/2" • 4 ' Cnyn'


y* ^ + Cai/^^ i- .. . + / (^)-
Multipliquemos a primeira equação por an ,a segunda por «n-i»
a penúltima por e juntemos. Obtém-se
♦(n)
EQUAÇÕES D IFEREN CIA IS 101

dado queí/i» í/2» • • •» í/n são soluções particulares da equação homo­


génea e que, por conseguinte, as somas obtidas, juntando entre si os
termos duma mesma coluna, são nulas.
Por conseguinte, a função y ^ = Cij/i + . . . + Ci, ...,
Cn são funções de x determinadas pelas equações (4)) é uma solução
da equação não homogénea (1), e como contém n constantes arbi­
trárias Cl, C2, . . é a solução geral.
A proposição está assim demonstrada.
Por vezes, é mais fácil encontrar soluções particulares duma equa­
ção não homogénea de ordem n de coeficientes constantes (confrontar
§ 24). Assim é quando:
I — Suponhamos que o segundo membro da equação diferencial
é da forma f (x) — P (x) sendo P (x) um polinómio em jc;
convém distinguir dois casos:
a) se a não é raiz da equação característica, procurar-se-á uma
solução particular sob a forma

y = Q W e“*,
em que Q(x) é um polinómio do mesmo grau que P ( jc), mas com
coeficientes indeterminados;
b) se a é raiz de ordem de multiplicidade /j. da equação caracte­
rística, procurar-se-á uma solução particular da equação com segundo
membro sob a forma
y. = x*‘Ç (x)e“*,

sendo Q{x) um polinómio do mesmo grau que P(x).


II — Suponhamos o segundo membro da forma
f (x) = M COS px + sen
onde M Q N são constantes. Determina-se, então, a solução particular
como se segue:
a) se pi não é raiz da equação característica, a solução particular
é da forma
y* = A COS Pa: + B sen Px,

em que A e B são coeficientes constantes indeterminados;


b) se pi é raiz de multiplicidade /x da equação característica,
tem-se
y* = x'^ {A COS px - f sen px).
III — Seja
f{x) = P (x)'e** COS px + (? (x) e“ * sen px,
102 CALCULO D IFEREN CIA L E INTEGRAL

em que P(x) e Q (x) são polinómios em jc. Tem-se:


a) Se a + pi não é raiz de ordem de multiplicidade fi da
equação característica, procura-se uma solução particular sob a forma
y * = U (x) COS ^x + V (a:) sen ^x,
em que U (x) c V (x) são polinómios cujo grau é igual ao grau mais
elevado de P(a:) e de Q(x).
b) Se a +. pi é raiz de ordem de multiplicidade fi da equação
característica, procura-se uma solução particular sob a forma
y* = x^[U (x) COS Px + F (x) sen Px],
em que í/ ( jc ) e F (x) têm o mesmo significado que para o caso a).
Nota geral para os casos II e III — Se o segundo membro da
equação contiver sòmente cos px ou sen px, será preciso quando muito^
procurar uma solução sob a forma indicada, isto é, com um seno ou
um cosseno. Noutros termos, pelo facto do segundo membro não
conter cospx ou senpx não resulta, de forma alguma, que a solução
particular não contenha estas funções. Pode-se comprovar isso mesmo,
considerando os exemplos 4, 5 e 6 do parágrafo anterior e ver-se-á
no exemplp 2 deste parágrafo.
Exemplo — 1. Determinar a solução geral da equação
y ^^ — y = x ^ + i .
Resolução — A equação característica â:* — 1 = 0 tem como raízes
^i = lt ^ 2 = —1» k2 = i , k ^ = —-i.
Achemos a solução geral da equação homogénea (ver exemplo 4, § 22):
y = Cie* + C2«“* + C3 cosa: + C4 sen x.
Tomar-se-á uma solução particular da equação completa sob a foitna
y* = A qX^ + AiX^ + A 2X+ A^.
Derivemos quatro vezes e substituamos as expressões obtidas na equação
dada. Obtém-se:
—A qx^—Aix^ —A 2 X— As = x^-\-í,
Igualemos os coeficientes das mesmas potêiicias de x. Tem-se:
— A o = í ; —^ , = 0 ; —^2 = 0 ; — ^3 = 1.
Por conseguinte,
y *= —a:3— 1.
Encontra-se o integral geral da equação completa sob a forma
l/ = y + y*» ou seja
y= COSX+ C4 sen x —x^ — 1.
Exemplo — 2. Resolver a equação
y^^ — y = 5 cosx.
EQUAÇÕES D IFEREN CIA IS 103

Resolução — A equação característica — 1 = 0 tem por raízes Afi = l*


— 1, *3 = 1, ^4 = — í. A solução geral da equação homogénea é, pois,
^ + C 2e ~ ^ + C 3 COS X + C 4 , sen x .
O segundo membro da equação proposta é da forma
t {x) = M z o s x + N sen x,
com M = 5, TV = 0.
Como í é uma raiz simples da equação característica, procura-se uma
solução particular sob a forma
y* = x ( A COS x + ^ s e n x).
Substituindo esta expressão na equação, obtém-se:
4 4 sen X—4B cos x = 5 cos x ,
donde
44 = 0, — 45 = 5
A solução particular da equação diferencial proposta é, pois, A = 0, B =
_ 5
4 * y* = — ^ X sen X
e a solução geral

y + + cosX-I-C4 sen r— ^ x senx.

§ 26. Equação diferencial das oscilações mecânicas


O objecto deste parágrafo e dos parágrafos seguintes é o estudo
dum problema de mecânica por meio de equações diferenciais lineares.
Consideremos uma massa Q colo­
cada sobre uma mola em espiral (fig. Í68).
Seja y o desvio desta massa a partir
da sua posição de equilíbrio. O desvio
para baixo será considerado como posi­
tivo, o desvio para cima será negativo.
Na posição de equilíbrio a força de
gravidade que age sobre a massa é
compensada pela elasticidade da mola.
Suponhamos que a força de chamada
é proporcional ao desvio, isto é, que
se exprime por — ky em que k é uma
dada constantes chamada «Rigidez» da Fig. 268
mola (*).
Suponhamos que se opõe ao movimento da massa Q uma força
de resistência proporcional à velocidade do movimento em relação ao

(♦) Uma mola cuja força de chamada é proporcional à deformação


diz-$e «característica linear».
104 CALCULO D IFEREN CIA L E INTEGRAL

ponto mais baixo da mola isto é, uma força —Xv = —X ^ , em que


at
X = const. > 0 (um amortecedor).
Escrevamos a equação diferencial do movimento. Em virtude da
segunda lei de Newton

^ dl^ dt ( 1)

(k e X são números positivos). Obtivemos uma expressão diferencial


linear homógenea de segunda ordem de coeficientes constantes.
Escrevamo-la sob a forma

tdt + w = o. ( 1' )
dt^
com
X k
Q
Além disso, suponhamos que o ponto inferior da mola efectua
um movimento vertical que obedece à lei z = (0- Tal é o caso se
a mola estiver fixada pela sua extremidade inferior a um rolo, des­
crevendo um contorno dado (fig. 269).

A força de chamada.será então, não — ky mas —k [i/ + tp (01>


a força de resistência será —X [y' + <p' (í)l obtém-se, em vez da
equação (1)

n i í ^ + X - ^ + Â :i/= -A :c p (t)-V (0 (2)


dt^ dt
ou

(2')
EQUAÇÕES D IFEREN CIA IS 105

onde se faz
^ < p (o + v (< )

Obtivemos uma equação diferencial de segunda ordem com


segundo membro.
A equação (1') chama*se equação das oscilações livres, a equa­
ção (20 é a das oscilações forçadas.

§ 27. Oscilações livres


Consideremos, em primeiro lugar, a equação das oscilações livres
y"" + p y ' + qy = 0.
Escrevamos a correspondente equação característica

+ pk q = Q
e achemos as raizes:

1. Seja q^ As raízes A:i e kz são, então, números reais


negativos. A solução geral exprime-se por exponenciais:
y = Cic"'‘ + C2e'‘‘‘ (Al < 0, *2 < 0). ( 1)
Resulta desta fórmula que a distensão y tende assintòticamente para
zero quando r o o , quaisquer que sejam as condições iniciais. Não
há oscilação no caso dado, porque a força de travagem é grande em
relação ao coeficiente de rigidez da mola k.
D
2. Seja - ^ = g; tem-se, então, uma raiz dupla k i= k^ = — — .
* ^
A solução geral escreve-se, pois:
_ __p_ pt

y = C,e ^ '+ C ,te = {C, + C ,i)e ( 2)

Aí ainda a distensão tende para zero quando t oo, mas menos


depressa do que no caso anterior (dado o factor Ci + Czt).
3. Seja p — 0. isto é. que se supõe a ausência de travagem.
A equação característica escreve-se
k^ q = 0,
106 CALCULO D IFEREN CIA L E INTEGRAL

e tem por raízes ki = pi, = —Pi, com p = A solução geral é

y = Ci COS Pí + Ca sen pi. (3)


Substituamos nesta fórmula as constantes arbitrárias Ci e C 2 por
outras, A Q <po ligadas a Ci e C2 pelas relações:

Cl = A sen (po, Cg = A cos (po.

Daí resulta A e <po em função de Ci e C2, como se segue:

A = V C l + Ca, cpo = are •


C'2fl
Substituindo as expressões de Cj e C2 na fórmula (3), obtém-se

y = A sén q)o cos pi ^ (po senPi

1/ = ^ sen(P í + (po). (3')

Tais oscilações dizem-se harmónicas. As curvas integrais são sinu­


soides. O intervalo de tempo T no qual a quantidade pt + (po varia

ou

2n
de 27t chama-se período das oscilações', no nosso caso 7* = -5- . Cha-
P
mamos frequência das oscilações ao número de oscilações no tempo 2tt\
no nosso caso a frequência é p, a. constante A, que represento a dis­
tensão máxima a partir da posição de equilíbrio, chama-se amplitude
do movimento oscilatório; ç»o é a fase inicial.
Representa-se o gráfico da função (3') na fig. 270.

4. Seja p 0 e -^ < g.
As raízes da equação característica são. então, complexas:
EQUAÇÕES D IFEREN CIA IS 107

fci = a + ip, = OL — i p ,
em que
a = - ^ < 0 , P =

O integral geral é da forma

y = e“ ‘ (Cl COS pí + Cz sen PO

1/ = i4 e “ ' sen (pí + q>o). (4)

Somos obrigados a tomar aqui, para amplitude, a quantidade


que depende do tempo. Como a < 0, ela tende para zero quando r -» oo,

isto é, que estamos na presença de oscilações amortecidas. A figura 271,


representa o gráfico de tais oscilações.

§ 28. Oscilações forçadas


A equação das oscilações forçadas escreve-se
y” py' + qy = f {t).
Consideremos o caso importante da prática em que a força per­
turbadora exterior é representada pela função periódica

j {t) = a sen coí;


a equação torna-se. então,
í/" + p y + q y = d sèn d)
108 CALCULO D IFEREN CIA L E INTEGRAL

1. Suponhamos, em primeiro lugar, que p =7^ 0 e ^ < g, isto é,


que as raízes da equação característica são números complexos a ± pi,
A solução geral da equação homogénea escreve-se. então, (ver fór­
mulas (4) e (4'), § 27)
y = A é^^stn + W-
Procuremos uma solução particular da equação com segundo
membro sob a forma
y =z M COS co/ + sen (út. (3)

Substituindo esta expressão de y* na equação diferencial inicial,


encontra-se M e N:

M = ____ __________• N= .
(í-o Y + p V (g _ co Y + p W
Antes de substituir as expressões de Af e na igualdade (3),
introduzamos as novas constantes A* e (p*, fazendo
M = A*sen cp*, N = A*coscp*,
ou seja
^ M
a * = ^ V a j ^ + jv^ = tl^o) = ---- .
V (g _ o )Y + /)W N

Poder-se-á, então, escrever a solução particular da equação não


homogénea sob a forma
3|c :|e a(e :|e :|e :|t
y — A sen 9 COS coí + ^ cos (p sen (út = A sen (co/ + cp )
ou, finalmente.

y =: sen(o)í + ).
V(q — (o Y +

O integral geral da equação (1) é igual z. y = y y * , oa seja

i/ = ^e'*‘ sen(pí + cpo) + sen (o)í + <P ).


V (q — (ú')" +
O primeiro termo do segundo membro fa solução da equação
homogénea) representa oscilações amortecidas. Decresce quando t cresce
e, por conseguinte, ao fim dum certo tempo, é o segundo termo, que
representa oscilações forçadas, que desempenha o papel principal.
A frequência a> destas oscilações é igual à frequência da força exte-
EQUAÇÕES D IFEREN CIA IS 109

rior /(O; a amplitude das oscilações forçadas é tanto maior quanto


mais pequeno for p e que esteja na vizinhança de q.
Estudemos, em detalhe, a dependência entre a amplitude das
oscilações forçadas e a frequência w para diferentes valores de p.
Designemos para este efeito a amplitude das oscilações forçadas
por D(ü>):
D (( d) = . “
V(g — + p 2 (02

Façamos q = (para p = 0 Pi seria igual à frequência das osci­


lações próprias). Tem-se

Façamos
— = X- -^ =
Pl ’ Pl

em que A é o quociente da frequência da força perturbadora e da


frequência das oscilações livres do sistema, não dependendo a cons­
tante \ da força perturbadora. A amplitude exprime-se, então, pela
fórmula
Õ (^) = — ^ ° (4)
P iV (i +

Achemos o máximo desta função. Corresponde, evidentemente,


ao valor de \ para o qual o denominador é mínimo. Mas o mínimo
da função _____________
V(1_A,=*)2 + yV (5)
é atingido para
.= Y i -

e é igual a

V ' - 4 -
Por conseguinte, a amplitude máxima é igual a
a
D.
PiY 7 ^
110 CALCULO D IFEREN CIA L E INTEGRAL

O gráfico da função D (K) para diversos y está representada na


figura 272 (para fixar ideias, fez-se a construção das curvas cor­
respondentes a = 1. = 1). Estas curvas chamam-se curvas de res­
sonância.

Resulta da fórmula (5) que para y pequenos o valor máximo da


amplitude é atingido para valores de A. vizinhos da unidade, isto é.
quando a frequência da força coercitiva é vizinha da frequência das
oscilações livres. Se y = 0 (logo p = 0, isto é, se não houver resis-
EQUAÇÕES D IFEREN CIA IS 111

tência ao movimento, a amplitude das oscilações forçadas cresce inde­


finidamente quando A 1, isto é, quando o) =z= Y q \
lim D (A.) = oo.
(v=0)
Quando = q, há ressonância.
2. Suponhamos, agora, que se tem p = 0, isto é, que conside­
raremos a equação das oscilações elásticas sem resistência em presença
duma força coercitiva periódica:
y'" qy = a sen (ot. (6)
A solução geral da equação homogénea é
^ = Cl C O S p^ - f C2sen. p^ (p^ = q),
Se isto é, se a frequência da força coercitiva não é igual
à frequência das oscilações próprias, a solução particular da equação
não homogénea escreve-se
y* = M COS (oí + iV sên cot.
Substituindo esta expressão na equação proposta, vem
M = 0, N = —^ , .
q — iú
A solução geral é

y = A sen (Pí -|- 9o) “h ■ ' sen coí.


7 — (O
O movimento resulta, pois, da sobreposição das oscilações pró­
prias de frequência p e das oscilações forçadas de frequência ü>.
Se = ü), isto é, se a frequência das oscilações próprias coincide
com a frequência da força coercitiva, a função (3) não é solução da
equação (6). Procurar-se-á, então, em virtude dos resultados do § 24,
uma solução particular sob a forma
y = t{ M COS Qòt-\-N StVi (út). (7)
Encontra-se M e N substituindo esta expressão na equação
diferencial;
M = - — ; N = 0.
2(ú

Por conseguinte.
y = — ^ teos(út
zp
112 CALCULO D IFEREN CIA L E INTEGRAL

A solução geral é da forma

y = Asèn (pí + q ,„) _ — tcospt.


2 (0

O segundo termo do segundo membro mostra que a amplitude


das oscilações aumenta indefinidamente quando t oo. Este fenómeno.

Fig. 273.
que tem lugar quando a frequência das oscilações próprias do sistema
e a frequência da força coercitiva coincidem, chama-se ressonância,
O gráfico da função y* está representada na figura 273.

§ 29. Sistem as de equações diferenciais

Na resolução dum grande número de problemas pede-se para


encontrar funções yi = yt (x), y 2 = y 2 {x), , , , , yn = yn {^) que satis­
façam a um sistema de equações diferenciais que contém a variável x,
as funções desconhecidas yi, yz....... yn e suas derivadas.
EQUAÇÕES DIFEREN CIA IS 113

Consideremos o sistema de equações diferenciais de primeira


ordem:

dyi
= h { x . Ui, Vz, . . . . yn),
dx

= Vit í/2» •••» í/n)»


( 1)

dyn
dx = fn (x , yu í/2» . . . » yn).

onde, yu ........ yn são funções desconhecidas e x a variável.


Um tal sistema, resolvido em relação às derivadas primeiras,
chama-se sistema normal.
Integrar este sistema, é determinar funções yu ^2. . .. yn qüe
verifiquem as equações (1) e que satisfaçam às condições iniciais dada$:

( í^ l ) 3c= x o — í/io » {y^x=X(i — 2/20» . . . » ( 2/n )x = x o — Vno* ( 2)


Integra-se o sistema (1) como se segue.
Derivemos a primeira das equações (1) em relação a jc*

dfi dfi dyi dfi dy„


dx* dx dyi dx dyn dx

Substituamos as derivadas , dyn pelas suas expres-


dx dx ” dx
sões ju fo> .... fn tiradas de (1).
Obtém-se a equação

d^yi
dx^ = P2Í^y yu yn)-

Derivando a equação obtida e procedendo da mesma maneira,


acha-se:

È ^ = z F z i x , yu yz, . . . . y„).
dx"

Assim, continuando, encontra-se finalmente


cTyi
iPn{x, yu . . . . Vn)-
dx"
114 CALCULO D IFEREN CIA L E INTEGRAL

Obtém-se, assim, o sistema seguinte:

= yi, . . . . yn),

^ U l t • • •> í/n)»
da^ (3)

d"yi
= Pn{x< Uu ■■•. Vn)-
dx^

Obtém-se das n — 1 primeiras equações (admitindo que isso seja


possível) >>2. ........... y^ exprimindo-as em função de x, yi e das deri-
Íií! .
dx dx^ ■ ■ ■’ dx"-i •

yi = <P2 {x, yi, y\, .. .. y ' r \


yj = (p3(x, yi, y\, .. y 'r \
(4)

yn==tpn(a;, y i, y i, •
Substituindo estas expressões na última equação (3), obtém-se
uma equação de ordem n baseando-se em y i’.

= y i, y i, • • • . y i" (5)
dx^
Determina-se resolvendo esta expressão:
— ^2» • • •» ^n)* ( 6)

Derivemos esta expressão n — 1 vezes; encontram-se as derivadas

í *§•.. .................
Substituamos estas funções na equação (4). Encontram-se y2 ,
ys....... Vn :
V2 ''1^2 ^1» ^2» • * 1
(7)
yn = ^ n {^ . Cl, C2, . . C J. I
Para que a solução obtida satisfaça às condições iniciais dadas (2),
não resta mais do que determinar ém (6) e (7) os valores das cons-
EQUAÇÕES D IFEREN CIA IS 115

constantes Cu C 2 ........ (como se fez no caso duma só equação


diferencial).
N ota — 1. Se o sistema (1) for linear em relação às funções des­
conhecidas, a equação (5) será também linear.
Exemplo — 1. Integrar o sistema

- ^ = y+ 2 + x, - ^ = —4í^—3z + 2x (a)
com as condições iniciais

Resoluções: ( í') * = o - l. (* )x = 8 -0 . (b)


1. Derivando em relação a jc a primeira equação, tem-se
d^y dy ^ dz , ^
dx^ dx dx
dy dz
Substituindo, nesta última, as expressões c obtidas das equa­
ções (a), obtém-se:

- ^ = (» + * + *) -f-(— 4 y — 3s + 2*) + l
ou
d^y
- 3y—2z -j- 3x -j-1. (c)
dx^
2. Deduz-se da primeira equação (a)
dy
(d)
Substituindo esta expressão em (c), obtém-se
d»y

ou
----
dx^ + 2 - 3 j - f y = 5 * - f l. (e )
A solução geral desta última é
y = (Ci + C2x) e-* + 5 x - 9 (f)
e tendo em conta (d)
z = (C2 —2(7i — 2C2X) e-* - 6x + 1 4 . (g)
Escolhamos as constantes Ci e Cs de maneira a satisfazer âs condições
iniciais (b):
( i/ ) x = o = f» ( ^ ) x = o = ^«

Deduz-se, então, das igualdades (f) e (g)


1 = C^— 9, 0 = C2—2Ci + 14,
donde ^ = 1 0 e C-. = 6. /
Por conseguinte, a solução que satisfaz às condições iniciais (b) escreve-se
y = (10 + 6x) e-* + 5x—9, z = (— 14 — 12x) e-^ — 6o: + 1 4 .
116 , CALCULO D IFEREN CIA L E INTEGRAL

Nota — 2. Suposemos, nos raciocínios anteriores, que se podia tirar


as funções .... das n — 1 primeiras equações (3). Mas pode-se.
por vezes, tirar yz. . Vn de menos de n equações. Obtém-se. então,
para determinar y, uma equação diferencial de ordem inferior a n.
Exemplo — 2. Integrar o sistema
dx dy dz ,
-d t= ^+ « -
Resolução — Derivando em relação a / a primeira equação, encontra-se
d ^ x _dy dz
=(i + z) + (i + y),
dV^ dt ' dt
d^x
2x-\-y -\~z.
dt^
Eliminando as variáveis y e z das equações
dx d2x .
dt^ y + 2,
obtém-se uma equação de segunda ordem em relação a x:

__ 2 i - 0
dí2 dt
O integral geral desta última é
x = Cie-^-\-Coe^K (a )
Donde
e t j / = - ^ _ z = _ c , e - í + 2 C2e2t _ z . (P )
Substituindo as expressões acima de x e y na terceira equação do sistema
proposto, obtém-se uma equação que permite determinar z:
dz

Integrando esta equação, encontra-se


^— (y)
Mas tem-se, então, em virtude de ip):
í / = - ( C i + C3)e-í + C2^2í. (ô)
As equações (a ), (5), (y) dão a solução geral da sistema proposto.

Pode suceder que as equações diferenciais dum sistema contenham


derivadas de ordens superiores a um. Por conseguinte, a ordem do
sistema eleva-se.
Assim, o problema do movimento dum ponto material solicitado por
uma força F reduz-se a um sistema de três equações diferenciais dc segunda
ordem. Sejam F^, F^ as projecções da força F sobre ^ s eixos coordenados.
EQUAÇÕES D IFEREN CIA IS 117

A posição do ponto em cada instante / é definida pelas suas coordenadas x,


y, z. Resulta daí que x, y» z são funções de /. As projecções do vector
velocidade do ponto material sobre os três eixos são -4^-, .
dt dt dt
Suponhamos que a força F e, por conseguinte, as suas projecções Fx, F,V'
Pz . dependem do tempo /, da posição jc, y, z e da velocidade .
dt dt dt
As funções que se procuram neste problema são
x = i(í), v = v(t), z = z(t).
Determinamo-las a partir das equações da distância (lei de Newton):
d^x „ í^ dx dy dz \ \
'” ■*2 l* ’ *’ ~dt' H F ’ ~dr) ’
d^y „ / dx dy dz \
-d t' s r )' \ w
d^z _ / dx dy dz \
i T ' -S T ' - w ] -
Obtivemos um sistema de três equações diferenciais de segunda ordem.
Se o movimento é plano, isto é, se a trajectória é uma curva plana (por
exemplo, do plano Oxy) obtém-se um sistema de duas equações para determinar
x{t) fí y (/):
d^x / dx dy ^
> dt i (9)

( dx dy
-F 1 ( 10)
* d r '' dt
Pode-se resolver um sistema de equações diferenciais de ordem n redu­
zindo-o a um sistema de equações de primeira ordem. Mostremos como se
procede com o exemplo (9) e (10). Introduzamos as notações:

dx dy
dt dt "
Tem-se
d^x du dv
dt^ dt dt^ 1T
O sistema de duas equações de segunda ordem (9) e (10) a duas fun­
ções desconhecidas x (/) e y é substituído por um sistema de quatro equações
de primeira ordem com quatro funções desconhecidas x, y, u, v:
dx
-=u,
dt
dy = u,
dt
du
m - ji i - = F x { t , X, y, lí, u),
dv
m — = Fy{t, X, y, u, v).
118 CALCULO D IFERENCIA L E INTEGRAL

Indiquemos, para terminar, que o método geral examinado de resolução


de sistemas de equações diferenciais pode ser substituído, em certos casos con­
cretos, por processos artificiais que permite chegar mais ràpidamente ao fim.
Exemplo — 3. Determinar a solução geral do sistema de equações dife­
renciais
d^y
dx^
d^z
dx2 = y-

Resolução — Derivemos duas vezes em relação a jc os dois membros da


primeira equação:
d^y _ d^z
dx^ ~ dx2
Ora,
d^z = logo se obtém a equação de quarta ordem
d^y
dx^ =y-

Por integração, obtém-se a solução geral desta equação (ver t. II, cap. XIII,
§ 22, exemplo 4):
y= + COS + senx.

Tiremos
0X2 desta equação e substituámo-lo na primeira equação do
sistema proposto. Determina-se
z= + — ^3 COS X—C4 sên x.

§ 30. Sistem as de equações diferenciais lineares


de coeficientes constantes
Seja dado o sistema de equações diferenciais
dx-^
— ^11^1 + ^12^2 H“ • • • +
dt
dx^
— ^ 2 1 *^ 1 “ h ^ 2 2 -^ 2 + • • • + ^2 n ^n »
dt
( 1)

dXn , , ,
—; — + a^^nX^ + . . . annXjii
dt
em que os coeficientes au são constantes. Aqui t designa a variável
independente, Xi (t), jcz (í).......... (0 as funções desconhecidas. O sis­
tema (1) chama-se sistema de equações diferenciais homogéneas de
coeficientes constantes.
EQUAÇÕES D IFEREN CIA IS 119

Como indicamos no parágrafo anterior, este sistema pode ser


resolvido reduzindo-o a uma equação do grau n, ordem que no caso
presente será linear (já o haviamos notado na nota 1 do parágrafo
precedente). Ora o sistema (1) pode igualmente ser resolvido por um
outro método, sem o reduzir a uma equação de ordem n. Este método
permite analisar mais concretamente o carácter das soluções.
Procuraremos a solução particular do sistema sob a forma seguinte:
= X2 = ol2 6 ' , Xn = ar,e^\ (2)
Deve-se determinar as constantes a i, ag, .... e â: de modo
que as funções aie'", a jc " ', .... anC*' verifiquem o sistema de equa-
ções (1). Substituindo-as no sistema (1). obtemos:

kai6^^ = (aiiai + flizaz + . + «man)


= (^21^1 + 022^2 + + <h.n^n)

Dividamos por Passando para o primeiro membro todos


os termos e pondo em evidência os coeficientes de «i, a j,
obtemos o sistema de equações

(ajl — /c) «1 + 012^2 + • • • + = 0»


+ (®22 — A:) a2 + ... + ^2n^n = 0.
(3)

^n l^l + (^n2^2 + • • • H“ (^nn — = 0-


Escolhamos a i, « 2»•••• ^ ^ maneira que seja verificado o
sistema (3). Este sistema é um sistema linear de equações algébricas
em relação a a i , a g ,..., a^.Formemos o determinante do sistema (3):
aii — k «12 • ^l7l
«21 «22 — k . , «2n
A(A) = (4)

^nl an2 • (flnn — k)

Se A: é tal que o determinante A é diferente de zero, o sistema (3)


não possiri «enão uma solução nula «i = «2 = ... = a^i = 0, e, por
conseguinte, as\ fórmulas (2) apenas dão as soluções triviais:
120 CALCULO D IFEREN CIA L E INTEGRAL

Assim, apenas poderemos obter soluções não triviais (2) para os


valores de k para os quais o determinante (4) se anula.
Obtemos uma equação do grau n para determinar k:
lll— fc ãi2 CL^n
^21 U>22 k ,. ^2n = 0. (5)

^nl ^ m2 ••• dnn— k


Esta equação é chamada equação característica do sistema (1).
As suas raízes chamam-se raízes da equação característica.
Consideremos alguns casos.
I — As raízes de equação característica são reais e distintas.
Designemos por ku kz, kn, as raízes da equação característica.
Para cada raiz /c; escrevamos o sistema (3) e determinemos os coeficientes

a ii) (i)
1 í ^2 » aU)
Pode-se mostrar que um dentre eles é arbitrário. Pode-se estimá-lo
igual à unidade. Assim, obtemos:
para a raiz ki a solução do sistema (1)
x„ = a , e

para a raiz a solução do sistema (1)


x.<r2) _= ar e xf =

para a raiz k^ a solução do sistema (1)


^(n) A,,/ (n) hnt
-- LCj c , 0^2 -- ^2 ^ » • • •» ^
Pode-se verificar, por substituição directa nas equações, que o
sistema de funções

+ C^afe"'-^ + . .
( 6)

Xj^ = + C2^n^'^^ + • ■
em que Ci, Cz.......Cn são constantes arbitrárias, é a solução do sistema
de equações diferenciais (1). É a solução geral do sistema (1). Mostra-se
fàcilmente que se pode encontrar para as constantes valores tais que a
solução verifica as condições iniciais dadas.
EQUAÇÕES D IFEREN CIA IS 121

Exemplo — 1. Achar a solução geral do sistema de equações

- ^ = 2x1+ 2x2, - ^ = xi + 3x2.

Resolução — Formemos a equação característica


12 —A: 2
= 0
1 3 — A:
cm que —5A: + 4 = 0. Encontramos as raízes
*1 = 1» *2 = ^-
Voltemos a procurar as soluções sob a forma

Componhamos o sistema (3) para a faíz /ci = l e determinemos c

(2 -l)aií> + 2a^i> = 0, \
laiD + (3-l)aS5i> = 0 /
ou
aii> + 2a^i) = 0,
aii> + 2a^i) = 0,
1 1
donde ^ Façamos = 1. Obtemos = — g -. Obtivemos, assim,
a solução do sistema
xil> = e^ X^l>=— y e'.

Componhamos, em seguida, o sistema (3) para a raiz *2 = 4 e deter­


minemos e :
—2aí2) + 2a^2)^0,
a<2)-2a^*> = 0,
donde = e = a^^^ = l. Obtemos a segunda solução do sistema:

A solução geral do sistema será [ver (6)]

X 2 = — +

II — As raízes da equação característica são distintas, mas alguma


delas são complexas.
Suponhamos que entre as raízes da equação característica existem
duas raízes complexas conjugadas:
fel = a + ip , fe« = a —- if
122 CALCULO D IFEREN CIA L E INTEGRAL

A estas raízes corresponderão as soluções


xf = (y = 1, 2, .. ., n), (7)
x f = a ‘/ V “ " (;• = 1, 2, . . ., n). (8)
Os coeficientes aj^ e são determinados a partir do sistema
de equações (3).
Do mesmo modo que no § 21 (t. II. cap. III) pode-se mostrar
que as partes reais e imaginárias da solução são também soluções.
Obtemos, assim, duas soluções particulares:
{ 7Í f COS sên p z )
sen pz - f cos pz) (9)

em que A."’, são números reais definidos por meio


de «ji** e afK
As combinações correspondentes das funções (9) entrarão na
solução geral do sistema.
Exemplo — 2. Encontrar a solução geral do sistema
dxi
"dT = — +
dx2 = —2xi —5x2.
dt

Resoluçõo — Formemos a equação característica


‘ _0
_2 —5—A:
A2 + 12A: + 37 = 0 c calculemos as _|uas^juízes;
^2——^ —í*
Fazendo k ^ = —6 + i no sistema (3). achamos:

Escrevamos a solução (7): '


= l^(-6+Í)f, == (1 + i) (7)
Fazendo ^ 2 = —^ ^ sistema (3), achamos:
a^2^^= í — i.
Obtemos o segundo sistema de soluções (8):
(8')
Voltemos a escrever a solução (73:
= (cos í + i sen í), ^-6/ (cos í + i sent)
OU
x ju = cos t + ie~^^ sen í,
= ( c o s í — sen (cos í + sen t ) .
EQUAÇÕES D IFEREN CIA IS 123

Voltemos a escrever a solução (8') ‘.


COS t —ie"® * sen í ,
x^2) _ g - 6 í (cos í — sení) — (cos í + sen t).
Podemos escolher como sistema de soluções parciais separadamente as
partes reais e imaginárias
= e“®^ COS í, = e"®^ (cos t — sen í),
(9')
sen f, = e"®* (cos f + sén t).
A solução geral do sistema será:
xj = cos t + ^2^“®^ sen t,
x2 = Cic"®^ (cos í —sen + (cos í + sen r).

Pode-se encontrar, por um método análogo, a solução dum sis­


tema de equações diferenciais lineares de ordens superiores de coefi­
cientes constantes.
Em mecânica e teoria de circuitos eléctricos estuda-se. por exemplo,
a solução do sistema de equações diferenciais de segunda ordem

■aiiX-\-ãizy.
dt^
<fy (10)
■Ü2\X -|“ ^222/*
de
Procuremos de novo a solução sob a forma

x = ae kt y = ^e^\

Substituindo estas expressões no sistema (10) e dividindo por


obtemos um sistema de equações para determinar a, p t k:

(«11 — k^)a-\- «12P = Q, y


(1 1 )
«21^^ -f" (^22 — P = 0* /
Os valores a t p não serão diferentes de zero senão no caso
em que o deteribinante do sistema for igual a zero:

«11 — k «12
:0. ( 12 )
«21 ^22 — ^
É pre<|isamente a equação característica para o sistema (10); é
uma equação de quarta ordem em relação a k. Sejam kx, k 2 , k^ e k 4
as suas raízes (supondo-as distintas).
124 CALCULO D IFEREN CIA L E INTEGRAL

Por cada raiz ki do sistema (11) encontramos os valores a e p.


A solução geral análoga a (6) será da forma

y= + C3p<=*^e*>‘ + C iP ^ V * '.
Se certas raízes forem complexas, a cada par de raízes complexas
corresponderá, na solução geral, uma expressão do tipo (9).
Exemplo — Encontrar a solução geral do sistema de equações diferenciais
d^x
dt^ = x — Ay,
d^y
dt^ -a:+y.
Resolução — Escrevamos a equação característica (12) e achemos as suas
raízes:
1_A:2 _4
= 0,
—1 1—A;2
k^ = i, k 2 = —í, A;3= 1/3, ki^=—l/S .
Voltemos a procurar a solução sob a forma
i<i) = a<i>eí',
l(2) = (x(2)e-*', y(2) = p(2)e^^\
i(3)=^a(3)e ^3t ^ yi3) = p(3)e^^\
x(4) = a í í ) í - ’^^', y(4) = p(4)g-
11) tiramos a^íi e pu> :
a<i> = l,

a<2) = l,

a<3) z= 1, P<3> = - 1 .

a<4) = l, = .
Escrevamos as soluções complexas:
a;íi> = e~^^ = cos / + í sen ^ = y (cos í + í sen/),
1
X(2) ; - i t = COS t — i^ í, í/<2)= -_ (cos t — i stn t).
As partes reais e imaginárias tomadas separadamente serão também
soluções: __ _ ^
j;(l) - : cos /, ^(1) — COS / ,

1
x(2) —sen r, sén t.
EQUAÇÕES D IFEREN CIA IS 125

Podemos, agora, escrever a solução geral:


X= C^ COS t -j- C2 sen t -f- V 3í - j- (J

y = ^i~2 ^ ^ V lt.

N o t a — Não consideramos neste parágrafo o caso das raízes múl­


tiplas da equação característica.

§ 31. Noção sobre a teoria da estabilidade de Liapounov


Como as soluções da maior parte das equações diferenciais e dos
sistemas de equações elementares não se exprimem por meio de funções
elementares ou por quadraturas, recorre-se igualmente a métodos de
integração aproximada. Deu-se uma ideia destes métodos no § 3 (t. II,
cap. X lll); além disso, vários destes métodos serão examinados nos
§§ 32-34 e também no capítulo XVI.
O defeito destes métodos, é que eles apenas dão uma solução
particular; para obter outras soluções particulares é preciso refazer
todos os cálculos. Conhecendo uma solução particular não se pode
pronunciar sob o carácter das outras soluções.
Em muitos problemas de mecânica e de técnica, importa conhecer
não os valores concretos da solução correspondente a valores concretos
da variável, mas o comportamento da solução quando a variável varia,
em especial quando tende para infinito.
É, por exemplo, importante saber se as soluções que satisfazem
às condições iniciais dadas são periódicas ou se elas tendem assintòtica-
camente para uma função conhecida, etc. A teoria qualitativa das
equações diferenciais tem por objecto estas questões.
A questão da estabilidade duma solução ou dum movimento é
uma das questões fundamentais da teoria qualitativa; esta questão foi
estudada em detalhe pelo eminente matemático russo A. Liapounov
(1857-1918).
Seja o sistema de equações diferenciais

É ^ = fi{t, X, y ) ,
di
(1)
^ = X, y ) .
dt
Sejam x = x { t ) e y = yU) as soluções deste sistema que satis­
fazem às condições iniciais
^t=0 = ^0> 1
yt= o= yo- > ’
126 CALCULO D IFEREN CIA L E INTEGRAL

Sejam ainda a: = x {t)Qy = y {t)as soluções deste sistema (1) que


satisfazem às condições iniciais

^t= 0-- ^0i 1


(11)
yt=o = yo- )
Definição — As soluções ar = aí (/) e y = y{t) que satisfazem às
equações (1) e às condições iniciais (!') dizem-se estáveis no sentido de

Liaponnov quando / oo se, para todo 0 > 0 arbitràriamente pequeno,


existe 8 > 0 tal que se tenha para todo / > 0 as desigualdades

|x(<) — x ( í ) | < e ,
( 2>

desde que as condições iniciais satisfaçam às desigualdades

I ío — ^ 0 1 < ô .
(3>
I ^0 — í/o I < ô .
Interpretemos esta definição. Resulta das desigualdades (2) e (3)
que as soluções variam pouco, qualquer que seja t positivo, quando
as condições iniciais variam pouco. Se o sistema de equações diferen­
ciais é o dum movimento, o carácter do movimento varia pouco
quando as condições iniciais variam pouco se as soluções forem estáveis.
Vejamo-lo com o exemplo duma equação de primeira ordem.
Seja a equação diferencial

dy
-Í/ + 1 . (a)
dt
A sua solução geral é
(b)
EQUAÇÕES D IFEREN CIA IS 127

Achemos a solução particular que satisfaça à condição inicial


»(=0 = ^- (c)
É evidente que esta solução y — i corresponde a C = 0 (fig. 274). Ache­
mos em seguida a solução particular que sattsfaça à condição inicial
yt=o = yo-
Achemos o valor de C na equação (b):

yo=c+u
donde
C=yo~í.
Substituindo este valor de C na igualdade (b), obtém-se

P = (yo— 1) e"' + l .
É evidente que a solução >^ = 1 é estável. Com efeito
y— y= [(íío— 1) «■' + !] — l = (ío —l ) « “ ' —^ 0
quando t —> oo.
A desigualdade (3) é, pois, verificada qualquer que seja c desde que
se tenha
(í/o—l) = ô < e.

Consideremos, em seguida, o sistema de equações:

dx
= cx-\- gy,
dt
(4)
- ^ = ax + by,
dt
supondo que os coeficientes a, b, c. g são constantes e gz^O ,
Vejamos a que condições devem satisfazer os coeficientes para
que a solução jc = 0, >^ = 0 do sistema (4) seja estável.
Derivemos a primeira equação e eliminemos y. Obtém-se uma
equação de segunda ordem:
(Px dx du dx . , .
^ ^ c — + g ^ = ^ c — + g (a x+ b y )= =
dr dt dt dt

dx
= c -------1- agx + b
dt
ou
dí^x , . dx
+ ---- (ag — bc) x = 0. (5)
dt
128 CALCULO D IFEREN CIA L E INTEGRAL

A equação característica escreve-se


- ( b + c)K - { a ^ - b c ) = 0. ( 6)
Designemos as raízes da equação característica por Ai e A2. Os
casos seguintes se podem apresentar:
1. As raízes da equação característica são reais, negativas e
distintas:
<C 0, X2 <C 0, X2 .
Então,

y = [ C l (^ 1 - c) + C 2 (X 2 - c) ^ .

A solução que verifica as condições iniciais


x|i= o = ^o, y |í= o = í/o

c x ^ -\-g y ^ — X it I ^0^1 — ^^0 — V o g j . 2 t


X ---- 6 -1 6 ,
A-i —A2 Âi —
1
y= — (7)
g 'K
^0^1 — ^*^0 — í/o^
{K - c)
^
Resulta destas últimas fórmulas que, para todo 8 > 0, se pode
escolher e yo suficientemente pequenos tais que se tenha para todos
os r > 0:
I I < I !/ (0 I < dado que
< 1 ^e < 1.
Resulta daí que neste caso a solução jc = 0, y = 0 é estável
2. Sejam Xi = 0, A2 < 0. Tem-se
x = C^-\-

y = j[ C 2 Í X . - c ) e ’'^* -c C i]
e, como anteriormente, a solução é estável
3. Seja Al = A2 < 0. Tem-se
x= {C ^ -\- C 2O 'i

í/ = — ^ ^ [ C l ( À i — c) - I - C 2 (1 4 "
EQUAÇÕES D IFEREN CIA IS 129

Dado que
te^it 0 e »0 quando í —> oo,
ter-se-á para Ci e Co. suficientemente pequenos (isto é, quando JCo e yo
sejam suficientemente pequenos) | x (í) | < e e | i/ (0 I < ® qualquer
que seja í > 0. A solução é estável.
4. Seja Al = zA. = 0. Tem-se

a: = Cl +
1
y =■ — [— cC\ H" C 2 — CC 2Í].
g
Vê-se que por mais pequeno que seja Cj 7^ 0. jr e y tendem para
o infinito quando t-> co, isto é. que a solução é instável.
5. Suponhamos que uma das raízes Ai c Ao é positiva, por
exemplo, Ai > 0.
Resulta da fórmula (7) que por mais pequenos que sejam jc e >^, se
+ gyo — ^0^2 ¥= 0,
isto é. que se Ci =?í= 0, \ x (t) | 00, quando t - ^ 0 0 ,
Por conseguinte, a solução é instável neste caso também.
6. As raízes da equação característica são complexas e a parte
real é negativa:

A.1 — cc i'P, 1
. í « <0.
A,2 = a — íP, J
Neste caso
X= sen (Pí + ô),
1
y = j cé^ [(a - c) sen (pí + ô) + p cos (pí + 6)]

Ê evidente que para todo e > 0 se pode tomar Xo e yo de modo


1 ( 8)

que se tenha | C | < ; e e f ^^ I <; e e, portanto,


I“ 1
I X (í) I < 8 e I 1/ (0 | < 8.
A solução é estável.
7. As raízes da equação característica são números imaginá­
rios puros:
= p í, Xa = — p t.
130 CALCULO D IFEREN CIA L E INTEGRAL

Neste caso
x = C sen(pt + ô),
1
y = — C [P COS -j- ô) — c sen (pí -j- ô)],

isto é. que x ( t ) q y (t) são funções periódicas de t. Verifica-se. como


anteriormente, que a solução é estável.
8. As raízes da equação característica são complexos e a parte
real é positiva {a > 0).
Resulta das fórmulas (8) que por mais pequenos que sejam Xq e yo
(isto é, que para C 0 arbitrariamente pequenos) as quantidades
I jc (/) I e I .V(/) I podem tomar valores arbitràriamente grandes quanto t
cresce, dado que -> oo quando oo. A solução é instável.
Para dar um critério geral de estabilidade da solução do sis­
tema (4). procederemos como se segue.
Escrevamos as raízes da equação característica sob a forma
complexa
^1 = -|- ^^1 í
^2 = ^2 H” 1^ 2

(se as raízes forem reais. ÀJ* = 0 e À** = 0).


Representaremos as raízes da equação característica por pontos
no plano da variável complexa. Partindo dos oito casos examinados
acima, pode-se formular a condição de estabilidade da solução do
sistema (4) como se segue.
Se alguma das duas raízes \ i e X2 da equação característica (6)
não se encontrar à direita do eixo imaginário e se uma raiz pelo menos
for diferente de zero, a solução é estável; se houver uma raiz à direita
do eixo imaginário ou se as duas raízes forem nulas, a solução é
instável.
Consideremos agora o sistema de equações mais geral:

dx
— = cx + g y -[-P {x , y),
dt
(4')
- ^ = ^ a x - \ - b y -\-Q {x, y).
dt
Salvo casos excepcionais, a solução dum tal sistema não se exprime
por meio de funções elementares e quadraturas.
Para estabelecer a estabilidade da solução deste sistema, compara-se
às soluções dum sistema linear. Suponhamos que quando jc -> 0 e y 0
EQUAÇÕES D IFEREN CIA IS 131

as funções P ( jc , y ) c Q ( ac, y) tendem igualmente para zero mais


depressa do que p = ; noutros termos.

p-^0 P p-^0 p

Demonstra-se, então, que além dum caso excepcional, a solução


do sistema (4') é estável ou instável ao mesmo tempo que o do sistema

dx
= cx-\- gy,
dt
(4)
du . .
— = ax by,
dt

Exceptua-se o caso em que as duas raízes da equação caracterís­


tica se encontram sobre o eixo imaginário; então, é mais difícil de
decidir da estabilidade ou da instabilidade da solução do sistema (4').
A. Liapounov estudou a questão da estabilidade das soluções de
sistemas de equações sob hipóteses bastante gerais.

§ 32. Solução aproximada das equações diferenciais


de primeira ordem peio método de Euler
Consideraremos dois métodos de resolução numérica duma equa­
ção diferencial de primeira ordem. Neste parágrafo abordaremos o
método de Euler,
Achemos a solüção aproximada da equação

|= /( » , y) ( 1)

sobre o segmento [xo, 6], que verifica a condição inicial y = yo para


JC = jCo. Decomponhamos o segmento [aco, b ] com a ajuda dos pontos
ACü. ACi. ACo....... Xr^ = b H partes iguais (aqui aco < aci < X2 < ... < x ^ ) -
Introduzamos a notação aci — Xo = aco — aci = ... = fe— = Ax = /»>
por conseguinte,
, b - X o

Seja y = 9 ( x) uma certa solução aproximada da equação (1) e


U o^fpi^o), yi = (p{^i), y„ = 9(x„).
Introduzamos as notações
A l/o í/o . ^ y i — Ví — í / l .......... A y „ _ i = í/n
132 CALCULO D IFEREN CIA L E INTEGRAL

Em cada um dos pontos jco, jci........ da equação (1) substi-


tuámos a derivada pela relação das diferenças finitas:

í), ( 2)
l\x
^ y = f{x , y )S x . (2')

Para x = x», teremos


Ayo = f{xo, yo),
—^ Ayo = /(^ o , !/ o) A x
Ax
OU
Uí — yo — / (^0» í/o)
Nesta igualdade jco. yo. h são conhecidos, por conseguinte, encon­
tramos:
yi = yo + f{xo, yo)h
Para x = JCi a equação (2') será da forma
Ayi = /(x j, 1/,)A
ou
y 2 — yi = f(X i, yt)h, y 2 = yi + f{Xi, yt)h.

Ti, yi, h são aqui conhecidos, por conseguinte, determina-se y^.


Do mesmo modo achamos:
í/3 = í/2 + / (^2» í/2)

yk+i = yk + f{^k. yh)h,

yn — yn—l “í” / {^n—íi í//»—l)


Encontramos assim os valores aproxi-
Xo Xí Xz X3 X niaàos da solução nos pontos Xo, Xi,
Fig 275 Reunindo sobre o plano das coordenadas os
pontos (xo, yo). (xi, yO ........... por
segmentos de recta obtemos uma linha quebrada que é a representação
aproximada da curva integral (fig. 275). Esta linha chama-se linha
quebrada de Euler,
Nota — Designemos por ^ = (p/^ (o:) a solução aproximada da
equação (1) correspondente à linha quebrada de Euler para- Ax = h.
Pode-se demonstrar que se existir uma solução única y = <p* (x) da
EQUAÇÕES D IFEREN CIA IS 133

equação (1) que satisfaz às condições iniciais e determinada sobre o


segmento [xo, b] então lim | (p/^ (x) — (p* (x) | = 0 para todo o x do
/i->0
intervalo [xo, b].
Exemplo — Achar o valor aproximado para x = 1 da soluçfto da equação

y' = y + «
que verifica a condição inicial: >^o ^ 1 para Xo = 0.
Resolução — Dividamos o segmento [0, 1] em 10 partes iguais com a
ajuda dos pontos Xo = 0; 0,1; 0,2; 1,0. Por conseguinte, h = 0,1. Procuraremos
os valores y^y •••yVn com o auxílio das formulas (2 ')
byh = {yh+^h)b
ou
yh+i = y h + (v k + x k )h .
Obtemos assim: yi = l.+ (l + 0)*0,1 = 1 + 0,1 = 1,1,
!/2= 1,1 + (14 + 0, 1) . 0,1 = 1, 22,

No decurso da resolução formamos o quadro:

yh y i& ^ h

xo = 0 1 ,000 1,000 0,100


xi = 0 , l 1,100 1,200 0,120
X2 — 0 , 2 1 ,220 1 ,4 2 0 0 ,1 4 2
X3 - 0 , 3 1,362 1,620 0,162
^4 = 0 , 4 1,524 1,924 0 ,1 9 2 4
— 0 ,0 1,7164 2,2 1 6 4 0 ,2 2 1 6
X 6 --0 ,6 1,9380 2,5380 0 ,2 5 3 8
X7 = 0 , 7 2,1 9 1 8 2 ,8 9 1 8 0,2 8 9 2
Xg = 0 , 8 2,4810 3,2810 0,3281
X9 - 0 , 9 2,8 0 9 1 3,7091 0 ,3709
xi o - 1,0 3 ,1 8 0 0

Encontramos o valor aproximado y |x=i = 3,1800. A solução exacta da


equação dada, que verifica as condições iniciais citadas anteriormente, será

y = 2e ^ — X — 1.
Por conseguinte.
y |3ç_ 4 = 2 (e — 1) = 3 ,4 3 6 6 .

O erro absoluto é igual a 0,2566, o erro relativo a


0 ,2 5 6 6
= 0 ,0 7 5 ^ 8 % .
3,4366
134 CALCULO D IFEREN CIA L 1 INTEGRAL

§ 33. Solução aproximada das equaçSes diferenciais


pelo método dos diferenciais finitos baseados na aplicação
da fórmula de Taylor. Método de Adams
Voltemos de novo a procurar a solução da equação
y ' = f { x , y) (1 )

sobre o segmento [;co. b] que verifica a condição inicial: para x = Xq,


y = >0. Introduzamos as notações que nos servirão para a exposição.
Os valores aproximados da solução nos pontos
••m
serão
Í/01 yu í/2i • . -1 í/n-
As primeiras diferenças ou as diferenças de primeira ordem serão:
^ ^ 0 = ^1 — Í/01 ^ í / l = í/2 — í/l» • • • 1 ^Vn-i — Vn Vn-i-
As segundas diferenças ou diferenças de segunda ordem são:
^ y o = ^ y \ — ^yo = y 2 — 2^1 -|-
= Ay 2 — Ayi = ^3 — 2^2 + yu

^^Vn-2 = ày„-i — A y„-o = y„ — + ^„_2.


As diferenças das diferenças de segunda ordem chamam-se dife­
renças de terceira ordem. etc. Designemos por yl, y[^ .... y,; os valores
aproximados das derivadas e por yl, y[, .... os valores aproximados
das derivadas de segunda ordem. etc. Duma maneira análoga se deter­
minam as primeiras diferenças das derivadas
Al/Ó = .ví — Í/Ó, A i/; = 1/2 — VÍ . • • •, ^y'n-i = yn — y'n-i,
e as segundas diferenças das derivadas
= ^y'^ — Ai/i, AV i = Ai/á — ^ y \, . . .
• . •• A V .,-2 = A v;_i — Al/;,-2, etc.
Escrevamos, em seguida, a fórmula de Taylox para a solução da
equação na vizinhança do ponto x = Xn (t. I, cap. IV, § 6, fórmula (6)):
, X — Xq , {x Xo) —
1/ i/o -j------ ^ I/o "
Vo+

...
I
+
1 ' 1-2
(x — Xo)
+ yo+ R n ( 2)
1 -2 - •m
EQUAÇÕES D IFEREN CIA IS 135

Nesta fórmula yo é conhecido e achamos os valores das deri­


vadas a partir da equação (1) da maneira seguinte. Substi­
tuindo no segundo membro da equação (1) os valores iniciais JCo» yo
encontramos
y a = f(x o , yo)-
Derivando os termos da equação (1) em relação a x, obteremos:
.. df , df .
y = — + — í/- (3)
dx dy
Substituindo no segundo membro os valores xq, yo, y'o, encon­
tramos:
.. í df ^ df \
í/o = l -T— )
\ OX oy / x = xq, í/= y o . V'=V q-

Derivando ainda uma vez a igualdade (3) em relação a jc e


substituindo os valores Xq, yo, y^, y l encontramos y'^". Prosseguindo (*(*)
assim podemos determinar os valores das derivadas em qualquer ordem
para x = Xo. Todos os termos do segundo membro da fórmula (2).
exceptuando o termo resto Jijn são conhecidos. Assim, despre?ando
o termo resto, podemos obter os valores aproximados para qualquer
valor de jc ; o seu grau de exactidão dependerá da grandeza | jc — jCo |
e do número de termos do desenvolvimento.
No método considerado mais abaixo não se determina com a
ajuda da fórmula (2) senão alguns primeiros valores de y quando ! jc — jcq |
é pequeno. Determinaremos os valores y^ e yz por jCi = jCo + A e
Xo = Xo + 2/i, tomando quatro termos do desenvolvimento (yo é conhe­
cido dos dados iniciais):

h r ,,,
= ^0 + — í/o + —-r í/o + — ^0 , (4)
1 1-2 3!

, 2A , , (2 h f , {2 h f
^2 = yo + 7 -^ 0 + í/o + - ^ í/o (4')
1 1*2 3!
Supomos, assim, conhecidos três valores (*♦) da função yo, yi. y2.
Na base destes valores, determinemos, utilizando a equação (l):
yo = f{xo, yo ). y't = f{xi, y i) , yz = í{xz, yz).
( *) Suporemos, no que se segue, que a função f { x , y) é dcrivável em
z t y tantas vezes quantas o exijam os raciocínios.
(**) Para um^çi solução dum grau de exactidão mais elevado deveríamos
calcular mais que os três primeiros valores de y.
136 CALCULO D IFEREN CIA L E INTEGRAL

Conhecendo y'» y'v U^podcmos determinar Ay'. Ayí. Agru­


pemos os resultados dos cálculos no quadro seguinte:

y y' Ay' A2y'

^0 yo yí

A{/i

Xj — Xq-j~ ^ Ul yí A«yí

Ay;

X2= xq-\-2h V2 ví

... ... ... ... ...

y h -2 y 'k -2

^y'k—2

;to-}-(A: — 1) h U k -i y 'k -i a - í/;,_2

^ y 'h -i

Xk=^XQ-]-kh Vk y'k

Suponhamos, agora, que conhecemos os valores da solução

í/o. yu y 2 , . . . . yh-

Baseando-nos nestes valores, podemos calcular utilizando a equa­


ção (1), os valores das derivadas

í/o» yi. ^2» yÁ,


EQUAÇÕES D IFEREN CIA IS 137

e. por conseguinte.
Ayó, Ayí.........

® AVo', AV í , . . . . A^y;_2.

Determinemos o valor de j/a+ i , segundo a fórmula de Taylor,


fazendo a — a;*, x = x^+i = x* + fe:
7^3 Lm
Vh+i = — í/fe + T-Z Vh + ...
1 1 *2 1 .2 .3 m!

Limitemo-nos, neste caso, a quatro termos do desenvolvimento:


h?
^/l+l = í/fe + — í/fe + T—Z í/fe “t 4 0 0 Vh- (5)
1 1 *2 1 * 2 -o
Nesta fórmula as incógnitas são e que procuramos
determinar com a ajuda das diferenças conhecidas de primeira e
segunda ordens.
Exprimamos, em primeiro lugar, com o auxílio da fórmula de
Taylor. fazendo a = x — a = ^h:

, , i-h ) „ A - h f
Vh-i = y* H— z— Vk + ' ■• • - Vh. ( 6)
1 1 -2
ey'h- 2 >fazendo a = X/,, x — a — — 2 h :
{ - 2 h)r „ I, { - 2 h f
yh- 2 = Vh Uh + yh> (7)
1.2
Da igualdade (6) tiramos
/ f jk r %
1 h ff,
Vh — y h - i = ^ y k - i (8 )
1 1-2
Subtraindo os termos da igualdade (6) pelos da igualdade (7),
obtemos: \
y ^ -, - = S y \_2 = A y l - ^ y -
(9)

De (8) e (9) tiramos

Ayft-i — Aj/ft_2= AVÁ-2 = feV r


ou
4
i í/ft = - ^ A (10 )
138 CALCULO D IFEREN CIA L E INTEGRAL

Substituindo o valor de V k’ na igualdade (8), obtemos:


^^y'k- 2
( 11)
h 2h

Achamos, assim, e Substituindo as expressões (10) e (11)


no desenvolvimento (5). obtemos:

, h r , h . , . .0 '
Uk+l — + z9" A/Va-I “1“ J12z ^ y h - 2 - ( 12)
1
É a chamada fórmula de Adams para quatro termos. A fór­
mula (12) permite, conhecendo y^, yk- 2.^ determinar y^ +i- Assim,
conhecendo j,,. >’i e y>> podemos encontrar y\i e. em seguida. >^4» l/s, •••
N ota— 1. Indiquemos, sem o demonstrar, que se existir uma
solução única da equação (1) sobre o segmento [x,. h] que verifica as
condições iniciais, então, o erro dos valores aproximados determinados
pela fórmula (12) não excede em valor absoluto M/z‘ em que M é
uma constante que não depende do comprimento do intervalo e da
forma da função / (x, y) e independente da grandeza h.
Nota — 2. Se quisermos reduzir a margem do erro. convém tomar
um maior número de termos no desenvolvimento (5) e modificar, por
conseguinte, a fórmula (12). Assim, se em vez da fórmula (5) tomar­
mos uma fórmula cujo segundo membro contenha cinco termos, isto é,
se acrescentarmos um termo de ordem h \ obteremos, duma maneira
análoga, em vez da fórmula (12) a fórmula

Ufi + i — + ^Í/A + y ^ y h -l “I” Í2 ^ ^ ^ yk-3 ‘

Aqui yh+i é determinado a partir dos valores í//t, yu-i^ ^A-aC y k - 3 -


Assim, antes de abordar os cálculos segundo esta fórmula, é preciso
conhecer os quatro primeiros valores da solução: >0, y^, y.v
No decorrer do cálculo destes valores com a ajuda das fórmulas
do tipo (4) convém tomar cinco termos no desenvolvimento.

Exemplo — 1. Achar os valores aproximados da solução da equação

y ' \ X

que verifica a condição yo = 1 para Xo = 0.


Determinar os valores da solução para x = 0,l; 0,2; 0,3; 0,4.
Resolução — Em primeiro lugar determinemos yu >2 com a ajuda das
fórmulas (4) e (4'). Obtemos da equação e das condições iniciais

yl)--- iiQ ; 0^-1 f-0r=i.


EQUAÇÕES D IF E R E N a A IS 139

Derivando esta equação, obtemos


í/'' = í/' + l.
Por conseguinte.
y ;= (y ' + i U o = i + i = 2 -
Derivemos ainda uma vez
y = y ■
Por conseguinte,
yó'' = yõ=2.
Substituindo na igualdade (4) os valores yot !/ó» !/õ e /i = 0,1, obtemos:

Duma maneira análoga encontraremos para h = 0,2:


y 2 = l + - ^ - l + - ^ - 2 + l ^ ' 2 = l,2427.
Conhecendo yo, yi, >^2 obteremos a partir da equação:
yó = yo+^o = l I
y\ — —1,1103-|-0,1 = 1,2103 \
y2~y2~\~^2~ 1 , 2 4 2 7 0 , 2 = 1,4427 \
Ayi = 0,2103;
Ay; = 0,2324;
A2yí = 0,0221.
Com os valores obtidos, teremos o quadro seguinte:

.V V y' Ay'

xo = 0 yo= 1,0000 » i= i

Ayi = 0,2103

xi = 0,l yi = 1,1103 yi = 1,2103 A2í/í = 0,0221

Ay; = 0,2324

x2= 0.2 j/2= Í>2.427 y; = 1,4427 A2j/; = 0,0244

Aj/i = 0,2568

= 0,3 í/3= 1,3095 y i= 1,6995

J4= 0,4 1/4= 4,5833


140 CALCULO D IFEREN CIA L E INTEGRAL

Tiramos da fórmula (12):

í/3 = 1,2427 + - ^ • 1,4427 + - M . 0,2324 + - ^ 4 ^ • 0,0221 = 1,3995.


1 Z IZ
Em seguida, encontramos os valores ^^y[- Depois, com o auxílio
da mesma fórmula (12) encontramos >^4:

j/4 = 1,3995+^ . 1,6995 + M . 0,2568 + - ^ - 0 , 1 -0,0244 = 1,5833.

A expressão da solução desta equação é:


y= 2e^ — x — í.

Por conseguinte, 4= —0,4— 1 = 1,58364.0 erro absoluto é 0,0003;


0 0003
o erro relativo: —:------- 0,0002 = 0,02 %. (O erro absoluto do valor y4 cal-
1,583Ô4
culado pelo método de Euler é de 0,06: o erro relativo: 0,038 3,8 %).
Exemplo — 2. Achar os valores aproximados da solução da equação
y ' = y ^ + x ^ j

que verifica a condição inicial:. yo = 0 para Xo = 0.


Determinemos os valores da solução para a: = 0,1; 0,2; 0,3; 0,4.
Resolução — Achamos:

yJ = 02 + 02 = 0,
{^^=0=i^yy'+ 2^)*=o=0*
yí'=0 = (2í/'* + 2yí/‘'+ 2 U o = 2 -
Das fórmulas (4) e (4'), obtemos:

= 2 = 0,0003, = 2 = 0,0027.
Da equação tiramos!
yó = 0» yí = 0,0100, í^2= 0,0400.

Com o auxílio destes valores compomos as primeiras linhas do quadro,


depois, determinamos • os valores de e 3^4 segundo a fórmula (12).
Assim,

ys = 0,0027 + . 0,0400 + • 0,0300 + - ^ • 0,1-0,0200 = 0,0090,

í/4 = 0,0090 + • 0,0901 + - ^ - 0.0501 + - ^ - 0,1-0,0201 =0,0214.

Notemos que para ^4 os quatro primeiros números exactos depois da


vírgula são: >^4 = 0,0213 (pode-se obtê-los por outrOs métodos mais precisos
que permitem avaliar o erro).
EQUAÇÕES D IFEREN CIA IS 141

Ay' A2y'

Xq= 0 í/o= 0 í/i = 0

Ay; = 0,0100

xi = 0,l 1/1 = 0,0003 í/| = 0,0100 A2i^i = 0,0200

Ay; = 0,0300

3^2= 0,2 y2 —0,0027 y2 = 0,0400 A2y;:z^ 0,0201

^!/2 = 0,0501

^3 = 0,3 y3 = 0,0090 y ' = 0,0901

í *4= 0,4 y 4 = 0,0214

§ 34. Método aproximado de integração dos sistem as


de equações diferenciais de primeira ordem

Os métodos de integração aproximados das equações diferenciais


considerados nos §§ 32 e 33 podem ser aplicados, igualmente, para
a resolução dos sistemas de equações diferenciais de primeira ordem.
Consideremos, agora, o método das diferenças para a resolução de
sistemas de equações. Conduziremos os raciocínios para um sistema
de duas equações que comporta duas funções desconhecidas. Pede-se
para procurar as soltí^es do sistema de equações

^ = / . ( x , y, z), (1)

■ ^ = Í 2 Í^, y, z), ( 2)
dx
que verifica as condições iniciais: para x = x., > = >’o , z = Zo.
Determinaremos os valores das funções y t z para os valores da
variável independente Xq, x \, x ^, . .
142 CALCULO D IFEREN CIA L E INTEGRAL

Seja de novo
—h {h — 0, 1, 2y • • . t ^ 1). (3)
Os valores aproximados da função serão notados por

í/o» y • • •» Í/A» Í/A+ 1> • • •»


e, respectivamente.
Zq» Z/t, 2^+ 1í • • •» Z,i.
Escrevamos as fórmulas de recorrência do tipo (12) do § 33:
. h , }i . , , 3 , .2 '
Vh + i — ^A + -j“ í/A + -^ ^ V h -i + Í/a 2i (4)

h , h . , i * ^ i a 2'
^A+ l = 2a + ^ — AZa-1 + hS. Z/e-2. (5)

Para abordar os cálculos segundo estas fórmulas torna-se neces­


sário conhecer, além dos 3^0 e Zo dados, yi, y 2 \ Zi. Z2 * encontraremos
estes valores pelas fórmulas do tipo (4) e (4') do § 32:

í/l = !/o + Y í/o + — í/o + — //o .

^ 2 h , . { 2 k f . ^ (2 h f ...
V2 = uo ~ yo — ~~ yo H— Y” ’

, h . . hr .. ...
21 =— 2q + Y ^0 + ~ 2q + ~ 2q ,

, 2h ^ . (2h f (2k f ^...


22 = + “ 2q H------- --- 2 q H----- 7— ^0 •
1 2 3!
Para aplicar estas fórmulas é preciso conhecer?/', y'o"^ ^ó’
Zo"que vamos agora determinar. Tiremos, agora, das equações (1) e (2)
Ui) / l (*^0» !/()í 2 q),

"0 /2 (*^Ü’ yOy 2 q).

Derivando as equações (1) e (2) e substituindo os valores Xo. yo, Zo*


. encontramos:

//õ = (//").v-A„ = ( " 'V


T l + ÍÍÍ l „ ■ + ^ A ,
r/r (lif r>z /.\=Ao

r/./’ ay úz / -v=.vn
EQUAÇÕES D IFEREN CIA IS 143

Derivando, ainda uma vez, obtemos y '" e 2 ^", Conhecendo 3^1,


y 2 . Zi, Z2. tiramos das equações (1) e (2)
l/íy UZy ^1, Z2y Az/q, A^Í, A^^q, Az^, A z[, A^Zq,
O q u e n o s p e r m i t e c o m p o r a s p r i m e i r a s c in c o l in h a s do q u a d ro

X V V* A y' A2y' Z z' A z' A 2z'

^0 yo yi ZQ ^ 0'

^ y 'o A z ’o

yi yí A ^y'o Zi Z'l

Ayi A zí

Xo y2 y '2 A V i ^2 Z2

A yí A 22

•^3 y3 y ’z Z3 Z3

Das fórmulas (4) e (5) obtemos 3^3 e z» e das equações (1) e (2)
y's ® ^3- Calculando Ay' A^^', Az', A^z' encontramos, utilizando de
novo as fórmulas (4) e (5). >4 e >5 e assim sucessivamente.
Exemplo — Encontrar os valores aproximados das soluções do sistema
z' = y
para as condições inic)áis: y.» = 0, Zo = 1 para a- = 0.
Calcular os valores das soluções para jc = 0; 0,1; 0,2; 0,3; 0,4.
Resolução — Tiramos destas equações:

í/o = ^ac=o= 1»
^ó = í/x=o = 0.
Derivando estas equações teremos:
yo = {ynx=o=(z^)x=o = 0 y
2S= (z'')3:=0 = (y')x=0 = l,
í/Í:' = (í/'")x=0=(O*=0==1,
2i:'=(-")x=o = (y")x=o = o.
144 CALCULO D IFEREN CIA L E INTEGRAL

Aplicando as fórmulas do tipo (4) e (5), obtemos:

n , ü.l . , (O.ip n ■ -------.1=0,1002,


!/i=o+— 3!
( 0. 2)^ (0,2)3
j,2 = 0 + - ^ - l - 1-2
0+
3!
1 = 0,2013,

0,1 ( 0, 1)*
. 0 = 1,0050,
3i = 1 + ^ - 0 + V 1-2
t9— 1 3!

Dos dados obtidos, tiramos:


y{ = 1,0050, í/2 = 1,0200,
zí = 0,1002, 22= 0,2013,
= 0,0050, A2^ = 0,1002,
Ayí = 0,0150, A2Í = 0,1011,
A2yJ = 0,0100, A2::i = 0,0009

e compomos as cinco primeiras linhas do quadro:

X y V’ Ay' A 2 í/ '

Xq = 0 !/o = 0 í/i = l

A j/i = 0 ,0 0 5 0

ii= --- 0, l yi = 0,1002 í/i = 1 ,0050 A-í/,', = 0,0100

Ayí = 0 ,0 1 5 0

^2 = 0 . 2 ^2 = 0 ,2 0 1 3 1/2= 1,0200 A 2yJ = 0,0102

Aí/Ó = 0 ,0 2 5 2

13 = 0,3 í /3 = 0 ,3 0 4 5 í/i = 1 ,0 4 5 2

^4 = 0 ,4 !/4 = 0 ,4 1 0 7
EQUAÇÕES D IFEREN CIA IS 145

X z 2' Az' A3z'

Xq = 0 Zq = Í * i= o

A«i = 0,1002

= 0,1 zi = 1,0050 z ; = 0,1002 A% = 0,0009

A « ;= 0,1011

12 = 0,2 Z2= 1,0200 «2 = 0,2013 A2«i = 0,0021

Azí = 0,1032

X3 = 0,3 23 = 1,0452 «i = 0,3045

0:4 = 0,4 Z4= 1,0809

Com a ajuda das fórmulas (4) e (5) encontramos;

JÍ3 = 0,2013 + . 1,0200 + . 0,0150 + . 0,1 -0,0100 = 0,3045,

Z3 = 1 , 0 2 0 0 + - ^ • 0,2013 + - ^ - 0,1011 - f - ^ • 0,1-0,0009= 1,0452

e de maneira análoga:
= 0 , 3 0 4 5 - 1 , 0 4 5 2 + ^0,1 - 0,0252 + • 0,1-0,0102 = 0,4107,

Zi = l,0 4 5 2 + - ^ 0 ',3 0 4 5 + - 0,1032 -0,0021 = 1,0809.

Ê evidente que as soluções exactas do sistema dado de equações que


verificam ás condições iniciais serão:

!/ = | - («* —«“*). z= «■*)-


Eis porque os quatro primeiros algarismos exactos, após a vírgula, serão:

V4 = Y («® = 0,4107,

z*=4-(e®-^ + «"®-^) = 1,0811.

10
146 CALCULO D IFEREN CIA L E INTEGRAL

Nota — Como as equações de ordem superior e os sistemas de


equações de ordens superiores se reduzem em numerosos casos a um
sistema de equações de primeira ordem, o método exposto é igualmente
Aplicável à resolução destes problemas.

Exercícios

Mostrar que as funções abaixo dependentes de constantes arbitrárias


satisfazem às equações diferenciais em frente.
Funções Equações diferenciais

1. y = sen x — 1 + + í/ COS X = Y sen 2x.

2* y = Cx C— .

3. y^ = 2 Cx + C^,

4. y‘i = Cx^-
1+ C
d^y . 3 d^y
5. y = C\X-\--- ^ + ^3- =0.
dx3 X d x 2
d2y
6. j/ = (Ci+C2x)e'‘* + (^c—1)2 * dx2 -2fc-^
ax
+ fc2jí = e*.
dy
7. y = C^e',a arc senx_|_ arc senx
dx
diy 2 dy
8 . dx2 ' X dx
0.

Integrar as equações de variáveis separáveis.


9.! y dx—x d y = 0 . Rcsp. y = Cx.
10. (l + w )y d u + (l — v) u dv = 0 . Resp. Log. uv-{-u— v = C.
11. (l + i/) dx —(1 —x) di/ = 0. Resp. (1 + í/) (1 —x) = C.
12. (í2— i / 2 )_____
4 r +U í® + tía;2=0.
T-a _I_ r 'A = n Resp. - ^ :^-------
K e . K n . ---- ^ +U L o g ^y = C.
l ncr
dt

13. {y — a) dx-fa;2 di/ = 0. Resp. — a) = Ce^


‘14. z dí —(/2—-a2) dzz=0. Rcsp. z2a_(7 / ^
t a
dx _ l + x2
45. . Resp. X =
dy ~ í + y^ i — Cy
16. (1 + í2) dt — l / F ds —■0. Resp. 2 tg s = C.
17. dp + p tg 0 d0 = O. Resp. p = C COS 0.
18. sen 0 COS q)d0 — cos 0 sen q>dq) = 0. Resp. cos(p = C cos0.
19. sec2 0 tg (p d0 +sec2 (p tg 0 dq)= 0. Resp. tg 0 tg q ) = C.
20. sec2 0 tg(p dcp + sec2 <p tg 0 dÒ= 0. Resp. scn2 0_|..sen2 cp= c.
21. (l + x2) dy-—~ ^í — y^ d x = 0 . Resp. arc sen y— arc tg x = C *
EQUAÇÕES D IFEREN CIA IS 147

22. V l - a ; 2 d y —V l —í/2dx = 0. Rcsp. y V l —x2 —x -í/^ =


23. 3 « * tg y d x + (l —«*)sec2i/dy = 0. Resp. tg y = C (1 —
24. (x — í/2jj) dx + (y — x2y) dy = 0. Resp. x2 + y2 = x2y2 C,

Estabelecimento de equações diferenciais


25. Mostrar que a curva cujo declive da tangente em cada ponto é proporcional
à abcissa do ponto de contacto é uma parábola. Resp. y = ax2-|-C.
26. Determinar uma curva que passa pelo ponto (0, — 2) tal que o declive
da tangente em cada ponto seja igual à ordenada correspondente aumentada
de 3 unidades. Resp. y =
27. Determinar uma curva que passe pelo ponto (1, 1) tal que o declive da
tangente em cada ponto seja proporcional ao quadradc da ordenada desse
ponto. Resp. k (x — í) y — y + 1 = 0 .
28. Determinár uma curva cujo declive da tangente em cada ponto seja n
vezes maior que o da recta que reúne este ponto à origem das coordenadas.
Resp. y = Cx^.
29. Fazer passar pelo ponto (2, 1) uma curva cuja tangente em cada ponto
1
coincida com o raio vector traÇado da origem a esse ponto. Resp. y = x.

30. Encontrar em coordenadas polares a equação duma curva tal que em cada
ponto a tangente do ângulo formado p^lo raio vector e a tangente à curva
seja igual ao inverso mudado do sinal do raio vector. Resp. r(0-[-C7) = l.
31. Encontrar em coordenadas polares a equação duma curva tal que em cada
ponto a tangente do ângulo formado pelo raio vector e a tangente à curva
seja igual ao quadrado do raio vector. Resp. r^ = 2.
32. Mostrar que a curva que goza da propriedade de todas as suas normais
passarem por um ponto fixo é um círculo.
33. Achar uma curva tal que em cada ponto a subtangente seja igual ao dobro
da abcissa. Resp. y = C “|/x .
34. Determinar uma curva cujo raio vector seja igual à porção de tangente
compreendida entre o ponto ^ langência e a sua intersecção com o eixo Ox.

Resolução — De aCqr^ com as condições do problema "j/l-j-y'2 =


= t/ x2-|- iy2 donde ^ ^ ^ ^ Integrando>o, obtém-se duas famílias de
V i f y X■
^ C
curvas: y = Cx e y = — .
X

Segundo a lei de Newton, a velocidade de arrefecimento dum corpo qual­


quer no ar é proporcional à diferença de temperaturas entre o corpo e
o meio. Sendo a temperatura do ar 2C° C, o corpo arrefece de 100° a 60° C
no espaço de 20 minutos. Quanto tempo demorará a temperatura a baixar
a 30° C?
dT
Resolução — A equação diferencial do problema é — 20). Inte­
grando, vem r —20 = Ce'*'; 7 = 1 0 0 quando í = 0 ; 7 = 60 quando <= 20;
148 CALCULO D IFEREN CIA L E INTEGRAL

logo C = 80, donde 40 = 67^20/1^ 20^ por conseguinte, J = 2 0 +

+ 80 Fazendo T = 30, vem / = 60 minutos.

36. Considere-ss um funil cónico de ângulo 60^ no cimo e de altura 10 cm^


Ao fim de que tempo T o funil ficará vazio, sabendo que a água passa
por uma abertura de 0,5 cm-^ no fundo?
Resolução — Calculemos de duas maneiras diferentes o volume de água que
corre entre os instantes t e r + Ar. À velocidade constante v, escapa em
um segundo uma coluna de água de secção 0,5 cm^ de altura h. Escapa-se,
pois, no tempo Ar uma quantidade de água de dv
— dt)= — 0,5i) <ít= — 0,3 *).
Por outra via, diminuindo a altura com o escoamento, o seu acréscimo dh
é negativo e tem-se:
— dv = Jir* áfc= 4 (* +

De modo que
■^(ft + 0,7)*dA= —0,3 V 2 g h d t,
O
donde
t = 0,0315 (10*/*—ft*/*)+ 0,0732 (10®/*— fe®/*)+ 0,078 ( V 10 — Y h ) .
Fazendo li = 0, obtém-se o tempo de escoamento T = 12,5 segundos.
37. A travagem dum disco que gira num líquido é proporcional à velocidade
angular de rotação o). Achar a dependência entre a velocidade angular e
o tempo, sabendo que a velocidade angular do disco baixa de 100 r/m
para 60 r/m no espaço de um minuto. Resp. cd= 100(3/5)^ tr/m n.
38. Suponha-se que a pressão de ar vertical numa dada secção depende da
pressão das massas de ar superiores. Encontrar a dependência entre a
pressão e a altitude, sabendo que a pressão é de 1 kg/cm* ao nível do
mar e de 0,92 kg/cm2 a 500 metros de altitude
Indicação — Servir-se da lei de Mariotte em virtude da qual a densidade
dum gás é proporcional à sua piessão. A equação diferencial do problema
é d p = — kp dh, donde p = e“0,00017/1. Resp. p = ^-0,00017/1.
Integrar as equações homogéneas seguintes:
39. (y — x )d x + (y + x)dy = 0, ReSp. y^ + 2xy — x ^ = C ,
40. (x-\-y) d x -^ x d y = 0 . Resp, x^-{- 2 xy = C.
41. {x-\-y) dx + (y — x) dy = 0 .R tsç . Log (x2 + i/2)^/2__arc t g = C.

^ 2 . X dy — y dx= ~\/x'^-^y^ dx. Resp. í + 2Cy— C^x^ = 0.)


43. (S y + í0 x )d x + (ò y + lx )d y = 0.Resp. (x + y)^ {2x+y)^ = C.

(*) A velocidade de escoamento v da água por uma abertura que


se encontra à distância h da superfície livrcr 6 dada pela tórmula v = 0,6 ~[/2 gh,
cm que g é a aederação no campo da gravidade.
BQUAÇOES DIFEREN CIA IS 149

(2 V®7 —s) dí + í ds = 0. Rcsp.K te^ Vi-* = C ou s = t Log2 — . Rcsp.

T C
5 . (r — s ) d í + í ds = 0. Rcsp.
45. = C ou s = íL og —
Rcsp.
46 xj/2 dy = (a:3 j,3) dx. Rcsp. y = x f 3 Log Cx.
47 X COS — (y dx + x dy) = y sen — (X dy — y dx). Rcsp. xy c o s = = ^ *

Integrar as equações diferenciais seguintes, reduzindo-as a equações homo­


géneas:
48. (3 y — 7 x - f 7 ) d x— (3x — 7y — 3) dy = 0. Rcsp. (x + y — í)^ (x —[/ —1)2 = C.
49. (x + 2 í / + l ) d x — (2 x + 4 j/ + 3) dy=^0. Rcsp. Log (4 x + 8 i/ + 5) + 8 y — 4 x = C .
50. {x + 2y + l) dx—(2x —3) dy = 0. Rcsp. Log (2x —3) — =
51. Determinar a curva cuja subnormal é a média aritmética entre a abeissa
e a ordenada do ponto da curva considerada. Resp. (x — y)2 (x + 2 y ) = C.
52. Determinar a curva cuja relação do segmento cortado pela tangente sobre
o eixo Oy pelo raio vector é uma constante.
_ ÉL
^ ^ dx f x \ ‘ni / C
Resolução — Por hipótese, tem-se :^ = = = = | = m, donde ^“g " j

53. Determinar a curva cuja relação do segmento cortado pela normal sobre
o eixo Ox pelo raio vector é uma constante.
, dy
Resolução — Por hipótese = m, donde x2-f ^2= ^2 (x—C)2,
'V x^+ yi
54. Determinar a curva cujo segmento cortado pela tangente sobre o eixo Oy
é igual a sec G em que 0 é o ângulo entre o raio vector e o eixo Ox.

Resolução — Como se tem tg 0 = -i^ e, por hipótese.

y = x ^dy- = a sec a
0,
dx
obtém-se
y - x - f - = a-^-----
dx X
donde j
- ( t + o)

55. Determinar a- curva sujo segmento cortado pela normal sobre o eixo Oy
é igual à distância do ponto considerado ]ài origem das coordenadas.
150 CALCULO D IFERENCIA L E INTEGRAL

Resolução — O segmento cortado pela normal no eixo Oy é igual a à 1/ + - ^ ;


logo, por hipótese, tem-se

donde
x2 = C(2|/ + C).
56. Achar a forma de um espelho tal que os raios provenientes dum ponto O
sejam reflectidos paralelamente a uma dada direcção.
Resolução — Identifiquemos esta direcção com o eixo Ox e seja O a origem.
Sejam OM o raio incidente, MP o raio reflectido, MQ a normal à curva
procurada:
a = P; OM = OQ, N M = y,

NQ==NO + O Q = - x - { - y x ^ + y ^ = y cotg P = ! / ^ .
donde
y dy = ( — x + ~ y y ^ ) dx \
por integração, encontra-se
y^ = C^-\-2Cx.
Integrar as seguintes equações diferenciais lineares:
2y (x-Ll)». Resp. 2y = (x + l)* + C ( x + l)2 .
57. y
X + Í

y x - ^ í .Resp. i/ = C'x®+
X 1
58. y' — a i —a
59. (x — x3) y ' + (2x2— 1) y —ax3 = 0. Resp. y = ax + Cx~[/í — x^,
60. COS t-\-s sen t = 1. Resp. s =sen t-\~C cos t .
dt
—sen t
61. - ^+ s COS í = -^sen 2t. Resp. s= sén í — 1 + Ce
dt 2
62. y ' ---- íí- y = Resp. y = x‘^ (e^-\-C ).
X

63. y ' + — y = - ^ .R e s p .. x^y = ax + C.


X x '‘

64. y' + y = ^ . R e s p . e^y = x + C


e
1 —2x
65. r2 y —1 = 0 . Resp. y = X®(1 +
Integrar as equações de Bernoulli:
66 . y ' xy = x3y3. Resp. y2 (x^ + l + = 1.
67. (1 — x2) y ' — xy — axy^ = 0. Resp. {C \ — x^—a ) y = l.
3y2y'_fly3__x— 1 = 0 . Resp. = (x 4 -l) — 1.
- 1/2
69. w'(x*í/S+a;i/) = l. Resp. X [{2 — +C]=e^
EQUAÇÕES D IFEREN CIA IS 151

70. {y\jOgx — 2 ) y d x = x d y , Resp. y (Cx + Log


1) = 1.
tg x + sec X
71. z/ —y'COSx = ^2 cosa: (1 —senx). Resp. y = -
scnx + C
Integrar as seguintes equações de diferenciais totais:
72. (x2^ y ) dx + {x— 2y) dy = 0. Resp. + y x — y^ = C,
73. (y —3x2) dx— (Ay — x) dy = 0. Resp. 2y2 —x y + x®= C.
74. (y3—x) y' = y. Resp. y^ = 4xy + C.

Resp. Log = c.
X X— y

76. 2 (3 x y 2 + 2 x 3 ) d x 3 ( 2 x 2 ^ ^ 2 ) dy = 0. Resp. x^ + 3 x 2 j/2 i^3—


X d x + (2x + y ) dy
77. = 0. Resp. L o g (x + y )- =c.
78. ( i + - 5 r ) **+!'’*
79. f Í f c i í ! Í f = 0. Resp. ___ ^
{x — y)i x —y

80. xdx-\-ydy = ^ - ^ ^ ^ .'Resp. i2 + y * —2arc t g ~ = C.

81. Encontrar as curvas que gozam da propriedade segundo a qual o produto


do quadrado da distância dum ponto qualquer tomado sobre a curva
considerada na origem pelo segmento cortado pela normal sobre o eixo
das abcissas é igual ao cubo da abcissa do ponto. Resp. y^ (2x2 _f_ ^2) = c ,
8 Encontrar os envoltórios das famílias de curvas seguintes: a) y = Cx-\-C^,
Resp. i» + 4 j, = 0. b) j, = - ^ + C 2 . Resp. 27*2 = 4j/3. c) 4 -— I t-= 2 .
O c
Resp.27í^ = **. d) C*x+Cj/— l = 0.Resp. j,í + 4x = 0. e) ( i — C)*+(j/—C)* = C*.
Resp. 1 = 0 ; y = 0 . f) (í —C)2 + y* = 4C. Resp. i,2 = 4x4-4. g ) ( i_ C ) * +
+ (íí— C)3 = 4. Resp. {x— y)i = 8 . h) Ci3-j-C2i/ = 1. Resp. i« + 4y = 0.
83. Uma recta desloca-se de tal maneira que a soma dos segmentos que ela
corta sobre os eixos de coordenadas é igual a uma constante a. Procurar
o envoltórm desta família de rectas. Resp. = (parábcrta).
84. Achar o envoltório duma família de rectas tais que os ei^os de coordenadas
cortem sobre estas rectas, segmentos de comprimehto constante a,
Resp. x*/3_j_^V3_fl*/3^ j
8Õ. Achar o envoltório duma família de círculos, óujos diâmetros sejam duplos
das ordenadas da parábola = 2px. Resp. y2 = 2/> +

86. Achar o envoltório dum?, família de círculos centrados sobre a paróbola


= Ipx è que passe pelo vértice da parábola. Resp.
«cissoíde». x3 -f. ^2 (x 4- 2/?) = Q.,
152 CALCULO D IFEREN CIA L E INTEGRAL

87. Achar o envoltório duma famflia de círculos cujos diâmetros sáo as curvas
-_0
da elipse 62^2+ a2y2 =-^252 perpendiculares ao eixo Ox, Resp. + 62
4- - ^ = 1
^ 62
88. Achar a evoluta da elipse x262 + a2y2_a252 como envoltório das suas
normais. Resp. =
Integrar as seguintes equações (Equações de Lagrange):
89. ]/ = 2 x y '4 -y '^ . Resp. x = - ^ — ^ p ; y= Í£ ^ Z fl .

90. y = x y '* + y '* . Resp. y = (V * + l-T -C )* . Integral singular : y = 0.


91. y = x ( l f y ') + (y')*. Resp. x = Ce~P— 2p + 2 ; y = C ( p + í) c - P — p*-^2.
92. y = yy'^ + 2 x y \ Resp. 4Cx = 4C2— y^.
93. Determinar as curvas à normal constante. Resp. (x—C)2+y2 = a2. Integral
singular: y = ± f l.
94. y = xy' + i/' — y'2. Resp. y = Cx-\-C — C2. Integral singular: : 4y =
= ( x + l) 2 .
95. y = x y '+ l / l — y'2,Resp. y = C x + I n t e g r a l singular : y2—x 2 = +
96. y = xy' + y '. Resp. y = Cx-\-C,
1 1
97. y = x y ' 4 — . Resp. y = Cx + — Integral singular : y2 = 4x.
y O

y^33z=----
= 27
1 1 „
—x^-
98. y = x y ' — . Resp. y = C x— ^ . Integral singular 4
99. A área do triângulo formado pela tangente a uma curva e os eixos de
coordenadas é constante. Achar essa curva. Resp. As hipérboles equiláteras
4xy = i t l>em como as rectas da família y = Cx ± a “^/C.
100. Achar uma curva tal que o segmento da sua tangente compreendida entre
os eixos de coordenadas tenha um comprimento constante a. Resp. y = C x ±
aC
± . Solução singular : x*/3_j_^2/3_^2/3
101. Achar uma curva tal que a soma dos segmentos cortados pelas suas tan*
gentes sobre os eixos coordenados seja igual à contante 2a. Resp. y = Cx —
— _2oC Solução singular: (y — x — 2a)2 = 8ax.
1 —C
102. Achar as curvas tais que o produto das distâncias de dois pohtos dados à
tangente seja constante. Resp. Elipses e hipérboles (trajectórias ortogonais
e isogonais).
103. Achar as trajectórias ortogonais da família de curvas y = ax^. Resp. x^ +
-f ny- = C.
104. Achar as trajectórias ortogonais da família de parábolas y* = 2p(x — a)

(a é o parâmetro da família). Resp. y = Ce


105. Achar as trajectórias ortogonais da famílias das curvas x* — y* = a (sendo
a o parâmetro). Resp. y = — .
106. Achar as trajectórias ortogonais da família de círculos x* + y* = 2ax. Resp.
Os círculos;
y = C (x 2 + y 2 )
EQUAÇÕES D IFEREN CIA IS 153

107. Achar as trajectórias ortogonais de parábolas iguais às tangentes nos seus


vértices a uma recta dada. Resp. Se o parâmetro das parábolas for 2p e
2 / 2~
se Oy for a recta dada, a equação das trajectórias será y-{-C = -^ 1 / —
3 r p
x3
108. Achar as trajectórias ortogonais das cissoídes i/2 = . . Resp. (Jt2 + y2)2 =
2a — X
= (jc^ - y^)a\
109. Achàr as trajectórias ortogonais das lemniscatas (jc* + y^y = (jc* — y^)á^.
Resp. (x^J^y2)^ = Cxy.
110. Achar as trajectórias isogonais da família de curvas' jc2 = 2a (y — X T/3)
em que a é um parâmetro variável, sabendo que o ângulo entre as curvas
e as suas trajectórias é w = 60°.
Resolução — Acha-se a equação diferencial da família de curvas y' = -—

— ~[/S e substitue-se y' pela expressão ç = — ----


í + y' tgo) *
Se « = 60°, tem-se ç = y '- V 3 e obtém-se a equação diferencial
1 + 1 /3 y'
y '—V 3 -2i/ -,/í
i + y 'V s X
O integral geral i/^ = C {x —y l/W já á a família das trajectórias procuradas.
111. Achar as trajectórias isogonais da família de parábolas y^ = 4Cx, sabendo
arc tg-, 2y-3c
que ú) = 45°. Resp. y^— xy-{-2x^ = Ce x v7
112. Achar as trajectórias isogonais da família de rectas y = Gjc para o) = 30°, 45°.
2 V3 arc tg ^

1
X^ —1—m2 —g

^ y
2arc tg —
a;2_i_ ^2 — g
113. y = C|í* 4- Eliminar Cj e C2. Resp. y" — y = 0.
114. Escrever a equação diferencial de todos os círculos dum plano. Resp.
(l+y'2) y>"-3y'y"2=0.
115 Escrever a equação diferencial de todas as cónicas com centros, nos
eixos principais Ox, Oy. Resp. x (yy"-\-y'^)—y'y = 0.
116. Dá-se a equação diferencial yw_2y''__y'4 -2 y = 0 ® ^ ^ua solução geral
y==Cie^-\- ^2^“*-]-C3^2*.
Pede-se: 1) verificar que a família de curvas dadas é precisamente a solução
geral; 2) encontrar a solução particular correspondente a jc = 0; y |= 1;
y’ = 0, y" = - 1. Resp. jr = - l (9e* + e-*—4e**). '

117. Dá-se a e^quação diferencial y’*= 1— \


e a sua solijção geral y = ± -2^ ( x -b

1) Verificar que a família de curvas dadas é precisamente o integral geral.


2) Achar a curva integral que passa pelo ponto (1, 2) e cuja tangente neste
ponto
R » p -forma
» - !com
/ -o= +
eixo
4 - positivo Ox um ângulo de 45°.
154 CALCULO D IFERENCIA L E INTEGRAL

Integrar as seguintes equações diferenciais simples, reduzindo-as a equações


de primeira ordem:
118. ^y'" = 2, Resj). y = x^Logx-]-CiX^-\-C 2X + C ^\ dar a solução particular que
satisfaz às condições iniciais: x = l, y = í, = j/" = 3.
119. = ■f + . . . + C„_1I + Cn.

120. y“= a ^y . Resp. a x= Log (ay + Va*!/® + C^)+ ou j/= Cie“» + Cje"®*.

121. y" = - ^ . Resp. (CiX+C 2)^ = Ciy<‘— a.


Nos exemplos' 122-125, escrever a solução particular que satisfaz às con­
dições iniciais: x = 0, y = — 1, y' = Q.
122. XI/'' — = Resp. y = e* (x— í) + C ^x^+C 2- S o l u ç ã o p a r tic u la r:
y = e ^ (x -í).
123. yy" — (y')^ + (y')^ = 0 - y + y = ^ + ^2- $ o l u ç ã o particular:

124. / + y' tg x = s e n 2 x . Resp. y = Í72+Ci sénx —x —y sen2x.Soluções p a rti­


c u la re s : y = 2 sen X—sen XCOS x —x — 1.
125. (y")^4-(y')^ = ®^* y = C2 — acos(x-\-C^). Solutions particulières :
y = a — 1 — a c o s x ; y = a cosx — (a + 1). (Indicação —'Forma paramétrica
y" = aco stj y' = ã se n f.)

í26. y" = . Resp. y = ± (x + + C2.

127. y'" = y"2. Resp. y = (Ci —x) [Log (Ci —x) — 1] fC 2X + C3.
128. y'y'" —3y"2 = 0. Resp. x = C'iy2 + C2y + f^3.
Integrar as equações lineares diferenciais seguintes de coeficientes constantes:
129. y" = 9y. Resp. y = €^€'^^ + € 26-^^.
130. y "-f y = 0. Resp. y = ^ cosx-j-B sen X.
131^ y'" —y' = 0. Resp. y = C'i + C'2e*.
132. y" + 12y = 7y'. Resp. ^ = Cie3* + C2^4^.
133. y" —4y' + 4y = 0. Resp. y = (C1 + C2X)
134. y" + 2y'-4-10y = 0. Resp. y = (/I cos 3x + B sen 3x).
-3+KÍ7 -3 -V \7
135. y"-f-3y' —2y = 0. Resp.. y = CiC ^ +C2e ^
136. 4y" — 12y' + 9 y = 0. Resp. y = (Cj 4-C2X)

137. y" + y' + í/ = 0. Resp. y = e ^ ^ cos x ) + B sen ( ^ í ^ x ) J


138. Duas massas idênticas estão suspensas numa mola em espiral. Suponha-se
que uma das massas se destaca e se pede para encontrar o movimento da outra.
Resp. x = a cos ^ - y í j , em que a é o alongamento da mola sob a
acção duma só massa em repouso.
EQUAÇÕES D IFEREN CIA IS 155

139. Um ponto material de massa m é solicitado por dois centros, sendo as


forças proporcionais à distância. O factor de proporcionalidade é k. A dis­
tância entre os dois centros é 2c. O corpo encontra-se no instante inicial
sobre a linha dos centros à distância a do meio. A velocidade inicial é
nula. Achar a lei do movimento do ponto. Resp. x = a cos

140. 5i/'' + 4y = 0. Resp. y = Cie^ + C2e~^ + C2e^^ + C^e~^^.


141. y”' —2y"— y '- | - 2 í / = 0. Resp. y — C .
142. y " " — + —a^y — 0. Resp. y = { C C 2^^) c®*.
143. j, = Ci + C2a: + C3x2 + C4c2* + C5C-2*.
144. yiv = 0. R e s p . y = (Cj cos l / ^ + ^2 s c n l/^ ) +
+ ( C 3 COS V 2 X + C 4 s c n " \/ 2 a :) c * .

145. y^^ — + 16y = 0. Resp. 1/ = U1^2* _j_C2c"23c ^ C3XC2*+ Ci^xe-^^,

146. !/^^ + y = 0. Resp. y = e ^ ^CiC 0S : ^ + C 2 sen— ^ +

147 , y^^ — a^i/ = 0. Encontrar a solução geral e pôr cm evidência a solução


particular que satisfaz às condições iniciais: a;o = 0, .v= l, t/' = 0, y" =.
= — t/'" = 0. Resp. Solução geral: y = C^e'^^-\-C2<s~^^+C^cosax -\-Cz
Solução particular: yo = cosax.
Integrar as equações diferençais com segundos membros; achar a solução
geral:
148. V " - l y ' + \2y = x. Resp. y = Cie3«-f-Cae«»+ •
144
149. s"—fl2s = <+ l. Resp. S = Cte°< + c^e-ot — L + i .
«2
150. 2/*+ ^ '—2y = 8 se n 2 í. R e ^ . y = Cie*+C2e-**—-^ (6 sen2x+ 2 cos2z),
151. y"—y = 5z + 2. Resp. y = Cie*+C2e“*—5z—2.
152. s ' —2as' + a2s = e< (a 1). Resp. « = Cie<*‘ + C2fe»‘ +
(a -l)í •
153. y" + 6y ' + 5y = e2*. R esp. y= C je -* + C 2 e “ 6* _ ^ A ^2 * .

154. í/" + 9 y = 6 c 3 * . R e s p . í/ = C i COS 3a: + C 2 s e n 3 a ; + - ^

155. —3i/' = 2--6a:. Resp. y = Ci + C2c3*4-x2.


156. i/" + 3y = e"* COS a:. Resp. y = c* (^4 cos ^ 2 a:+ B sen. I / 2 x) -f
+ - ^ (5 COS a:—4 sen x).

157. í/" + 4í/=-=2sén2x. Resp. > = >1 sé* 3c + 5cof2ec—y cos2ác.


156 CALCULO D IFEREN CIA L E INTEGRAL

158. y»" — —2y = 2x + 3.Resp. y = (Ci + C'2x) c* + C'3e2* —x —4.

159. y ^ a^y = sen ax. Resp. y = {Ci — sen ax) e®* + C2C"®* + C3 cos a x +
+ C4 senax.
160. y^^ 4- 2fl2y" -|- a^y = 8 cos ax. Resp. y = (Cj + ^2^) cos ax 4- (C3 + C4X) X
x2'
X sen ax ----- ^ cosax.
a"
101^ Determinar a curva integral da equação y^-|-A2y = 0 que passa pelo ponto
M (^0» 1/0) e tangente neste ponto à recta y = ax. Resp. y = yo cos Ac(x— xq) +
_L ® sen (x —Xq).
' k
162. Achar a solução da equação y" + 2Ay' + /i2j/==0, que satisfaz às condições
y = a, y '= C para x = 0. Resp. Si h< ^n,
y = e~^ cos ‘X/n^— h^x-\—:j^ Í i^ = ^ s e n "yn^ — h^x^ ;

si h = n, y = \(C + ah) x + a] ; si n,
C + a ( f e + y / t 2—ra8) (h _ y p r;^ )X
2 V W ^i
C + a ( h — 1/fe^—n^) ^ -(h + y siU S )*
2 y ft2 _ » a
163. Achar a solução da equação y^ + n^y = A sen p x ( p :^ n ) , que satisfaça
às condições: y = a, y ' = C quando x = 0.
C(/i2—p2)-_ Ap - , A .
Resp. y = a cos nx 4----^ H — 5----- 5“ sin px.
164. Um peso de 4 kg ligado a uma mola distende-a de 1 cm. Achar a lei do
movimento, sabendo que a extremidade superior efectua oscilações harmó­
nicas y = sen “j/lOOgí, sendo y a distensão vertical.
Resolução — Seja x a coordenada vertical do peso contado a partir da
posição de repouso. Tem-se:
4 d2jj
- k ( x — y — l),
g dt^
em que / é o comprimento da mola distendidade t k = 400, como resulta
das condições iniciais. Deduz-se 100^ x = 100^ sen ‘\/íÕÕgt + 100/^.
Procurar-se-á um integral particular desta equação sob a forma
t (C1 cos ~l/í00gt 4- C2 sen VlOOgí) 4- g,
dado que o primeiro termo do segundo membro da equação entra na
solução da equação homogénea.
165. No problema 139, a velocidade inicial é igual a e está dirigida pcr-
pcndicularmente à recta dos centros. Encontrar a trajectória.
Resolução — Tomemos a origem das coordenadas no meio do segmento í
ligando os dois centros: as equações diferençais do movimento escrevem-se:

= + m ^ = - 2 ky.
EQUAÇÕES D IFEREN CIA IS 157

Condições iniciais no instante r = 0:


dx ^ r. dy

Encontra-se, integrando:

x= a c o s ( | / ^ / ) ,

donde
x2 ym
- = 1 (elipse).
a2
166. Um tubo horizontal gira em torno dum eixo vertical com uma velocidade
angular constante <o. Uma esfera desliza no tubo sem atrito. Achar a
lei do movimento da esfera, sabendo que no instante inicial se encontra
sobre o eixo de rotação e a sua velocidade inicial é (segundo o eixo do
tubo).
d^r
Indicação — A equação diferencial do movimento é -j-r- = o)2r. Condições
dr
iniciais; r = 0, = vq quando / = 0.

integrando-se, obtém-se: = +
Aplicar o método da variação das constantes na integração das seguintes
equações diferenciais:
5 sen x-\-7 cos x
167. — 7i/' + 6i^ = sena;. Rcsp. y = C^e^-\-C2e^^-
74
168. y " - j - y = sec x , Resp. y = Ci cos x + C2 sen x - \ - x sen x + q o s x Log cos x.
1
169. i / + y = - . R esp. = cosa: + C 2 s e n x — l / c o s 2 i:.
cos 2x cos 2x
Integrar as seguintes equações diferenciais de tipos diversos:

170. y y ''= y 'i + l.R tsp . j/ = - 5 ^


2Ci
x2 dy — y2 dx xy
171 =0. Resp. = C.
(X—y)2 ‘ x—y
1 7 2 . í/ = a : y '2 + {/'*. Resp. jí = ( V Í T Í + C ) ® .
Soluções singulares: í/ = 0 ; x + l = 0.
173. y " + y = sec X . Resp. y = C i cos x - ( - C 2 s é n x + x se n x + cos x Log cos x .
1 7 4 . ( 1 + x 2 ) i/' — x i/ — a = 0. Resp. y = ax + C V l+ x*y^

U du l!
175. Xcos — = í/c o s---- X . Resp. xe = C.
X dx X
\e2x
176. —4i/ = í2*sen 2x. Resp. v = Cie-2* + C2e**— (scn2i 4-2 cos 2i).

177. + y L o g z = 0. Resp. (Logx + l + Ci) i/ = l.


178. (2i + 2í^—l ) d i + (a: + íí—2)dj/ = 0. Resp. 2x + y —3 Log ( x + í/ + 1) = C.
179. 3e* tg dx + (1 — e*) sec* j/dj/ = 0. Resp. tgj/ = C (1 —e*)®.
158 CALCULO D IFEREN CIA L E INTEGRAL

Integrar os sistemas de equações diferenciais:

180. -4 ^ = y + l» = ^+ Indicar a solução particular que satisfaça às con-


at at
dições iniciais x = — 2, y = 0 para / = 0. Resp. y = Ci cos f -(-'C'2sén t,
x = (Ci-\-C 2) c o s í+ (C 2 —Cj)sen t. Solução^particular
x* = cost — sení, i/*=:cosf.
181. —^ = x — 2y, —^ = x —y. Indicar a solução particular correspondente às
dt dt • •
condições iniciais jc = 1 y = \ para / = 0. Resp. y=C^ cos t-\-C2 sen/,
x = (C i-r^2) COS í-f (C2 —Cj) sen /. Solução particular: y* = cos/ — sen /,
y* = cos /.
dx dy

I
Resp. x = C ie -' + C2e-3‘,
182.
dx , y= C ^2^”^^ “}~cos t.
— + y = cos/.

( d^y = x,
I d/2 Resp. x = fC2«"^ 4-Ç3 cos Í + C4 sén í,
183.
y = C^e^ fC 2 e“ ^—C3 cos í —C4 sèn t,
] Ü í. y-
V d/2
Resp. x = C^ + C2t-\-C^t^ — ^ / 3 + e^
d^x , dy
dt^ ir
184. y = C4 - (Cj -f- 2C3) í — L (C 2 -1) í* -
dx d^y
= 1.
dt dt^

dy
= z — y, Resp. y = (Cj + C21) e“3x^
dx
185.
dz z = (C2—Cl —C21) e-3*.
= — y — 3z.
dx

Resp. y = Cie**-rC2e“3*,
186.
dz 2= -2(Ciér2* —C2«-**).
+ 4^=0.
dx
dy
- ~ - \- 2 y - \- z = » a x , Resp. y = Cl -j- C/^x 2 sen x,
187. 2 = —2Ci—C2 (2i + l) —
dz , «
------- 4 v — 2 z = cos X. —3 sèn X—2 cos x.
dx ^
dx
= y4-z,
d/ Resp. j: = Cie“ *-f C2e3‘.
188.
dy = x + z, y = C3e-‘ +C2e3‘,
dt
z=_(C i-i-C3) e-< + C2«3'.
dz
= x+ y.
dt
EQUAÇÕES D IFEREN CIA IS 159

C*
\ dx z ’ Resp. z ^ C ^ e
189.
I ^ ___ i _ £/=X-
N dx ~ y — X ‘ c .c
1^2
dy ^ X
190.
{ dx
dz _ X
yz ’
Resp. — = Ci,
y
2^2— | - x2= C2.
*

Estudar a estabilidade da solução x = 0, y = 0 para os sistemas de equação


diferençais seguintes:
dx
~ = 2 x -3 y ,
191.
I ^

^
= 5x+6y.

= _ 4 x - 1 0 ,,
Resp. Instável.

192.
I iisT
- ^ = i2x + iSy,
Resp. Estável.

193.

194.
I ^ = -8 x -i2 y .
Resp. Instável.

Achar os valores aproximados das soluções da equação y' = + x que


verificam a condição inicial: y = 1 para jc = 0. Encontrar os valores das
soluções para os valores jc = 0,l; 0,2; 0,3; 0,4; 0,5. Resp. z/3c=o,s = 2,114.
195. Achar o valor aproximado Vx—i,k da solução da equação y '+ y = g3c
que verifica as condições iniciais: y = 1 para jc = I. Comparar o resultado
obtido com a solução exacta.
196. Achar os valores aproximados e yt=uk das soluções do sistema de
equações ^ . = y — x, = — x — 3y que verificam as condições iniciais:
X = 0, y = 1 para / = 1. Comparar os resultados obtidos com os valores
exactos.
Capítulo XIV

INTEGRAIS MCLTIPLOS

§ 1. Integral duplo

Seja no plano Oxy um domínio fechado (*) D limitado por uma


curva L.
Seja dado no domínio D uma função contínua
z = f ( x , y).
Dividamos o domínio D em n domínios parciais por curvas
quaisquer:
Asi, A^2f AS3, • • *9 As^
(fig. 276). Para não complicar a escrita, designaremos igualmente por
Asi, Aí^ as áreas destes pequenos domínios. Escolhamos em cada
um ponto Pi arbitrário (interior ou
sobre a fronteira); ter-se-á, pois, n pontos:
■Pl» *^2» • • •» -^71•
Sejam /( P i) , / ( ^ 2)» • • •» / (^n) os
valores da função nestes pontos: formemos
a soma dos produtos f (Pi) A sj:

y „ = /(i>0 As, + / ( í '2)A s2 + . . .

+ /(/>„) A í „ = S / ( P í ) A s í (1)

que se chama soma integral da função


/(jr, >^) no domínio D.
Se / > 0 em D, poder-se-á representar geomètricamente cada termo
/ (Pj) Asi como o volume do cilindro elementar de base Aí ^ e de
altura / (P,).
A soma é a soma dos volumes dos cilindros elementares, isto é,
o volume do corpo em «escada» representado na fig. 277.

(*) Um domínio D diz-se fechado se está limitado por uma curva


fechada e se se considera que as pontos fronteiros pertencem ao domínio.
INTEGRAIS MÚLTIPLOS 161

Consideremos uma sequência arbitrária de somas integrais for­


madas pela função f(x, y) no domínio D
Vn„ Vn„ nk* (2)

por diversos cortes de D em domínios parciais Asi. Supor-se-á que


o maior diâmetro dos Asj tende para zero quando oo.
Tem-se, então, o seguinte teorema que não demonstraremos.
Teorema— 1. Sendo contínua a função f(x, y) no domínio /e-
chado D, a sequência (2) de somas integrais (1) tem um limite quando
o maior diâmetro dos domínios parciais As tende para zero e quando

n - ^ oo. Este limite é o mesmo qualquer que seja a sequência (2),


isto é, que não depende nem do modo do çorte de D em domínios
parciais Asi nem da escolha do ponto Pi em Ast.
Este limite chama-se integral duplo da função /(x, y) sobre o
domínio D e designa-se por

l \ f { P ) d s ou y) dxdy,
D D
isto é.

lim 2 / (Pi) [x, y) dxdy.


diam A Si - > 0 i = l D

D chama-se o domínio de integração.


Se f (jc, y) > 0, o integral duplo da função f (x, y) sobre o domí­
nio D é igual ao volume Q do corpo limitado pela superfície z = f(x, y),
o plano z = 0 e a superfície cilindrica cujas geratrizes são paralelas
ao eixo Oz e se apoiam sobre a fronteira de D (fig. 278).
Consideremos ainda os teoremas seguintes sobre o integral duplo.
11
162 CALCULO D IFEREN CIA L E INTEGRAL

Teorema — 2. O integral duplo da soma de duas funções tp (;c, y)


^ y) no domínio D é igual à soma dos integrais duplos de cada
uma das duas funções consideradas nesse domínio:

1 5 [q> {=>:, y) + ^ (^. í')] f y'» + y ^


Teorema — 3. Pode-se separar um factor constante para fora do
sinal de integração dupla:
se a = const., tem-se
5 J a c p ( x , y ) d s = a l l (f ( x , y) ds.
D D

Demonstram-se estes dois teoremas exactamente como os teoremas


correspondentes sobre os integrais definidos (ver tomo I, § 3, cap. XI).
Teorema — 4. Se o domínio D for constituído por dois domínios
parciais Di e D 2 , sem ponto interior comum e se f (x, y) for contínua
em todos os pontos de D, tem-se
J J / ( x , y ) d x d y = \ J f { x , y)dxdy +

D2
Demonstração — Pode-se representar a
soma integral em D sob a forma (fig. 279)
2 / (PO = 2 /(^ 0 +

+ 2 / ( ^ i ) A S i, (4)
Di
contendo a primeira soma os termos relativos aos domínios parciais
de Dl e a segunda os termos relativos aos domínios parciais de D^,.
Como o integral duplo não depende de modo de corte, cortaremos
o domínio D de tal maneira que a fronteira comum de Di e D 2
seja também uma fronteira dos domínios parciais As^-. Passando a
limite a igualdade (4) quando As^ -^0 , obtém-se a igualdade (3). Este
teorema subsiste quando D é formado de vários domínios disjuntos
ou sem pontos interiores comuns.
§ 2. Cálculo dos integrais duplos
Consideremos um domínio D do plano Oxy tal, que qualquer
paralela a um dos eixos coordenados, por exemplo a Oy, e que passa
por um ponto interior (♦) ao domínio, corte a sua fronteira em dois
pontos e N 2 (fig. 280).(*)
(*) Um ponto interior é um ponto que não se encontra na fronteira.
INTEGRAIS MÚLTIPLOS 163

Suporemos que, no caso considerado, D está limitado pelas curvas


y = (pi (x ), y = (p2 (x) e das rectas jc = a , x = b e que
(pi (o;)<(p2(^)»
sendo as funções <pi ( jc ) e (pz (x) contínuas sobre o segumento [ a , b].
Convencionaremos chamar a este domínio regular segundo o
eixo Oy. Do mesmo modo se define domínio regular segundo o eixo Ox.
Um domínio, regular segundo os
dois eixos de coordenadas dir-se-á, sim­
plesmente, domínio regular, A fig. 280
dá um exemplo de domínio regular.
Suponhamos / ( jc , y) contínua no
domínio D,
Consideremos a expressão
h (P2(x)
I d = \ ( i / ( ^ . y) dy) dx
a q>i (x)
que chamaremos integral duplo ou soma
dupla da função / ( jc , >^) sobre D, Nesta
expressão, calcula-se em primeiro lugar
o integral entre parêntesis, sendo a integração feita em 'relação a y
e sendo jc considerado como constante. Acha-se, após integração, uma
função contínua (*) de j c :
q>2^x)
(D (a;)= I f{x, y)dy.
(Pl (x)
Integremos agora esta função em relação a jc entre os limites
a c b:
b
/ ^ = J O (x) dx.

Por fim, encontra-se um número constante.


Exemplo — Calcular o integral duplo
1 x2
0 0

Resolução — Em primeiro lugar, calculemos o integral interno (entre


parêntesis):

rí-l-

(*) Não demonstraremos a continuidade da função O (a:).


164 CALCULO DIFEREN CIA L E INTEGRAL

Integremos, agora, a função obtida de 0 a 1:

) ( * * + x ) ‘^ ^ = l “ 5 ~ + 3 : 7 j o 5 + H *= 105-
0
O domínio de integração D é o domínio limitado pelas curvas (fig. 281)
y = 0y x = 0y i/ = a:2, x = l .
Sucede que o domínio D é tal que uma função y = (fi (jc), y =
não pode ser dada por uma única expressão analítica em todo o

intervalo de variação de x (de x = a a x = b). Seja, por exemplo,


a <c < b e
^)^ (x) = (x) sobre o segmento [a, c],
cp^ (j :) = X {x) sobre o segmento [c, ò],
sendo e x U ) funções dadas analiticamente (fig. 282).
Escrever-se-á, então, o integral duplo como se segue:
h (po(x)
í ( í /( ^ . V)dy)dx =
a (pi(x)

= í ( í y)dy)dx+ l (J /(x , ij )dy)dx =


a qpiC\) c (Pt(x)

= n í y ) dy ) dx - \ - 5 ( J f (x, y)dy) dx.


a itC.x) c xix)

Escreve-se a primeira igualdade em virtude da conhecida pro­


priedade dos integrais definidos e a segunda porque se tem (x) = ^ (x)
sobre o segmento [a, c] e <pi (x) = x (^) sobre [c, b].
Uma transcrição análoga para o integral duplo tem lugar quando
a função <p2 (x) se decompõe em diferentes expressões analíticas sobre
o segmento [a, b \
Estabeleçamos algumas propriedades dos integrais duplos.
INTEGRAIS MÚLTIPLOS 165

Propriedade— 1. Se se divide um domínio D regular segundo Oy


em dois domínios Dx e por uma paralela ao eixo Oy ou ao eixo Ox,
o integral duplo sobre D é igual à soma dos integrais análogos
sobre Di e D 2 :
I d = ^Di "h (^)
Demonstração — a) Suponhamos que a recta jc = c (a < c < b )
divide o domínio D em dois domínios regulares segundo Oy (*) Z>i e Dz.
Então,
h (P2(x) b
I d = \ { J J { x , y)dy) d x = \ ( b { x ) d x =
a q>i ( X ) a

c 6
= Í O ( x ) á c + j(I>(x)da: =
a

C <P2 (x)
=S( S /(^, !/)dy)dx +
a <Pi ( X )

b (P 2 ( x )

+ S( I y) d y ) d x = lD, + / dj.
C (P i ( x )

b) Suponhamos que a recta y = h divide o domínio D em dois


domínios regulares segundo Oy Dx e Dz como a figura 283.
Designemos por Aíi e Mz os pontos de intersecção da recta y = h
com a fronteira L de D. Designemos as abcissas desses pontos por
e bz*
O domínio D é limitado por curvas contínuas:
1. y = W ;
2. a curva AxMxMzB de que escrevemos convencionalmente a
equação sob a forma
y = (p*(a:),

tendo em vista que (x) = «p, ( ^ ) q u a n d o a - ^ x < ; a i e


e que _
<p*(x) = h q u a n d 9 ^ < x < 6,;
3. as rectas jc = n. x = b.
O domínio Dz é limitado pelas curvas
y = (pí (x), y = (p2 (x), em que Uj < x < òj.
(*) O facto de uma parte da fronteira do domínio D ser um segmento
vertical não impede que este domínio seja regular segundo Oy; porque se
exigia para esse efeito que qualquer vertical que passe por um ponto interior
do domínio não cortasse a fronteira em mais de dóis pontos.
166 CALCULO D IFEREN CIA L E INTEGRAL

Escrevamos a identidade seguinte aplicando ao integral interior


o teorema sobre a decomposição do intervalo de integração:
h (poíx)
I d = 1 { í /(^ . y ) d y ) d x =
a (pi(3c)

h q)*(x) (Í.2Í31')
í ( y)dy+ S f{x, y ) d y ) d x =
a ffiix) <p*(x)
(p*(x)
b í;,o( x)

= í ( í y ) d y ) d x + J ( I f{x, y) dy) dx.


a (pi(x) a cp*(.x)
1

Decomponhamos o último integral em três integrais aplicando o


mesmo teorema integral exterior:
h q>2(x) a, q)2<x)
] ( i y)dy)dx=^ y)dy)dx+
a q)*(x) a (p*(x)

bj q>2(x) b q>2(*^)
+ 5( i /(^- y ) d y ) d x + 5 ( J /(x , y ) d y ) d x \
Cj q>*(x) bi (p*(x)

Como (x) = (x) no segmento [a, b\ e no segmento [f>„ b\


o primeiro e o terceiro integral são idênticamente nuIo$. Por coQseguinte
qp*(x)
1
jj b| <P2^^^
I d = ] { I /{^. y ) d y ) d x + l ( i /(^> y) dy) dx.
a (PiCx) Qj (p*(x)

O primeiro termo é aqui um integral duplo estendido sl Di t


o segundo, um integral estendido a Dz. Por conseguinte,

A demonstração é análoga qualquer que seja a secante AfiAfz.


Se a recta AfiAfz divide D em três domínios ou mais, obtém-se uma
relação análoga a (1) com um número correspondente de termos no
segundp membro.
Corolário — Pode-se dividir cada um dos domínios regulares
segundo Oy por uma paralela a Oy ou Ox e aplicar-lhe a proprie­
dade (1). Por conseguinte, pode-se dividir o domínio D por paralelas
aos eixos coordenados num número arbitrário de domínios parciais
regulares:
Dj, £>2, Dg, . .
INTEGRAIS MÚLTIPLOS 167

e poder-se-á, sempre, afirmar que o integral duplo alargado ao domí­


nio D é igual à soma dos integrais duplos alargados aos domínios
parciais (fig. 284)
/ d= H " -^^8 "1" (^)
Propriedade — 2. (Avaliação dos integrais duplos). Sejam m e M

o mínimo e máximo valor da função f(x, y) do domínio D. Seja S


a área de D, Tem-se a desigualdade
b CP2 ^ x )
( I y)dy)dx^MS. (3)
a (P i ( x )

Demonstração — Calculemos o integral interior que designaremos


por 4» (jc):
(P2(x) <P2(x)
(x) = J f{x, y ) d y ^ I M d y = M [((>2 {x) — (pi (x )].
(x) <Pi (x)
Tem-se:
b (p2 (x) b

/ f l = I ( S /(a:, i/)dí/)dx< 5 M[(p2 (x) — çi(x)]dx — M5,


a (Pi (x) a

isto é,
(3')

Duma maneira análoga


92 <P2 ( x)

0(x)= í / (x, y) dy ^ J m d x = m [(pz (x) — q)j (x)],


<Pi (x; (p^ { x )
b b

/ ^ = J o (x) d x : ^ l m [(p2 (x) — (Pi (x)] dx = mS,


1G8 CALCULO D IFEREN CIA L E INTEGRAL

isto é, que ID TflS. (3")

A desigualdade (3) resulta das desigualdades (30 e (3^0*

Interpretaremos geomètricamente este teorema no parágrafo seguinte.


Propriedade — 3. (Teorema da média). O integral duplo I d duma
função contínua f(x, y) é iguçd ao produto de S pelo valor da função
num certo ponto P do domínio D:
h (p2 (ac)

í ( í /(^ . y ) d y ) d x = f { P ) S . (4)
a ( P i( 3 c )

Demonstração — Deduz-se de (3):


1
TO< — / d < Aí.
U

O número ^ d está compreendido entre o maior e o menor


valor da função f(x, y) no domínio D, Em virtude da continuidade
de f(x, y) em D, ela toma num certo ponto P do domínio D o
i
valor / d , isto é. que
o

donde-
ÍD = f { P ) S . (.=»)

§ 3. Cálculo dos in teg rais duplos {continuação)

Teorema — O integral duplo duma função contínua f (jc, y) esten­


dido ao domínio regular D tem por expressão (♦)
h (po(^)
J J / (x, (/) dard{/ = J ( ] / {x, y) dy)dx.
D a (Pi(x)

(*) Supõe-se, de novo, que o domínio é regular segundo O y e limitado


pelas curvas y = (p, (i), y = <p2 (i), i = a, i = 6.
INTEGRAIS MÜLTIPLOS 169

Demonstração — Cortemos o domínio D por paralelas aos eixos


coordenados em n domínios regulares (rectangulares):
Asi, As2) • • •}
Tem-se. em virtude da propriedade 1 [fórmula (2)] do parágrafo
anterior.

/d= •••+ S
i= l

Transformemos cada termo da direita pela aplicação do teorema


da média sobre os integrais duplos
/ A ,.= /(P ,) As,.
A igualdade (1) transforma-se em

/ d = / (Pi) Asi + / (Pi) Asj + . . . + / (P„) As„ = S / (Pi) Asi, (2>


i=l
onde P i é um ponto em As,. Tem-se à direita uma soma integral para
a função / (x, y) sobre D. Segundo o teorema da existência dos inte­
grais duplos, resulta que o limite desta soma, quando n - ^ oo e que
o maior diâmetro dos domínios parciais As^ tende para zero, existe e
é igual ao integral duplo da função / (;if, y) sobre D. O valor numérico
de do primeiro membro da igualdade (2), resultante de duas
integrações simples sucessivas, não depende de n. Passando a limite
em (2). obtém-se
I d = lim S / (Pi) Asi = n / (^< y) ^y
diam As, -►o D
OU

H / (^. y) dy = ^D- (3>

Por fim, obtém-se:


b (P2<»)
y ) d x d y = . l A f ^ f(x, y)dy)dx. (4>
D a tpi(x)j

N ota— 1. Quando f(x, >^) > 0, a fórmula (4) a^piite uma inter­
pretação geométrica simples. Consideremos o corpo delimitado pela
superfície z = f (x, y), o plano z = 0 e a superfície cilíndrica cujas
geratrizes são paralelas a Oz e se apoiam sobre a fronteira do domínio D
(fig. 285). Calculemos o volume V deste corpo. Indicamos acima que
o volume deste corpo era igual ao integral duplo de /(jc, y) sobre D:

H y) dxdy. (5)
D
170 CALCJULO D IFEREN CIA L E INTEGRAL

Calculemos agora o volume deste corpo utilizando os resultados


do § 4, cap. XII, tomo I, sobre o cálculo do volume dum corpo em
função das áreas de secções paralelas. Tracemos o plano secante
X = const. (a < c < b). Calculemos a área 5 (jc) da figura obtida no

plano X = const. Esta figura é o trapézio curvilíneo delimitado pelas


curvas z = f(x, y) {x = const.), z = 0, y = n W , y = y>2W- Por con­
seguinte, esta área, exprime-se pelo integral
(P2(X)
5 ( x ) ,= \ f{x, y ) d y )
(Pi(oc) ( 6)

Conhecendo as áreas das secções paralelas, encontra-se fàcilmente


o volume
h
V = l S ( x) dx

ou, substituindo a expressão (6), pela área 5 (x), encontra-se:


b q>2 ( x )

= 5 ( í /(^ , y ) d y ) d x . (7)
a <Pi ( x )

Os primeiros membros da fórmula (5) e (7) são iguais e. portanto,


o mesmo se diga dos segundos membros
b (P 2 ( x )

n /{ ^ > y ) d x d y = l ( J f (x, y ) dy) dx.


D a <Pi (x)
INTEGRAIS MÚLTIPLOS 171

Agora não é difícil dar o sentido geométrico do teorema sobre


a avaliação dos integrais duplos (propriedade 2 do parágrafo anterior):
o volume V do corpo delimitado pela superfície z = f (jc, y), o plano
z = 0 e a superfície cilíndrica que tem por directriz a fronteira do
domínio D é superior ao volume do cilindro de base S e de altura m,
mas inferior ao cilindro de base 5 e de altura M (sendo m t M o

F i g. 287.

menor e o maior valor da função z = f (x, y) no domínio D (fig. 286).


Isto resulta do facto de o integral duplo Ij) ser’igual ao volume V
deste corpo.
Exemplo — 1. Calcular o integral duplo

X X (4—«2 —ya)dxíiy,
D
3
sabendo que o domínio D está limitado pelas rectas x = 0, x = l, y = 0, y =
Resolução.

8 /2 1

0 0 u

Exemplo — 2. Calcular o integral duplo da função f(x, y) = 1 + x + y


sobre o domínio limitado pelas curvas x = " l/y , y = 2, z = 0 (fig. 287).
172 CALCULO D IFEREN CIA L E INTEGRAL

Rm o Iuçõo.

2 Vv
j [ j (l + x + !/)(ix J á y = j ^x + x y + ^ ~ ^ ^ d y =
0 - V 0

= 1 [ ( V í/ + y V Í / + - |- ) — .V® =

2 i i
f r-y/- , T/- , í/® 1 J
I J 3^2 , 2i/2 y3 -12 4 4 „
= ) L V i ; + y + y V ! / ^ j < í í / = [ ^ + - r + - ^ + — J o = i5 ^ 2 + ^
0
A^o/a — 2. Suponhamos um domínio D regular segundo Ox deli­
mitado pelas curvas
a; = ^5i(í/). a; = i|)2(i/), y = c, y = d,
com ,t i (y ).< t z (y) ( % 288).
Tem-se, então, evidentemente
d i|J2(i^)
J J / ( x , y ) d x d y = l ( I f{x, y) dx ) dy . (8 )
D c ll?i(í/)

Para calcular um integral duplo, aplicar-se-á, segundo o caso,


a fórmula (4) ou a fórmula (8). A escolha é indicada pela forma do
domínio D ou da função a integrar.

Fig. 288 Fig. 289


Exemplo — 3. Inverter a ordem de integração em
1 Vx
/ = H I /(X, y)dy)dx.
0 3C
Resolução — O dornínio de integração é limitado pela recta y ^ x c
pela parábola y = '\ /x (fig. 289).
Qualquer recta paralela ao eixo dos x corta a fronteira do domínio
em dois pontos ou mais; poder-se-á, pois, aplicar a fórmula (8); fazendo
^i(y) = í/2, t|)2(y) = y, 0 < í/< l;
INTEGRAIS MÚLTIPLOS 173

tem-se
1 V
(í ^y-
0 yf

Exemplo — 4. Calcular
_y_

sabendo que o domínio D é o triângulo limitado pelas rectas y = x, y = 0,


X = 1 (£ig. 290).
Resolução — Apliquemos as fórmulas (4). (Se se aplicasse a fórmula (8),

ser-nos-ia preciso integrar a função e ^ em relação a x\ mas este último integral


não é integrável por meio de funções elementares):

D 0 0 0
1 1
= ^ x ( e - i ) d x = ( e - í ) - ^ \ = - ^ = 0 ,8 5 9 ...
0 0
Nota — 3. Se o domínio D não for regular nem segundo Ox
nem segundo Oy (isto é, se existirem verticais e horizontais que passem

pelos pontos interiores do domínio e que cortem a fronteira do domínio


cm mais de dois pontos), não se pode, então, integrar sem precaução.
Sc se chega a cortar o domínio irregular D em um número finito de
domínios regulares segundo Ox ou Oy, Di D 2 , .... integrar-se-á
cm cada domínio parcial e far-se-á a soma dos resultados obtidós.
Na figura 291, tem-se um corte dum domínio irregular D em
dois domínios regulares £>i e D 2 .
174 CALCULO D IFEREN CIA L E INTEGRAL

Exemplo — 5. Calcular o integral duplo


lle^^yds
D
em referência ao domínio D compreendido entre dois quadrados centrados na
origem e cujos lados são paralelos aos eixos coordenados sabendo que os
lados são» respectivamentc» iguais a 2 e a 4 (fig. 292).
Resolução — O domínio D é irregular. Corta-se em quatro domínios
regulares Di, £>2, D», D í pelas rectas x = — 1 e x = l. Tem-se, pois,
f t e*+i/ds= C ( e*+i'(is+ ( f e*+Wíís+f f e»+i/ds4 -^ J e^+V ds.
V DÍ oi 03 Oi
Tem-se, sucessivamente,
-1 2 12
I (I e ^ +Vd y ) d x + \ (l e=^*Vdy) dx +
D —2 -2 —1 1
1 -1 2 2
+ í (I dy)dx-^ ^ dy)dx =
-1 -2 1 -2
(tf2_ e - 2) ( e - l _ e - 2) + (e2—e) (« _ « -!) + (e-l —«-*) («— í" !)+
^ ( e 2_ e - 2) (e2_ e ) = (e3_ e -S ) (e—e-i) = 4 sh 3 sh 1.
Nota — 4. No seguimento, omitiremos os parêntesis no integral
duplo,
h (P2(3:)
^ D = l { I f i x , y ) dy) dx,
a q)j(x)
e escreveremos, simplesmente:
6 (Pgíx)
Id= 1 I f {x, y) dydx,
a (Pi(3c)

sendo a integração feita pela ordem como são escritos os diferenciais


das coordenadas (’").

§ 4. Aplicação dos integrais duplos ao cálculo


de áreas e volumes
1. Volumes — Vimos no § 1 que o volume V dum corpo limi­
tado por uma superfície z = / (a:, y \ onde f (x, y) é uma função não
negativa, o plano z = 0 e a superfície cilíndrica de geratrizes paralelas
a Oz e cuja directriz é a fronteira de D, é igual ao integral duplo
de / (x, y) sobre D:
v=

(*) É por vezes cómodo escrever


b tp2 b (j)2

^D = j ( j / (^, í/) di/j d x = ^ d x ^ /(X, y) dy.


a <pi a q>i
INTEGRAIS MÚLTIPLOS 175

Exemplo —Calcular o volume do corpo limitado pelas superfícies x = 0,


>^ = 0, j: + y + z = l, z = 0 (fig. 293).
Re«dução.
^ = n (1—*—») ***.
D

em que D é o domínio triangular do plano Oxy limitado pelas rectas x 0,


y = 0, jc + y = l; é o domínio tracejado da figura 293. Tem-se:
1 1—X 1 ^
^ (1—*— ^(1 —*) y— *d* =
0 0 0

Tem-se, pois, V a — da unidade de volume.


6
N ota— 1. Se o corpo de que se procura o volume é limitado
superiormente pela superfície z = ^ 2 (x, y) ^ 0 e inferiormente pela
superfície z = (x, y} > 0, sendo a projecção destas duas superfícies

F i g. 294.

sobre o plano Oxy um mesmo domínio D, o volume V deste corpo


será igual à diferença dos volumes dos corpos «cilíndricos»; o pri­
meiro cilindro tem por base D e é limitado superiormente pela super­
fície z = ^ 2 (x, y): o segundo cilindro tem igualmente por base D e
é limitado superiormente pela superfície z = (Xy y) (fig. 294).
O volume V é, pois, a diferença de dois integrais duplos:
r = n < i > 2(^. y)ds.
D D
OU

<t>i{x,y)]ds. (1)
176 CALCULO D IFEREN CIA L E INTEGRAL

É fácil de demonstrar que a fórmula (1) é verdadeira não só


qiiàndo {x, y) e ^2 U, y) são funções não negativas, mas também
quando (jc, >?) e ^2 (x, y) são funções contínuas arbitrárias que
satisfaçam à relação
y)-
Nota — 2. Se / ( jc, y) muda de sinal em D, dividir-se-á D em
dois domínios: 1) £>i com / (jc, y) > 0; 2) Dz com / (x, y) < 0.
Suponhamos Di e Dz tais que os integrais duplos sobre estes domínios

existam. O integral sobre £>i é. então, positivo e representa o volume


do corpo que se endontra por cima do plano Oxy, O integral sobre Dz
é negativo e o seu valor absoluto representa o volume do corpo que
se encontra por cima do plano Oxy. Por conseguinte, o integral sobre D
representa a diferença dos volumes correspondentes.
2. Ãreas planas — Se se formar uma soma integral para a fun­
ção f (jc, >^) = 1 definida no domínio D, obtém-se a área

s = S l- A s i,
i=l
qualquer que seja o corte Passando a limite nó segundo membro,
obtém-se
S=l^dxdy.
D
INTEGRAIS MÚLTIPLOS 177

Se o domínio D é regular (ver. por exemplo, fig. 280), a área


exprime-se pelo integral duplo
h (p2(^)
> 5 = 1 ( 5 dy)dx.
a (Pi(ac)

Tem-se, após integração do integral interno,


h

>5 = 5 [q >2 {x) — (^ )J dx


a
(comparar § 1, cap. XII, tomo I).
Exemplo — 2. Calcular a área do domínio limitado pelas curvas
y = 2 — x2 y = x .
Resolução — Determinemos os pontos de intersecção das curvas dadas
(fig. 295). As ordenadas das duas curvas são iguais num ponto de intersecção;
x = 2—
donde
x2 + x—2 = 0
x i = —2,
X2=í.
Obtivemos dois pontos de intersecção: ( — 2, 2), A/o(l, 1).
A área procurada é, pois,
1 2-3c2 1

-2 X -2 /
§ 5. Integrais duplos em coordenadas polares
Consideremos, em coordenadas polares 0, p um domínio D tal
que todo o raio procedente da origem e que passa por um ponto
interior do domínio corta a fronteira de D em dois pontos ou mais.
suponhamos que D é lipritado pelas curvas p = Oi (0), P = O , (0)
e os raios 0 = a e 0 = p, com (pi (0) < Og (0) e a < p (fig. 296).
Diremos, então, que um tal domínio é regular.
Seja em D uma função contínua das coordenadas 0 e* p:
z = F { e , p).

Decomponhamos arbitràriamente D em domínios parciais

Formemos a soma integral

V n = ^F{P,)As,, (1)
Jt—1
em que Pfi é um ponto tomado em As;^.
12
178 CALCULO D IFERENCIA L E INTEGRAL

Resulta do teorema de existência dos integrais duplos que quando


o maior diâmetro dos ASf^ tende para zero, a soma integral (1) tem
um limite V. Ele dá por definição de integral duplo de F (0, p) em D:

7 = n / ’ (e , P)ds. (2)
D
Ocupemo-nos do cálculo de um tal integral duplo.
Como o limite da soma integral não depende do modo do corte
de D em domínios parciais cortá-lo-emos, por razões de como­

didade, traçando raios 0 == 0 q, 0 = 0i, 0 = Bg. 0 = 0^ (em que


00 = a , 0 n = P , 0 o < 01 < 02 < • • • < 6n)e circunferências concên­
tricas p = po, p = Pií . • p = Pm onde [po é o menor valor da
função (0) e Pm o maior valor de ^2 (6) no intervalo fechado
a < 0 < P ; ••• < Pi < . . . < p^ l.
Seja ASij^ a área delimitada pelas linhas de coordenadas
p = p f-i. p = p/, 0 = 0 / ,_ i , 0 0^.
Haverá 3 espécies de domínios parciais :
1. Domínios inteiramente interiores a D\
2. Domínios inteiramente exteriores a D;
3. Domínios que invadem a fronteira de D.
A soma das áreas que invadem a fronteira tende para zero
quando 0 e A p /^ ^ O ; desprezar-se-á, pois, estas áreas. As
áreas parciais As^j^ exteriores a D não entram na soma integral con-
INTEGRAIS MÚLTIPLOS 179

siderada e não apresentam interesse. Poder-seá, pois, escrever a soma


integral sob a forma

em que P,* é um ponto arbitràriamente tomado em


A soma dupla exprime que somamos em primeiro lugar sobre
o índice i considerando k fixo (isto é, que fazemos a soma das áreas
compreendidas entre dois raios vizinhos (*)). O sinal da soma exterior
exprime que adicionamos as somas que resultam da primeira soma
(somamos sobre k).
Achemos a expressão da área dum domínio parcial que
não invade a fronteira de D. É a diferença das ^reas de dois sectores:

= — (Pi “f" ^Pi) ^ Pi = ^Pi H— Api

ou
Aí?ife = p?Ap,. A0ft, oú P i < p ? < p i + Apf.
A soma integral escreve-se, pois(**),

h=i i
em que P (6J, p?) é um ponto tomado em As^^.
Destaquemos, agora, o factor A0;^ da soma interior (o que é
legítimo, porque é um factor comum a todos os termos desta soma):

= 2 [S ^ P?) P?^P‘]
k=l i
Suponhamos que Ap,- 0 e que A0/i é constante. Então, a
expressão entre parêntesis tenderá para o integral

<i>2(ej)
I F (6ft. P) P dp.
«Kl (e*)

(*) Notemos que somando sobre o índice / este índice não tomará,
forçosamente, todos os valores de 1 a m, dado que todos os domínios parciais
compreendidos entre os raios 0 = 0^^ e 0 = 0ft+i, não pertencem, forçosa-
m;nte, a D.
(**) É permitido considerar uma soma integral sob esta forma, dado que
o limite da soma não depende do ponto escolhido no domínio parcial.
180 CALCULO D IFEREN CIA L E INTEGRAL

Supondo agora que AB^ 0 obtém-se, por fim (*):


P <I>2(0)
V = S ( J F(9, p)p d p )d 0 . (3)
a <Di(0)
A fórmula (3) serve para o cálculo de integrais duplos em
coordenadas polares.

Fig. 297

Se se integrar primeiro sobre 0, depois sobre p. tem-se a fórmula


(fig. 297):
P2 0)2(p)
F = J ( J F{e, p)d 0 )p d p . (3')
Pi Wi(p)
Seja calcular o integral duplo da função f(x, y) sobre o domí­
nio D, sendo este integral escrito em coordenadas rectangulares:

y)dxdy.

Se D é um domínio regular em coordenadas polares 0, p, poder-


-se-á passar nos cálculos às coordenadas polares.

(*) A nossa dedução da fórmula (3) não é rigorosa: em primeiro lugar


temos feito tender Apj para zero conservando invariável, e sòmente depois
é Que temos feito tender para zero. Isto não corresponde completamente
à definição de intecral duplo que consideramos como limite de somas integrais
quando o maior diâmetro dos domínios parciais tendesse para zero( aqui seria
preciso fazer tender para zero, simultâneamente, A0^ e Apj). Entretanto, apesar
desta falta de ricor, o resultado está certo (isto 6, que a fórmula (3) é leeítima).
Poder-se-ia estabelecer esta fórmula rigorosa como para o integral duplo em
coordenadas rectangulares. Indicaremos que será estabelecida também no § 6
partindo de outras considerações (como caso particular da fórmula geral de
transformação de coordenadas num integral duplo).
INTEGRAIS MÚLTIPLOS 181

Com efeito, tem-se,


a: = pcos0, i/ = psen0,

f {x, y) — /[p co sS , psene] = / ’(6, p),

por conseguinte,
p 02(0)
J J / ( x , y )d a :d y = J ( 5 /[p c o s 0 , p se n e ]p d p )d 0 . (4)
D a <D:(0)

Exemplo — 1. Calcular o volume V do corpo compreendido entre a esfera

-j-
e o cilindro
x2_|_y2— 2a y - 0 .
Resolução — Poder-se-á tomar por domínio de integração a base do cilindro
.x2-j-y2— 2a y= isto é, o círculo do centro (0, a) e de raio a. Pode-se escrever
a equação deste círculo sob a forma x^-\-(y— a)2 = (fig. 298).
Calculemos um quarto do volume procurado V (metade está representao
na fig. 298). Tomar-se-á, eqtão, por domínio de integração o semi-círculo definido
pelas equações
í^= <Pi (í/) = 0, X= (f2{y) = y 2ay—yi,
y = 0, y = 2a.
A função sob o sinal soma é

2 = /(^ , y) = — —
Por conseguinte,

T-J( í 2a V 2ay—y2
l/4 a 2 — x2— ^2 dy.

Passemos a coordenadas polares G, p:


x = p c o s 0, í/ = p s e n 0.
Determinemos os limites de integração. Para esse efeito escrevamos a
equação do círculo dado em coordenadas polares: como
x2 + í/2 = p2^
í/ = p sen 0,

tem-se
p2— 2ap sen 0 = 0
ou
p = 2íiscn0.
182 CALCULO D IFEREN CIA L E INTEGRAL

A fronieira do domínio em coordenadas polares, escreve-se, pois, (fig. 299):

p = a>i(0)=O, p = ®2(Ô) = 2a séne, o = 0 , P=

A função a integrar transforma-se em


F ( e , p ) = V4a*—p«.
Obtém-se, por conseguinte:
n JC
2 2 a se n 6 2

T - $ ( I <ie=
0 0 0
71
2
= ~ j ^ [(4a®—4a»seiií 0)* /t_ (4 ,*)Vtj ^0 =
0

2
bá^ p 4
=— ^ (l-cos«0)d0 = ~ a8(3„«.4).
0
Exemplo — 2. Calcular o integral de Poisson

Resolução — Calculemos, primeiramente, o integral ** ^*dxdy,


D

F i g. 299.

endo o domínio de integração o círculo (fig. 300)


X*-f- y*eaií8.
Passemos a coordenadas polares 0, p:
2n R 2«

/ „ = 5 ( 5 e - P * p i p ) á 0 = - |- S «-•’*) áe = n ( l — -'*V
0 0 0 ó
INTEGRAIS MÚLTIPLOS 183

Fazendo lender R para infinito, isto é, fazendo crescer indefinidamente o


domínio de integração, tem-se
2n oo 2n R
j ( í e P*pdp)d0= lim J (J « ^*p^p)^®= ^ ^ ® ) = n.
í 'i*
0 0
' D_^nrk ív
0 0
n. R—
¥CO

Mostremos que o integral J J e ** dx dy tende para tt quando se


D'
alarga o domínio D' de modo que qualquer ponto do plano se encontre defi­
nitivamente em D' «convencionalmente escreveremos D' oo).

F i g. 301.
Sejam e i?2 a maior e a mais pequena distância da fronteira de D'
à origem das coordenadas (fig. 301).
Como e*’**” í'* > 0 para todo valor, tem-se

D'
ou
n (1 — e” ^^) * < 5 5 tf"'**—''* dxdy (1 — e” ^^).

Como D ' —^ co, tem-se i? i—>-oo e e os membros extremos


da desigualdade tendem para um só e único limite •tt. O mesmo se diga, pois,
do termo intermediário:

lim I I e-^^'-y^dxdy = n. (5)


D'—
¥oo

Suponhamos, em especial, que D' seja um quadrado de lado 2a e de


centro na origem. Ter-se-á
a a

^ ^ e~^^~y^dxdy= J J e~^^~y^dxdy =
^jy

“Í n n

5
e y^dxdy—
—a
J ^ J

—a
e dx^ dy.
184 CALCULO D IFEREN CIA L E INTEGRAL

Ponhamos fora o factor e do integral interno (o que é permitido,


porque não dei>ende da variável de integração x). Tem-se
a a

J J dy= J ^ j dx^ dy,


D' —a —a
n
Façamos ^ dx = Ba- É um número constante (depende sòmente de a)\
—a
tem-se, pois,

j J d x d y = I e-y^Bady = Ba j e~y^ dy.


D' -a _a
a

Mas este último integral é também igual a ( porque j dx =


—o
a

= I dy^ ; por conseguinte,


—a

D'

Passemos agora a limite fazendo tender a para infinito nesta igualdade


{D' alargou-se, então, indefinidamente):
a

lim f f dxdj/=-- lim = lim f f


D'->oo J J a->cx) 0-+00 L J J

—oo
Mas viu-se que (5)

lim f f dx dy — Tí.
D '-^ o o J J
D'
Por conseguinte.

[j

ou
00
1 dx =

Este integral encontra-se muitas vezes em probabilidades e em estatística.


Notemos que nos teria sido impossível calcular directamente este integral, dado
que a primitiva de não se exprime por meio de funções elementares.
INTEGRAIS MÚLTIPLOS 185

§ 6. Mudança de variáveis num integral duplo


{caso geral)

Consideremos um domínio D do plano Oxy limitado por uma


curva L. Suponhamos que as coordenadas x c y s&o funções de novas
variáveis u e v:
X = (p{u, u), v), ( 1)

onde as funções <p(u, v) e v) são unívocas, contínuas e possuem


derivadas contínuas num certo domínio D' que definiremos no segui­

mento. Corresponde, então, segundo as fórmulas (4) a qualquer par


de valores w e v, um único par de valores x c y. Suponhamos, além
disso, as funções <p e ^ tais que se der a jc e valores definidos do
domínio £>, lhes corresponde, então, valores determinados de m e v
segundo as fóriíHilas (1).
Consideremos o sistema de coordenadas cartesianas Ouv (fig. 302).
Resulta do que precede que a qualquer ponto P{x, y) do plano
Oxy (fig. 303) corresponde univocamente um ponto P ( « , v) do plano
Ouv de coordenadas m , v definidas pelas fórmulas (1). Os números u
e V chamam-se coordenadas curvilíneas de P.
Se no plano Oxy o ponto P descreve a curva fechada L que deli­
mita o domínio D, o ponto correspondente descreve no plano Ouv
uma curva fechada U que delimita um certo domínio £)': corresponde,
então, a qualquer ponto de D' um ponto de D.
Assim, as fórmulas (1) estabelecem uma correspondência biunívoca
entre os pontos dos domínios D e D \ ou, como se costuma dizer,
representam biunivocamente D sobre D \
Consideremos em E/ uma recta u = const. Em regra, as fór­
mulas (1) fazem-lhe corresponder no plano Oxy uma linha curva.
186 CALCULO DIFEREN CIA L E INTEGRAL

Do mesmo modo, corresponderá a qualquer recta v = const. do


plano Ouv uma certa curva no plano Oxy.
Cortemos o domínio D' pelas rectas u = const. e v = const. em
pequenos domínios rectangulares (não teremos em conta os rectângulos
que invadem a fronteira de D'). As curvas correspondentes do domínio
D cortam este último em quadriláteros curvilíneos (fig. 303).
Consideremos no plano Ouv o rectângulo As' limitadopelas rectas
u = const., lí + Au = const., v = const. v + Av = const.eo quadrilá­
tero curvilíneo correspondente a A^ no plano Oxy,
Designaremos as áreas dos domínios parciais igualmente por A^
e As, Tem-se, evidentemente,

às = àuàv.

As áreas As e As' são, em regra, diferentes.


Suponhamos dada em D uma função contínua

Z = j{x, y).

A todo o valor da função z = f(x, y) do domínio D corresponde


o mesmo valor de z = F ( m, v) em Z)', em que

F{u, v) = f[(p{u, v), i|3(u, y)].

Consideremos as somas integrais da função z no domínio D.


Tem-sê. evidentemente, a igualdade seguinte:

2 /(a:, i / ) A s = 2 ^ ( “ . v)As. ( 2)

Calculemos A;, isto é, a área do quadrilátero curvilíneo P1P2F3F4


no plano Oxy (ver fig. 303).
Determinemos as coordenadas dos vértices:

Pi {xi, yi),.xi = (p (u , v), 1/1 = ^ (u, v),


Pi {Xi, 1/2). ^2 = cp(u + Au, y), 1/2= Tl?(u + Au, y),
(3)
PiiXi^ys), a:3 = cp(u + Au, y + A y ) , y3 = T|3(u + Au, y + Ay),
P4 yò, x^ = (ç{u, y + Ay), j/4 = ip (u, y + Ay).

Assemelharemos no cálculo da área do quadrilátero P1P2P3P4.


os arcos P1P2, P2P3, P3P4. P4P1 a segmentos de rectas paralelas: substi­
tuiremos, além disso, os acréscimos das funções pelos seus diferenciais.
INTEGRAIS MÚLTIPLOS 187

Quer dizer, fazemos abstracção dos infinitamente pequenos de ordem


mais elevada que e Av. As fórmulas (3) transformam-se, então, em
X, = (p(u, v). yi = i|)(u, v).
5<p
X 2 = 9 ( ií, y) + - ^ A u ,
du du
(30
Xj = <p(u, v) + - ^ A u + - ^ Av, yj = 1|) (u, v) + - ^ A u + - ^ Av,
du dv du du

X4= <p(u, v) + - ^ A v , y4 = ’l’ (w. v) + - ^ A v .


dv dv
Sob estas hipóteses, o quadrilátero curvilíneo PxP2 PJPa pode ser
assemelhado a um paralelogramo. A sua área A5 é aproximadamente
igual ao dobro da área do triângulo P1P2P3. que se calcula aplicando
a fórmula correspondente da geometria analítica:
— t ó — ( * j— *s)fes — íl)| =

= I f i í , A p) ií.A:,-i» f ü Au + ü A »)I =
IV du dv / dv dv \ du dv / 1

Ü Ü A u Ap — ? ^ - ? Í . A u A u
du dv dv du

d(p dq)
d<p d»|) d<p dt|) du dv
Au Av = Au Av *).
du dv dv du dij} di|)
du dv
Façamos
d(p dç
du dv
= /.
d»j) d<|)
du dv

Por conseguinte,
As I/ 1As'. ( 4)

O determinante I chama-se determinante funcional ou jacobiano


(do nome do matemático alemão Jacobi) das funções (p (w, v) e ^ (u, v).
(*) O duplo traço vertical significa que se toma o valor absoluto do
determinante.
188 CALCULO DIFEREN CIA L E INTEGRAL

A igualdade (4) apenas é aproximada, dado que nos cálculos da


área às desprezamos os infinitamente pequenos de ordem superior.
No entanto, à medida que as dimensões dos domínios elementares às
e são mais pequenas, mais se aproxima da igualdade. A igualdade
tem lugar quando se passa a limite, tendendo os diâmetros dos domínios
elementares às e A' para zero:

|/|= lim
dlamAs'-^0 A s '

Apliquemos agora a igualdade obtida ao cálculo do integral


duplo. Em virtude da igualdade (2) pode-se escrever
2 / ( ^ . y) As í t í F (u, i ; ) | / | A s '
(a soma integral da direita estende-se a ly). Passando a limite quando
obtém-se a igualdade rigorosa
n / ( x , y ) d x d y = I I F(u, v ) \ I \ d u d v . (5)
D D'

Tal é a fórmula, de transformação das coordenadas em integral


duplo, Ela permite reduzir o cálculo dum integral duplo num domínio D

Fig. 304 Fig. 305

ao cálculo dum integral duplo num domínio D', o que pode simplificar
o problema. A primeira demonstração rigorosa desta fórmula deveu-se
a M. Ostrogradsky.
Nota — A passagem das coordenadas cartesianas às coordenadas
polares, examinada no parágrafo anterior, é um caso particular de
mudança de variáveis num integral duplo. Neste caso, tem-se w = 0,
V = p:
x=pcos9, y = pscnQ-
INTEGRAIS M ÚLTIPLOS 189

O arco de círculo A B (p = pO do plano Oxy (fig. 304) é repre­


sentado pela recta A 'B ' no plano OQp (fig. 305). O arco de círculo
DC (p = pi) do plano Oxy 6 representado pela recta D'C' do plano
oep.
As rectas AD e BC do plano Oxy estão representadas pelas
rectas A'£y e B'C' do plano OOp. As curvas Li e L , estão representadas
pelas curvas e L'^.
■ Calculemos o jacobiano da transformação das coordenadas car­
tesianas X e em coordenadas polares O e p :
dxdx
d0dp - p sen 0COS 0
/ =
p COS 0 sen 0
50 dp
= — p sen^ 0 — p cos^ 0 = — p-

Tem-se, pois, | / 1= p e
3 <i>2(e)
llf(x, y )d x d y = \{ p)pí^p)<í®-
D a O i (0)
VoIta-se a encontrar a fórmula esta­
belecida no parágrafo anterior.
Exemplo — Seja calcular o integral duplo

J J (y —x) I x d y
D
em que JD é o domínio do plano Oxy limitado
pelas rectas

y= j/ = x —3, y~- 1 ^

y = — g-*-!-5.
O cálculo directo deste integral é bas­
tante fastidioso, mas uma mudança de variá­
veis simples permite reduzir este integral à integração num rectângulo cujos
lados são paralelos aos eixos coordenados.
Façamos ^
u = y — x, v = y + - ^ x . (6)

Então, as rectas y = jc + l, y = x — 3 são representadas, respectivamente,


1 7
pelas rectas m = 1, « = — 3 do plano Ouv; as rectas y = — õ" * + - õ"*
1 y d d
y=a— 5-X+ 5 tem por imagens as rectas i; = — , i;=a5.
3 3
O domínio D será, pois, representado pelo domínio rectangular D' da
figura 306. Resta, pois, calcular o jacobiano da transformação. Exprimamos para
190 CALCULO D IFEREN CIA L fi INTEGRAL

esse efeito jc e em função de 1/ e v. Resolvendo o sistema de equações (6)»


obtém-se
3 ,3 1 .3
X = - — U + — V^ y = - u + - ^ V .

Por conseguinte.
dx dx 3 3
du dv 4 4 _3_
1=
dy dy 1 3 16 ■ 16
du dv 4 4
e o valor absoluto do jacobiano é | / | = — . Logo

J J (y-x)dxdy= J J [ ( + x “+ T ‘' ) “ ( ~ T “ + T ‘' ) ] t ‘^“ ‘^'' =

~ ^ í -^ududz;= ^ ^ u d u d v — — 8.
d"
3

§ 7. Calculo das áreas de superfícies


Seja calcular a área limitada por uma curva T traçada sobre
uma superfície (fig. 307); a superfície é dada pela equação z = f(x, y)
onde /(jc, y) é contínua e possui derivadas parciais contínuas.
Seja L a projecção de T sobre o plano Oxy. Designemos o domínio
do plano Oxy limitado por L por D.

Cortemos arbitràriamente D em n domínios elementares A^i,


A^2........ Tomemos em cada domínio elementar um ponto
arbitrário Pi (1^, y\i).
Corresponde ao ponto Pf um ponto sobre a superfície

r]h / ( Í m y]i)l
INTEGRAIS MÚLTIPLOS 191

Tracemos o plano tangente à superfície no ponto Aí|. Tem por


equação
Z — Zi = f^ (li, T)i) (x — li) + f y (li, Tli) {y — lli) ( 1)

(ver § 6, cap. IX, t. I). Delimitemos sobre este plano o domínio Aa^
tendo Asj como projecção sobre o plano Oxy, Consideremos a soma
de todas as áreas correspondentes Aoi

2 Aa<.
í=»i
O limite a desta soma quando o maior dos diâmetros dos
tende para zero será, por definição, a área da superfície:
n
a= lim 2 (2)
d i a m A a i “^ 0 i = i

Calculemos agora a área da superfície. Designemos por yi o


ângulo entre o plano tangente e o plano Oxy. Sabe-se de geometria
analítica que (fig. 308)
Asi = Aoi COS Yi
ou
A$i
Aoi =
COS Yi (3)

O ângulo Vi é também o ângulo entre o eixo Oz e a normal


ao plano (1). Tendo-se em conta (1) e por aplicação da correspondente
fórmula de geometria analítica, obtém-se:
1
COS Vi = ;
Vl + / ^ ^ ( 5 i , 'ni)+/y(?i» 'Hi)

Por conseguinte.
Aoi= v i . + r : a i , ti o+ t, í) así.

Substituindo esta expresão na fórmula (2), tem-se:

o= lim 2 V l + f i i h , T,i) + fy^ilt, r\i) ASi.


d i a m Asi-^O i = l

Como o limite da soma integral do segundo membro desta última


igualdade, é, por definição, o integral duplo

II + + (0 ) ^ tem-se, por fim.

"“[í ^‘+('fr+(í-) (4 )
192 CALCULO D IFEREN CIA L E INTEGRAL

Tal é a fórmula que permite calcular a área da superfície z = f(x , y).


Se a equação da superfície é dada sob a forma
x = ii{y, z)outmelbor y = X z),
as fórmulas correspondentes do cálculo da área transformam-se em

(3 ')

“ ! 1 &)

onde D ' e D" são domínios dos planos Oyz e Oxz sobre os quais se
projecta a superfície dada.

Exemplo — 1. Calcular a área da esfera


+ =
Resoluçõo — Calculemos a área do hemisfério superior

Z= l/i?2 — —J/2
(fig. 309). Tem-se:
dz X
dx y m — ifi — yi '
dz y
ày -|/ií2_a:2_j,2
Por conseguinte.

y dx ) dy ) y J t i - x i - y i - y fl 2_ a : 2_ y 2
INTEGRAIS MÚLTIPLOS 193

O domínio de integração é determinado pela condição

Logo, em virtude da fórmula (4), tem-se:


R
dx.
4 "= $ ( S

Para calcular este integral duplo, passemos a coordenadas polares.


A equação da fronteira do domínio de integração torna-se, então, em p = R,
Por conseguinte,
2n R
0= 2 J f — P*^p) <je= 2 i? J l- V Ã ã H ^ l? d e = 2fl Ji?d0=4nil*.
0 0
2n
= 2R j R de= istfíi.

Exemplo — 2. Achar a área da parte do cilindro


-j_ y2 =

cortada pelo cilindro

Resolução — Na figura 310 tem-se representado a oitava parte da super­


fície em questão. A equação da superfície é
; = l / a * —I*,
donde
dy -• -^ = 0 -
dx Va® —; :ã ’ dz

O domínio de integração é o quarto de círculo da equação


*^ + 2*<a2, ® > 0 , z > 0 .
Por conseguinte,
a y^a2—jc2 /a2-:«2
T ”- í ( J y jfe l* )'''— "]/a2—~x2
dx =

= a J dx = a2, a=8a2.

13
194 CALCULO D IFEREN CIA L E INTEGRAL

§ 8. Densidade de distribuição de matéria e integral duplo


Suponhamos, distribuído num domínio D uma certa matéria,
de modo que cada unidade de área deste domínio contenha uma certa
quantidade desta matéria. Falaremos no seguimento da distribuição
de massa, se bem que os raciocínios que se seguem, sejam válidos
quando se trata de distribuição de carga eléctrica, de quantidade de
calor, etc.
Consideremos um elemento de área arbitrário ^s do domínio D.
Seja Am a massa da matéria distribuída sobre este elemento. Chama-se,
então, à relação densidade superficial média da matéria no
domínio ^s,
Suponhamos agora que a área A5 se encerra em volta do ponto
P {x, y). Consideremos o limite lim . Se ele existir, dependerá,
em regra, da posição do ponto P, isto é, das coordenadas x t y.
É, pois, uma função /(P ) do ponto P. Chamaremos a este limite den-
sidade superficial da matéria no ponto P;

lim 4 ^ = /(/> ) = /( X , y). ( 1)


As~^0 As
Assim, a densidade superficial é uma função f(x , y) das coorde­
nadas do ponto considerado no domínio.
Tnversamente, supomos dado no domínio D a densidade super­
ficial duma certa matéria como função contínua f (P) = f (x, v); pede-se
para determinar a quantidade total de matéria M contida, em D.
Cortemos o domínio D em áreas parciais As^ (i = 1, 2, . . n) e
tomemos em cada área um ponto P i ; / (P,) representa, então, a
densidade superficial no ponto
O produto f (Pi) Asi representa, então, a menos de um infini­
tamente pequeno de ordem superior, a quantidade de matéria contida
na área As,-, e a soma

i= l
exprime aproximadamente a auantidade total de matéria distribuída
no domínio D. Ora, é uma soma inteeral para a função f (P) em D.
Obtém-se um valor exacto passando a limite quando
Por conseguinte (*),

lim ^ f{P i) A s i = l U ( P ) < ^ = l U i ^ ’ (2)


A sj - ^ 0 i= i D D

(♦) A espressão Asi 0 significa, aqui, que o diâmetro do elemento de


área Asi tende para zero.
INTEGRAIS MÚLTIPLOS 195

isto é, que a quantidade total de matéria no domínio D é igual ao


integral duplo sobre D da densidade f (P) = f (x, y) desta matéria.
Exemplo — Determinar a massa duma placa circular de raio R , sabendo
que a densidade sup:írficial f ( x , y) do material em cada ponto P {x , y) é
proporcional à distância do ponto (jc, y ) ao centro do círculo:.
/(z , y )= k l / a :2 + y2 .
Resolução — De acordo com a fórmula (2), tem-se:

Af = J ^ d x dy^
D

onde o domínio de integração D é o círculo +


Passando a coordenadas polares, obtém-se: •

2n R 1

M = k \ ppdpj d0 = A :2 n ^ •knR^.
0 0 (

§ 9. M omento de inércia dum a figu ra plana

Chama-se momento deinércia / dum ponto material M de


massa m em relação a um ponto O ao produto desta massa m pelo
quadrado da distância r do ponto M ao ponto O:
I = mr^.
O momento de inércia dum sistema de pontos materiais jiíu
ni2 , .... em relação a O é a soma dos momentos de inércia
dos diversos pontos do sistema:

1=
i= i

Determinemos agora o momento de


inércia duma figura material plana D.
Suponhamos D contido no plano Oxy.
Determinemos o momento de inércia desta
figura em relação à origem O supondo a
densidade superfcial constante e igukl à
unidade.
Cortemos D em áreas elementares Í^Si (/ = 1, 2....... ri) fig. 311).
Tomemos em cada elemento de área um ponto Pi de coordenadas
T]/. Chamamos momento de inércia elementar da área A A',- ao
produto da massa A5j pelo quadrado da distância r? = |f + Ti?:
196 CALCULO D IFEREN CIA L E INTEGRAL

e formemos a soma de tais momentos:

i= l
Ela define uma soma integral para a função f (x, y) =
no domínio D,
Definamos o momento de inércia da figura como limite desta
soma integral quando o diâmetro de cada elemento A5j tende para zero:

/o = lim
diam ASf-^o i= l
Mas o limite desta soma é o integral duplo H + 3^^) X dx dy.
Por conseguinte, o momento de inércia da figura £>. em relação à
origem das coordenadas, é
■^0= + l^)d x d y , ( 1)
D

sendo D o domínio definido pela figura plana dada.


Os integrais
/ * * = í í y^d xd y, ( 2)
D

Iyy — ^ ^ ^ d^ dy (3)

chamam-se, respectivamente, os momentos de inércia da figura D em


relação aos eixos Ox e
Exemplo— 1. Calcular o momento de inércia do círculo cheio D de
raio R em relação ao seu centro O.
Resolução — Segundo a fórmula (1), tem-se:

^ 0 = j j (3:2 + y2) dxdy.

Pa^^sando a coordenadas polares 0, p, a equação deste crfrculo trans­


forma-se em
p = i?.
Logo
2 jt R

nR*
^0= j ( j p^p^p)

Nota — Se a densidade superficial y não é igual à unidade mas


é uma função de x e y, isto é, y = y (x, y), a massa de área íiS i será
igual, a menos de um infinitamente pequeno de ordem superior, a
INTEGRAIS MÚLTIPLOS 197

Y (li, r\i)A Si e o momento de inércia duma figura plana em relação


à origem se transforma em

v)(3:^ + y^)dxdy. ( 1')

Exemplo — 2. Calcular o mamento de inércia da figura material plana D


limitada pelas curvas — x \ a; = 0, y = 0 em relação ao eixo Oy» sabendo
que a densidade superficial em cada ponto é igual a y (fig. 312).
Resolução.
1 V i-x 1
•2|í2
Vl-X
dx-
0 0 0

Elipse de inércia — Determinemos o momento de inércia duma


figura plana D em relação a um certo eixo OL passando por um

ponto O que tomamos para origem das coordenadas. Seja <f o angulo
formado pela recta OL com a direcção positiva do eixo Ox (fig. 313).
A equação normal da recta OL é
X sèn 9 — y COS (p = 0.
A distância r dum ponto qualquer M {x. y) a esta recta é igual a
r = I X sèn 9 — 1/ COS 9 |.
198 CALCULO DIFF^RENCIAL E INTEGRAL

O momento de inércia / da figura plana D em relação à recta OL


é, por definição,
/ = JJ dx dy = X5 (a: sèncp — 1/ COS dxdy =
D D

= sen^ ^ I I ^ dx dy — 2 sen q) cos ^ H xy dxdy cos^ V

Por conseguinte.
I = I sen (p — 21xy sen cp cos (p + Ixxcos (p, (4)

onde ^í/í/ = í I x^ dx dy é o momento de inércia da figura em relação


ao eixo y e Ixx = I í dx dy o momento de inércia em relação ao
D

eixo x; pusemos, além disso, I^y = H xy dx dy. Dividamos todos os


D
termos da igualdade (4) por /; obtém-se:

/cos . / sén cp Y
X

Tomemos sobre o eixo OL um ponto


A ( X . Y) tal que

OA = ^ .
V l
Corresponde a diversas direcções OL,
isto é. a diversos ângulos p, diversos valores /
e logo diversos pontos A. Procuremos o lugar geométrico dos pontos A.
Obtém-se, evidentemente,
1 1
X = —= cos cp, y = —z: sen cp.
V/ V l

Em virtude da igualdade (5). as quantidades X t Y estão ligadas


entre si pela relação
1 = Ixx^^ — 2 IxyX Y + ly y ^ ^ ‘ (^)
O lugar geométrico dos pontos Á (X, Y) é, pois, a curva do
segundo grau (6). Mostremos que é uma elipse.
INTEGRAIS MÚLTIPLOS 199

Tem-se a igualdade seguinte, chamada de Bouniakovsky (*)


(matemático russo):
(J x ^ d x d y )( ll y^dxdy)
OU

Jxx^yy lly > 0 .


Assim, o descriminante da curva (6) é positivo, o que mostra
que é uma elipse (fig. 314). Chama-se elipse de inércia, A noção de
elipse de inércia é fundamental em mecânica.
Notemos que os comprimentos dos eixos da elipse de inércia e
a sua disposição no plano dependem da forma da figura plana dada.
Como a distância da origem das coordenadas a um ponto arbitrário A
{*) Para demonstrar a desigualdade de Bouniakovsky, consideremos a
desigualdade evidente:

H [/ (*. í/) —^<p {x, !/)]* dx dy > 0,

onde X é uma constante. A igualdade não é possível senão quando / (x, y) —


— X<p (jc, y) = 0, isto é, se f {x, y) = X<p (jc, y). Se se suposer que
ter-se-á sempre uma desigualdade. Obtém-se, pois, desenvolvendo os parêntesis
sob o sinal de in ^ ra l:

J J /*(•*. y ) d x d y — 2X J J /(x , {/)q>(x, y ) d x d y +


D D

I I y)dxdy-;>0.
D

Consideremos a expressão do primeiro membro como função de X. É um


polinómio do segundo grau que não se anula: as suas raízes são, pois, com­
plexas, o que implica que o descriminante formado com os coeficientes do
polinómio do segundo grau é negativo, isto é, que
( J J /q) d l d y Y — J ^ p dx dy ^ J <p2 dxdy<^ 0
D D D

(55 /(pdxdyj < 55 /2 dx dy ^ ^ (f2 dx dy.

Ê a desigualdade de Bouniakovsky.
X
No nosso caso, / (x, y) = x, (p (x, y) —y, — const.
A notável desigualdade de Bouniakovsky intervém, constantemente, cm
matemáticas. Em muitas obras chama-se injustamente desigualdade de Schwarz.
Bouniakovsky publicou-a (bem como outras desigualdades importantes) cm 1859
c Schwarz em 1875.
200 CALCULO D IFEREN CIA L E INTEGRAL

da elipse é igual a y = , onde / é o momento de inércia da figura


relativamente ao eixo O A, ao construir-se a elipse, calcula-se fàcilmente
o momento de inércia da figura D em relação a uma recta qualquer
que passe pela origem das coordendas. Em particular, é fácil de ver
que o momento de inércia da figura é mínimo em relação ao eixo
maior da elipse e máximo em relação ao eixo menor.

§ 10. Coordenadas do centro de gravidade duma figura plana

Indicamos no § 8 do capítulo XII (tomo I) que as coordenadas


do centro de gravidade do sistema de pontos materiais Pi, P2, Pj^
de massas mi, 17 12 ....... rrin eram dadas pelas fórmulas

(*)
S». ■

Determinemos agora as coordenadas do centro de gravidade duma


figura plana £>. Cortemos estas figuras em áreas elementares muito
pequenas Se se suposer que a densidade superficial é igual à
unidade, a massa do elemento parcial será igual à sua área. Além
disso, se se suposer, em primeira aproximação, que toda a massa da
área elementar Aí,- está concentrada em qualquer um dos seus pon­
tos Pi (Çf, Tii), poder-se-á assemelhar a figura D a um sistema de
pontos materiais. Em virtude das fórmulas (1) as coordenadas do centro
de gravidade da figura serão, então, determinados aproximadamente
pelas igualdades:

a5,
i= i______ i=l______
i= n Vc i= n

S a5 í
í= 1 i= l

No limite, quando A*?; -> -0,as somas integrais dos numeradores


e dos denominadores definem os integrais duplos e obtemos fórmulas
exactas para o cálculo do centro de gravidade duma figura plana:

I J xdxdy J I ydxdy
D _______________ , D _______________
Vc- ( 2)
J i dxdy n ^dy
D
INTEGRAIS MÚLTIPLOS 201

Estas fórmulas, que foram estabelecidas para uma figura plana


de densidade superficial igual a um. subsistem para uma figura cuja
densidade fosse uma constante y.
Se a densidade superficial é variável:

T = Y (a:, y),

as fórmulas correspondentes tomam, então, a forma

11 Y (^. y ) x d x d y SI Y(^. y ) y d x d y
D _____________________________ .
yc-
11 Y (^. y )d x d y II Y(^. y )d x d y
D

As expressões
= 11Y (^. y ) ^ ^y
D

M x = M y i.x , y )y d x d y
D

são chámadas momentos estáticos da figura plana D em relação aos


eixos lç>y e Ox.
o integral J J 7 (x, y) dx ãy exprime a massa da figura considerada.

F i g. 315.

Exemplo — Determinar as coordenadas do centro de gravidade dum quarto


de elipse (fig. 315)

— + iíi = l
a2 ^ 62
supondo a densidade superficial igual a 1.
202 CALCULO D IFERENCIA L E INTEGRAL

Resolução — Segundo as fórmulas (2):

a a Va2-x2
í ( j xdy^dx — f ~\/a% — x ' ^ x d x
_0_____ 0_____________ ü
a 4" -4- nab
4

j ( í

4a

—41 nab. 3 ji ’

a a
í ( J ydy)dx
i/c=- ü Ab
1 3 jt

4
nab
§ 11. Integrais triplos
Consideremos um domínio V do espaço limitado por uma super­
fície S. Seja / (jc, y, z) uma função em que x, y, z são as coordenadas
rectangulares dum ponto do espaço, definida e contínua em K e sobre
a sua fronteira. Para fixar ideias, quando f(x , y, z ) > 0 . poder-se-á
supor que está função representa a densidade de distribuição duma
certa matéria em V.
Cortemos o domínio V arbitràriamente em domínios parciais Aui.
onde Aui representará igualmente o volume do pequeno domínio cor­
respondente. Tomemos um ponto arbitrário em cada Aui e designe­
mos por f (Pi) o valor da função f nesse ponto. Formemos a soma
integral ^
IifiP ô A v i (1)

e aumentemos o número de domínios parciais Avi de modo que os


seus diâmetros tendam para zero (*). Se a função / (jc, y, z) é contínua,
o limite das somas integrais (1) existe ídá-se aqui. ao limite, o mesmo
sentido que para os integrais duplos (**). Este limite, que não depende
nem do modo da divisão do domínio V nem da escolha dos pontos Pi,
é designado pelo símbolo 1 1 i / (P) dv e chama-se integral triplo.

(*) Chama-se diâmetro do domínio Ai;j à maior distância entre os pontos


da sua fronteira.
(**) Não demonstraremos este teorema de existência do limite das somas
integrais (teorema de existência de integrais triolos) que tem lugar para qualquer
função contínua num domínio fechado V (compreendendo a fronteira).
INTEGRAIS MÚLTIPLOS 203

Tem-se, então, por definição


. lim ^ f ( P d à v ,= ^^U (P )d v
diam A v i -►o y

^ ^^ f (P) dv = ^^ ^ f {x, y, z)dxdy dz. (2)


V V
Se se considera que / (jc, y, z) é a densidade especial da distri­
buição duma matéria num domínio V, o integral (2) dá a massa de
toda a matéria que se encontre em V.

§ 12. Cálculo dos integrais triplos


Suponhamos que um domínio espacial (tridimensional) V limitado
por uma superfície fechada S goza das seguintes propriedades:
1. Qualquer paralela ao eixo Oz que passe por um ponto interior
(isto é. não tangente à fronteira 5) de V corta a superfície S em dois
pontos;

F i g. 317.

2. Todo o domínio inteiro V tem por projecção sobre o plano


Oxy um domínio regular D (a duas dimensões);
3. Qualquer parte de V obtida cortando V por um plano paralelo
a um plano de coordenadas quaisquer (Oxy, Oxz, Oyz) goza igualmente
das propriedades 1. e 2.
Um domínio com três dimensões que goza das propriedades indi­
cadas, dir-se-á regular.
Tais são, por exemplo, a elipsoide, o paralelipedo recto, o tetrae-
dro, etc. Dá-se um exemplo de domínio irregular a três dimensões na
fig. 316. Neste parágrafo, apenas consideraremos domínios regulares.
204 CALCULO DIFEREN CIA L E INTEGRAL

Suponhamos ijue a superfície que limita o domínio V tem por


equação na sua parte inferior z = x U, >^) e na sua parte superior
z = ^{x, y) (fig. 317).
Vamos dar um processo de cálculo de um integral triplo l y
num domínio V para uma função de três variáveis / (jc, y, z), definida
e contínua em V. Suponhamos que a projecção D t V sobre o
plano Oxy é limitado pelas curvas

í/ = 9i(^ ). y = x = a , x = b .
Tem-se, então.
b (P 2 ( x ) \|) ( 3 C , y)

' ^ v = í l S [ í /(a ;, y . z)d z\d y]d x . (1 )


a (P i(x ) x (3 c, y)

Observemos que após integração em relação a z e substituição


dos limites nos parêntesis de (1), obtém-se uma função x t y.
Resta, então, um integral duplo sobre D que
se sabe integrar,.
Consideremos um exemplo de cálculo dum
integral triplo.
Exemplo — 1. Calcular o integral triplo da
função / (jc, y, z) =xyz no volume V limitado pelos
planos
x = 0, y = 0 , z = 0, x-\-y + z — í .

Resolução — Este domínio é regular. É limi­


tado sup^riormente e inferiormente pelos planos
z = 0 e z = I — z — y e a sua projecção sobre o
plano Oxy é um domínio regular D que é o
triângulo limitado pelas rectas x = 0, y = 0, y = l —jc (fig. 3Í8). Por conseguinte,
i- x - y

J J J xyz d z j da.

Introduzamos os limites de integração no integral duplo sobre o domínio D:


1 1—ac 1—ac—y
dx =

\-x
=j {1
1—ac
xyz^
7 = 1 —ac—y

2=ü
dyj- dx =

Consideremos agora algumas propriedades dos integrais triplos.


INTEGRAIS MÚLTIPLOS 205

Propriedade— 1. Se se cortar o domínio V em dois domínios


Vx e V 2 por um plano paralelo a um plano de coordenadas quaisquer,
o integral triplo em V é a soma dos integrais triplos em Vx e V 2 *
A demonstração desta propriedade é análoga em todos os pontos
dos integrais duplos. Não há lugar, pois, a repetição.
Corolário — Qualquer que seja a divisão do domínio V em
número finito de domínios Vx, V 2 ....... . tem-se

/ y = /v j + "í" •••
Propriedade — 2. (Teorema sobre a avaliação dum integral triplo).
Sendo m e M o mínimo e máximo valor de f {x, y, z) em V, tem-se
a igualdade
m V ^Iy^M V ,

onde V é o volume do domínio dado e l y o integral triplo de t(x , y, z)


em V.
Demonstração — Calculemos, em primeiro lugar, o integral interno
y)
no integral triplo J J / (x, y, z) dz\ d o :
D x(x, V)
^ (*, y) 'f (*. y) <*. w>
I í{x , y, 5 M dz = M \ dz =
X(x, y) X (x. y) X (x, y)
Mj ( x , y)
= Mz 1 =M[<lp(x, y ) - x { x , y ) l
y)

O integral interno não é, pois. superior à expressão M [ ^ (x , y ) —


—X >^)]* conseguinte, em virtude do teorema do § 1 sobre os
integrais duplos, obtém-se (designando por D a projecção de V sobre
o plano Oxy):

/^ = J J J f{x , y, z) dz] d a < í i' M[y\> {x, ij) —


D X ^ x , y ) -D

— %(x, i/)]da = M n y) — x(^.


D

Mas este último integral duplo é igual ao volume do domínio


compreendido entre as superfícies z = >^) ® z = ^ (x, y), isto é,
ao volume do domínio V, Logo, tem-se
ly ^ M V .
Demonstra-se duma maneira análoga que / y > m F. A proprie­
dade 2 está, assim, demonstrada.
206 CALCULO D IFEREN CIA L E INTEGRAL

Propriedade — 3. (Teorema da média). O integral triplo l y duma


função contínua /(jc, y, z) num domínio V é igual ao produto do
seu volume V pelo valor da função num certo ponto P do domínio:
b q>2 y)
! [ í f{ x , y, z)d z]d y ]d x = f{ P )V . ( 2)
V : (3 c) y)

A demonstração desta propriedade é análoga à da propriedade


correspondente dos integrais duplos [ver § 2, propriedade 3, fórmula (4)].
Podemos, agora, demonstrar ò teorema sobre o cálculo dos integrais
triplos.
Teorema — O integral triplo duma função / ( jc, y, z) num domínio
regular V tem por expressão
h <p2 ( x ) y)

í n / (^. y. z) = n í [ í f ( ^ , y, z)d z]d y}d x.


V a q>i(x) x(x, y)

Demonstração — Cortemos o domínio V por planos paralelos aos


planos de coordenadas em n domínios regulares:

Designemos, como acima, por l y o integral triplo de / (jc, y, z)


em V ' e por / a u ^. o integral triplo desta função no elemento de
volume /SkVi, Pode-se escrever, em virtude da propriedade 1 (do seu
corolário):
l y = /Ayj4- /Ay2+ • • • + I (3)
Transformemos cada termo do segundo membro, segundo a
fórmula (2):
l y = / ( P , ) ^V^ + / ( P 2) A i ;2 + . . . + / {Pn) Al ^n, ( 4)
onde Pi é um ponto de
Tem-se, no segundo membro desta igualdade, uma soma integral.
/ ( jc, y, z) é, por hipótese, uma função contínua no domínio V t o
limite desta soma, quando o maior diâmetro dos ^Vi tende para zero,
existe e define o integral triplo de / ( jc, v, z ) em V. Obtém-se, pois,
passando a limite na igualdade (4) quando o diâmetro 0:

= H í / (^. y. z) dv.
O U seja, ainda.
h (p 2 (a c ) \ |) ( x , y)

J j j / 2) = í I í [ í / (j c , y, z) dz\dy] dx,
V a <P|U) x(’c.
O teorema está demonstrado.
INTEGRAIS MÚLTIPLOS 207

Aqui z = x(^> y) e z = ^ (x, >^) são equações das superfícies que


limitam o domínio regular V inferiormente e superiormente. As curvas
y = (Pi (x), y = (f2 W , X = a, y = b delimitam o domínio D, projecção
de V sobre o plano Oxy,

Nota — Tal como para os integrais duplos, pode-se formar inte­


grais triplos com ordens diferentes de integração em relação às variáveis
e com outros limites, se porventura a forma do domínio o permitir.

Cálculo do volume dum corpo por meio dum integral triplo — Se


a função V integrar é f ( x / y , z) = 1, o integral triplo no domínio V
exprime o volume V deste domínio:

V = l l l d x d y dz. (5)
V

Exemplo — 2. Calcular o volume do elipsóide

a2^ ^ c2

Resolução — O elipsóide (fig. 319) é limitado inferiormente pela superfície

2 = _c 1/ 1__— ____— c superiormente pela superfície z = c 1 / 1 — —— — .


" V r 1)2
_2 jj2
A projecção deste elipsóide sobre o plano Oxy (domínio D) é 2l elipse =
208 CALCULO D IFEREN CIA L E INTEGRAL

Tem-se, pois, reduzindo ao cálculo dum integral triplo:

-í a

í I í ^ dx =

■ v< -^
-2. j j ____ y fl2 52 dy dx.

Quando se calcula o integral interno considera-se x constante. Façamos


a mudança de variável

y = 6 |/ ^ 1 — ^ sen í, dy = b j / ^ 1— - ^ c o s t d L

^2 Jl
A variável 3^ vai de — 6 1 / 1------- a 6 1 / 1------ ^r-, pois t varia d e ---- —
r w 2
a . Substituindo estes novos limites no integral, obtém-se:
n

V -2 . I [ j
—a Ji
2
n
2
X b J / 1— COS í dí J d x = 2c6 — J c o s^ /d íjd x ^
—a X jx
” 2
a
Anabc
= j (a2 —x2) (il = -

Assim,
F = — Jia6c.
o

Se a = b = c, obtém-se o volume da esfera:

V = fn a ^ .

§ 13. Mudança de variáveis num integral duplo


1. Integral triplo em coordenadas cilíndricas — Chamam-se coor­
denadas cilíndricas aos três números 0, p, z que definem a posição
num ponto P do espaço, sendo 6 e p as coordenadas" polares da pro-
INTEGRAIS MÚLTIPLOS 209

jecção do ponto P sobre o plano Oxy e z a cota de P, isto é, a sua


distância ao plano Oxy tomado com o sinal mais se o ponto se encontra
acima do plano Oxy e com o sinal menos no caso contrário (fig. 320).
Corte-sc o domínio espacial dado V em volumes elementares
pelas superfícies de coordenadas 0 = 0^, p = pj, z = (semi-planos
que contêm o eixo Oz, cilindros circulares de eixo Oz, planos perpen­
diculares a OzX Um volume elementar é. então, um «prisma» curvilíneo
(representado na fig. 321). A área da base deste prisma é igual, a menos

de um infinitamente pequeno de ordem superior, a pA0Ap, sendo a


sua altura ^z (omitimos os índices /, /, k para abreviar a escrita). Tem-se,
pois, Av = pA0ApAz. O integral triplo da função F (0, p, z) no domí­
nio V escreve-se, então.

/ = H Í ^ ( 0 . P- z )p d e d p d z. (1 )
V

Os limites de integração são determinados pela forma do domínio V.


Se o integral triplo de f(x , y, z) é dado em coordenadas rectan-
gulares. é fácil dar a sua expressão em coordenadas cilíndricas. Com
efeito, tendo em consideração que ^

x = p co s0 ; y = p se n 0 ; z = z,
obtém-se:
z ) d x d y d z = l l l F ( Q , p, z )p d 0 d o d z,
V y
onde
/(p c o s0 , psen0, z) = F(Q, p, z).
Exemplo — Determinar a massa M dum hemisfério de raio R e âc
centro na origem das coordenadas, sabendo que a sua densidade F é propor-
porcional em cada ponto {x, y, z) à distância deste ponto à base: F ^ kz>
14
210 CALCULO D IFERENCIA L E INTEGRAL

Resolução — A equação do hemisfério superior

Z = — y 2

escreve-se em coordenadas cilíndricas


—p2.
Por conseguinte,
2n R V m -pz

J J J í [ 1 ( j Ajzííz j p d p j d0 =
V 0 0 0
2n R V R 2-p2 2nR

“ K í'^
0 0
I P‘*p]‘í®= í [ f
0 0 0
2n
4

2. Integral triplo em coordenadas cilíndricas, — Em coordenadas


esféricas, a posição dum ponto P no espaço é definida por três números

Fig. 323

0, r. <p em que r é a distância do ponto à origem das coordenadas,


chamado também raio vector do ponto, <p o ângulo entre o raio vector
e o eixo 02 e 0 o ângulo entre a projecção do raio vector sobre o
plano Oxy e o eixo Ojc calculado no sentido trigonométrico (no sentido
contrário dos ponteiros de um relógio) (fig. 322). Tem-se, para qualquer
ponto do espaço:
0< r< oo, 0<q><n; O<0<2n.
INTEGRAIS MÚLTIPLOS 211

Cortemos o domínio dado V em elementos Av por superfícies


de coordenadas r = const. (semi-plano que passa pelo eixo Oz)- A menos
dc um infinitamente pequeno de ordem superior, pode-se considerar que
o domínio elementar Av é um paralelepípedo de arestas Ar^ rAq),
r sen ^A0. O volume elementar exprime-se, então, (ver fig. 323):
Au = r^sén cp Ar A0 Aq).
O integral triplo da função F (0, r, (p) no domínio V, escreve-sc
r, (p)r"sénq)drd0dq). (!')
V

Os limites de integração são determinados pela forma do domínio V.


Deduz-se, fàcilmente, da figura 323, as expressões das coordenadas
cartesianas em função das coordenadas esféricas:
x = r sen q)cos0,
^ = rsen (p sen0,
z = rcosq).
A fórmula que permite passar de um integral em coordenadas
cartesianas a um integral em coordenadas esféricas é, pois,

5 n / ( ^ . y. z )d x d y d z =
V

= J n / [^ s®” COS0, r sen 9 sen 0, r cos cp] r^sen (p dr dQ dqi.


V

3. Mudança das coordenadas gerais num integral triplo — A pas­


sagem d e s o rd e n a d a s cartesianas para coordenadas cilíndricas ou
esféricas num integral triplo é um caso particular da transformação
geral das coordenadas no espaço.
Suponhamos que as funções
x = ^ { u , t, w),
j/ = iHu, t, w),
z = X (“ . ^

representam biunivocamente o domínio V em coordenadas cartesianas


X, y, z no domínio K' em coordenadas curvilíneas u, t e w.
Suponhamos que o elemento de volume Av do domínio V é
representado pelo elemento Av' do correspondente domínio V ' e seja

Aü -►o A v
212 CAIXrULO DIFEREN CIA L E INTEGRAL

Então,
y. Z )d x d y d z =
V

= J J Í /[ q ) ( u , t, w), iH m, t, w), x (u , t, w )] \I\d u d td w .

Do mesmo modo que para os integrais duplos, aqui ainda I


se chama jacobiano da transformação: do mesmo modo que para os
integrais duplos, mostra-se que o jacobiano está representado pelo
determinante de terceira ordem:

dx dx dx
du dt dw
dy ày dy
/ =
du dt dw
dz dz dz
du dt dw

Assim, no caso de coordenadas cilíndricas:


X— p co s6 , y = p se n 0 , z = z (p = u, 0 = í, z = u > ) ;
COS0 — psen0 0
/= sen0 pco s0 0 =p.
0 0 1
Em coordenadas esféricas:
x = r s e n <pcos0, y = rsen <ps«i0, z = rcoscp {r = u, ==.í, 0 = u>);
senq)cos0 rcos(pcos0 — rsen<psen0
/= senq>sen0 rcos(psen0 rsencpcos0 = /^sen (p.
COS (p — r sen q) 0
§ 14. Momento de inércia e coordenadas do centro
de gravidade dom corpo
1. Momento de inércia dum corpo — Os momentos de inércia
dum ponto material M (x, y, z) de massa m em relação aos eixos
coordenados Ox, Oy, Oz (fig- 324), exprimem-se, respectivamente, pelas
fórmulas ^ , 2 . 2.
Ixx = (y + ^ ) m ,
lyy = (•** -|- z^) m, Izi = (x* -}- y*) m.
INTEGRAIS MÚLTIPLOS 213

Os momentos de inércia dum corpo exprimem-se pelos integrais


correspondentes. Assim, o momento de inércia dum corpo em relação
ao eixo Oz exprime-se pelo integral iH X 7
y
y, z) dx dy dz, onde y {x, y, z) é a, densidade da matéria.
Exemplo— 1. Calcular o momento de inércia dum cilindro circular recto
de altura 2h e de raio R em relação a um diâmetro da sua secção média,
sendo a densidade Yo constante.

Fig. 324 Fig. 325


Reeoluçõo — Escolhamos um sistema de coordenadas como se segue: iden­
tifiquemos o eixo Oz com o eixo do cilindro e tomemos a origem no centro
de simetrià^s({ig. 325).
O problema reside em procurar o momento de inércia do cilindro em
relação ao eixo Ox:
^xx = í n (y^+^^)yodxdydi.
V
Passemos a coordenadas cilíndricas:
2n R h
^X X = Yo í { I [ 1(**+p® **“* ®) p <í>j- <Í0=
2n R
= Yo I 1 1 [ - ^ + 2Ap2 sen* o j p d p | dB =

2n
2A3 R2 2hR^
= ,H sen* 6 I d e=
4
214 CALCULO D IFEREN CIA L E INTEGRAL

2. Coordenadas do centro de gravidade dum corpo — Tem-se


fórmulas análogas às do centro de gravidade das figuras planas dadas
no § 8. cap. XII, tomo I;

SH y* z )d x d y d z
V _____________________________ .
x^ =
H í yi^y Vy z )d x d y d 2 ’

S H yyi^y Vy z )d x d y d z
yc= -
í H V (^. y. 2) f^y '

ZY (x, y, z )d x d y d z
V
z, =
í n Y (^. y. 2) dx dy dz
V

onde y{x, y, z) é a densidade.


Exemplo — 2. Determinar as coordenadas do centro de gravidade da
metade superior duma esfera de raio R e de centro na origem. Considera-se
constante a densidade yo.
Resolução — O hemisfério é limitado pelas superfícies

*= z =. 0.
A cota do centro de gravidade é dada pela fórmula
í H *V0 àx dy dz
V

I I l y o d x d y dz
V

Passando a coordenadas esféricas, tem-se:

ac = -
2ji
Yo Híü 2

0

0
r COS q)ra sen (p d r j d0

2n 2 R

Yo Ç ^ t q) dr J áq) I" d0
b b 0

_ -T 3
~ 2 P3 ■" 8

Em virtude da simetria do hemisfério, tem-se, evidentemente, Xc = yc = 0.


INTEGRAIS MÚLTIPLOS 215

§ 15. Integrais que dependem dum parâmetro

Consideremos o integral seguinte, dependendo do parâmetro a:


b
^ (a) = í / (^. a) dx.
a

(Considerámos tais integrais no § 10, cap. XI, tomo 1). Indiquemos


sem demonstração que se a função f { x , a) é contínua em relação a x
no segmento [a, b\ e em relação a a no segmento [ai, «2]» a função

I ( a ) = \ í { x , a)dx

é contínua no segmento [au ccz]. Poder-se-á, pois, integrar a função / (a)


em relação a a no segmento [«i, a2]:

j / ( a ) d a = J (S /(a:, a )d x )d a .
0C{ a
A expressão do segundo membro é o integral duplo de f(x, a)
no rectângulo correspondente do plano Oxy, Pode-se inverter a ordem
de integração:
tt2 6 h
y ( y / ( x , a )d x )d a .= l ( í / ( x , a )d a )d x ,
cti a a c tx

o que mostra que basta integrar em relação ao parâmetro a sob o


sind soma. Esta fórmula serve também para o cálculo de certos inte­
grais definidos.
Exemplo — Calcular
00
dx (a > 0 , 6}>0).

Não se sabe calcular este integral por meio de funçSes elementares.


Mas partamos do seguinte integral, fácil de calcular:

J (o > 0 )

Integrando esta igualdade de a = a a a = ò, obtém-se:

a 0 a
216 CALCULO D IFER EN C IA L B INTEGRAL

Mudando a ordem de integração, no primeiro membro, tem-se:


oo b

Í [ i *"“* * ']
b a
Calculando o integral entre parêntesis rectos, obtém-se:

í
oo
e -a x ^ g -b x h
-----------------d x = Log-----.
X

Exercidos
^ a

Calcular os integrais (*):


1 2

1. j J (x^ + y i)d x d y . Reip.


0 1
4 2
2.
íí
- - (®+y)® ■
3 1
dy dx T o g 25
«Resp. L -^

2 xVZ
3. J j xy dx dy , Re*p. — .
1 X
2n a

4. f f r dr d0. Resp. - ^ jia 2 .


0 a sen 6

=-íí U Jt

0 x^
X dy dx
xa + ya •
p 3Xfl . 1
Resp. — ---- a arc tg — .
4 ® fl

ífV5 0 v —a
xydxdy, Resp.
24 •
Ji
b 2
7. j I p d 0dp. Resp.

(*) Como indicamos mais adma, a ordem de integração em


N L

j j ^ dx é a dos diferenciais, isto é, que


^ ^ N L N L

J J/(*> í ( f/(ft v)^\dy.


M K M k
INTEGRAIS MÚLTIPLOS 217

Definir os limites de integração para o integral ^ ^ / (x, y) dx dij, sendo

o domínio limitado pelas curvas:

3 5
8. x = 2y x = 3 , í / = — 1, i/ = 5. Resp. j j / (x, y) dy dx.
2 -1
1 í-x2
9. y = 0, i/= l —x2. R esp. J j / (x , I/) d l/ d x .
-1 0
a Va2-x2
10. x2 + z/2 = a2. Resp. j j / (x, y) dy dx.
- a _ Ya2-x2

1 1+3C2

í' ~ T + Í 2 > v = ^^- j [ /( ^ . y)dydx.


-S Xi
a i/-f2a
12. y = 0, y = a, i/ = x, y = x — 2a. Resp. j j / (i, y)dxdy.
0 V
inverter a ordem de integração nos seguintes integrais:

2 4 4 2
, s . J Í f( x , y)dydx. Resp. f Ç / (a;, y)dxdy.
1 3 3 *1
1 Vx _ 1y i
I j / (*. y) dy dx. Resp. ^ j ^ l')
0 x3 b Í/2
a V 2 Õ y '^ 2 a a

j í /( * ' V)dxdy. Reg?. j j / (a:, y) dy dx.


° ® 0 o_ /aa_5cj
1 V1-** 1 y~ r^
J J /(*> y)dydx. Req>. \ \ /(* , y)dxdy.
-1 0 i -v^T qfã
1 1-p 0
J J
0 —/ l_y2
/(* . y)d x d y . Resp.
-1
j j
0
/ (®. y)dydx-\-

1 1-X

+ j j i ( x , y) dy dx.
0 0
218 CALCULO D IFERENCIA L E INTEGRAL

Calcular os seguintes integrais passando a coordenadas polares:


ji
a x2 2 a
<8. j J V « . _ a ;2 — y2dydx, Resp. J f P^P
0 0 0 0
Ji
a Va2-y2 2 a
19. J ^ ( x ^ y ^ ) dx dy, Resp. J ^ p3dpd0 = -^!^
0 0 0 0
Ü2
oo oo oo
20. j j da;. Resp. ^ ^ g” ^*p dp dO = - ~ .

2a V 2ax—x2 2 2a cos 0
21. I j dydx, Resp. | j pdpd0 = i ^ .

Transformar os seguintes integrais duplos introduzindo as novas variáveis u


e V, ligadas pelas fórmulas x = u — uv, y = uv:
3 e
« 3* 1+3 l-i>
22. 1 f / (a;, y) dy dx, Resp. f f f (u — uv, uv) u du du.
0 CMC a ,0J
1+q
b c
b-j-c i —v
23- j j / (x, y) dff dx. Resp. j j f (u— uv, uv)ududv~r

1 V

+ J — uv)ududv.
b 0
5+c

Aplicação do integral duplo ao cálculo das áreas


24. Calcular a área da figura limitada pela parábola y* = 2jc e a recta y = x.
Resp. I - .
25. Calcular a área da figura limitada pelas cuvas y* = Aax, x + y =? 3^, y = 0.
Resp.

26. Calcular a área da figura limitada pelas curvas ^1/2 ^


Resp.
3 •
INTEGRAIS MÚLTIPLOS 219

27. Calcular a área da figura limitada pelas curvas y = sen x, y = cos x, x = 0.


Resp. 1 /2 — 1.
28. Calcular a área do arco da curva p = a sen 20. Resp. .
o
29. Calcular a área limitada pela lemniscata cos 2tp. Resp. a^.
x2 \ 2 2xy
30. Calcular a área da «boucle» da curva ( ——
[ a2 ^ b2 J c2 •
Indica
Indicação — Passar as novas coordenadas x = pa cos 0 e y = pb sen 0.
Resp.
a2b2
■ê2~*
Cálculo de volumes
Calcular os volumes dos corpos limitados:
31. Pelas superfícies ± + + x = 0, y = 0, z = 0. Rép.
32. z = 0, x + í/ + 2 = 3. Resp. 3jt.
Resp.
33. (x — l ) 2 + = i^ xy = z, Z = 0. Resp. Jt.
(y _ l)2
QO
34. x2_l_y2— 2flX= 0, z = 0, Íc2-|-y2=:z2, Resp. —^ a ^ .

35. y = x2^ X= y2^ Z= 0, Zz=i2 + y — x2, Resp. .


140
36. Pelos planos coordenados, o plano + 3y — 12 = 0 e o cilindro z = - ^ y2,

Resp. 16.
37. Pelo cilindro circular recto de raio a e cujo eixo se identifica com Oz,
X Z / 3X 1 \
os planos coordenados e o p la n o ---- 1----= 1. Resp. (-7-------- I .
a ' a V4 3 /
38. Pelos cilindros ®2-|-j,2 =a2, xi-\-z^ = a2. Rép. — a*.
3
39. y 2 ^ z 2 = x, x — y, z = 0. R e s p . .
d4
40. x2-\-y2j^z2 = a2^ x 2 + y 2 = R 2^ fl > i?. Resp. A ji [a^-~Ç]/a 2 — R 2)^ ,

41. az = x 2 -{-y2 ^ z = 0, x , ^ = 2ax, Resp.


42. p2 = a2cos20, x 2 y 2 ^ z2 z=a2^ z = 0. (Calcular r» volume interior do
(cilindro.) Resp. ( 3 jx + 20— 16 V ^)-

Áreas de superfícies
43. Calcular a área da parte do cone = ^2 cortada pelo cilindro
x2-\-y2 = 2ax, Resp. 2na2 I /2 .
44<, Caiw'dar a área da parte do plano ;c + y + z = 2^z que se encontra no
primeiro triedro formado pjlos eixos coordenados e limitada pelo cilindro
+ y2 = a 2 . Resp. 1/ 3.
220 CALCULO DIFEREN CIA L E INTEGRAL

45. Calcular a área do segmento esférico (do pequeno), sendo o raio da esfera a
e o raio da base do segmento b. Resp. 2jx (a^— a "l/a2 — b^),
46. Calcular a área da parte da esfera + q u e é cortada pelo
cilindro = 1 (a > 6). Resp. — Sa^ — arc sen
^ ' a
47. Achar a área da superfície do corpo que é formado pela intersecçâo de
dois cilindros = y ^ -\-z ^ = a^, Resp. 16 í| 2.
48. Calcular a área da parte da süperfície do cilindro = l a x compreen­
dida entre o plano z = 0 e o cone Resp. %à^.
49. Calcular a área da parte da superfície cilíndrica compreendida
entre os planos z = m x e z = 0. Resp. 2m á^.
50. Calcular a área da parte do parabolóide jc^ + z^ = 2ajc compreendida entre
o cilindro parabólico y^ = «jc e o plano x = a. Resp. J - jia^ (3 — 1).
3
M assas, centro de gravidade e m o m e n to de inércia
de figuras planas

(Suporemos, nos problemas 51 a 62 e no problema 64, que a densidade


superficial é constante e igual a 1)
51. Qual é a massa de um disco circular de raio a sabendo que a densidade
em cada ponto P é inversamente proporcional à distância ao centro
(designar-se-á por K o coeficiente de proporcionalidade). Resp. traK .
Calcular as coordenadas do centro de gravidade dum triângulo equilátero.
52. Identificar-sc-á o eixo O x com a altura e o vértice com a origem.
Resp. x = ^ - ^ , y = 0.
53. Encontrar as coordenadas do centro de gravidade dum sector circular de
raio a. Identificar-se-á a bissectriz do ângulo ao centro (2a) com o
eixo Ox. Resp. Jg — — ^ “ , ííg = 0.
54 . Encontrar as coordenadas do centro de gravidade do semi-círculo superior
da equação x^ + y^ = d^. Resp. Xc = 0, .
3ji
55. Determinar as coordenadas do centro de gravidade da área definida por um
arco de cicloide x = a (/ — sen 0, y = fl (1 — cos t \ Resp.
= aji, y^ = — .
6
56. Determinar as coordenadas do centro de gravidade da área da «boucle» da
curva = a ^ c o s lQ . Resp. = , yc = 0.
o
57. Achar as coordenadas do centro de gravidade da área interior à cardióide
p = a (1 + COS 0). Resp. ig = , j/g = 0.
58. Calcularo momento de inércia da área do rectângulo limitado pelas
rectas x = 0, x = a, y = 0, y = 6 em relação à origem de coordenadas.
Resp. a b (a ^± ^^
INTEGRAIS MÚLTIPLOS 221

3j2 ji2
59. Calcular o momento de inércia da elipse — +
a) em relação ao eixo Oy;
b) em relação à origem de coordenadas.

Resp. a),, na^b
— — ; b) nab( ^, 2„ +i 62).
—^ to\

60. Calcular o momento de inércia do círculo cheio p = 2a cos 0 em relação ao


3
pólo. Resp. — na*.

61. Calcular o momento de inércia da área da cardióide p = n:(l — cos 6) em
relação ao pólo. Resp.
35jia^
16
62. Calcular o momento de inércia do disco (x—a)2-f(y —6)2 = 2a2‘cm relação
ao eixo Oy. Resp. 3'Tra'*.
63. A densidade em cada ponto duma placa quadrada de lado a proporcional
à distância deste ponto a um vértice do quadrado. Calcular o momento
de inércia da placa em relação a um lado que passa por aquele vértice.
Resp. ■ ^ /2 + 3 L o g (V 2 + l)], onde Ar é o coeficiente de pro­
porcionalidade.
64. Calcular o momento de inércia da área da figura limitada pela parábola
w
= ajc e a recta x = u em relação à recta y = — a. Resp. — a*.
5
Integrais triplos
dx dy dz
65. Calcular
limitado
limita
5ÍÍ - sabendo que o domínio de integração é
pelos planos coordenados e o plano jc -I- y + z = 1.
Resp. Log 2 5
” 2 16 *
M ^ y
66. Calcular J ^ xyzdzj dx. Resp. .
0 0 0
67. Calcular o volume do corpo limitado pela esfera + y^ + z^ = 4 e o
parabolóide + y^ =3z- Resp. ji.
D

68 (*) Calcular as coordenadas do centro de gravidade e os momentos de inércia


X y z
da pirâmide formada pelos planos x = 0, i/ = 0, z = 0 ; — + — + — = 1 .
Resp. = v —— r — T r r T
4 ’ 4 ’ ^ 4 ’ * 60 ’ 60 ’ —
~
^9. Calcular o momento de inércia dum éone circular recto em relação ao
seu eixo. Resp. - L nhr* onde /i é a altura e r o raio do círculo da base.

(*) Nos problemas 68, 69, 72 e 73 supõe-se que a densidade é cons­


tante e igual à unidade.
222 CALCULO D IFEREN CIA L E INTEGRAL

70. Calcular o volume limitado pela superfície de equação (x2 + y2_^22)2 = a3x.
Resp. -1
à

71. Calcular o momento de inércia dum cone circular em relação ao diâmetro


da sua base. Resp. _ _ (2 fc 2 + 3r2).

72. Calcular as coordenadas do centro de gravidade limitado por uma esfera


de raio ^ e um cone de ângulo no vértice 2a, coincidindo o vértice com
3
o centro da esfera. Resp. Xc = 0, í/ c= 0, Zc = õ" ®(1 + cos o) (identificou-sei
o

o eixo do cone com o eixo Oz e colocou-se o vértice na origem).


73. Calcular as coordenadas do centro de gravidade do corpo limitado por
uma estera de raio a e por dois planos que passam pelo centro e formam
q jr
um ângulo de 60°. Resp. p = _ _ a ^ 0 = 0, (p = — (a recta de intersecção
iD ^
dos planos foi tomada para o eixo Oz, o centro da esfera serviu de origem
de coordenadas esféricas p, 0, <p).
oo
74. Sirva-se da igualdade f da ( a > 0 ) , para calcular os
-\/x V ji J
oo OO __ __
f COS X d x Ç sen X d x í ^ ^

J - T 7 T ‘ J - Í T T • 1 ^ T ' K t -
Capitulo XV

INTEGRAIS CURVILÍNEOS E INTEGRAIS DE SUPERFÍCIE

§ 1. Integral curvilíneo

Consideremos um ponto P(jc, >^) movendo-se sobre uma curva


plana L dum ponto M a um ponto N. O ponto P é solicitado por
uma força t^que varia em grandeza e em direcção quando P se desloca,
isto é, que ela é uma função das coordenadas de P:
F=F{P).
Calculemos o trabalho A da força quando o ponto é deslocado
de M para N (fig. 326). Cortemos para esse efeito a curva M N em n
partes arbitrárias pelos pontos Mo = M, Mi, M2, ...» = N par­

tindo de M para N e designemos por o vector Designemos


por F f a intensidade da força F no ponto M j. Pode-se, então, consi­
derar que o produto escalar F^Asf representa aproximadamente o
trabalho F ao longo do arco
A i ^ F iA Si.
Seja
F = X ( x , y ) i + Y{ x , y)J,
224 CALCULO D IFEREN CIA L E INTEGRAL

em que X (jc, y) t Y (jc, y) são as projecções do vector F sobre os


eixos Ox e Oy, Designando por e A//f os acréscimos das coorde­
nadas Xi e i/i quando se passa de a Afi+i, obtém-se:

A si = A x ii + Ayt j.
Por conseguinte.
F tA Si = X(Xi, yt) Axi + Y (x,-. yt) Ayt.
O valor aproximado do trabalho A da força F' ao longo da
curva MN é

A ÍV S = s [X (Xi, Ui) àXi + r (Xí, yt) Aí/i]. (1)


i=l i=i

Sem fazer raciocínios rigorosos, indiquemos atendendo que se


o limite da expressão do segundo membro existe quando (é.
então, evidente que Axi - í- 0 e Ayt 0), ele exprime o trabalho da
força F ao longo da curva L entre os pontos M e N:

^ = lim 2 [X {Xi, Ui) Ax; + Y (Xi, i/,) Aj/J. ( 2)


Axi-^O i=i
Ai/j-^0
O limite (*) do segundo membro chama-se integral curvilíneo
de X (x, y) e Y (x, y) ao longo da curva L e é designado por

A = l X { x , y ) d x - \ - Y (x, y) dy (3)

OR

A= 5 X ( x , y)d x + Y (x, y) dy. (3 ')


(Af)

Encòntra-se muitas vezes limites de somas (2) em matemáticas


e física, sendo X ( jc , y ) & Y ( x , y ) funções de duas variáveis no domínio D.
As letras M c N no' integral (3') foram postos entre parêntesis
para indicar que não são números mas os extremos da curva na qual
é estendido o integral curvilíneo. O sentido de M e iV ao longo da
curva diz-se sentido de integração.

* Dá-se aqui ao limite da soma integral o mesmo sentido que para


o integral definido, ver § 2, Cap. XI, t. I.
INTEGRAIS c u r v il ín e o s E INTEGRAIS DE SU PE R FÍC IE 225

Se L é uma curva empenada, define-se duma maneira análoga


o integral curvilíneo das três funções X {x, y, z \ Y {x, y, z), Z {x, y, z)
J X {x, y, z ) d x - \- Y {x, y, z) dy + Z (x, y, z) dz =
L
11
= lim 2 ^ i^h, Vk, h ) + y Uh, Z a ) A i//, +
A.v/i-^O f i ~ i
Ajfk^O
Azfi~>^0
+ Z { x ^ , yn, Z /,)A z /,.
A letra L sob o sinal soma indica que o integral é estendido
à curva L.
Indiquemos duas propriedades do integral curvilíneo.
Propriedade— 1. Um integral curvilíneo é definido pela expres­
são sob o sinal soma. a forma da curva de integração e o sentido
de integração.
O integral curvilíneo muda de sinal ao mesmo tempo que o
sentido de integração, dado que o vector As e. por conseguinte, as
suas projecções Ax e Aj mudam de sinal.
Propriedade — 2. Cortemos a curva L em
duas partes Li e Lo de modo que M N =
= M K ! KN (fig, 327). Resulta, então, directa-
mente da fórmula (1)
(N) (K)

J X dx ~ \-Y d y = J X dx
(Aí) (Aí)
(AO
+ Ydy+ J X d x + Ydy,
--- - (K)

parciaiís.
^Indiquemos ainda que o integral curvilíneo conserva o seu sentido
quan^do a curva L é fechada.
A origem e a extremidade da curva coincidem, então. Já não se
(V)
pode escrever no caso duma curvjr fechachr X dx + Y dy, mas
(i/)
I X dx -\ Y dy e será preciso mdkar, forçosamente, o sentido de
L
percurso ao longo da curva fechada L. Designa-se também frequente­
mente um integral curvilíneo sobre uma curva fechada L pelo símbolo
j ^ X dx f Y dy.
Nota — Fomos conduzidos à noção de integral curvilíneo consi­
derando o problema do trabalho duma força F sobre um percurso
curvilíneo L.
15
226 CALCULO D IFEREN CIA L E INTEGRAL

Considerava-se, então, que a força F era uma função vectorial


das coordenadas do ponto de aplicação {x, y); as projecções do vector
variável F sobre os eixos de coordenadas são iguais às funções escalares
(isto é, numéricas) X (x, y) e Y (jc, y). Pode-se, pois considerar um
integral curvilíneo da forma dx + Y dy como integral da função
vectorial F dada pelas suas componentes X c Y.
O integral da função vectorial F sobre a curva L é designado
pelo símbolo
iF d s .
L
Se o vector F é determinado pelas suas componentes X, Y, Z,
e^te integral escreve-se
] X dx Y dy Z dz.
L
Em especial, se o vector se encontra no plano Oxy, o integral
deste vector reduz-se, então, a
I X d x + Ydy.
L

Quando o integral curvilíneo duma função vectorial é estendido


a uma curva fechada L, chama-sc ainda a circulação do vector F
sobre o contorno fechado L.

§ 2. Ofcleulo do integral curvilíneo


Propomo-nos, neste parágrafo, precisar a noção de limite da
soma (1) § 1 e, do mesmo modo. teremos precisada a noção do inte­
gral curvilíneo e indicaremos um processo de cálculo.
Suponhamos a curva L dada sob
a forma paramétrica:
^ = <p(0. y = ^(t).
Consideremos o arco de curva M N
(fig. 328). Sejam a e p os valores do
parâmetro correspondente nos pontos
M e N. Dividamos o arco M N em par­
tes Asj pelos pontos Mi (Xu yO, Mg (Xz,
Fig 328 >'='>........ yn) e façom os X i =

Consideremos o integral curvilíneo definido no parágrafo anterior


j X (x, y)dx + Y (X, y) dy. ( 1)
INTEGRAIS CURVILÍNEOS E INTEGRAIS DE SU PE R FÍC IE 227

Enunciemos, sem demonstrar, um teorema sobre a existência dos


integrais curvilíneos. Se as funções <p{t) e tf/ (t) forem contínuas e pos­
suírem derivadas contínuas (p' (/) e (/) sobre o segmento [a, p] e se
as funções de t X [p (t), ^ (01 ^ ^ [<p(0> ^ (0 } forem contínuas sobre
segmento, os limites existem.

lim 2 ^ (^i. Vi) Aa:; = J X {x, y) dx,


( 2)
lim 2 y ( i i , y,) Aj/i = I X (íi y)dy
Ayi->0 i = l

sendo Xi e as coordenadas dum ponto do arco As^-. Estes limites que


não dependem do modo da decomposição da curva em arcos parciais
Asf quando As,; 0 , nem da escolha do ponto M i {Xi, yi) sobre o
arco Asj-, chamam-se integrais curvilíneos e designa-se-los por

lim 2 A" (Xi, yi) Axi = ^ X (x, y) dx,


Aa'/-^0 i= i L

n
lim
Aj/^-^O í = l
2 y (^h yd Ai/j =
L
y y {x, y) dy.

Nota — Resulta do teorema que tendem também para este mesmo


limite (isto é, para o integral curvilíneo) as somas definidas no parágrafo
anterior, onde os pontos {xi, y^) são as extremidades do arco As^-,
sendo arbitrária a decomposição de L em arcos parciais.
O teorema que acaba de ser formulado dá um processo de cálculo
dos integrais curvilíneos.
Assim, por definição:
(N)
J X {x , y ) d x = lim ^ X{xi, y;)Axi, (3)
(M) Axi->0 i = l

onde
AX; = Xi — = (f- (ti) — (p ( í i - ,)•

Apliquemos a fórmula dos acréscimos finitos de Lagrange


Axi = 9 (t i ) — (p (ti-i) = (p' (Ti) (t i — ti-^) = cp' ( t O Á ti,

sendo um certo valor de t compreendido entre os valores


e ti. Sendo o ponto Xi, ^i arbitrário sobre o arco A^,, escolhamo-lo
por forma a que as suas coordenadas correspondam ao valor do
parâmetro Xf:
í i = íp ^ ("Ti)-
228 CALCULO D IFEREN CIA L E INTEGRAL

Substituindo os valores encontrados de Xj, yi e na fórmula (3),


encontra-se:

J X {x, y) d x = lim X ^ [<P ('^í) 9 ' (t^í)


ÍM) Mi-^0i = l
O segundo membro representa o limite de uma soma integral
para a função contínua de uma só variável X (0. ^ (0] / ( O sobre o
segmento [a , /?].
Por conseguinte, este limite é igual ao integral definido desta
função:
(.V) P
J A’ (x, y)dx = ^ X[(p (t), (í)] (p' (t) dt.
(31) a

Duma maneira análoga, obtém-se a fórmula


(-V) p
J Y {x, i/)rfí/= J F [(p (0 , ^{t)]yl;i' {t)dt.
(M) a

Somando membro a membro estas igutádádes, obtém-se:


(-V)
I X (x, y ) d x + Y (x, y) dy =
(M)
P

= [fp (^). ^ (019 (0 + >^19 CO. ^ (0]^’ (0l (4)

Tal é a fórmula que permite calcular um integral curvilíneo.


Calcula-se do mesmo modo o integral curvilíneo
^ X dx -f- Y dy -j- Z dz
ao longo duma curva empenada definida paramètricamente: x = ^ ( t ) ,
y = (0 . z = x (0 .
Exemplo — 1.Calcular o integral curvilíneo a respeito das funções
3zy- (isto é, sobre a função vectorial
— x^y — x ' ^ y k ) ao longo
do segmento de recta que vai do ponto Af (3, 2, 1) ao p o n t o N ( 0, 0, 0) (fig. 329).
Resolução — Para encontrar as equações paramétricas da recta de inte­
gração, escrevamo-la sob a forma:
JL- —
2 ■ 1 ’

Designando por t o valor comum destas relações, obtém-se a equação


paramétrica da recta:
x = 3t^ y = 2t, z = t.

Corresponde à origem do segmento M N o valor do parâmetro / = 1 c


à extremidade o valor t = 0. Encontra-se, fàcilmente, as derivadas de x, y, z
INTEGRAIS c u r v il ín e o s E INTEGRAIS DE SU PE R FÍC IE 229

em relação a t (tem-se necessidade para calcular o integral curvilíneo):


—3, y^=2i, = 1.
Calcula-se, agora, o integral curvilíneo proposto com a ajuda da fórmula (4):
(V) 0
j x3dx-\-3zij^dy— x i y d z = ^ [(3<)® 3-l-3< (2 í)2 -2 -
(■'/)
U
87
(3<)*-2M] d<= j S7t^dt=— ^ .
4
Exemplo — Calcular o integral curvilíneo para o par de funções 6x^y,
1Gjc>'2 sobre a curva plana y = entre os pontos A í(l, 1) e N {2, 8) (fig 330):
Resolução — Para calcular o integral
proposto
(V)

f 6 x ^y dx-]-íO xy'^ d y

(M)
é preciso ter as equações paramétricas da
curva. É evidente que aqui x pode servir dc
parâmetro
x = x, y=-x^.

Fig. 329 Fig. 330


O parâmetro x varia de x i = I a Xt = 2, As expressões das derivadas
em relação ao parâmetro são:

Por conseguinte,
(N) 2
^ ^x'^y d x i O x y " ^ d y = ^ + =

I ^
= j (6x5_1_ 30i9) dx = [i« + 3x10)2 ^ 1084.
230 CALCULO D IFEREN CIA L E INTEGRAL

Vamos dar, agora, algumas aplicações dos integrais curvilíneos.


1. Expressão da área dum domínio limitado por uma curva em
função dum integral curvilíneo — Seja dado no plano Oxy um domí­
nio D limitado por uma contorno L tal que qualquer paralela a qual­

quer dos eixos coordenados que passa por um ponto interior do


domínio corte a fronteira L em dois pontos no máximo (isto é, que
o domínio é regular) (fig. 331).
Seja [a, b] o segmento do eixo Ox sobre o qual se projecta o
domínio Z>, limitado interiormente pela curva (/i):

y = yi (^).

e. superiormênte, pela curva

y = yz{^),
[í/i (^) < yz W].

A área do domínio D é, então, igual a

b b
s = ^ yz(x) dx — I yi (x) dx.

Mas o primeiro integral é um integral curvilíneo ao longo da


curva 1, {MPN), dado que y = y 2 {x) é a equação desta curva; por
conseguinte: ,,
I Uz (x) d x = I y dx.
M P N
INTEGRAIS c u r v il ín e o s E INTEGRAIS DE SU PE R FÍC IE 231

O segundo integral é um integral curvilíneo estendido à curva


h {MQN) : ^
lyi{x)dx= J yd x.
a MQn

Em virtude da propriedade 1 dos integrais curvilíneos, tem-se:

MPN N PM
Por conseguinte.
S = — ^ ydx— J y d x = — I ydx, (5)
N PM MQn l

sendo L percorrido no sentido inverso dos ponteiros dum relógio.


Se uma parte da fronteira L é constituída por um segmento M-M
(M)
paralelo ao eixo Oy, tem-se \ y dx = Q e a igualdade (5) é, ainda,
(Ml)
verdadeira (fig. 332).
Pode-se mostrar duma maneira análoga que
X dy. ( 6)
L
Juntando membro a membro (5) e (6) e dividindo por 2, obtém-se
ainda uma fórmula para calcular a área 5:

S = x d y — y dx. (7)
Exemplo — 3. Calcular a área da elipse
^ ' a: = a co sí, y — hstnt.

Resolução — De acordo com a fórmula (7), encontra-se:


2Jt

= y ^ [a COS t b COS t — bsen t (— a sén í)] d t ^ ::ia b .


b
Notemos que a fórmula (7) bem como as fórmulas (5) e (6)
se aplicam também para a área de domínios cujas fronteiras são
cortadas pelas paralelas aos eixos de coordenadas em mais de dois
pontos (fig. 333). Para o demonstrar, partamos o domínio dado
(fig. 333) em dois domínios regulares por meio da curva /*. A fór-
imlta (7) é verdSOeira para cada um dáes. Juntandò memhüo a mem­
bro, obtém-se no primeiro nwmbro a área do domínio dado e no
segundo o integral curvilíneo (precedido do coeficiente ^ ) estendido
a toda a fronteira, dado que o integral sobre a linha de divisão l*
é tomada duas vezes, no sentido directo e no sentido inverso, e
anula-se, portanto.
232 CALCULO D IFEREN CIA L E INTEGRAL

2. Trabalho duma força variável J f sobre um caminho curvi-


líneo L — Indicamos no começo do § 1, que o trabalho de uma
f o r ç a F = X {x, y, z) i ^ Y {x, y, z ) j - V Z {x, y, z) k ao longo duma
curva L = M N era igual ao integral curvilíneo:
( N)
A= í X ( x , y, z ) d x - { - Y (x, y, z) dy + Z {x, y, z) dz.
(M)
Tomemos um exemplo concreto do cálculo do trabalho de uma
força.
Exemplo — 4. Calcular o trabalho A da força de gravidade F que desloca
uma masa m do ponto M\ cj) ao ponto M2 («2» ^2» ^2) longo do
caminho arbitrário L (fig. 334).

Resolução — As projecções da força de gravidade F sobre os eixos de


coordenadas são
-T = 0, y = 0, Z = — mg,

O trabalho executado é, pois,


(M a) ('2

^ X dx-\-Y dy-\-Z d z — ^ (— mg) dz = mg (c^—


(Ml)

Vê-se que, no campo da gravidade, o trabalho não depende do caminho


seguido, mas sòmente do ponto inicial e do ponto final. Mais exactamente, o
trabalho da força de gravidade apenas depende da diferença dos níveis deter­
minados pelo ponto final e o ponto inicial.

§ 3. Fórmula de Green
Mostremos que um integral duplo num domínio plano D exprime-se
por um integral curvilíneo tomado ao longo da fronteira L deste
domínio.
Seja D um domínio do plano Ojc3^ limitado pelo contorno JL,
sçndo D regular, quer em relação a Ox quer em relação a Oy.
INTEGRAIS CURVILÍNEOS E INTEGRAIS DE SU PE R FÍC IE 233

Suponhamos este domínio limitado inferiormente pela curva y = yi(x)


e superiormente pela curva y = y\ i ^ ) < y 2 W < b)
(fig. 331).
Conjuntamente, estas curvas formam o contorno fechado L, Sejam
em D duas funções contínuas X (jc, y) e Y (jc, y) dotadas de derivadas
parciais contínuas. Consideremos o integral

Tem-se:
íí D
dX{x, y)
by
dxdy.

b
dx
( =
Viix)
D a yiix) a
b
= j [X (x, z/2 (x)) — X {x, i/i (x))] dx. (1)
a

Notemos que o integral

Í X ( x , y2Íx))dx
a
é numèricamente igual ao integral curvilíneo
I X{x , y)dx,
( M P N )

ao longo da curva MPN de equações paramétricas


x= = x, y = y 2 Íx),
sendo x o parâmetro.
Então, tem-se
b
^ X (x, 1/2(x)) d x = I X (x, y) dx. ( 2)
a M P N

De maneira análoga, o integral


b
J X (x, yt (x)) dx
a

é numèricamente igual a
b
í X (x, y^ (x)) dx = J X (x, y) dx. (3)
a (.m Q N )

Substituindo as expressões (2) e (3) na fórmula (1). obtém-se:

í X (x, y ) d x - j X{x,y)dx. (4)


M P N MQN
234 CALCULO D IFERENCIA L B INTEGRAL

Ora,
j X (x, y ) d x = — 5 X {x, y) dx
MQN NQM

(ver § 1, propriedade 1). Pode-se, pois, recopiar a fórmula (4) sob a


forma:

11 ^ j ^ (^. y )d x -\- j X {x, y) dx.


D M P N N Q m

Mas a soma dos integrais curvilíneos do segundo membro é


igual ao integral curvilíneo sobre o contorno L completamente percor­
rido no sentidodos ponteiros dum relógio. Pode-se, pois, pôr esta
igualdade sob a forma

í í = j X {x, y) dx, (5)

onde L indica que o contorno fechado L é percorrido no sentido


dos ponteiros dum relógio.
Se uma parte da fronteira é constituída por um segmento /&
paralelo ao eixo Oy, tem-se ^ X {x, y) dx = 0 e a igualdade (5)
permanece verdadeira.
Do mesmo modo se encontra:

^ ^ ^ d x d y = - ^ Y { x , y)dy. ( 6)
D L

Deduzindo (6) de (5). encontra-se:

D L

Se se percorrer o contorno L no sentido inverso dos ponteiros


de um relógio, tem-se (*(*)

D ^ L

É a fórmula de Green (matemático inglês, 1793-1841) (♦♦).


(♦) Quando num integral curvilíneo sobre um contorno fechado L,
não ss indica o sentido de integração, subentende-se que se irata do sentido
inverso dos ponteiros dum relógio. Se o percurso tiver lugar no sentido dos
ponteiros, é preciso ter cuidado em especificá-lo.
(**) Esta fórmula é um caso particular duma fórmula mais geral esta­
belecida pelo matemático russo M. Ostrogradsky.
INTEGRAIS CURVILÍNEOS E INTEGRAIS DE SU PE R FÍC IE 235

Suposemos o domínio D regular. Mas, tal como para o cálculo


duma área (ver §2). pode-se mostrar que esta fórmula permanece
verdadeira para qualquer domínio que admita um corte regular

§ 4. Condições para que um integral curvilíneo não dependa


do caminho de integração

Consideremos o integral curvilíneo


(N)
I X d x + Ydy,
(M)
estendido a uma curva plana L que reúne os pontos M t N Supor-se-á
que as funções X (x, y) Q Y (x, y) possuem derivadas parciais contínuas
no domínio D. Vejamos em que condições o integral curvilíneo não
depende da forma da curva L, mas sòmente da posição dos pontos
M e N,
Consideremos essas curvas arbitrárias MPN e MQN do domínio
considerado D reunindo os pontos M e N (fig. 335).
Seja
í X d x + Y d y = J Xdx-^Ydy, (1)
M P N m Q N

isto é.
í X d x-\-Y d y- I X d x + Y d y = 0.
M P N M Q N

Em virtude das propriedades 1 e 2 dos integrais curvilíneos (§ 1),


pode-se escrever
fi ^ X dx -{ -Y dy J X dx - \- Y dy = 0^
M P N n q m

que representa o integral curvilíneo sobre o con­


torno fechado L
Fig. 335 ^ X d x - \ - Y dy = 0. ( 2)

Nesta última fórmula, o integral curvilíneo é tomado sobre um


contorno L, constituído pelas curvas MPN e NQM. É evidente que
este contorno pode ser considerado como arbitrário.
Por conseguinter-cesuka 4a -condição de o ixUiegui sobfc mna
curva que reúne dok pontos arbitrários M e N não dqpender dò
caminho seguido, mas sòmente destes dois pontos, que este integral
é mdo sobre qmiqtter contorno fechado.
O recíproco é verdadeiro: se um integrsd curvilíneo é nulo qual­
quer que seja o contorno fechado, não dq>ende do caminho de
236 CALCULO D IFEREN CIA L E INTEGRAL

integração entre dois pontos, mas somente da posição destes dois


pontos. Com efeito, a igualdade (2) implica (1).
No exemplo 4 do § 2 o integral curvilíneo não depende do
caminho de integração: no exemplo 3 depende do caminho, dado
que o integral sobre o contorno fechado considerado não é nulo. mas
dá a área limitada por este contorno; nos exemplos 1 e 2 os integrais
curvilíneos dependem igualmente do caminho de integração.
Põe-se, naturalmente, a pergunta: a que condições devem satisfazer
as funções X (x, 3^) e y (x, y) para que o integral curvilíneo \ X dx +
Y dy seja nulo qualquer que seja o contorno fechado?
O teorema seguinte responde a esta pergunta.
Teorema — Sejam X (x, y) e Y (x, y) duas funções contínuas num
domínio £), bem como as suas derivadas parciais e
dy dx
Para que o integral curvilíneo sobre qualquer contorno fechado L
deste domínio seja nulo, isto é, para que se tenha
i X (x, y ) d x - \ - Y {x, y) dy = 0, (2 )

é necessário e suficiente que

dX _ d Y
dy dx (3)

em todos os pontos do domínio D.


Demonstração — Tomemos um contorno fechado arbitrário L num
domínio D e escrevamos a fórmula de Green correspondente a este
contorno:

D ^ L

Se a condição (3) for satisfeita, o integral duplo da esquerda é


idênticamente nulo e tem-se

^X dx-i-Ydy= = 0.

Demonstrou-se. pois, que a condição_(3) é suficiente.


Mostremos q u ê ^ la è nec^sâriSi i*to é, que le a iguaMade (2')
tiver lugar para qualquer, contorno fechado L em D, a condição (3)
tem forçosamente lugar em cada ponto do domínio.
BTTSORAIS CURVILÍNEOS E INTEGRAIS DE SU PE R FÍC IE 237

Suponhamos, pelo contrário, que tem lugar a igualdade (2'):

mas que a condição (3) não tem lugar, isto é, que

dx dy
se verificaria apenas um único ponto. Seja, por exemplo, num ponto
PUo. >^o)

dx dy

Como se tem no primeiro membro uma função contínua, ela


é superior a um certo número 8 > 0 em todos os pontos dum domínio
suficientemente pequeno £>' que contenha o ponto P(xo, Tomemos
dY d\
o integral duplo da diferença-------------s sobre este domínio. Ele é
dx dy
positivo. Com efeito.

í í ^ ^ ^ 6 d xd y = ò ^ ^ dxdy = bD' >Q.

^ O ia ; segundo a .fórmula de X3reen, o primeiro membro desta


fitim a desigualdade é-igual aa~integral curvflíneo sobre a fronteira U
do \ domínio D', que por bqjótese é nula. Logo. esta desigualdade
1 dY dX
contradiz a condição (T) e a suposição de que ---------- — é diferente
dx dy
de zero» não o seria senão num ponto» é falsa. Tem-se» pois»
dY
- ^ = 0
dx dy
em todos os pontos do domínio D.
O teorema está completamente demonstrado.
Mostramos, no § 9. cap. XIII. que a condição
d Y {x, y) d X {x, y)
dx dy
traduzia o facto de que a expressão X dx + Y dy é o diferended total
duma certa função u (x, y). isto é. que
X dx + Y dy = du {x, y)
238 CALCULO D IFERENCIA L E INTEGRAL

com

dy
Mas, então, o vector
, du . , du .
f’= x í + y i = ^ . + ^ -j

é o gradiente da função u {x, y); a fünção u {x, y) cujo gradiente é


o vector X i + Y j\ chama-se potencial deste vector. (n)
Mostremos que. neste caso, o integral curvilíneo 1= I
{M)àx dy
sobre uma curva anbitrária que reúne os pomos M e N é igual à
diferença dos valores da função u nestes pontos:
( N ) (N )

J X d x + Y d y = J du(x, y) = u ( N ) - u { M ) .
(M) (M)

Demonstração — Se X dx + Y dy é o diferencial total da fun


ção u (x, y), tem-se X — ^ , Y = ^ c o integral curvilíneo escreve-sc

( N )

, ,d u
dx+ -dy
~ ] dx
(M)
Para calcular este integral, escrevamos as equações paramétricas
da curva L que reúne M e N:

x= y = ^(t).

Admitiremos que corresponde ao valor t = to, d o ' parâmetro o


ponto Af e ao valor t = T o ponto N. O integral curvilíneo reduz-^e,
então, ao integral d^inido
r
du dy 1 ,
dx dt ^ dy d t \

A expressão entre parêntesis é uma função de t que exprime a


derivada total da função u[^(r), f (01 em relação a i.
INTEGRAIS CURVILÍNEOS E INTEGRAIS DE SU PE R FÍC IE 239

Por conseguinte.

= I ---- d t = u [<p(0 , 1*5(0] |ío =


J dt

= u [<f (0, 1|5(Oj — w [<p (<o), (<o)] = u{N) — u (M).

Vê.se que o integral curvilíneo dum diferencial total não depende


do caminho de integração,
O integral curvilíneo estendido a uma curva empenada goza da
mesma propriedade (ver §7).
Nota — Tem-se, por vezes, de integrar o integral curvilíneo duma
função X (jc, >^) em relação ao arco da curva de integração L\

í X {x, y ) d s = lim 2 ^ !//) (4)


L A s;-^0 i = l

sendo ds o diferencial do arco. Calcula-se estes integrais como os inte­


grais curvilíneos considerados acima. Suponhamos a curva L dada pelas
equações paramétricas
^=<í>(0 . !/ = ll’ (0 ,
sendo f(t). ^((), /(0> ^ ' ( 0 funções continuas de t.
Sejam a e os valores do parâmetro t correspondentes às extre­
midades do arco L.
Como
ds = (p’ { t f - f a|)' ( t f dt,

obtém-se a seguinte fórmula para calcular o integral (4):

5 X (z, y ) d s = l X [(f) (t), ij5(í)] V cp' ( t f + ij)' ( t f dt.


L CL

Pode-se considerar o integral curvilíneo em relação ao arco da


curva empenada x — <f{t). y = (0. z = x W

J X (x, y, z ) d s = ] X [<p (t), \|5 (t), X (í)] X

X K <p' (tf + {t f + X {t f dt.


Com a ajuda dos integrais curvilíneos com fclemento diferencial,
o arco ds, pode-se determinar, por exemplo, os centros de gravidade
de curvas pesadas.
240 CALCULO D IFEREN CIA L E INTEGRAL

y Raciocinando como no § 8, cap. XII, (tomo I,) obtém-se as


^guintes fórmulas para o cálculo das coordenadas do centro de gra­
vidade duma curva empenada:
I x ds lyds I zds
L L L
^c- , Vo- . (5)
L L L

Exemplo — Encontrar as coordenadas do centro de gravidade duma eq>ira


da hélice,
x = a c o s í, y = ascnt, z = b t (0 < ! í < 2 ji),
sabendo que a sua densidade linear é constante.
Resolução — Aplicando-se a fórmula (5), obtém-se:
2Jt ___________________
J a COS í ^+ cos2 í + 62 d í

2ji
J V«^sen2 í-f-a2 cos2 í + 62 dí
2n
J a COS í l / a 2 - | - 6 2 d í _______
b____________________ a ’\/a2_j_62.0
= 0.
/ ---------- 2 ji + 62

IXima ^nraneira análoga = 0,

[ 0/ ~|/a2sen2 í-|-a 2 cos2 í-j-ò2 dí ______


b ò-2ji2 l / a 2-(- Ò2
■= ------------------= JXO.
2ji V a 2 + 62 2 ji Y a ^ + b^

Tem-se, então, para coordenadas do • centro d( gravidade duma espira


ro de
da hélice
^c = 0, yc = 0y Zc = nb.

§ 5. Integrais de superfície
Seja dada em coordenadas rectangulares Oxyz um domínio V,
Em V é dada uma superfície a limitada por uma curva empenada A.
Relativamente à superfície a, suporemos que se definiu em cada
ponto P um sentido positivo indicando a normal unitária n (P) cujos
cossenos directores serão funções contínuas das coordenadas dos pontos
da superfície.
Consider^nos em cada fxmto da superfície um vector
F = X (.r, y, z ) i - \ - Y (x, y, z) j + Z (x, y, z) k,
seflPdo X, Y, Z funçõâ^ contínuas das coordenadas.
INTEGRAIS CURVILÍNEOS E INTEGRAIS DE SU PE R FÍC IE 241

Dividamos arbitràriamente a superfície em áreas elementares Aa,-.


Tomemos um ponto arbitrário Pi em cada elemento e consideremos
a soma
S ( F ( /» ,) w ( P ,) )A a ,. (1)

sendo F {Pi) o valor do vector F no ponto de n {Pi) o


vector unitário da normal nesse ponto, F n o produto escalar destes
vectores

O limite da soma (1) relativo a todas as áreas Ao,- quando o


maior diâmetro destas áreas tende para zero .é, por definição, um
integral de superfície que se designa pelo símbolo

J 5 F n da.
a
Então, por definição í*), tem-se
lim 2 = J J F n da. ( 2)
diam Aai->0 ^

Cada termo da soma (1)


F ifiià C i = F i A U f COS ( n | , jP ,) (3)

admite a seguinte interpretação: este produto é igual ao volume dum


cilindro elementar de base Aa^ e de altura Fi cos (tii, Fi), Se F
for a velocidade dum fluído que atravessa a superfície a, o produto (3)
é igual à quantidade de fluído que atravessa o elemento Aa^ na
unidade de tempo na direcção Ui (fig. 336).
C") Se a superfície a admite em cada ponto um plano tangente que
varia conlinuamente com P e se a função vectorial F é contínua nesta superfície,
csle limite existe (admitiremos sem demonstração, este teorema de existência dos
integrais de superfície).
16
242 CALCULO D IFEREN CIA L E INTEGRAL

A expressão f I F n do dá a quantidade total do fluído que


a
atravessa na unidade de tempo a superfície a no sentido positivo,
sendo F o vector velocidade do fluído no ponto dado. Eis porque
o integral de superfície (2) se chama ainda fluxo de campo vectorial F
através superfície a.
Resulta da definição de integral de superfície que se se dividir a
superfície o em partes Oj, Og, . . a ,te r-s e -á
^ J F n da = ^ J F n da + ^ ^ F n dor + . . . + J ^ F n do,
a Oi O2 Ok

o vector unitário n escreve-se:


n = cos(/i, .r) i-f-COS (n, y )j- \-c os (n , z)k.
Substituindo no integral (2) as expressões dos vectores F e n
em função das suas componentes, obtém-se:
J J F n da = J J [X
COS {n, x) + Y cos (n, y) +
a a
+ Z cos(ai, z)]do, (2')
o produto A(7 cos (n, z) é a projecção da área Aa sobre o plano
Oxy (fig. 337); tem-se igualmente para os outros produtos:
A acos(n, x) = íiOy^, Aacos(Ai, y) = ^Oxz, àocos(n, z) = àOxy, (4)
onde AOy^y Aa^^, Aa^^y são as projecções da área Acr sobre os cor­
respondentes planos coordenados.
Sendo assim, o integral (2') escreve-se ainda sob a forma

J J F n da = J J [X cos {n, x) Y cos {n, y) +


a a
-f- Z COS z )] da = X dy dz Y dzdx Z dx dy. {T)

§ 6. Câlciilo dos integrais de superfície


O cálculo de um inteeral sobre uma superfície empenada reduz-se
ao cálculo dum integral duplo sobre um domínio plano.
Indiquemos um processo de cálculo do integral
J J Z cos {riy z) do.
a

Suponhamos a superfície o* tal que qualquer recta paralela ao


eixo Oz o corta num só ponto. A equação de superfície pode ser
posta, então, sob a forma
^= y).
INTEGRAIS CURVILÍNEOS E INTEGRAIS DE SU PE R FÍC IE 243

Designando por D a projecção da superfície a sobre o plano Oxy,


tem-se (em virtude da definição dos integrais de superfície):
] l Z { x , y, z) COS (ra, z)da =

— lim '^Z{Xi, y i , Zi) COS (rii, z ) A o i .


d i a m Aai->0 i = í

Em seguida, tendo em consideração a última fórmula (4) do § 5,


obtém-se:
J ^ Zcos(A^, z) do =
a
n

=^, lim Hl Z{Xi, yi, f{Xi, yi))(Aa^ )i =


diam Aoxy-^0 i= l
n
S ^ (^í. í/í. / yi)) I A a * „ |i,
d i a m Aa-vo i = i

sendo a última expressão uma soma integral para o integral duplo da


função Z (x, y, f (u, y)) no domínio D. Por conseguinte.

J j2 c o s ( n , z ) d a = ± ^ ^ Z { x , y, f{x, y))dxdy.
a D

O sinal mais corresponde a cos («, z) > 0 e o sinal menos a


CO S (n, z) ^ 0.
Se a superfície a não satisfaz à condição indicada no começo
deste parágrafo, corta-se-la em partes que satisfaçam a esta condição
e calcula-se o integral, separadamente, em cada parte.
Calcula-se. duma maneira análoga, os integrais
^J X C O S {n, x)do; Jí cos {n, y) da.
a a

Assim se encontra justificada a éxpressão dum integral de super­


fície sob a forma (2'0 do §5.
Poder-se-á considerar, então, que o segundo membro da igual­
dade (2") é uma soma de integrais duplos sobre as projecções cor­
respondentes do domínio o-, sendo os sinais destes integrais duplos
(ou. como se diz ainda, os sinais dos produtos dydz, dxdz, dxdy)
tomados de acordo com a regra indicada.
Exempla— 1. Consideremos uma superfície fechada a cortada em dois
pontos no máximo por qualquer paralela ao eixo Oz.
Consideremos o integral

H z COS (n, z) d o .
244 CALCULO D IFEREN CIA L E INTEGRAL

Escolheremos como normal positiva a normal exterior. Podemos cortar


esta superfície em duas partes, inferior e superior de equações:
2 = /l(* . í/) et z = h { x , y).
Designemos por D a projecção a sobre o plano Oxy (fig. 338); tcm-se:

j ^ z COS ( n , z)do= ^ ^ Í2 (X, y) dx d y — f f


*^0 D D

O segundo integral foi afectado do sinal menos, porque no integral de


superfície o produto dxdy para a superfície z = fi {x, y) deve ser precedido
do sinal menos, dado que cos(n, z) é negativo.

iZ

Ora, a diferença dos integrais do segundo membro representa o volume


limitado pela superfície a. Logo, o volume do corpo limitado pela superfície a
6 igual ao integral de superfície

F = ^ ^ 2 COS ( n , z) do.
o

Exemplo — 2. Uma carga eléctrica positiva e colocada na origem das


coordenadas origina um campo vectorial cuja distribuição do vector F é
dada cm cada ponto pela lei de Coulomb:

sendo r a distância do ponto considerado à origem e r o vector unitário do


raio vector dirigido em direcção ao ponto considerado (fig. 339); A: é um factor
constante.
Calcular o fluxo do campo de vectores através de uma esfera de raio R
•centrada na origem.
INTEGRAIS CURVILÍNEOS E INTEGRAIS DE SU PE R FÍC IE 245

Resolução — Considerando que r = R = const., tem-se:

k ^rn d a =^ ^ ^ r n d o .
O a
Mas o último integral é igual à área u da esfera. Com efeito, o produto
escalar r n é constantemente igual à unidade e fica

íí do = a.

Por conseguinte, o fluxo procurado é

§ 7. Fórmula de Stokes
Seja dada uma superfície a tal que qualquer paralela a Oz a
corta num único ponto. Designemos por k a sua fronteira. A normal
positiva à superfície n será a que forma com Oz um ângulo agudo
(fig. 340).
Seja z = f(x, y) a equação da superfície. Os co^enos directores
da normal à superfície têm por expressão (ver § 6, cap. IX. tomo I):

dx
C O S ( n , x) =

+(^)
ày ( 1)
cos(n, y) =

cos(w, z ) = -

^{ ^ )
Suporemos que a superfície <r se encontra completamente num
domínio espacial V. Seja dada em V uma função contínua X (x, y, z)
com as suas derivadas parciais de primeira ordem. Consideremos o
integral curvilíneo tomado ao longo do contorno A.
I X {x, y, z) dx.
246 CALCULO D IFEREN CIA L E INTEGRAL

Tem-se, ao longo de A z = f (x, y), sendo x, y as coordenadas


dos pontos da curva L. projecção de A sobre o plano Oxy (fig. 340).
Por conseguinte, pode-se escrever
í X (x, y, z ) d x = [ X {x, y, f {x, y)) dx. (2)
\ L

Este último integral é um integral curvi­


líneo tomado ao longo de L. Transformemo-lo,
aplicando a fórmula de Green, fazendo
X{x , y, f{x, y)) = X{x , y), 0 = Y ( x , y).
Substituindo na fórmula de Green X
e Y pelas suas expressões, obtém-se:
_ JJ dX( x, y J ( x _ , j ^ ^ ^ ^ ^ _

\
= \ X ( x , y, f {x, y)) dx. (3)

sendo o domínio D limitado por L. Derivemos a função composta


A' (jc, y, f (x, y)) em relação a y :
dX (x, y, f (x, y)) ^ dX (x, y, z) dX {x, y, z) dfjx, y)
dy dy dz dy
Substituindo a expressão (4) no primeiro membro de (3), obtém-se:

dX{x, y, z) ^ d X ( x , y, z) dfjx, y)
dxdy =
- íJJí [ dy dz dy

= ^ X (a-, I/, / (x, y)) dx.


L

Tendo em conta (2) esta última igualdade escreve-se:

^ X (a:, y, z) dx = — ^ ^ dxdy dx dy. (5)

Os dois últimos integrais transformam-se em integrais de super­


fície. Com efeito, resulta da fórmula (2") do § 5 que se tem para
qualquer função A (x, y, z) a igualdade
H ^ 2) COS (n, z)da — A d x dy.
INTEGRAIS CURVILÍNEOS E INTEGRAIS DE SU PE R FÍC IE 247

Sendo assim, os integrais do segundo membro de (S) transfor­


mam-se como se segue:

j J j - ^ c o s ( n , z)da,
D a

( 6)
[ [ d X df , , [ { d x df , _
J J oz oy J J dz du

Transformemos o último integral aplicando as fórmulas (1) do


presente parágrafo; calculando o quociente da igualdade (1) pela
terceira, encontra-se:
cosjn, y) ^ df
cos{n, z) dy
ou
fif
-----cos(n, z) = — cos(n, y).
dy
Por conseguinte,

D
dz dy
dxdy
-H ? a

Substituindo as expressões (6) e (7) em (S). obtém-se:


COS ( n , y) da. (7)

^ X (x, y, z ) d x = — I ^ C O S (n, z ) da -f-

+
!!S C O S (n , y) da.

O sentido de percurso de X deve ser tal que um observador


(8)

atravessado de pés à cabeça pela normal n veria o contorno percorrido


no sentido inverso dos ponteiros de um relógio.
A fórmula (8) é verdadeira para qualquer superfície podendo
ser cortada em duas partes de equações da forma z = f(x, y).
Obtém-se, duma maneira análoga, as fórmulas

j Y (x, y, z ) d y = j | — - ^ c o s ( r a , x) + - ^ c o s ( r e , z)j da, (8')


X a

j z ( x , y, z ) d z = I ---- ~ c o s { n , y ) - f - ^ c o s ( n , x) j da. (8')


248 CALCULO D IFEREN CIA L E INTEGRAL

Somemos membro a membro as igualdades (8). (80 e (8^0;


obtém-se a fórmula

dx dy

+
(dZ
dy dz
I C O S (n, X) +
, í dX

Ê a fórmula de Stokes (matemático inglês (1819-1903)). Ela permite


(9)

transformar um integral de superfície a num integral curvilíneo tomado


sobre a fronteira X desta superfície, sendo o sentido de percurso da
fronteira o especificado mais acima.
O vector {B de componentes
dZ dY dY dX
B = — — — B^■.
dy dz ^ dz ■ dx dx dy
chama-se vector «tourbillon» ou rotacional da função vectorial
F = X i + Y j + Z k e escreve-se, simbòlicamente, rot F .
Por conseguinte, pode-se recopiar a fórmula (9) sob a forma
vectorial
^ F d s = l^nvotF da, (9')

e o teorema de Stokes, enuncia-se:


A circulação de um vector ao longo dum contorno fechado que
limita uma superfície é igual ao fluxo do seu rotacional através desta
superfície.
Nota — Se a superfície a é uma porção de plano paralelo a Oxy,
tem-se Az = 0 e encontra-se a fórmula de Green como caso particular
da fórmula de Stokes.
Resulta da fórmula (9) que se
dY ^ dZ dY ^ dX dZ
.0, ^ - ^ = 0, 0, ( 10)
dx dy dy dz dz dx
o integral curvilíneo é nulo sobre qualquer curva empenada fechada A:
^ X d x - \ - Y d y - \ - Z d z = Q. (11)

O integral curvilíneo não depende, pois, da forma da curva de


integração.
Como no caso duma curva plana, mostra-se que as condições
mencionadas (10) são não sòmente suficientes mas também necessárias
para ter a igualdade (11).
INTEGRAIS CURVILÍNEOS E INTEGRAIS DE SU PE R FÍC IE 249

Sendo estas condições satisfeitas, a expressão sob o sinal soma


é o diferencial total duma certa função u (x, y, z):
X d x - \ - Y d y -\-Z d z = du (x, y, z)
e, por consQguinte,
(N ) (N)
J X d x - \ - Y d y - { - Z d z = ^ du = u ( N ) — u{M).
(M) (M)
Demonstra-se-lo tal como para a fórmula correspondente no
caso de uma função de duas variáveis (ver § 4).
Exemplo— 1. Escrevamos as fórmulas fundamentais da dinâmica do
ponto material
dvx dUj. dvz
“ T - '''
Aqui m é 2l massa do ponto; X, Y, Z são as componentes da força que
solicita o ponto; 1;^ = - ^ , , Vy = - ^ as componentes da velocidade v.
at ^ dt dt
Multipliquemos os dois membros das
equações acima pelas expressões
Vxdt = dx, Uydt = dy^ Uzdt — dz.
Obtém-se, juntando membro a membro
as igualdades dadas:
m (vx d v x + dvy + Vz d v z ) =

= X dx-\-Y dy-\-Z dz :
d dx-\-Y dy -{-Zdz.

Como v%+ vy-\-vz — v‘^, pode-se escrever:

d (~ j ~ X dx-{-Y dy-\-Z dz. Fig. 341

Calculemos os integiais dos dois membros entre dois pontos e Afj sobre
a trajectória:
(M2)
1 1 (•
— — — m v\= - I X d x - \ - Y d i j - \ - Z d z^

(Ml)
sendo v, e v, as velocidades no ponto e M.^.
Esta ültima igualdade traduz o teorema das forças vivas; a variação
da energia cinética entre dois pontos é igual ao trabalho da força que age sobre
a massa m.
Exemplo — 2. Calcular o trabalho da força de atracção newtoniana
duma massa imóvel m que age sobre uma massa unitária deslocando-se entre
dois pontos A/j (fl^, hx, c ^2 (^2 , ^2» ^2)-
Resolução » Tomemos a origem das coordenadas no centro atractififo.
Designemos por r o raio vector (fig. 341) traçado da origem ao ponto onde
250 CALCULO D IFEREN CIA L E INTEGRAL

se encontra a massa unitária e seja o vector unitário desta direcção. Tem-se,


então, sendo k a constante universal de gravitação. As com-
ponentes de F sobre os eixos são, respectivamente,
-cr — — kni —r---- ;
.Y Y-.r 1
= — km ^ y ; ^ = — km -r- — .
Z
r2 r r2 r r2 r

O trabalho da força F sobre o arco é


(M 2) (M2) (M 2)
X dx-\-y dy-\-z dz
= — km ^ = j ^ = j ^ (y )
(Ml) (Ml) (Ml)
(dado que r2 = x 2 r dr = x dx-\-y dy-\-z dz). Designando por e
os comprimentos dos raios vectores dos pontos e Mj, obtém-se:

A = km ----- .
\''2 '*1/
Por conseguinte, aqui ainda o integral curvilíneo não depende do caminho
de integração. A função u — - ! ^ chama-se potencial do campo de atracção da
r
massa m. No nosso caso
Y_àu du ^ du
dx ’ dy ' dz ^
A = u {M2) — u {Mí),
isto é, que o trabalho para deslocar a massa unitária é igual à diferença do
potencial entre os pontos final e inicial.

§ 8. Fórmula de Ostrogradshy
Seja um domínio regular V do espaço a três dimensões, limitado
por uma superfície fechada a e tendo por projecção sobre o plano Oxy
um domínio regular D. Suporemos que se pode dividir a superfície
<T em três partes (Ti , Og» ^3 de mcKio que as equações das duas pri­
meiras se escrevam
z = fi(x, y) et z = tz{x, y),

sendo fi (jc, y) e /2 (jc, y) contínuas no domínio D e sendo a terceira


parte as uma superfície cilíndrica de geratrizes paralelas ao eixo Oz-
Consideremos o integral
dZ{x, y, z)
dx dy dz.
dz
INTEGRAIS CURVILÍNEOS E INTEGRAIS DE SU PE R FÍC IE 251

Integremos, em primeiro lugar, sobre os z:

'=íi( ID /,(.v, !/)


= j j 2 (z, Í/, fz (x, y))dxdy — ^ ^ Z {x, y, /,'(x, y)) dx dy. (1)
D D

Definamos sobre a superfície a a normal positiva, dirigida para


o exterior. Então, cos («, z) será positivo sobre a superfície ct2 e negativo
sobre a superfície <ti; é nula sobre a superfície ^3.
Os integrais duplos do segundo membro da igualdade (1) são
iguais aos integrais de superfície correspondentes
l l Z { x , y, Í 2 ÍX, y ) ) d x d y = l l Z { x , y, z)cos(n, z)da, (2')
D O2

I J Z (x, y, fi {x, y)) d x d y = l ' ^ Z {x, y, z) (—cos (n, z)) der.


D aj
Escrevemos no último integral (— cos(/i, z)) porque o elemento
de área A^i está ligado ao elemento de área ^s do domínio D pela
relação As = Aai [— cos (w, z)] dado que o ângulo (n, z) é obtuso.
Assim.
J y, fi(x, y))dxdy =
D

= — J J Z (x, y, /i (x, y)) cos (n, z) da. ( 2" )


<^t
Substituindo (20 e (2'0 em (1), obtém-se:

ííí dZ{x, y, z)
dz
dx dy dz =

= j | z ( x , y, z)cos(n, z)da +
02
íí Z(x, y, z)cos(«, z)dcr.

Para comodidade de cálculos a seguir, recoplemos esta última


igualdade juntando-lhe a quantidade J | Z (x, y, z) cos (n, z) do = 0
CT3
(tem-se cos («, z) = 0. sobre tr,):

ííídZ{x, y, z)
dz
•dx dy dz =

= I ^ Z cos (/z, z) da + U Z cos (n, z) da + j j Z cos (n, z) da.


02 ai a.
252 CALCULO D IFEREN CIA L E INTEGRAL

Ora. a soma dos integrais do segundo membro é igual ao integral


sobre toda a superfície c\ por conseguinte.

111 dx dy dz = I j Z (x, y, z) cos (n, z) da.


V o

Duma maneira análoga, obtêm-se as relações:

111 11

^^
V ^ a

111 dydz= (^. y> z) COS («, x) da.

Juntando membro a membro estas três últimas igualdades, obtém-se


a fórmula de Ostrogradsky (*):

ÍÍK dX
dx
. dY
dy
dZ
dz
I dx dy dz =

íí
= \ j (X COS {n, x) 4- y

riV ri''/
COS (n, y) + Z cos (n, z)) da. ( 2)

A expressão - — i---- --\------chama-se divergência do vector


dx dy dz

J-’ = X í + y^-f-ZA;
e escreve-se div h '\
.. ,, dX dY ^ dZ
dx dy dz
Indiquemos que esta fórmula é verdadeira para todo o domínio
podendo ser dividida em domínios parciais que satisfaçam às condições
mencionadas no começo deste parágrafo.
Vamos dar uma interpretação hidrodinâmica da fórmula estabe­
lecida.
Suponhamos que F — X I Y j + Z k é o valor velocidade dum
fluído que atravessa o domínio V. O integral de superfície em (2) é.
então, o integral da projecção de F sobre a normal exterior u ;
onde a quantidade de fluido saído do volume V durante a unidade
• Esta fórmula (por vezes, chamada fórmula de Ostrottradsky-Gauss) foi
descoberta pelo célebre matemático russo M. Ostrogradsky (1801-1861) que a
publicou em 1928 no seu artigo «Notas sobre a teoria do calor».
INTEGRAIS CURVILÍNEOS E INTEGRAIS DE SU PE R FÍC IE 253

de tempo (ou que aí entrou, se o integral é negativo). Esta quantidade


expriihe-se por meio do integral triplo de div F .
Se div F = 0, o integral duplo sobre qualquer superfície fechada
é nulo, a quantidade de fluído entrado ou saído é nula. Mais preci­
samente, a quantidade de fluído entrado no volume dado é igual à
quantidade de fluído saído.
Sob forma vectorial, a fórmula de Ostrogradsky escreve-se:

^^J div F d v = J J F n ds ( 1')

e enuncia-se: o integral da divergência dum campo vectorial F num


volume é igual ao fluxo do campo vectorial através da superfície que
limita este volume.

§ 9. Operador hamiltoniano e algumas aplicações


Seja dada uma função u = u(x, y, z). Em cada ponto do domínio
em que a função u (x, y, z) é definida e derivável, é determinado o
gradiente:
, ,du , ,du , ^ du
( 1)

Por vezes, o gradiente da função u(x, y. z) é assim designado:


_ .du . d u d u
Vu = t — + j ^ + k ; ( 2)
dx dy az
o sinal V lê-se «nabla».
1. É cómodo escrever sobre uma forma simbólica a igualdade (2):

{ .d , .d , - 5
Vu ( 2')
% s )“
e de considerar o símbolo

^ --- ^ (3)
dx dy--- dz

como um «vector simbólico». Este vector simbólico chama-se operador


hamiltoniano ou operador nabla (operador V ). Decorre das fórmulas (2)
e (2') que quando se «multiplica» o operador simbólico V por uma
função escalar u, obtêm-se o gradiente dessa função:
Vw = gradi/. (4)
254 CALCULO D IFEREN CIA L E INTEGRAL

2. Pode-se formar o produto escalar do vector simbólico V


pelo vector F = i X + j Y + k Z :

5 + + * • á) < « + J Y + ’‘^ =

dZ
:divi*’
dx dy dz dx dy dz
(ver § 8). Assim.
^F = à ivF . (5)
3. Formemos o produto vectorial do vector simbólico V pelo
vector F = íX + j Y + k Z :

I +•'l+*=á) =<<«+■'' + =
i j k
d d d d d d
ri
O 0r í . 0
= i dy dz - J dx dz + A* dx dy
dx dy dz
Y Z X Z X Y
X Y Z
dZ dY
/ \ dx dz / \ dx dy /

. ( dZ ay \ JdX dZ\- ^ ( âY dX\


rot F
“ dy “ “S " ) + v i r “ 'i r i + * A ir “ J“
(ver § 7). Assim,
V X F = rot F. ( 6)
Decorre do que acaba de ser dito que a utilização do vector
simbólico V permite exprimir sob uma forma muito sucinta as ope­
rações vectoriais. Consideremos ainda algumas fórmulas.
4. O campo vectorial F (x, y, z) — i X + j T + k Z diz-se
campo vectorial potencial se o vector F for o gradiente duma certa
função escalar u (x, y, z):
F = grad u
ou

F = i — -I- ^ -1_ l;
''d x ^ ^ d y + ^Jz-
INTEGRAIS c u r v il ín e o s E INTEGRAIS DE SU PE R FÍC IE 255

Neste caso as projecções do vector F serão

dx' ^ dy ’ dz'
Decorre destas igualdades (ver t 1, cap. VIU, § 12):
dX dY dY dZ dX dZ
dy dx dz dy dz dx
ou
ÊÃ dY dY dX dZ
^ = 0.
dy dx dz dy dz dx
Por conseguinte, para o vector F considerado
rot
Obtemos, assim:
ro t (grad u) = 0 (7)
Aplicando o operador V, pode-se escrever em virtude das fór­
mulas (4) e (6) a igualdade (7) sob a forma:
(V X Vu) = 0. (T)
Utilizando a propriedade que da multiplicação dum produto
vectorial por um escalar basta multiplicar por este escalar um dos
factores sòmente, escreveremos:
( V x V ) u = 0. (7” )
O operador V possui de novo as propriedades dum vector
usual: o produto vectorial do vector por si mesmo é nulo.
O campo vectorial F (x, y, z) para o qual rot F = 0 diz-se
irrotaciond. Decorre da igualdade (7) que todo o campo potencial é
é irrotacional.
O inverso é igualmente verdadeiro; por outras palavras, se um
certo campo vectorial F é irrotacional. é potencial. A validade desta
afirmação decorre dos raciocínios conduzidos no fim do § 7.
5. O campo vectorial F (x, y, z) para o qual
d iv F = 0 ,
isto é, o campo vectorial que não contém origens (ver § 8) chama-se
selenoidal oU tubular. Demonstraremos que
div (rot F ) = 0, (8)
por outras palavras que o campo rotacional não contém origens.
256 CALCULO D IFER EN C IA L E INTEGRAL

Com efeito, ^ F = i X + J Y + AjZ, então,

dX dZ ^ ' dY dX
\ dy dz) . dz dx ;! + * ' ( \ dx dy
e. eis porque.
dv) ' dX dZ )
ày dz í ' d y \ \ dz dx }1 +

dY dX
+ U
d z \ dx dy
0 )= .
A igualdade (8) escrever-se-á com a ajuda do operador V :
v ( v x i í ’) = o. (8:
o primeiro membro desta igualdade' pode ser considerado como
o produto misto vectorial escalar dos três vectores V, V, F de que
dois são idênticos. Este produto é. evidentemente, igual a zero.
6. Seja dado um campo escalar u = u(x, y, z). Definamos o
campo dos gradientes:
j .d u , ,du , j du
gmd

Achamos, em seguida,
,. , j . d ( du\ d ( du\ d ( d u \
d .v ( g r.d a ) = - y + - y +-(_— )

ou
,. , , , o2^U
o u . O^U . O
div(gradu) = — + — + — . (3)
dx dy dz

o segundo membro desta igualdade chama-se operador de Laptace


da fpnção u e nota-se por

Por conseguinte, a igualdade (9) pode-se escrever:


div (grad u) = Au. (11)
Com o auxílio do operador V a igualdade (11) escrever-se-á sob
a forma
(VVu) = Au. (11')
INTEGRAIS CURVILÍNEOS E INTEGRAIS DE SU PE R FÍC IE 257

Notemos que a equação


o2
dru ^2
o^u aru
= 0 ( 12)
do? d:?
ou
Aa = 0 (12')
se chama equação de Laplace. As funções que verificam a função de
Laplace chamam-se funções harmónicas.

Exercícios
Calcular os integrais curvilíneos seguintes:

1. J dx-\-2xy dy sobTQ o círculoa; = a cos 1, y = a sení. Resp. 0.

2. ^ y dx —2; (2^ sobre a clipsea: = fl cos í, y = b scn t, Resp. —2jiaò.

,
5 _X2__j_
r y ddx
í—^
x + xc
1/2
xd y
d x -----^ — —du sobre um círculo centrado na origem. Resp. 0.
x2 + y2 ^

;sobre a recta y = x.áe x = i à x = 2. Resp.Log 2.

5. ^ yz d x x z d y x y dz sobrt a hélicea: = a cos 1, i/ = asen í, z ^ k t ^ t variando


entre 0 et 2jc. Resp. 0.
Í 0
X dy — i/dx sobre a hipociclóide x = a cos^ í, y = ascn,^ t.Kesp. Jia^ (o

dobro da área limitada pela curva).


3at

J
y= . Resp. a2 (o dobro da área limitada pela curva).
xd y — y dx estendido à cu

8. J Xdy — y dx sobre a curva x = a (í —sen í), y = a (1 — cos í) (0 < ;í< ;2 n ).


Resp. — Srra^ (o dobro da área compreendida entre um arco da ciclóide
e o eixo Ox).
Demonstrar que:
9. grad (cq)) = c grad cp, onde c é constante.
10. grad (cjqp + C2\l)) = cj gr^d (p + C2 ^ ^ constante.
11. grad (cptl)) = (p grad grad (p.
12. Calcular grad r, grad r2, grad— , grad / (r), onde + +

Resp- : 2r ; ; / ' (r) .


13. Demonstrar que div ( ^ + J3) = div A + div B .
14. Calcular div r , ou r xí -\-yj-\-zk. Resp. 3.
15. Calcular div (^cp), onde .4 é uma função vectorial e 9 uma função escalar
Resp. (p div A + (grad (p4).
17
258 CALCULO D IFEREN CIA L E INTEGRAL

16. Calcular div (r«c), onde e é um vcctor constante. Rest>. .


17. Calcular div J3 ( r ^ ) , Resp. A B
Demonstrar que:
18. rot (ciAi + C2 A 2) = Ci rot A í -\-C2 rot ^2i onde et C2 são constantes.
19. rot (>!/*) = g ra d -4 X c,ondee é um vcctor constante.
20. rot rot iá = grad div 4 —A 4.
21. 4 X grad q) = rot (<p4).
Integrais de superfícies

22. Mostrar que J J cos (n, z)d o = 0 sobre uma superfície fechada.
23. Achar o momento de inércia dum segmento esférico de equaçSo -4-1/24.
+ cortado pelo plano z = H em relaçSo ao eixo Oz.
Resp. f í^ ( 2 i í 3 _ 3 / í 2 ^ _ |.^ 3 ) .
24. Achar o momento de inércia da porção do parabolóide de revolução
jç2 4. y2 = 2cz contido pelo plano z = c em relação ao eixo Oz. Resp. — c*.
3
25. Calcular as coordenadas do centro de gravidade da parte da superfície
R2 o
cónica a:2 + j/2 —_ ^ 2 cortada pelo plano z = H. Resp. O; 0 ; •—
26. Calcular as coordenadas do centro de gravidade do segmento da superfície
lesférica x2-f y2 ^2 = cortado pelo plano z = H. Resp.^0, — g— ) •

27. Calcular J J cos (nx) -\ y cos (ny)-\-z cos (nz)] da ; sobre uma superfíciei.

fechada. Resp. W , sendo V o volume interior à superfície


28. Calcular I I zd x d y , onde S designa o lado exterior da esfera x2-|-

+ J/2+z2 = /?2. Resp. *


• o
29. Calcular 1 l x’^ dy d z y ' ^ dz d x z ^ dx dyy onde S é o lado exterior da

superfície da esfera x^-{-y2-\-z^ = R2, Resp. 'ttR*.


30. Calcular j J ds ^ onde S designa a superfície lateral do cone
8
X
-2 [y2 z2 o 2na2-j/a2 + ò2
+ = 0 < z < 6 .R e s p . ---------^ . T

31. Com a ajuda da fórmula de Stokes transformar o integral ^ y dx-\-z dy-{-

+ xd2. Resp. ■“ j j (c o s a + c o s P + cos y) ds.


INTEGRAIS CURVILÍNEOS E INTEGRAIS DE SU PE R FÍC IE 259

Achar os integrais curvilíneos directamente e aplicando a fórmula de


Stokes:

32. í í (y + ^) + + + + L é o círculo x^-{-y^-\-z^ =


x + i/ + z = 0. Resp. 0.

’í
33. ^ dx-\-dy-\-z dz, onde L é o círculo x^-\-y^ = R^^ z = 0. Resp. —

Aplicando a fórmula de Ostrogradsky, transformar os integrais de superfície


em integrais de volume:

34. ^ ^ COS a + y cos P + z cos y) d s , Resp. ^ ^ J dz,


8 V

35. ^ ^ (x^-{-y^-]-z^) (dy dz-\-dx dz-]-dx dy). Resp. 2 J (x y z ) dx dy dz.

36. ^ ^ xy dx dy-]-yz dy dz-\-zx dz dx, Resp. 0.

l l ^ à y d z + ^ d x d z + ^dxdy.Rtsp.^^
S V
J(S+£^+S)
Aplicando a fórmula de Ostrogradsky, cúlcular os integráis seguintes:

38. j j (x c o s a + í/cosp + zcosv)ds, «onde 5 é a superfície de elipsóidc.

X^ 1/2 ^2
-^+ -b 2 + ~ ^-
39. ^ ^ (x3 COS a + í/3 COS P + z3 COS v) dí, onde 5 é a superfície da esfera

x2+ í/2_|_22 = í ?2, Resp.


O
40. ^ ^ x2 dy dz + y^ dz dx-\-z^ dx dy, onde 5 é a superfície do cone

z2 Tlã^b^
- - ^ = 0 (0<z<fc).Resp. —-—

41. ^ ^ X dy d z y dx d z z dx dy, onde 5 é superfície do cilindro x* + y* = «'

— H ^ z ^ H , Resp. Sna^H,

42. Demonstrar a identidade J J ^ ^ ^ o con-

du
torno que limita o domínio D e a derivada segundo a normal exterior.
on
260 CALCULO D IFEREN CIA L E INTEGRAL

Resolução:

= \ l — Y cos(s, x)-{-X scn(s, x)] d®,


C
onde (5. x) é o ângulo entre a tangente ao contorno C e o eixo Ox, Se
se designar por {n, x) o ângulo entre a normal e Ojc, tem-se sen (i, x) =
= cos (/z, X), COS (s, x) = — sen (n, jc). Por conseguinte,

I í í [AT COS («, x) + y scn(n, x)] ds.


D
Fazendo X = — , y = — , obtém-se:
dx dl/

D c

OU

í J ( S + S ) ‘“ ' " ” Í - 3 Í ' “ -


D C

d‘^u d'^u
A expressão chama-se operador de Laplace,
43. Estabelecer a identidade (chamada fórmula de Green)

J J j (v^u-u^v)dxdyd^= J J ~ ] áa.
V a
sendo u e v contínuas e tendo derivadas de segunda ordem contínuas no
domínio D. Os símbolos Aiz e Av representam:
_ d ^ u d'^u d^u * _d2í; d^i; d2y
'^“ ^d x 2 + ^ + '^ » . + d^ *
São os operadores de Laplace no espaço.
Resolução — Na fórmula

5jí f ê + i + f

= J J [ Z COS ( n , x ) + y COS ( n , y)-\-Z cos (n, z)\ db


façamos ^

X=zVUx-—UVx,
Y = ÜUy— UVy^
Z— VUz — UVz,
INTEGRAIS CURVILÍNEOS E INTEGRAIS DE SU PE R FÍC IE 261

Tem-se

dX . dY , dZ / ** t t ^\ t ^
^ ^ (U xx -f Uyy + Uzz) — U (V xx + + i^2z) = tAM— uAu^

JT COS (w, a:) + y COS (n, y ) - { - Z c o s { n, z) =

= V (Ux COS (n, x) + Uy COS (n, y) + Uz cos {n, z)) — u {vx cos (fi, x) +

+I V y' COS (n,


/ y )\ + ' cos i/n , z)\v) = v —----- u —dv .
Vz
On an
Por conseguinte.

44. Estabelecer a identidade

líí òiu d x dy d z = íí
a
do,

onde Au = ^ ^ + — -1-^^ (operador de Laplace).


dx^ ^ dy^ ~ âz2
Resolução — Façamos, na fórmula de Green, estabelecida no exemplo ante­
rior, V = 1. Então, Av = 0 e a identidade está demonstrada.
45. Se u (x, y, z) é uma função harmónica num certo domínio, isto é, uma
função tal que em cada ponto deste domínio é verificada a equação de Laplace
d^u d^u

tem-se, em toda a superfície fechada a

55 du
r—
ón
da = 0.
*

Resolução — Isso resulta directamente da fórmula do problema 44.


46. Seja u{x, y, z) uma função harmónica num domínio V e consideremos
em V uma esfera 6 de centro M (x^, iji, z{) e de raio R. Mostrar que

yu * i ) = - 4 í ^ { J
Resolução — Consideremos o domínio S2 limitado pelas duas esferas a ,
õ de raios R e p ( p < i? ) , de centros no ponto M y\, z^), Apli­
quemos a este domínio a fórmula, dc Green do problema 43, onde u
será a função indicada acima e v a função
1 _____________ 1____________
262 CALCULO D IFEREN CIA L E INTEGRAL

Confirmamos direçtamente, derivando e substituindo, que d^u d^u


d^u
= 0. Por conseguinte.

i+i
Ç Çf 1 ^
\ r dn “/
^ r)da=0,

ou

íí í í (l*-“ -=»■
Sobre as superfícies ct e a a quantidade _ é constante
“ r 7
pode-se-la fazer* sair do sinal de integração. Em virtude do resultado
estabelecido no problema 45:
(^' )
7 H du
dn
do = 0.

Por conseguinte,

“ í í í
mas

M r) J_
dn dr r2 •
Logo

+ J J u ^ d a - j J u-lda = 0

ou
_1_
JJ«da = ^ JJu d a . ( 1)
p2

Apliquemos o teorema da média ao integral da esquerda:

± J J J j ,. (2)
INTEGRAIS CURVILÍNEOS E INTEGRAIS DE SU PE R FÍC IE 263

onde (S, 0 é um ponto sobre a superfície da esfera de raio p e de


centro no ponto M {xi, z{).
Façamos tender p para zero: então, (6, il, D (^i» ^i) •

-L r f _ 4 ,.
P2 J J P*

Logo, quando p - » 0 , obtém-se:

I j “ « (*i. Vu *i) 4n.


ã
Além disso, dado que o primeiro membro da igualdade (1) não depende
de p, quando p —> 0, obtém-se, por fim:

~W f j Vi’ *i)
ã
ou

u(^i,yu * i)= 4 à 2 f f
Capíiuio XVI

SÊRIES

§ 1. Soma duma série


Definição— 1. Seja dada uma sucessão numérica infinita (♦)
Uiy lÍ2, • • «y Ujrij . . .
A expressão
+ ^2 + + •••+ • (1)
chama-se série numérica, sendo os números Ui, U2........ ... os
termos da série.
Definição — 2. À soma dos n primeiro termos de série chama-se
soma parcial :
+ ^2 + • • • +
Consideremos as somas parciais:
Si =

^2 = ^1 + ^2,
53 = -|- 1^2 + Z^3,

”f" ^2 "l~ ^3 “h • • • ~ \ ~ U n .

Se o limite seguinte existir e for finito:


s = lim
n-^oo
chama-se soma da série (1) e diz-se que a série converge.
Se lim não existe (por exemplo 00 quando n 00 ),
n-> oü

diz-se que a série (1) diverge e que não tem soma.*

* Diz-se que uma sucessão é dada qtiando se conhece a lei que permite
calcular qualquer termo u m vez dado n.
SÉRIES 265

Exemplos — Consideremos a série


a-\-aq-^aq^-\-. . . . ( 2>

É uma progressão geométrica de primeiro termo a c de razSo q ( a ^ 0 ) .


A soma dos n primeiros termos da progressão geométrica (quando q ^ l )
é igual a -
a — aq^
Sn —■ 1—g
OU
aq^
g 1 -g *
1. Se | g | < l , q ^ ^ O quando n -^ o o e, por conseguinte,

lim s„ = litn [ ---------- ^ — .


n-> oo n-> oo g 1 9 / 1 9
Assim, quando q < 1, a série (2) converge e a soma é
a
s=
1—g
2. Se q > 1 , I g ^ l ^ o o quando n->co e ——---->-±00 quando
' 1— g
n-^co,, isto é, que lim não existe. Assim, quando q > 1, a série (2)
n-> oo
diverge.
3. Se q = 1, a série (2) escreve-se

“1“ a -|- a -j- . . .


Por conseguinte
Sn = na, lim Sn = co,
n-^ao
a série diverge.
4. Se q = — 1, a série (2) escreve-se
a —a + a —a + . . .
e
I* 0 se n é par
—I
L a se n é ímpar,
não tem limite, a série diverge.
Assim, a progressão geométrica (de primeiro termo não nulo) converge,
se, e somente se, a sua razão for menor que a unidade em valor absoluto.
Teorema— 1. Se a série obtida, suprimindo em {l) vários termos,
converge, a série proposta converge igualmente,
Reciprocameníe, se a série proposta converge, a série obtida supri­
mindo vários termos converge igualmente. Por outras palavras, não
se afecta o carácter de convergência duma série suprimindo um número
finito de termos.
Demonstração — Sejam Sj^ a soma dos n primeiro termos da
série (1), , a soma dos k termos suprimidos (notemos que se n for
266 CALCULO D IFERENCIA L E INTEGRAL

suficientemente grande, todos os termos rejeitados estão compreendidos


na soma s^) e seja ainda ^n-h a soma dos termos da série que entram
em Sn mas não em Tem-se
+ or^-Af
sendo <^k um número constante não dependente de n.
Resulta desta última relação que se lim On-h existe, então, lim $n
n->oo n-¥oo
também existe; se lim existe, o mesmo se diga de lim On-hy O que
n-^oo n->0Q
demonstra o teorema.
Para terminar este parágrafo, indiquemos duas propriedades ele­
mentares das séries.
Teorema — 2. Se a série
+ (3 )
converge e se a sua soma é s, a série
Cãi -f- Cã2 (^)
em que c é um número arbitrário fixo, converge também e a sua
soma é cs.
Demonstração — Sejam e . respectivamente, as somas par­
ciais de (3) e (4). Tem-se
= cai + • • • + CUn = C(«1 + • • • + ^n) =
Resulta que a soma parcial de (4) existe, porque

lim On = lim {cSn) = c lim Sn = cs.

Assim, a série (4) converge e a sua soma é cs.


Teorema — 3. Se as séries
+ ^ + (5)

+ ^2 + • • • - ( 6)

convergem e têm por somas s e s, as séries

(«1 + Ò,) + («2 + 62) + • • • (7)


e
(ai — 6j) + («2 — ^^2) + • •. ( 8)

convergem igualmente e lêm por sòmas 5 + s e 5 — s.


SÉRIES 267

Demonstração — Mostremos a convergência da série (7). Designemos


a sua soma parcial de índice n por e as somas parciais de (S)
e (6), respectivamente, por Sn e Sn\ tem-se
0^71 = (^1 + ^l) + • • • + (^n + bn) =

= («1 + • • • + «n) + (^1 + • • • + fcn) = ^71 +


Passando esta igualdade a limite quando n oo, tem-se

lim On = lim (Sn -j- Sn) = Hm Sn + Hm Sn = S + S .

Assim, a série (7) coqverge e tem por soma s-j-s.


Duma maneira semelhante se demonstra que a série (8) converge
também e que tem por soma s —~s.
Diz-se que as séries (7) e (8) foram obtidas, somando e subtraindo
termo a termo as séries (5) e (6).

§ 2. Condição neceâsãria de convergência de um a série


Quando se estuda uma série, uma das questões fundamentais é
a da convergência ou da divergência dessa série. Mais abaixo estabele­
ceremos critérios suficientes que resolvem esta questão. Propomo-nos,
agora, estabelecer um critério necessário de convergência.
Teorema — Se uma série converge, o seu termo geral Un tende
para zero quando n tende para infinito.
Demonstração — Suponhamos que a série
+ 2^3 + • • • + + ••*
converge, isto é, que se tem a igualdade
Hm Sn =
Tl-^OO
em que f e a soma da 'série (um número finito fixo); mas, então,
tem-se, igualmente, a igualdade
lim s „ -i = s,
7l->00
porque n e (n — 1) tendem para infinito ao mesmo tempo. Subtraindo
termo a termo a segunda igualdade da primeira,. obtém-se:
lim s„ — lim s„_i = 0
Tl-^oo
ou
lim (s„ — s„_i) = 0
n-^oo
268 CALCULO D IFEREN CIA L E INTEGRAL

Ora,
^71—1—
Logo
lim Un = 0,

c. q. d.
Corolário — Se o termo geral duma série não tende parq zero
quando n-> oo, a série diverge.
Exemplo — A série

1+A+1+, n ,
3^ 5 ^ 7 ^ 2n -|-1
diverge, porque
lim u„ = lim ( = l.=jí=0.
n-^oo n->oo \ / Z
Notemos que o critério examinado dá uma condição necessária,
mas não suficiente, isto é, que uma série pode bem divergir mesmo
que o seu termo de ordem n tenda para zero.
Assim, a série seguinte, dita harmónica

diverge, se bem que


1
lim Un = lim — = 0.
j^_>oo ri
Para o demonstrar, recopiemos um maior número de termos da
série harmónica:
i 1 1 1 1 1 1
^ + Y + 3 ’+ 4 + T + ' 6 + y + ^ +

1 1 1 1 1 1 1 1 1
+ y + ro + n + 12 + Í3 + 14 + Í5 + 16 + 17‘+ (1)

Escrevamos, ainda, a série auxiliar

. , 1 . í1 1 . 1 . 1 , 1 , 1 ,
l + ^ + ^4 + ^ + ^ + ^ + - õ - + - ^8 +

1 1 1 1 1 1 1 1
+ F 6+ F6+ F 6+ F6+ F 6+ 1 6 + 1 6 + 1 6 +
16 termos

+ .32 + ' " + 3 2 + (2)


SÉRIES 269

Constrói-se a série (2) como se segue: o seu primeiro termo é


1 1
igual a 1, o segundo igual a _ , o terceiro e o quarto igual a — ,
2 4
1 1
O S quatro seguintes são iguais a -5-, os oito seguintes a — os dezas-
1 o 16
seis seguintes iguais a ^ , etc.
oZ
Designemos por Sn’ a soma dos n primeiros termos da série
harmónica ( 1) e por a soma dos n primeiros termos da série (2).
Como cada termo da série (1) é maior que o correspondente
termo da série (2) ou lhe é igual, tem-se para n > 2

S n> sf- (3)

Calculemos as somas parciais da série (2) para os valores n iguais


a 2 \ 2^ 2^ 2». 2*:

S2=l + | - = 1 + 1 - | - ,

«4= 1 + y + ( - ^ + ^ ) = 1 + y + - |- = 1 + 2 ~ ,

í8=l + y + ( y + y ) + ( y + y + y + y ) = 1 + 3 .1 ,

S i 6 = l+ l + ( l+ l ) + ( l+ ... + l ) +

4 tci mos

+ (i^ + ..+ i^ )= i+ ^ 4 ’
8 termos

S 3 2 - l+ l + ( l + l) + ( l+ ... + l) +

4 lermos

+ ( i^ + - . + 4 ) + ( 4 + - + 4 ) = ^ + ^ 4 =
8 termos 16 termos

1 1
calcula-se, do mesmo modo, «26 = 1 + 6--^ , «27 = 1 + 7 - c, em
A Z Z
geral. «24 = 1 + * . . ^ .
270 CALCULO D IFEREN CIA L E INTEGRAL

Por conseguinte, as somas parciais da série (2) podem ser supe­


riores a qualquer número positivo, fazendo k suficientemente grande,
isto é, que
lim s^n = oo,

mas resulta, então, da relação (3) que


lim
n-^oo
isto é, que a série harmónica (1) diverge.

§ 3. Comparação das séries com termos positivos


Sejam duas séries de termos positivos

+ ^2 + ^3 + • • • + + ••M (1)
+ ^’3 + • • • + + ••• (2)
Temos os seguintes teoremas.
©
Teorema— 1. Se os termos da série (1) não forem superiores
aos correspondentes termos da série (2), isto é, se
Un<.Vn{n = \, 2, . . . ) , (3)
e se (2) converge a série (1) converge também.
Demonstração — Designemos por e o„ as somas parciais da
segunda série:
n
^
i= l
Resulta da condição (3) que
Sji On* (4)

Como a série (2) converge, as suas somas parciais têm limites


lim a„ = o.
n-*-oo

Sendo positivos os termos das séries (1) e (2), tem-se On d o,


e. em virtude da desigualdade (4),
s„<a.
Assim, demonstramos que as somas parciais s,^ são limitadas
Notemos que, quando n cresce, a soma parcial s,^ cresce, e resulta
SÉRIES 271

do facto que a sucessão das somas parciais é limitada e cresce


quando tem um limite (*)
lim Sn = S,
n-*oo
e, evidentemente.
5<a.
O teorema 1 permite pronunciarmo-nos sobre a natureza de certas
séries.
Exemplo — 1. A série

converge, dado que os seus termos são mais pequenos que os termos cor­
respondentes da série

f + -p--!-“2 3 - + - ^ + ••• + - ^ + * • •

que é uma série geométrica da razão a partir do segundo termo. A sua


soma é l A1 , A série proposta converge^ e a sua soma é menor que i ^1 .

Teorema — 2. Se os termos da série (1) não forem inferiores


aos correspondentes termos da série (2), isto é, se
Un > (f>)
e se a série (2) diverge, a série (1) diverge igualmerue.
Demonstração — Resulta da condição (5) que
Sn>On- (6)
Como os termos da série (2) são positivos, a sua soma parcial fJn
cresce com n, e como diverge
lim = oo.
n -^o o

Mas, então, em virtude da desigualdade (6),

l i m Sn = oo,

a série (1) diverge.

* Para se confirmar que a variável tem um limite lembremo-nos


dum critério de existência do limite duma sucessão (ver cap. II, t. I): tuma
variável limitada e crescente tem um limite». No nosso caso, a sucessão das
somas 5^ é limitada e desce, logo tem um limite, a série converge.
272 CALCULO D IFEREN CIA L E INTEGRAL

Exemplo — 2. A série

Yn
diverge, porque, a partir do segundo, os seus termos são superiores aos termos
correspondentes da série harmónica

qu2 como S2 sabe diverge.


Noía — Os dois critérios que se acabam de dar (teorema 1 e 2)
apenas são legítimos para as séries de termos positivos. Continua a
vigorar quando íaltam vários termos numa ou noutra série. Todavia,
estes critérios já não são verdadeiros se uma qualquer das séries possuir
termos negativos.

§ 4. Regra de Alembert
Teorema (regra de Alembert) — Se numa série de termos positivos

+ ^ + + ••• + + ••• (1)


a relação tiver um limite finito I quando n-> oo:
Un
= (2)
n->oo Un

1. a série converge quando / < 1,


2. a série diverge quando / > 1
(se / = 1, nada se pode dizer).
Demonstração— 1. Seja / < 1. Consideremos o número q tal que
K q < \ (fig. 242).

q-L q-L
l' Jinfjll I

Fig. 342

Resulta da definição dos limites e da relação (2) que se tem


para todos os n a partir dum certo número N, isto é, para n ' ^ N ,
a desigualdade
Un +i
<q- ( 2')
SÉRIES 273

Com efeito, como a quantidade —^ tende para o limite /, pode-se


tornar a diferença entre íííLíl e o número / (a partir dum certo N)
Un
inferior em valor absoluto a qualquer número positivo, em particular
2i q — U isto é, que
Un+i - I <q —L
Un
Esta última desigualdade implica (2'). Escrevamos esta igualdade
para diversos n a partir de N:

^N+2<^q^N +i
(3)

Consideremos agora as duas séries


Ui U2 Un Un +í + Í^A'+2 + • • ( 1)
Un “h ÇUn q^UN "}■••• ( 1 ')
(10 é uma progressão geométrica de razão positiva ^ < 1. Logo
converge. Os termos da série (1). são a partir de u^+i^ inferiores
L-f L-f

Fig. 343

aos termos da série (10- Resulta do teorema 1 § 1 que a série (1)


converge.
2. Seja / > 1.
Resulta de lim = / ( / > ! ) que a partir dum certo número N,
n-*-oo Ufi
isto é. para n ' ^ N , sa tem a desigualdade
«n+l.
>1

(fig. 343), ou U n + i > Un para todos os n ' ^ N . Mas isso dizer que os
termos da série crescem a partir do índice iV + 1, logo o termo
geral não tende para zero. A série diverge
18
274 CALCULO D IFEREN CIA L 'E INTEGRAL

Exemplo — 1. Estudar a natureza da série

1 . 1 . 1
1.2.3...
Resolução — Tem-sc
1 1 1 1
1.2 .. •n n! ’ 1.2.....n(«4-l)' (« - h l) I ’
J^n+l _ n 1 1
u,n (n + 1) ! «+ 1 '
Por conseguinte,
lim — lini =0 < 1.
n -|-1
A série converge.
Exemplo — 2. Estudar a natureza da série
2 22 23 2^
T + ^ + - r + - + ^ + -
Resolução — Aqui, tem-se
2^ 2^+1 .2 —^ ; l i m - í ^ = Um 2 - ^ = 2 > l .
Wn = í n+ l ’ u„ ^« +
4 "11 n-> oo n-> oo ^

A série diverge, de resto o seu termo geral tende para infinito.


Nota — A regra de Alembert permite ver se uma série positiva
converge sòmente no caso em que lim existe e é diferente de 1.
n-voo ^71

Se este limite não existir ou se lim = 1, a regra de Alembert


n -v o o n
não permite concluir que a série converge ou que diverge, porque
pode, então, tanto convergir como divergir.
Para determinar a natureza de uma tal série, recorrer-se-á a
um outro critério.
n-ri
Notemos, no entanto, que se lim 1 e se o quociente
n -c o U Un
for maior que a unidade para n suficientemente grande, a série diverge.
Com efeito, se > 1, então Un +i Un e o termo geral
Un
não tende para zero quando n oo.
Ilustremos o que foi dito com exemplos.
Exemplo — 3. Estudar a convergência da série

'2 + ' 3 + T + " ' n+l + ■


SÉRIES 275

Resolução — Tem-Se

lim ^n-H
n-*>oo Uji 1V-H30 ^ n-¥CO
w-j- 1

A série diverge, porque i í í i ± i > l para todo o n:


un
^n+l _ n^-\-2n-\-í
Un «2 _|_ 2n > 1 .
Exemplo — 4. Apliquemos a regra de Alembert à série harmónia
1 1 1
1 + 2 + T + -+ I + -
Tem-se = —- e, por conseguinte,
^ /I ’ ^n+i n + i

lim -íííí±L= lim = 1.


n->oo **7i n->oo 'í + 1

Portanto, a regra de Alembert não permite dizer nada sobre a con­


vergência ou divergência da série harmónica.
Mas estabeleceu-se doutro modo que esta série diverge.
Exemplo — 5 Estudar a convergência da série
1
——h
1 .2 ^ 2—.3H
^—-—h
3 -4 ^ • • • • n (/i+ 1)
Resolução — Tem-se
.. 1 .. 1
**"~n(n + l ) ’ ( n+l ) ( r a + 2 ) ’

lim Jía± L = lim Um = 1.


n -.o o n -.o o ( ^ “h 1 ) ( ^ “1” n->oo n + 2

A recra d’ Alembert nada dá. Mas oode-se demonstrar que esta série
converge por outras considerações. Com efeito,
1 1 1
/i(n+l)~n n+ 1 ’
e pode-se recopiar a série dada sob a forma

(t “ t ) + (t “ t ) + (t ~ t ) + • •• + ) + • ••
Reduzindo os termos semelhantes, obtém-se a expressão da soma parcial Sn :

fi -|“ 1
Por conseguinte,
\ ,
l i m s , i = l i m ( l -----
n-¥co n-¥co
n->oo V
\ /
a série converge e a sua soma é igual .a 1.
276 CALCULO D IFEREN CIA L E INTEGRAL

§ 6. Regra de Cauchy
Teorema (regra de Cauchy) — Sendo dada a série
+ ^2 + + •••+ + ••• ( 1)

de termos positivos, se a quantidade Y tiver limite finito I quando


/i->oo, isto é, se se tiver
n ---
lim y Un = l,
n->oo

1. a série convege se I < l


2. a série diverge se l > l.
Demonstração — 1. Seja / < 1. Seja q um número tal que / < ^ < 1.
Ter-se-á, a partir dum certo n = IV

\V
Dal resulta que
VUn < q
ou melhor
Un<q"
para todos os m M
Consideremos, agora, as duas séries:
4 " ^2 “h ^3 “I” • • • “I” “h ^JV + l 4 “ ^N+2 4 “ • • ( 1)
N , JV+ 1 , N+2 I
q +q +q 4 - ... ( 1')
A série (10 converge porque os seus termos formam uma pro­
gressão* geométrica decrescente. Os termos da série (1) são. a partir
de inferiores aos respectivos termos de (10- Logo a série (1) converge.
2. Suponhamos / > 1. Ter-se-á, a partir dum certo n = N
Vu„>l
ou mrihor
u„ > 1 .
A série diverge, evidentemente.
Exmnple — Estudar a convergência da série

Resolução — Apliquemos a regra de Cauchy:

7,— n r
n->co ?i-»-oo ' \ i / n V = 2 < ‘-
A série converge.
SÉRIES 277

Nota — Tal como para a regra de Alembert o caso em que

lim Z=1
n-^oo
exige um estudo particular. A série pode. então, tanto convergir eomo
divergir. Assim, para a série harmónica (que. como se sabe. diverge)

lim V lü ^ = lim 1 / ~ = 1.
n-^oo n->-oo ' W
VT
Para nos assegurarmos disso, mostremos que lim Log 1 / — =»= 0.
n-+oo y
Com efeito,
X 1 /1 1. — Logn
hm Log 1/ — = l i m --------------- .
n-^oo ^ n 71-^00 n

O numerador e o denominador desta fracção tendem para infinito.


Apliquemos a regra d’Hospital:
_ _ J.
Ikn Log lim u„ — 1=0.
n-*-oo ’ n n->-oo n n-*<» 1

Assim, Log 1/ ^ ^

lim ] / ^ = í .
n-^ 0 0 ! n
O mesmo se diga da série
1 .1 .1 _r
n-2
para a qual

lim n . = lim l / X - lim i T - ! - - = !•

Mas esta série converge porque a partir do «agundo, os seus


termos serão inferiores aos da série convergente
1 1 1
— + - ^ + . . . H ------- -------- !-•••
l-2~2-3~ «( n + 1)
(ver exemplo 5, § 4),
278 CALCULO D IFERENCIA L E INTEGRAL

§ 6. CcHnparação com um integral


Teorema — Seja a série de termos positivos não crescente
+ ^^2 + ^3 + • • • + (1)
isto é,
^ • •I
e seja / (;c) uma função contínua não crescente tal que
f{ í) = Ui; /(2) = U2: • • • : f{n) = u . . (2)
Pode-se, então, afirmar que:
1. se o integral
oo
J / (x) dx

converge (ver § 7, cap. XI. t. I). a série íl) converge igualmente:


2. se o integral diverge, a série (1) converge.

Demonstração — Representemos os termos da série geomètrica-


mente, reportando ao eixo das abcissas, os números dos termos 1,
1, 3, n, ^ + 1. ... e verticalmente os seus valores Wi, 1/2........
(fig. 344).
ConstruSmos, sobre a mesma figura, o gráfico da função não
crescente
y = f (^ )

que satisfaz à condição (2).


Nota-se, na figura 344, que o primeiro rectângulo construído tem
por base 1 e por altura f (1) = Ui. A área deste rectângulo é, pois, u^,
A área do segundo rectângulo é Uo, e a área do de ordem n e último
rectângulo construído é A soma das áreas dos rectângulos cons­
truídos é igual à soma dos n primeiros termos da série. Por outro
SÉRIES 279

lado a figura em escada formada por estes rectângulos contém o


domínio limitado pela curva y = f(x) e as rectas jc = 1, ac = /i + 1,
Tl-f"1
y = 0; a área deste domínio é igual a j / (x) dx.
'i
Por conseguinte.
n + l

s»> j i(x) ãx. (3)


1
Consideremos, agora, a fig. 345. Aqui, o primeiro rectângulo
construído tem por altura e a sua área é também Wo. A área do
segundo rectângulo é M3, etc. A área do último rectângulo construído
é ^ 1+1* Por conseguinte, a área de todos os rectângulos construídos
é igual à soma dos ( n + l ) primeiros termos da série menos o pri­
meiro, ou seja. Sn+i — Ui. Por outro lado. como é fácil de ver,
a figura em escada formada por estes rectângulos está compreendida
no trapézio curvilíneo formado pela curva y = f(x) e ãs rectas x = 1,
n + l

x = n + l , y = 0. A área deste trapézio curvilíneo é igual a | / (a:) dx.


Logo n+l

«n+l — » 1 < í f(x )d x ,


1

donde n+l

» n + l < í f(x )d x - j^ U ,. (4)


1

Consideremos, agora, os dois casos.


00
1. Suponhamos que o integral I f (x) dx converge, isto é, que
1
tem um valor finito.
Como n+l 00
j f (x) dx < y (x) dx,
i 1
tem-se, em virtude da desigualdade (4),
00

1
isto é, que a soma parcial é limitada qualquer que seja n. Ora,
ela cresce com n, porque todos os Un são positivos. Logo tem
um limite finito quando n oo

lim Sn
n-foo
a série converge.
280 CALCULO DIFEREN CIA L E INTEGRAL

2. Suponhamos em seguida que \ f {x) dx = oo.Tal quer dizer


n-hl í
que I f (:r) dx cresce indefinidamente com Mas, então, em virtude
i
da desigualdade (3), cresce também indefinidamente com n, k4o é,
a série diverge.
O teorema, está pois, completamente demonstracfo.
Exemplo — Estudar a convergência da série

1 1 1 1
IP n- 2p ^ 3P 4" • • • n- .

Resolução — Comparemos com o integral da função

que satisfaz a todas as condições do teorema. Considermos o integsal

= {Ní-p— 1) para p i.

1 VLogx |J^ = Log para


pa p =1.

Façamos tender N para infinito e estudemos a convergência do integral


segundo os casos.
Poder-se-á, então, julgar da convergência do integral segundo os valores de p.
oo
Se p > 1, j ^ ^ , o integral é finito, logo a série diverge.
1
oo
dx
Í - ^ = oo, o integral é infinito, a série diverge.

. r dx
Se P — 1, \ — —oo, o integral é infinito, a série diverge.

Notemos que nem a regra d*Alembert nem a dc Cauchy resolvem a


questão da convergência desta série. Com efeito.

lim ^n+í — lim : 1,


n -voo n —►oo

l/- ^ = lim = .i P ^ \ ,
n-^oo 71-^00 ^ 71-voo V r /i /
SÉRIES 281

§ 7. Séries alternadas. Teorema de Leibniz


Considerámos, até agora, séries de termos positivos. Neste pará­
grafo vamos considerar séries cujos sinais dos termos são alternados,
isto é, séries da forma
l/j — Uo W3 — “1“ • • •»
em que Wi. «2........ • • são positivos.
Teorema de Leibniz — Se numa série alternada
Ui — 1/2 + — W4 -f- • • . ( U n ! > 0 ), (1)

os termos vão decrescendo


l^l>U2>^/3>. . . (2)
e se
lim w „ = 0, (3)
n -^00

a série (1) converge, a sua soma é positiva e não é superior ao primeiro


termo.
Demonstração — Consideremos a soma dos n = 2m primeiros ter­
mos da série (1):
S2m — (Ui — U2)^{U q— Ui^-\- . . . + (í^2m-l — í^2m)«
Resulta da condição (2) que as expressões entre parêntesis são
positivas. Logo a soma ^2m é positiva

*2m > 0
e cresce com m.
Recopiemos, agora, esta soma sob a forma
S2m = “ 1— ( « 2 — M3) — ( « 4 — W5) — . . •

• • • — (W 2 m - 2 — ^2m-i) — “ 2m-

Em virtude da condição (2), cada expressão entre parêntesis é


positiva, logo, subtraindo todas as expressões entre parêntesis de Ui,
obtém-se um número inferior a «i, isto é, que
S2m < «1-
Por conseguinte, estabelecemos que cresce com m e é limitada
superiormente. Resulta que «zm tem um limite s:
lilU $2m =
Q m -.o o

0< s< uj.


282 CALCULO D IFERENCIA L E INTEGRAL

Todavia, não demonstrámos ainda que a série converge; demons­


trámos somente que a sucessão das somas parciais pares tem um
limite 5. Demonstremos agora que as somas parciais ímpares tendem
também para s.
Consideremos, para esse efeito, a soma dos n = 2m + ] primeiros
termos da série (1):
^ 2 m + l = = ^2m + ^ 2 m + l-

Como, segundo a condição (3). lim í^2m+i =- 0» tem-se


m -yoo

l i m S2 m+i = l i m S2 m + ü m U2 m+i = ü m S2 m = s.
Y fl-y -C K > m -> o o 771->00 jn -y -o o

Do mesmo modo, demonstramos que lim = s quer n seja par


n-y-oo
quer seja ímpar.
Logo a série converge.

^5 ^3
Fig. 346

N ota — 1. O teorema de Leibniz pode ser ilustrado geométrica­


mente como se segue. Reportemos ao eixo numérico as somas parciais
(fig. 346):
Sj = U i, $2 = Wi — U 2 = ^1 U 21 ^3 ^2 H“ ^3»

^4 = ^3 — ^4» ^5 = ^4 + ^5»
e assim sucessivamente.
Os pontos que representam as somas parciais tendem para um
ponto 5 que representa a soma da série. As somas parciais pares
encontram-se à esquerda de s, as somas parciais ímpares à direita de s.
Nota — 2. Se uma série alternada satisfaz à condição do teorema
de Leibniz. não é difícil avaliar o erro cometido quando se substituir
a sua soma s por uma soma parcial ^T^-Isto equivale a desprezar todos
os termos a partir de ^^77+1. Mas estes termos formam uma série
alternada cuja soma é, em valor absoluto, inferior ao primeiro termo
SÉRIES 283

desprezado (í^ + i). Por conseguinte, o erro cometido quando se substi­


tui s por Sj^ não ultrapassa em valor absoluto o primeiro termo des­
prezado.
Exemplo — 1. A série harmónicía alternada

1----- 4 - ^ ___ L_l


2 ^ 3 4
converge porque

2) lim Un= lim — = 0 .


n -fo o n-*oo ^

A soma dos n primeiros termos desta série

1 1 1 1

difere da soma s da série por uma quantidade inferior a


n+ í •
Exemplo — 2. A série

1---- 1—4 1 t----


11 L
2! ^ 31 41
converge em virtude do teorema de Leibniz.

§ 8. Séries de termos de sinais quaisquer


Convergência absoluta e semi-convergência

Uma série diz-se de termos de sinais quaisquer se se encontra


entre os seus termos, quer termos positivos quer termos negativos.
As séries alternadas do parágrafo anterior são, evidentemente,
um caso particular das séries de termos quaisquer.
Vamos examinar algumas propriedades das séries de termos de
sinais quaisquer.
Contràriamente à convenção adoptada no parágrafo anterior, admi­
tiremos daqui em diante que os números Wi, W2. ...» ... podem ser
quer negativos quer positivos.
Vamos dar, em primeiro lugar, um importante critério suficiente
da convergência das séries de termos de sinais quaisquer.

Teorem a— 1. Se a série de termos de sinais quaisquer

+ ^2 + • • • + + ••• ( 1)
284 CALCULO D IFEREN CIA L E INTEGRAL

é tal que a série formada com os valores absolutos dos seus termos
I I “h I ^2 I + • • • + I I “h • • • (2 )
converge, a série proposta também converge.
Demonstração — Sejam e as somas dos n primeiros termos
das séries (1) e (2).
Sejam ainda s'n a soma de todos os termos positivos e a soma
dos valores absolutos de todos os termos negativos contidos nos n
primeiros termos da série proposta
Sn = Sn — Sn\ CT^ = 5,^ + .
Por hipótese, tem por limite o ; e Sn são quantidades
positivas crescentes inferiores a a. Logo, têm limites s e i". Resulta
da relação = Sn — sú q u e também tem um limite que é igual
a / — "5, isto é, que a série de termos de sinais quaisquer (1) converge.
O teorema demonstrado permite julgar da convergência de certas
séries de termos de sinais variáveis. O estudo da questão da conver­
gência duma tal série reduz-se, então, ao estudo duma tal série de
termos positivos.
Consideremos dois exemplos.
Exemplo — 1. Estudar a convergência da série

sen a , sen 2a , sen 3a sen na


12 ' 22 f ...H- «2 (3 )
32
onde a é um número qualquer.
Resolufão — Consideremos, paralelamente à série proposta, as séries

sen a sen 2a sen 3a sen na


12 + + 32 + •- n2 (4)

1 1
12 ^ 22 ( 5)

A série (5) converge (ver § 6). Os termos da série (4) não são superiores
aos termos da série (5); logo (4) também converge. Resulta do teorema
demonstrado que a série (3) converge também:
Exemplo — 2. Estudar a convergência da série
JX ^ 3X _ JX
COS - 7- COS 3 COS 5 - 7- COS (2n — 1)
4 , 4 , 4
3 i 32 I 33 T ••• +I 37» ( 6)

Resolução — Consideremos, ao mesmo tempo que a série proposta, a série

3 -1
J3 2_ +r _3L3 +
- r • • • I 3« 0)
SÉRIES 285

Esta série converge, porque é uma progressão geométrica de razão — .


o
Resulta daí a convergência da série dada (6) porque os seus termos são
inieriores em valores absolutos aos termos da série (7).
Notemos que o critério de convergência demonstrado acima é
simplesmente suficiente para que uma série de termos quaisquer con-
verja, mas não necessária; existem séries com termos de sinais variáveis
que convergem, mas cujas séries dos valores absolutos divergem.
Daí, ser útil introduzir as noções de convergência absoluta e
semi-convergência para as séries de termos quaisquer e classificar assim
estas séries.
Definição — A série de termos de sinais variáveis
“h ^2 “h ^3 4“ • • • H“ ••• ( 1)

diz-se absolutamente convergente se a série formada com os vaíores


absolutos dos seus termos

I 1+ I I + I í + • • • + I I+ • • • ( 2)
converge.
Se a série (1) converge, mas se a série (2) diverge, a série pro­
posta (1) diz-se semi-convergente ou não absolutamente convergente.
Exemplo — 3. A série harmónica alternada
1 1 1
1---- - 4 - - ------—4 - . . .
2 ^ 3 4 ^

é semi-convergente, porque a série dos valores absolutos é a série harmónica


1 1 1
1+T+T +T + -
que diverge. A série harmónica alternada converge, como resulta do critério
de Leibniz.
Exemplo — 4. A série
1 1 1

é absolutamente convergente, dado que a série dos valores absolutos


1 1 1

converge, como ficou estabelecido no § 4.


Utilizando a noção de convergência absoluta, formula-se muitas
vezes o teorema 1 como se segue: uma série absolutamente convergente
é convergente.
Indiquemos para terminar (sem demonstração) as seguintes pro­
priedades das séries absolutamente convergentes e semi-convergentes.
286 CALCULO D IFEREN CIA L E INTEGRAL

Teorema — 2. Se uma série converge absolutamente, ela converge


absolutamente quando se muda arbitràrramente a ordem dos seus termos,
A soma duma tal série não depende da ordem dos seus termos,
Esta propriedade não se conserva para as séries semi-convergentes.
Teorema — 3. Se uma série é semi-convergente, pode-se reagrupar
os seus termos de maneira que a soma da nova série obtida seja igual
a um número A dado antecipadamente, Além disso, pode-se reagrupar
os termos duma série semi-convergente de maneira que essa nova
série seja divergente.
A demonstração destes teoremas sai do âmbito deste curso. Para
ilustrar o facto de que se pode reagrupar os termos duma série semi-
-convergente de maneira a modificar a sua soma, consideremos o
exemplo seguinte.
Exemplo — 5. A série alterada

1— 2—^+ 3—-----—+
4 ^
... ( 8)

não converge absolutamente. Seja s a sua soma. Tem-se, evidentemente, s > 0.


Reagrupemos os termos de (8) de modo que um termo positivo seja seguido
de dois termos negativos:
A A A A 4

. (9)

Mostremos que a série obtida converge, mas que a sua soma / é duas
A
vezes menor que a soma da série (8), isto é, que é igual a — s. Sejam c
2
as somas parciais das séries (8) e (9). Consideremos a soma dos 3k termos
da série (9):

"3'‘" + ••• + “

= (t ~ t ) + (t ~ t ) + •• • + =

' ' T [ ( ^ “ T ) + ( T “ T ) + "- + ( 2 f c ^ “ Í ) ] =


1 1 , 1 1 , , 1 1 1 .
’ 2 V ^~ 2 + 3 4 ■*■••• + 2fc — l 2fc 2 *2'''
Por conseguinte. 1 1
lim lim -5- í2ft = - ^
ft-fCO /í-VOO ^ ^

Depois 1 \ 1
lim s 1 = lim (s g
A->>oo k-*oo \ 3» ' 2k +
+ \l ) “ 2
1
lim s 2k f-2= lim I 5Ó
fe-voo k-^co \ ^ 2fe + l Ak + 2} ~ 2
SâR IE S 287

Logo, obtém-se
lim
n^oo ^

Vê-se que a soma da série mudou após reagrupamento dos seus termos
(diminuiu de metade).

§ 9. Séries de funções
Chama-se série de junções a toda a série na qual o termo geral
é uma função duma variável x.
Consideremos a série de funções

Uj (x) + U2 (x) + U3 (x) + . . . + (x) + . . . (1>

Dando a x diferentes valores numéricos obtém-se diferentes séries


numéricas que podem tanto convergir como divergir.
O conjunto dos Valores de x para os quais a série de funções
converge chama-se domínio de convergência dessa série.
É evidente que, no domínio de convergência duma série de funções,
a sua soma é uma certa função de x.
Eis porque se a designa por ^(jc).
Exemplo — Consideremos a série
l + a: + x2- f . . . + x ^ + . . .

Esta série converge para todos os jc no intervalo (— 1, 1), isto é,


para todos os x que satisfaçam à condição | j c | < l . Para todo o valor de x
deste intervalo, a soma da série é igual a (a soma duma progressão
í —x
geométrica, decrescente de razão x). Por conseguinte, a série proposta define
no intervalo (— 1, 1) a função

S ( l) =
1— X

que representa a soma da série, isto é, que


1 = l+ a; + x2-t.x3+.
í —x
Designemos por (x) a soma dos n primeiros termos da série (1),
se esta série converge e se a sua soma é s(x), então,
s(x) = s„(x) + r„(x),

onde r„ (x) é o resto da série (1):


r„ (x) = w„+i (x) + «„+2 (a:) + . . .
288 CÁL.CULO DIFERENCIAL. E INTEGRAL.

Temos para todos os x do intervalo de convergência lim s^, (x) =


n-^oo
= s (x), logo
lim r„ (x) — lim [s (x) — s„ (x)] = 0,
71-► oo

o que mostra que o resto (x)duma série convergente tende para


zero quando oo.

§ lO. Séries majoffrveis


Definição — A série de funções
U ,(x ) + M 2 (^ ) + « 3 ( ^ ) + • • • + W n (-Í^ ) + • • • (1 )
diz-se majorável num certo domínio de variação de x, se existir uma
série numérica convergente de termos positivos
^1 + 0^2 + 0^3 + • • • + + ••• (2)
tal que se tenha para todo o x do domínio considerado
|w i(x )|< a ,, |i/2 ( x ) |< a 2 , |U n(x)|< a„, . . . (3)
Por outras palavras, uma série é majorável se cada um destes
termos não for superior em valor absoluto ao termo correspondente
duma série numérica convergente de termos positivos.
Assim, a série
COS X c o s 2 .r , c o s 3 :r ^ cosnx
o2 I I •••
~ r~ 3"

é majorável sobre todo o eixo Ox. Com efeito, tem-se para todos os
valores de x a relação
cosnx 1
< (^ = 1, 2, ...) ,

e sabe-se que a série

converge.
Resulta imediatamente da definição que uma série majorável
num certo domínio é absolutamente convergente em todos os pontos
desse domínio (ver § 8). Além disso, numa série majorável goza da
importante propriedade seguinte.
SÉRIES 289

Teorema — Suponhamos que a série de junções


(^) + ^^2 (^) + • • • + ^ n W + • • •
é majorável sobre o se^menfo [a, h]. Sejam s (x) a soma desta série,
a soma dos seus n primeiros termos. Então, corresponde a
qualquer e > 0 arbitrar lamente pequeno um número N tal que para
todos os n '^ N
\s(x) — (x) I < e,

qualquer que seja x sobre o segmento [a, b \


Demonstração — Designemos por tr a soma da série (2):
a = a i + a 2 + as + . . . + a,i + a„ + i + . .
Tem-se
= CTn +
em que o,, é a soma dos n primeiros termos de (2) e o resto
desta série:
t'7, = ^n +i + OÍ71+2 + • • •
Como esta série converge, tem-se
lim 0,1 = 0
71-► CX'
e, por conseguinte.
lim 8,1 = 0 .
7í -► 0 0
Escrevamos, agora, a soma da série de funções (1) sob a forma
s (x) = Sn (x) + r„ (x),
em que
Sn ( X ) = U i (x) + . . . + Un (x),
fn i^) = “ n + 1 W + “ n + 2 (^ ) + U„+3 + •■ •
Resulta da condição (3) que
I ^ n + l (•^) I I ^ n + 2 (^ ) I O ín+2t • • •
e, portanto,
I (-^) I
para todos os x do domínio considerado.
Í s (-í;) — s „ ( x ) |< e „

para todo o jc do segmento [a, b] e e,^ -► 0 quando « oo.


Nota — O resultado obtido pode ser ilustrado geomètricamente
como se segue.
Consideremos o gráfico da função y = s Cr). Construamos uma
faixa de largura 2sn . sobre esta curva, isto é, construamos as curvas
y = 5 (x) + 8^ e y = s ( x ) - En (fig. 347).
19
290 CALCULO D IFERENCIA L E INTEGRAL

Nestas condições qualquer que seja o gráfico da função Sn (x)


ficará contido completamente nesta faixa que conterá igualmente os
gráficos de todas as somas parciais seguintes.
Nota — Numa série de funções arbitrárias convergindo sobre o
segmento [a, b], não goza forçosamente da propriedade demonstrada

no teorema. Mas existem séries não majoráveis que gozam da referida


propriedade. Toda a série que goza desta propriedade diz-se unifor-
memente convergente sobre o segmento [a, b].
Assim, a série de funções Ui (jc) + «2 W + ... 4- u,^ (jc) + ... diz-se
uniformemente convergente sobre o segmento [a, b] se corresponde a
todo o e > 0 arbitràriamente pequeno um número N tal que, para
todo o n '^ N, se tenha
\s(^) — S n { ^ ) \ < e
qualquer que seja x sobre o segmento [a, b \
Resulta do teorema demonstrado que uma série majorável é
uniformeménte convergente.

§ 11. Continuidade da soma duma série


Seja uma série de funções contínuas
u, (x) + U2(a:) + ,. . + Ií„ (x) + . . .
convergente sobre um segmento [a, 6].
Demonstramos no capítulo II (t. I) um teorema sobre a conti­
nuidade da soma de um número finito de funções continuas. Esta
propriedade já não é conservada para a soma duma série (que contém
SÉRIES 291

uma infinidade de. termos). Certas séries de funções contínuas têm


por soma funções descontínuas.
Exemplo — Consideremos a série
JL J_ JL ^ ± 1 1

Os termos desta série (que figuram entre parêntesis) são funções conti­
nuas qualquer que seja jc. Mostremos que esta série converge e que a sua soma
é uma função descontínua.

Achemos a soma dos n primeiros termos desta série:


1

Calculemos a soma da série:


Se ;c > 0 1
5 = lim (x )= lim (x 2’^'Hi —x) = l —x,
n —yoo n —yoo

Sc a: < 0 1
s = lim Sn {x)= lim (— | x \ — x) = — 1— x
n-¥co n-> oo

Se JC= 0, tem-se = logo s - lim s^ = 0. Tem-se, pois:


n-^co
S(i) = l _ x para ® > 0 ,
s(i) = — 1—I para i < 0 ,
s(x) = 0 para x = 0.
Assim, a soma da série considerada é uma função descontínua. Repre­
sentamos o seu gráfico na figura 348, bem como das somas parciais (x), (;c)
e í,W.
292 CALCULO D IFEREN CIA L E INTEGRAL

O teorema seguinte diz respeito às séries majoráveis.


Teomera — A soma duma série de funções contínuas majorável
sobre o segmento [a, b] é uma função contínua sobre esse segmento.
Demonstração — Consideremos a série de funções contínuas majo­
rável sobre o segmento [a, b]
Ui (x) + Uz (x) + U3 (x) + . . . (1)
Escrevamos a sua soma sob a forma
s(x) = s„ (x) -t-r„ (x),
onde
s„ (x = Uj (x) + . . . + u„ (x),
c
^71(^) ^71+ 1 (^) H” ^71+2 (X) -+“ • • •
Tomemos sobre o segmento [a, b] um x arbitrário e demos-lhe
um acréscimo Ax, tal que jc -f Ajc pertença a [a, b \
Introduzamos as notações
As = 5 (x + A:r) — s{x);
As^ = S n ( x + Ax) — Sn (x) |
então.
As = A s^ + rn{x + Ax) — (x),
donde
A sj < 1A sJ + I rn {x + Ax)\ + \ (x) |. (2)

Esta igualdade é verdadeira para todo o n.


Para demonstrar a continuidade de s(jc), é preciso demonstrar
que, qualquer que seja e > 0 arbitràriamente pequeno, antecipadamente
dado, existe 8 > 0 tal que, para todos os j A j c ! < 8, se tenha | As | < e.
Como a série proposta (1) é majorável corresponde a todo o
e > b um n tal que
e
I rn {^ ) I < (3)
3 ’

qualquer que seja jc sobre o segmento [a, b]. O número jc + A jc per­


tence ao segmento [a, 6], logo

i rjí (x + Ax) | < -^ . (3')

Por outra via, para o N escolhido, a soma parcial (x) é uma


função continua (a soma dum número finito de funções contínuas), e.
SÉRIES 293

por conseguinte, pode-se escolher 8 positivo tal que. para todo o Ajc que
satisfaça à condição | Ajc | < 8 se tenha

I Aspf I < • (4)

Resulta das desigualdades (2). (3). (3'). (4)

isto é.
I < e . visto que | Ax | < ô,
o que prova que 5(;c) é uma função contínua no ponto x (e, portanto,
em qualquer ponto do segmento [a, Z?]).
Nota — Resulta do teorema demonstrado que se a soma duma
série e descontínua sobre um segmento dado [a, b], a série não pode
ser majorada sobre esse segmento. Assim,, a série estudada como
exemplo não pode ser majorada sobre todo o segmento que contém
o ponto .X= 0 no qual a série é descontínua.
Notemos, por fim, que o recíproco não é verdadeiro; existem
séries não majoráveis sobre um segmento mas que convergem sobre
esse segmento para uma função contínua. Em especial, qualquer série
uniformemente convergente sobre o segmento [a, b] (mesmo se não
for majorável) tem por soma uma função contínua (se, bem entendido
todos os seus termos fòrem contínuos).

§ 12. In teg ração e derivação de séries


Teorema— 1. Seja a série de junções contínuas
Ui (x) + «2 ( i ) + • •• + “ n W + • •• ( 1)

majorável sobre o segmento [a, b] e seja s(x) a sua soma. O integral


de s (x) entre a e x, pertencente a [a, i], é igual à soma dos integrais
dos termos da série entre os mesmos limites:
X X X X
I S {x) dx = I U^{x) dx I U2 {x) dx I Un{x) dx .
a a a a
Demonstração — A função s{x) pode ser posta sob a forma
s(x) = s„^xj + r„(x)
ou
s(x) = Ui(x) + M2 (x ) + . . . -I- (x) + r„ (x).
294 CALCULO D IFEREN CIA L E INTEGRAL

Tem-se

5 s (x) dx = J (x) dr + J (^) d r + • • •


a a a
X X

. . . + ^ U n ( x ) d x + I r n( x) dx ( 2)

(o integral duma soma finita de termos é igual à soma dos seus


integrais).
Como a série proposta (1) é majorável, tem-se, qualquer que
seja X, I r„ (x) | < ; e„, onde e„ 0 quando « ^ oo. Logo

I f r„ (x) d x | < ± S I r„ (x) I d x < ± S e„ dx =


a cc cx

= ± e „ (x — a ) < e „ (6 — a).
Como e„ -► 0, tem-se
X

lim 5 r„ (x) dx = 0.
n-^oo a
Mas, deduz-se de (2)
* * * *
J r n ( x ) d x = J s { x ) d x — [ J i i i ( x ) d x + . . . + ^ u „ (x )d x l.
a a a a

Por conseguinte,
X X X

lim { j s (x) dx — [ J ui (x) dx + . . . -1- J u„ (x) dx ]) = 0


n-»»oo a
OU

lim [ j Ui (x) dx -f . . . + j w„ (x) dx ] = J s (x) dx. (3)


7i-^oo a

A soma entre parêntesis recto é uma soma parcial para a série


X X

í u, (x) dx - f . . . - f í u„ (x) dx - f . . . (4)


a a
Como as somas parciais desta série têm um limite, esta série
X

converge e a sua soma é igual, em virtude de (3), a | s (x) dx, ou seja


a
X X X X

5 s(x)dx= 5 u i ( x ) d x - f ^ U2 ( x ) d x - | - . . . - f ^ u „ (x) d x .
a a a a
£ a igualdade- que nos propúnhamos demonstrar.
SÉRIES 295

N ota— 1. Se a série não for majorável, não é sempre possível


X

integrar termo a termo, isto é. que o integral J s {x) dx da soma


a

da série (1) não é sempre igual à soma dos integrais dos seus
termos (isto é, a soma (4)).
Teorema — 2. Se a série
Ui (^) + (^) + • • • + (^) + • • • (5)
de funções que têm derivadas contínuas sobre [a, 6], converge sobre
este segmento para 5(jc) e se a série
Ui (^) + ^2 (*^) + • • • H“ (^) + • • •» (®)
formada com as derivadas dos seus termos, é majorável sobre este.
segmento, então, a soma da série das derivadas é igual à derivada da
soma da série proposta:
s (x) = u[ (x) + Ü2 (x) + u^{x) + . . . + Un(x) + . . .
Demonstração — Designemos por F(jc) a soma da série (6):
F (x) = u\ {x) + ^2 (o:) + . . . + Un (^) + • • •
e demonstremos que
F (x) = s (x).

Como a série (6) é majorável, tem-se, em virtude do teorema


anterior.
X X X X

I F (x) d x = ^ u'i (x) dx I Uzix) dx + . . . u’n {x) dx + . . .


a a a a

Integrando no segundo membro, obtém-se


ac
I F{ x ) d x = [ui (x) — ui (a)] +
a

[U2 (^) — U2 (oc)] -f" • • • “1” \Un {^) Un (ot)] 4" • • •


Mas
S{x)= {x) - |- U2 {x) “1- (^) “!” •••»

S (oc) = 1^1 (oc)>f‘ í^ 2 (tt) 4“ • • • “t” U ji (oc) 4" • • •»

quaisquer que sejam jc e a sobre o segmento [a, 6]. Por conseguinte.

] F( x ) d x = s{x) — s{a).
296 CALCULO D IFEREN CIA L E INTEGRAL

Derivando em relação a x, os dois membros desta igualdade,


obtém-se ,, ,
F{x) = s (x).

Por conseguinte, demonstramos que sendo as condições do teo­


rema satisfeitas, a derivada da soma duma série é igual à soma das
derivadas dos seus termos.
Nota — 2. É muito importante que a série derivada seja majorável,
porque se esta condição não for observada, a derivação termo a^*termo
pode-se tornar impossível.
Para confirmar este facto, consideremos um exemplo duma série
majorável que não possa ser derivada termo a termo.
Consideremos a série
senl^x sen . sen sen n x
+ — !-••• +
r
Esta série converge para uma função contínua, porque é majo­
rável. Com efeito, qualquer que seja x, os seus termos são inferiores,
em valores absolutos, aos termos da série numérica convergente
1 . 1 1
+ T r + T ^ + --- + - ^ + ---
1 O n
Escrevamos a série formada com as derivadas dos termos da
série proposta:
COS X + 2 ^ COS 2 ^ + • • • + cos n^x + . . .

Esta série diverge. Assim, quando x = 0. transforma-se em


1 + 2^ + 3^ + . . . + + ...
ípoder-se-ia demonstrar que diverge não sòmente para x = 0).

I 13. Séries inteiras ou séries de potências


Intervalo de convergência
Definição— 1. Chama-se série inteira ou série de potências a
uma série da forma
^0 H” “h 0,2^^ -f- . . . -f- üj^X^ -|- (1 )
em que ao, fli, a2. .... ... são constantes dadas, chamadas coeficientes
da série.
O conjunto dos pontos de convergência duma série é um intervalo,
que se pode reduzir a um ponto. Para nos certificarmos disso, demons­
traremos. em primeiro lugar, o teorema seguinte, fundamental na teoria
das séries inteiras.
SARIES 297

Teorema (d’Abel) — 1. a) Se uma série inteira converge para


um certo valor de jCo, não nulo, convergeabsolutamente para qualquer
valor de x tal que
I ^ I <C I ^ 0 1»

b) Se a série diverge para um certo valor de x^, diverge para


qualquer x tal que

Demonstração— 1. Como, por hipótese, a série numérica


^0 “1“ ^1^0 “1“ ^2*^0 “h • • • “h ■!“ ••• (1 )
converge, o seu termo -► 0 quando n -> oo, o que prova que
existe um número positivo M tal que todos os termos da série são
inferiores em valor absoluto a M.
Recopiemos a série (1) sob a forma

ÜQ+ ÜíXq ( ----^ + ^2^0 Í ----1 + • • • + Í ----1 + • • • (1^)

e consideremos a série dos valores absolutos dos seus termos:


2
X X
I^0 I + I<*1^0I + IÜ
2XqI + •••
Xq

• • • + I ^ 71^0 I + ... (2)

Os termos desta série são inferiores aos termos correspondentes


da série

M + M + M + ... + M (3)
Xq

Quando | jc | < | jco |. esta última série é uma série geométrica de


razao < 1, logo, converge. Como os termos da série (2) são
X q

inferiores aos de (3), resulta que a série (2) converge também, o que
significa que a série (la) ou (1) converge absolutamente.
2. Já não é difícil, agora, a segunda parte do teorema: suponha­
mos que a série (1) diverge num certo ponto Então, ela divergirá
também em qualquer ponto x tal que \ x \ > \ x^\- Com efeito, se
ela convergisse num certo ponto x que satisfaça a esta condição, em
298 CALCULO D IFERENCIA L E INTEGRAL

virtude da primeira parte do teorema, convergiria igualmente no


ponto xó, porque | xó | < 1^ 1- Mas, isto é contrário à hipótese que
a série diverge no ponto xó* Logo a série diverge também no ponto x,
O teorema está completamente demonstrado.
O teorema d’Abel permite julgar da disposição dos pontos de
convergência e de divergência duma série inteira. Com efeito, se Xo
é um ponto de convergência, todos os pontos do intervalo (— | Xo 1, \xo\)
são pontos de convergência absoluta. Se xó é um ponto de divergência,
toda a semi-recta à direita de | xó | e a semi-recta à esquerda de — | xó |
são constituídos de pontos de divergência.

a série converge

a série diverge a série diverge

Fig. 349

Isto permite concluir que existe um número R tal que os pontos


IXI < /? são pontos de convergência absoluta, e os pontos | x | > /?
pontos de divergência.
Tem-se. pois, o teorema seguinte sobre a estrutura do conjunto
dos pontos de convergência duma série inteira.
Teorema — 2. O conjunto dos pontos de*convergência duma série
inteira é um intervalo centrado sobre a origem das coordenadas.
Definição — 2. Chama-se intervalo de convergência duma série
inteira ao intervalo compreendido entre os pontos — R c + R tal que
a série converge, e mesmo absolutamente, nos pontos x desse intervalo
e diverge nos pontos x que lhe são exteriores (fig. 349). O número R
chama-se raio de convergência da série inteira.
Nas extremidades do intervalo (isto é. nos pontos x = /? e x = — /?),
a questão de convergência ou de divergência da série proposta deve
ser objecto dum estudo especial.
Notemos que para certas séries o intervalo de convergência se
reduz a um ponto (^ = 0) e para outros estende-se a todo o eixo Ox
(R = 00).
Indiquemos um modo para determinar o raio de convergência
duma série inteira.
Consideremos a série

ÜQ -f- ã \X - |- 4 “ • • • “I" “[” ••• (1)


SÉRIES 299

e formemos a série dos valores absolutos dos seus termos:


|aol + l«lII^I + l«2ll ^l' + l«3ll ^l' +
+ |a4 ||x |^ + . . . + | a J | x | " + . . . (4)

Para determinar a convergência desta última série (de termos


positivos) apliquemos a regra d’Alembert.
Suponhamos que existe o limite
n+ 1
lim = lim ^77+1 \x\ = L \ x \ .
Ur, n-^oc

Então, segundo a regra d’Alembert, a série (4) converge L \ x \ < 1,


isto é. para | x | < y - , e diverge quando L \ x \ > 1, isto é. para
^ 1
1^ l > | .
1
Por conseguinte, a série (1) converge absolutamente para| x | .
Se I X I > , lim I X I Z/ > 1, e a série (4) diverge, o seu
L/ n-*co
termo geral não tende para zero(*).
Mas, então, o termo geral da série inteira (1) não tende mais
para zero, o que significa, em virtude do critério de convergência
necessário, que esta série inteira diverge ^ quando | x I > ^ ) •

Resulta do que precede que o intervalo | — y» y j é o intervalo


de convergência da série inteira (1), isto é, que

R = — = lim
L ^n+l

Duma maneira análoga, pode-se também servir-se da regra de


Cauchy para determinar o intervalo de convergência duma série inteira.
Obtém-se, então.

R =
lim V\ ãr, \

(*) Lembremos que dürante a demonstração da regra d*AIembcrt (ver § 4)


notamos que se lim > 1, o termo geral crescia, logo não tendia para zero.
300 CALCULO D IFEREN CIA L E INTEGRAL

Exemplo — 1. Determinar o intervalo de convergência da série


1+ X+ X2+ X3+... ...
Resolução — Aplicando a regra d*Alembert, obtém-se

lim = \x \.
n -> -o o 1

Por conseguinte, a série converge para | j: | < 1 e diverge para 1JC| > 1.
A regra d*Alembert nada dá nos pontos fronteiriços do intervalo (— 1, 1). Mas
vê-se directamente que a série diverge nos pontos x = ± 1.
Exemplo — 2. Determinar o intervalo de convergência da série
2x (2x)2 , (2x)3
1 2 + 3
Resolução — Apliquemos a regra d*Alembert:
( 2 x )^ + i
tl —f- 1
lim = l im 12x I = I 2x I.
n->oo (2x)" n -^ o o n-{-i

A
A série converge quando | 2 x | < l , isto é, se | a; ; converge no
ponto x = —1 e diverge no ponto x = ---- iL , ^
2 2

Exemplo — 3. Determinar o intervalo de convergência da série


x2 x3
"+ ^+ 3 T + --+ in -+ -
Resolução — Apliquemos a regra d*Alembert:
X
l im — 11
— m
11111 — lim
n -j- 1
= 0 < 1.
n-v o o Uji n-^oo x ’^ ( n + 1)1 n-> oo

Como o limite não depende de x e como é menor que 1, a série converge


qualquer que seja x.
Exemplo — 4. A série 1 -f ^ + (2x)2 -|- (3x)3 + • • • diverge qual­
quer que seja x, excepto x = 0 , porque {nx)'^ —^ oo quando n —> oo qualquer
que seja x náo nulo.
Teorema — 3. Urna série inteira
Oo + OiX + OaX* + . . . + o„x” + . . . (1)
é majorável sobre o segmento [— p, p] contido no seu intervalo de
convergência.
Demonstração — Por hipótese, tem-se p < R (fig. 350) e, portanto,
a 94ri« numérica (de termos positivos)

1 f lo I + I 1 P + I < *2 I + ••• + I Ip" (5 )


SARIES 301

converge. Mas, quantlo \x \ < p, os termos da série (1) não são supe­
riores em valores absolutos aos termos correspondentes de (5). Logo
a série (1) é majorável sobre o segmento [— p, p].
Corolário— 1. A soma duma série inteira é uma junção contínua
sobre todo o segmento completamente contido no intervalo de con-
Intervalo de convergência

H-
-R -pv
Intervalo de majoração
Fig. 350

vergência, Com efeito, a série é majorável sobre este segmento e os


seus termos são funções contínuas de x. Por conseguinte, em virtude
do teorema 1, § 11, a soma desta série é uma função contínua.

Fig. 351

Corolário — 2. Se os limites de integração a, p pertencerem ao


intervalo de convergência duma série inteira, o integral da soma da
série é igual à soma dos integrais dos termos da série. Com efeito,
o intervalo de integração pode ser contido no segmento [— p, p], onde
a série é majorável (fig. 351) (ver o teorema 2. § 12 sobre a integração
das séries majoráveis).

§ 14. Derivação de séries inteiras


Teorema— 1. Se (— /?, /?) for o intervalo de convergência da
série inteira
S {x) = «0 + a^x + Ü2 X^ + a^x^ + ai,x" + . . . + a^x"^ + • • •. (1)
a série
(p {pc) = -f- 2 ^ 2 ^ ”1“ Sü^x . -|- rtUj^x -j- . . . ( 2)
deduzida de (1) por derivação termo a termo, admite o mesmo intervalo
de convergência: além disso, tem-se
(p (a:) = s' {x) quando | o: | < i?,
isto é, que no intervalo de convergência, a derivada da soma da
série inteira (1) é igual à soma da série obtida derivando termo a
termo a série (1).
302 CALCULO D IFEREN CIA L E INTEGRAL

Demonstração — Demonstremos que a série (2) é majorável sobre


todo o segmento [— p, p] que pertence completamente ao intervalo de
convergência.

0 I I --H
-/? -p X p ç Xz X/
Fig. 352

Tomemos um ponto | tal que p < i < R (fig. 352). A série (1)
converge neste ponto, logo lim = 0, e existe uma constante M
n -*o o
tal (}ue
|a „ r i< M (n = l. 2, ...).
Se IXI < p, tem-se
n — 1 I
raa„x" * K | « a „ p " ‘ l= n |a„^ <« >
onde
g = T < i-

Assim, quando | x | < p, os termos da série (2) são inferiores em


valores absolutos aos termos da série numérica positiva

^ ( í + 2q + 3 g ^ - i - . . . + + ...)•

Mas. como o mostra a regra d’Alembert. esta última série converge:


nq
lim = q< í.
n-^oo (n — 1) qji —2
Logo a série (2) é majorável sobre o segmento [— p, p] é. em
virtude do teorema (2). § 12. a sua soma é a derivada da soma da
série proposta sobre o segmento [— p. p]. isto é.
q) (x) = s ' (x).
Como se pode encerrar todo o ponto interior do interv?ilo
(— R, R) num certo segmento [— p. p]. resulta que a série (2) converge
em qualquer ponto interior do intervalo (— /l, R).
Mostremos que a série (2) diverge na vizinhança do intervalo
(— R, R). Admitamos que a série (2) converge para Xi > R. Inte­
grando termo a termo no intervalo (0. jcg) onde R < X2 < Xu concluir-
-se-ia que a série (1) convergiria no ponto Xz. o que contradiz as
condições do teorema. Por conseguinte, o intervalo (— / ? . / ? ) é o
SARIES 303

intervalo de convergência da série (2). O teorema está completamente


demonstrado.
A série (2) pode ser de novo derivada termo a termo, e será
lícito continuar este procedimento à vontade. De modo que:
Teorema — 2. Se uma série inteira converge no intervalo (— R, R),
a sua soma representa uma função que tem no intervalo de conver­
gência, derivadas de qualquer ordem n, sendo cada uma delas a soma
da série proposta; além disso, o intervalo de convergência de cada
série obtida por derivação é também o intervalo de convergência
da série proposta {— R, /?).

§ 15. Séries de potências áe x — a


Chama-se também série de potências a uma série da forma
^0 + — a) + a2 (:r — + •••+ + • • •» (1)
onde as constantes Oq, au . •> dn ••• são igualmente chamadas os
coeficientes da série. Os termos desta série contêm as potências cres­
centes d t X — a.

Se a = 0, obtém-se uma série de potências de x, que é, pois,


um caso particular da série (1).
Para determinar o domínio de convergência da série <1), façamos
a mudança de variável
X — a = X.
A série (1) transforma-se, após esta substituição,
ÜQ+ a^K + d2X^ + • • • + dnX^ + • • •» (2)
que é uma série de potências de X.
Seja — R < X < R o intervalo de convergência da série (2)
(fig. 353, a). Resulta daí que a série (1) convergirá para os x que
verificam a desigualdade — R < x — a < R ou a — R < x a a + R.
Como a série (2) diverge para | X | > R, a série (1) divergirá para
\ x — a \ > R, isto é, na vizinhança do intervalo a — R < x < a + R
(fig. 353, p).
304 CALCULO D IFEREN CIA L E INTEGRAL

Por conseguinte, o intervalo de convergência da série (1) é o


intervalo {a — R, a R) tendo por centro o ponto a. Todas as pro­
priedades duma série inteira em x no intervalo de convergência (— /?,/?)
são completamente conservadas para uma série inteira de x — a no
intervalo de convergência {a — R, a R). Assim, integrando termo a
termo a série inteira (1), os limites de integração pertencentes ao
intervalo de convergência (a — R, a + R), obtém-se uma série cuja

0 1 2 3
H--- 1--- 1----h-
Fig. 354

soma é igual ao integral da soma da série proposta (1). Se se derivar


termo a termo a série inteira (1), sendo x tomado no intervalo de
convergência (a — R, a + R), obtém-se uma série cuja soma é igual
à derivada da soma da série proposta (1).
Exemplo — Achar o domínio de convergência da série
( x - 2 ) + (x -2 )2 + (o :-2)3+ ... + ( x - 2 r + ...

Resolução — Fazendo x — 2 = X , obtém-se a série


X + X2 + X 3 + . . . + X ^ + . . .
Esta série converge para — 1 < X < -H 1. Logo, a série proposta converge
para x tais que — 1 < x — 2 < 1, isto é, para 1 < x < 3 fig. 354).

§ 16. Séries de Taylor e de Maclaurin


Mostrámos no § 6, cap. IV (t. I) que uma função f (x) que
possua derivadas até à ordem /i +-1 inclusivé na vizinhança do ponto
X = a (isto é, no intervalo que contém o ponto x = a) admitia nesta
vizinhança o desenvolvimento seguinte de Taylor

f j ^) = f(a) + ^ r (a) + (a) + .. .

• • • T ------W + (1 )
onde o resto Rn (x) era calculado segundo a fórmula

Rn = [a + e ( X - a)]. O < 0 < 1 .


(ra + 1 ) !
SÉRIES 305

Se a função f(x) é indefinidamente derivável na vizinhança do


ponto X = a, poder-se-á tomar n arbitràriamente grande na fórmula de
Taylor. Suponhamos que o resto tende para zero no domínio
considerado quando n -» oo:
lim Rn {x) = 0.
n -^cx5
Sendo assim, fazendo tender oo na fórmula (1), obtém-se
à direita uma série com uma infinidade de termos dita série de Taylor:
X — a
f(x) = f{a) I
i --------,---- /
A n ) / _\
W + * ..
I /o \
(2)
1 n\
Esta última igualdade apenas está certa se R^, {x) 0 quando
/!-> 00. Então, a série do segundo membro converge e a sua soma
é igual à função j{x). Mostremos que assim é:

í{x) = Pn{x) + R n i A
onde

Pn (X) = / (a) + ^
1!
/ ' («) + . . . +
rei
/" V ).

Como, por hipótese, lim (x) = 0, tem-se


n -v o o

f { x ) = lim P „(x).

Ora, Pn (x) é uma soma parcial da série (2); o seu limite é


igual à soma da série do segundo membro da igualdade (2). A igual­
dade (2) é, pois, legítima:

/ (X) = / (a) -f ^ r (a) + (fl) + ...


1 ' ' ' 2
(x — a)" ,(„)
•••+ ^ r '( a ) + ...
re!
Resulta, do que antecede, que a série de Taylor representa a
função dada f (x), se e só se, lim (x) = 0. Se lim (x) ¥= 0,
n -v o o n-»>oo
a série não representa a função dada, se bem que possa convergir
(para uma outra função).
Se, na série de Taylor, se fizer a = 0, obtém-se num caso par­
ticular desta série, chamada série de Maclaurin:

/ (X) = / (0) + - ^ / ' (0) + / ' (0) + . . . - f ^ (0) + . . . (3)


1 2! ni
20
306 CALCULO D IFEREN CIA L E INTEGRAL

Se se escrever formalmente a série de Taylor duma dada função


e se se quiser certificar que ela representa efectivamente esta função,
será preciso demonstrar que o resto tende para zero bem como ainda
comprovar, duma maneira ou doutra, que a série escrita converge
para a função dada.
Notemos que. para cada função elementar definida no § 8.
cap. I (t. I). existe um a g um R tais que. no intervalo {a — R, a R),
ela se desenvolve em série de Taylor ou (se n = 0) de Maclaurin.

§ 17. Exraoplos de desenvolvimento de fnnções em séries


1. Desenvolvimento em série de Maclaurin de f{x) = senx,
No § 7. cap. IV (t. I), obtivemos a fórmula
3 5 1
X . X . . . .vn-H OC
senX = X ------- ^---------777 + ^2n W-
3! 5! (2n - 1)!

Como se demonstrou que lim i?2n (^) = 0, obtém-se, tendo em


n-voo
conta o que foi dito no parágrafo anterior, o desenvolvimento de
sen X em série de Maclaurin;

sen X = a ; + ( - 1 ) " + * ^ ^ ------- + • • • (1 )


3! 5! (2n - 1)!

Como o resto tende para zero qualquer que seja jc, a série
proposta converge e a sua forma representa a função senjc qualquer
que seja x,
A figura 355 representa o gráfico da função senx e das três
primeiras somas parciais da série (1).
Recorre-se a esta série para calcular senjc para diversos valores
de X.
Calculemos, por exemplo, sen 10® a menos de 10"® Dado que
10® = ^ = 0,174533, tem-se

sen
Í8 ~ 5 f ( f s T + à ( f s T - l i ( i - s ) ’ + • •
Limitando-nos aos dois primeiros termos, obtém-se a igualdade
seguinte aproximada;
. Ji ji 1 / jt
18 ^ Í8 “ tT i i 8 j ’
SÉRIES 307

O erro S é. em valòr ^bsoluto, inferior ao primeiro termo desprezado:

ô < — f — y < — (0,2)® < 4 -10-«.


5! \ 1 8 / 120
Se se calcular cada termo da expressão do sen ^ tomando 6
decimais, obtém-se
Jl
sen .^ = 0473647.
lo

Pode-se garantir as quatro primeiras decimais.


2. Desenvolvimento em série de Maclaurin da função f (x) = e*.

é‘* = \. +. X - \ I----------------------------------h
_l_ I _L _L • • •.
( 2)
2! 3! n!
porque, como se demonstrou, lim (x) = 0 qualquer que seja x a
n-*-oo
série converge para todos os x e representa a função e^.
308 CALCULO D IFEREN CIA L E INTEGRAL

3. Desenvolvimento em série de Maclaurin da função f (x) = cos x.


Deduz-se do que foi dito no § 7, cap. IV, (t. I) que
2 4 6
c o s x = 1 --------- ---------------- [-•••» (3)
2! 4! 6!
a série converge para todos os jc e representa a função cosjc.

§ 18. Fórmulas de Euler


Até agora, consideramos séries de termos reais e deixámos à
margem as séries de termos complexos que sai fora do âmbito deste
livro, limitando-nos a examinar um exemplo importante.
Definimos, no cap. VII (t. I). a função pela igualdade
^ _j_ ^ g ç j j y Y

Fazendo jc = 0, obtém-se a fórmula de Euler:


^*^=z=cosi/ + isen y.
Se se definir a função exponencial de expoente imaginário
por meio da fórmula (2) do pàrágrafo 17. representando a função
sob a forma de série inteira, encontra-se a igualdade de Euler. Com
efeito, definamos substituindo na igualdade (2) do § 17 iy por x:

= 1 + ÍL { i yf { i yf iiy T
+ ( 1)
1! 2! 3! n\
Como j2 = _ 1, = — i, i* = 1, = i, i» = — 1, etc.,
obtém-se

1! 2! 3! 4! 5!
Separemos as partes reais e imaginárias desta série

V 2! 4! / V I! 3! 5! /

As expressões entre parêntesis são as séries inteiras de cos y


e de seny (ver as fórmulas (3) e (1) do parágrafo anterior). Por
conseguinte,
= cos y i sen y.

Voltamos a encontrar a fórmula de Euler,


SÉRIES 309

§ 19. Fórmula geral do binómio


1. Desenvolvamos em série jdc Maclaurin a função
f(2) = { í + x ) ^ ,
sendo m uma constante arbitrária.
Como o cálculo do resto apresenta algumas dificuldades, proce­
deremos doutro* modo para encontrar o desenvolvimento em série desta
função.
Tendo em conta que a função / (x) = = ( 1 4 - xy^ satisfaz à equação
diferencial
(1 + ^) í {^) = m f (x) (1 )
e à condição
/(0) = 1,
procuremos uma série inteira cuja soma s{x) satisfaça à equação (1) e
à condição 5(0) =-l:
s{x) = i + aiX + a^x^ + • . • + a^x" + . . . (♦). (2)

Substituindo esta série na equação (1), obtém-se:

(1 4“ (^1 “{“ 2^2^ “1“ 4~ • • • “í~ 4~ • • •)


= m (1 4" 4~ ^2^ ”1“ • • • “í~ CLji^ 4“ • • •)•
Identificando os coeficientes das mesmas potências de x dum
e doutro lado da igualdade, encontra-se:
rtj = m ; ai + 2ü2 = ma^; . . . ; nan + (w 4-1) í •••
Donde se obtém, para os coeficientes da série, a expressão

. a^im — 1) m {m — 1)
«0 = 1; a^ = m \ = ----- i = — 5— ---- >-■,

_a2 {m — 2)__ m{m — \ ) { m — 2)


3” - 2^3 ’

_m (m — 1 ) .. . [m /I + 1]
— 71 “õ
* 2 . . . Al ’ ‘ *

São os coeficientes da série do binómio.

(*) Tomamos o termo constante igual à unidade, vista a condição s (0) = 1.


310 CALCULO D IFEREN CIA L E INTEGRAL

Substituindo-os na fórmula (2), obtém-se:

1-2

m (m — — (ra — 1)1 „ ,
(3)
1-2...n

Se /n é um inteiro positivo, os coeficientes de e das potências


superiores são nulos e a série reduz-se a um polinómio. Se m é frac-
cionário ou um inteiro negativo, tem-se uma série com uma infinidade
de termos.
Determinemos o raio de convergência da série (3):

m ( m — í ) . . . [ m — n + i ]^n
“ n+l = ----------------- ;----------------^ .
n!

_T O (m — — w + 2] „_i
~~ ^ j
{n - 1)!

m ( m — í ) . . , (m — n í) {n — í)\
lim Un-hí = lim
n-^oo Un 7l->00 m {m — l ) . . . {m — n 2)nl

m —n í
= lim \x = X ,
n-*-oo n

De modo que a série (3) converge para |jc| < 1.


No intervalo (— 1, 1) a série (2) representa uma função í (jc) que
satisfaz a equação diferencial (1) e à condição

5(0) = 1.

Como existe apenas uma única função que satisfaz a equação


diferencial (1) com a condição inicial s (0) = 1, resulta que a soma
da série (3) é identicamente igual à função (1 + a:)”*, e obtém-se o
desenvolvimento
(1 + = 1 + ma: ■ m (m — í)-a:^2 -I-
1-2

m (m — 1) (m — 2) , ,
-------------------------X + . . . (3')
1-2-3
SARIES 311

Em e$.pecial. se m = — 1, tem-se
1
= 1 — X , (4)
1 + a:

Se m = i

V l + X= 1 + ------ —
1-3
-1----------- .
1-3-5
— X X^ X* + . . . (5)
2 2-4 2. 4-6 2-4.6.8

Se m = —

, 1 - 3 .5 - 7 .X4 —
(6)
Vi 2 2-4' 2-4-6 2 -4 -6 .8

2. Apliquemos o desenvolvimento do binómio ao desenvolvi­


mento doutras funções. Desenvolvamos em série de Maclaurin a função
/ [x) = arc sen x.
Substituindo x por — Jc^ na igualdade (6). obtém-se:

1 :^ ^ , 1-3-5.. . ( 2 » - l ) ^
2-4-6 2 - 4 - 6 . ..2ra
Em virtude do teorema sobre a integração das séries inteiras,
tem-se para | x | < 1

\ dx , 1 , 1-3 x'* , 1-3-5 x' .


\ ^ = arc sen x = x-\ ----------- ------------- h :r—;— ^ •
ò Y i-x^ 2 3 2-4 5 2-4-b 7

1 - 3 - 5 . . . ( 2 r a - l ) X-,2n + l
...+ + -••
2 - 4 - 6 . . . 2n 2w + l
Esta série converge no intervalo (— 1, 1). Poder-se-ia demonstrar
que a série converge igualmente quando x = ± 1 e que a soma cor­
respondente a estes valores representa arc sen x. Então, fazendo x = 1,
obtém-se esta fótmula para calcular i r ;
. ji 1 1 1-3 1 . 1 - 3 - 5 1
a rc s « i 1 = — = 1 H----------- ------------- \-----------------\ - . . .
2 2 3 2-4 5 2-4-6 7
312 CALCULO D IFEREN CIA L E INTEGRAL

§ 20. D esenvolvim ento d a função log (1 -|- x ) ^ série in trira .


C^dculo de logaritm os
Integrando a igualdade (4) do § 19 de 0 a x (com | x | < 1).
obtém-se:
X X

f _ d ^ ^ f (1 _ a; 4. . . .) dx
j 1+ ^ ^
ou

L o g (l-fx ) = x - ^ + | - - ^ + ... + ( - i r - " ‘- + ... (1)


2 3 4 n
Esta igualdade é verdadeira no intervalo (— 1, 1).
Se se substituir x por — x nesta fórmula, obtém-se a série:
^2 x^
X
Log (1 — ;r) = (2)
2 3 4
que converge no intervalo ( — 1, 1).
As séries (1) e (2) permitem calcular os logaritmos de números
compreendidos entre zero e dois. Indiquemos sem o demonstrar que
o desenvolvimento (1) subsiste para jc = 1.
Vamos dar uma fórmula que permita calcular os logaritmos
naturais dos números inteiros.
Como a diferença termo a termo de duas séries convergentes
é uma série convergente (ver § 1, teorema 3). obtém-se. subtraindo
termo a termo (2) de (1):

Log (1 + a:) — Log (1 — x) =

1 1 o r I I I
1 — X L 3 5 J

Façamos em s ^ i d a 1 X TI -|- 1 1
tem-se x = Para
1—X n 2n + l*
qualquer n > 0, tem-se 0 < x < 1, logo

L o g ^ = L o g ^ = 2 [ - j — -f - * .4
\ - x n l2n + l 3(2n-|-iy

1
+
5(2n + iy
SÉRIES 313

donde
Log(n + 1) — Logn =
1 1
= 2 + (3)
,2n + l ' 3(2n + l f 5(2« + l)®
Para w = 1, deduz-se
r 1 1 1
‘■”* ^ = n r 4 + ? ¥ * + ^ = ' + -

Para calcular log2 com a precisão desejada ô, é preciso calcular


a soma parcial Sp tomando um número p de termos tal que a soma
dos termos desprezados (isto é. o erro Rp cometido, substituindo por Sp)
seja inferior ao erro admitido 8. Calculemos, para esse efeito, o erro Rp :

1
R^ = 2 +
(2p + 1) 3 " ^ + ’ (2p + 3)

1
\+ •••
(2/7 + 5) 3‘2P+5'
Como os números 2p + 3, 2p + 5. ... são superiores a 2p + 1.
aumentamos o valor de cada fraeção quando substituímos estes números
por 2/ 7+1. Logo.

1
Rp<2 +
(2p + l)3"^+ ‘ (2p + l ) 3 ‘‘^ + ^

i
+ . . .
(2p + l)3"P+®
OU

Rp <C g2p+5
2p + l

Temos, entlre parêntesis recto, uma série geométrica de razão •


A soma desta série é

32P+1
R p< (4)
2P + l i _ l (2p + l)3=^'>-‘4 '
9
314 CALCULO D IFEREN CIA L E INTEGRAL

Se se quiser agora calcular log 2, por exemplo, a menos de


0,0000001, é preciso escolher p de modo que se tenha Rp < 0,0000001.
Chega-se lá, tomando p de modo que o segundo membro da desi­
gualdade (3) seja inferior a 0,0000001. Verifica-se que basta fazer p = i.
Assim, a menos de 0,0000001, tem-se
1 1
Log 2 » Sg = 2 Í ^ + - i + Ti +
U -3 3 - ; 3®'*’ 5-3®’^ 7 - 3 ’ 9-3® H -3

1
= 0,6931471.
13-3* 1 5 - 3 ‘* J

A resposta é log 2 = 0,6931471, com sete décimais exactas.


Fazendo na fórmula (3) n = 2, obtém-se:

Log3 = Log2 - f 2 [ 4 ' + 7 ^ + + • • •] = li098612, etc.


Lo 0*0 0*0 J

Podm os, pois, calcular os logaritmos naturais de qualquer inteiro.


Para obter os logaritmos decimais basta servir-se da r e la to
(ver § 8, cap. II, t. I)
log N = M Log N ,

com M = 0,434294. Obtém-se, assim, Log 2 = 0,6931472, log 2 = 0,30103.

§ 21. Airiicação das séries ao cálculo dos integrais definidos

Mostrámos, nos capítulos X e XI (t. I), que existiam integrais


definidos que, considerados como funções dos seus limites superiores,
não se exprimiam sob forma finita por meio das funções demmtares.
Por vezes, é cómodo calcular tais integrais por meio de séries.
Consideremos alguns exemplos.
1. ^ j a calcular o integral

A primitiva de não é uma função elementar. Para calcular


o integral, desenvolvamos em série, a fu n ^ o sob o sinal soma substi­
tuindo no desenvolvimento de e* (ver fórmula (2), § 17) x por — x*:
^4 0 271
= 1 - A - f — - — -f . . . 4 - ( - i r — + . . .
1! 2! 3! n\
SARIBS 315

Integrando os dois membros desta igualdade de 0 a n, obtém-se

a a a
+ •••
1 1|.3 2!.5 3!-7
Esta igualdade permite calcular o integral, qualquer que seja a.
com a aproximação desejada.
2. Seja calcular o integral
a
sen X
dx.
\
Desenvolvamos em série a função sob o sinal soma: como
X*.3 X X
s e n i = x --------- 1---------------- H
3! 51 7!
temse:
senx 3^ x ' ' X*
i --------- \---------------
3! 5! 7!
convergindo esta última série qualquer que seja x. Integrando termo a
termo, obtém-se:
a

í sen X ,
------- dx = a —
X
a**
31-3 5!-5 71.7
■f. ..

É fácil de calcular a soma desta série com a precisão desejada,


qualquer que seja a,
3. Calcular o Integral elíptico

5Vl — sen^cpdq) ( /c < l) .


0

Desenvolvamos a expressão sob o sinal soma em série de binómio,


1
com m = —, X = — k^sGn^<p (ver fórmula (5), § 19):

.2— 2 ^ _ I — A^sen^ (p — ^ —

1 1 3
316 CALCULO D IFEREN CIA L E INTEGRAL

Esta série converge qualquer que seja ^ e pode ser integrada


termo a termo, dado que ela é majorável em todo o intervalo.
Por conseguinte.
(p (p
j V l — sen^ (p dcp = (p — ^ sen^ Q) dfo —
0

1
— AM sen* çp dtp — A® j sen* (p dcp — . , .
0 0

Os integrais do segundo membro calculam-se elementarmente.


Para <p = ^
2 ■

í sen (p d(p j
1 - 3 . . . {2n — í) n
~2

(ver § 6. cap. XI, t. I) e. por conseguinte,


n
2

j V l — A ^ se n ^ (p d q )= ^ | l — ^k] —
0

íí-3Vk^ íl-S-òVk^ 1
V2. 4/ 3 \2.46/ 5 *‘J ’

§ 22. Aplicação das séries à integra^ ^ de equações


diferenciais
Se a integração duma equação diferencial não se reduzir a qua­
draturas, tem-se, então, de recorrer a métodos de integração aproximada.
Um destes métodos consiste em representar a solução da equação
sob a forma de série de Taylor; a soma de um número finito de
termos desta série será aproximadamente igual à solução particular
procurada.
Suponhamos, por exemplo, que se tem de procurár a solução
duma equação diferencial de segunda ordem
y' = F(x, y, y'), (i>
cmn as condições iniciais
(y)x=*o yo« (y )x=*o yo* ( 2>
SARIBS 317

Suponhamos que a solução >^ = / (jc) existe e pode ser repre­


sentada por uma série de Taylor (não nos aprofundaremos sobre a
questão das condições que devam ser verificadas para que tal tenha
lugar):

y = /(x) = /(x„)+^f"^»/'(:ro) + ^ ^ ^ ^ 'r (^ o ) + . . . • (3)


1 1 •z
Devemos encontrar / ( jco), f (xo), f'(xo), isto é, os valores
das derivadas da solução particular para x = jcq. Encontra-se-las por
meio da equação (1) e das condições (2):
f(Xo) = yo, f'(^o) = yol
deduz-se da equação (1):
r M ^ (^0, uo, .</ó).
Derivando os dois membros da equação (1) em relação a x
i/"' = F;. (ít. !/, y ) + F\j {x, y, y ) y + {x, j/, if) i f (4)

e substituindo x = Xo no segundo membro, encontra-se:


/ " ' W = (í/'")x=.v
Derivemos a relação (4) uma vez mais. Tem-se

e assim sqcessivamente.
Substituamos os valores das derivadas encontradas na igualdade (3).
Esta série representa a solução da equação proposta para os valores
de X para os quais ela converge.
Exemplo — Encontrar a solução da equação
í/a:2,
que satisfaz as condições iniciais
(í/)x=0 = 1» (í/')x=0 = 0.
Resolução — Tem-se:
/ (0 ) = z / o = l ; / '( 0 ) = yi = 0.

Deduz-se da equação dada (z/*')jc=o = r (0) = 0 ; depois


y - = _ y'x2- 2xy, {ynx= 0 = r (0) = 0,
4x1/' —2i/, (r/^^)^=,o= —2
c em geral, derivando k vezes os dois membros da equação pela aplicação
da fórmula de Leibniz, vem (§ 22, cap. III, t. I):
j/<fe+2)= A:(it—1)
318 CALCULO D IFEREN CIA L E INTEGRAL

Fazendo jc = 0, obtém-se:

ou, fazendo k + 2 = n.

Donde
!/r= -l-2 . í,i«=-5.6yíV_(_i)2(i.2)(5.6K
j,J* = -9.10j/^f> = ( - l ) 3 (1-2) (5-6) (9.10),
í/S'* = ( - l ) ' ‘ (1-2) (5-6) ( 9 .1 0 )... [(4 * -3 )(4 fc -2 )].

Além disso, j,(s) = o, = 0 , . . . , i/í,^'‘+i)==0.


y'o»•>= 0, i^^'»>= 0........ |/i<'‘+2>=0,
!/!,’»= 0, !,<>» = 0........ y<‘'‘+3)=0.

De modo que sòmente não se anulam as derivadas de ordens múltiplas


de quatro
Substituindo os valores das derivadas encontradas na série de MacLaurin,
obtém-se a solução da equação proposta

j, = 1 _ _ 1 . 2 + . ^ ( 1 . 2 ) ( 5 . 6 ) - j ^(1.2)(5.6)(9.10) + . . .

. . , + ( ^ i ) h — (i.2) ( 5 .6 ) ... [(4 ^ -3 ) (4 /c-2 )l + . . .

Verifica-se, por meio da regra d*Alembert que esta série converge para
todos os valores de x, logo ela é solução da equação difrencial.
Quando a equação é linear, é cómodo procurar os coeficientes
do desenvolvimento duma solução particular pelo método dos coefi­
cientes indeterminados. Para isso, «substitui-se» directamente a série
y = Uq a^x -f- . -f- CLn^^ “f" • • •
na equação diferencial e identificam-se os coeficientes das mesmas
potências de x dum e doutro lado da equação.
Exemplo — Achar a solução da equação
í/'' = 2xí/' + 4í/,

que verifica as condições iniciais


(y)3C=0 = 0, (y')^^_Q=l.
Resolução — Façamos
y = ao+ aiX+ 00^2 + 03^^+ . • • ...
Acha-se, tendo-se em conta as condições iniciai»
ao = 0, ai = l.
SâR IE S 319

Por conseguinte,
y = x + a2x"^-\-a^x^-\- . . . +anX^+ ... ,
í/' =--1 + 2ü2X+ 803x24- ... + +... ,
y”= 2^24"3•2 fl3 X4“ •••4“w —1) 4“ •••
Substituindo as expressões acima na equação proposta e identificando os
coeficientes das mesmas potências de x, vem:
2o2= 0, donde 02 = 0
3*203 = 24“4, donde 03 = 1
4 • 804 = 4 o2 4 “ ^^ 2 » donde «4 = 0

n ( o — l )o^ = (n— 2) 2o;i_24-4o;i_2, donde = .


/I— 1
Por conseguinte,

2*1 1
_ ! Í - T
_T! "9 = 4!
4 “ 2! ’ 6 3! ’

„ _ (A-1)! 1
2A) — A! ’
04 = 0 ; 00 = 0 ; ü2k —0.
Substituindo os coeficientes encontrados, emcontra-se a equação procurada
x3 r2fe+l
í/ = ^ + — + - ^ + - 3 T + - - - ' A:!
A série obtida converge qualquer que seja x.
Notemos que a solução particular encontrada se exprime por meio das
funções elementares; com efeito, se se puser x em factor, obtém-se o desen­
volvimento da função Logo,

y = xe^X2.

§ 28. E^çptapão de Bessel


Assim se chama a uma equação da forma
x^y’ + xy + (x^ — y= 0 {p = const). (1 )
Convém procurar a solução desta equação, como as soluções de
certas equações de coeficientes variáveis, não sob a forma de série
inteira, mas sob a forma de produto duma certa potência de x por
uma série inteira:
y —^ • (2>
A=0
320 CALCULO D IFEREN CIA L E INTEGRAL

É lícito tomar ao diferente de zero, dado que o expoente r é


indeterminado.
Recopiemos a expressão (2) sob a forma

,r+Ã
y=
k=0
e procuremos as suas derivadas:

y ' = S (r + A:)aAX^+*-‘ ;
k=0

y '= S (r + A) (r + A; — 1) a^x H-A-2


k=0
Substituamos estas expressões na equação (1):
uu
x^ (r + k)(r + k - l ) +
A=*0

X 2 j (^ + ^) * + (^ — P^) 2
A=0 ft=0
Anulando os coeficientes de x na potência r, r + l, r + 2,
r + k. obtém-se o sistona de equações diferenciais:

[r (r — 1) + r —p^J Oo = 0 ou [r^ — p^] = 0,


I(r + l ) r - f - ( r + l ) - p 2 ] a i = 0 ou [(r + 1 ) ^ = 0,
[{r + 2) (r + 1) + (r + 2 ) - p ^ ] + a» = 0
ou [(r -|- 2 f — p^] 02 + ^0 = 0, (3)
[(r + A) (r -1- A: — 1) + (r + A) —p^]a* + 0^-2 = 0
ou [(r + k f — p^] ük - f «*-2 = 0.

Consideremos a última igualdade:


[(r + k f — p^] a* + aft_2 = 0. (3')
Pode-se reç(^iá-la sob a forma
[(r -f. A —p) (r + A + p ) ] a* - f ak- 2 = 0.
Por hipótese. a ^ 0 \ por conseguinte,

logo. Ti = p OU melhor t2 = — p.
SÉRIES 321

Consideremos, em primeiro lugar, a solução correspondente a


ri = p > 0 .
Deduz-se, sucessivamente, do sistema de equações (3) todos os
coeficientes ai, ü2 ....... uq sendo Oo arbitrário. Tomamos, por exemplo,
flo = 1. Então,
____ ______________ O 'h - 2

k {2 p -\-k)

Dando a k vários valores, encontra-se:


fli = 0, fl3 = 0 et, em geral, fl2m+i = 0!
1
«4 = (4)
2(2p + 2) 2 -H 2 p + 2){2p + A)
1
fl2v = ( - i r ‘
2 - 4 - 6 . . . 2v {2p + 2) (2j5 + 4 ) . . . (2p + 2v)
Substituindo os coeficientes encontrados na fórmula (2). obtém-se:

Vi
2 (2 p -f2 ) 2-4(2p + 2)(2p + 4)

(5)
2 -4-b(2p + 2)(2p + 4)(2jD-t-6)
Todos os coeficientes C2v são determinados porque, por todo o k,
o coeficiente de na equação (3)

nao é nulo.
Assim, é uma solução particular da equação (1).
Procuremos, agora, em que condições todos os coeficientes
são determinados quando se considera a segunda raiz ra = — p. Para
isso, é preciso que seja verificada a desigualdade seguinte para todo k
positivo e par:

(6)

r2 + k ^ p .
Ora, p = Ti, por conseguinte,
r2 + k ^ V i .
Assim, a condição (6) é, neste caso, equivalente à s ^ i n t e :

^2 ^ k.
21
322 CALCULO D IFEREN CIA L E INTEGRAL

sendo k um inteiro par positivo. Mas

por c o n s ^ in te ,
^^ — ^2 = 2p.

De modo que, se p não é um número inteiro, pode-se escrever


uma s ^ n d a solução particular que se deduz da expressão (5) substi­
tuindo p por — p:

Vz = x ~ A \ ---------- +
I 2 (( -- 2 p + 2 ) 2 -4 (-2 p + 2 ) ( - 2 p + 4)

1
2 .4 . 6 ( - 2p + 2) ( - 2 p - f 4) ( - 2 p + 6) J- (5-)

As séries inteiras (5) e (50 convergem qualquer que seja x, o


que é fácil de verificar aplicando a regra d’Alembert. É igualmente
evidente que e yz são linearmente independentes (*).
A solução multiplicada por um certo factor, constante, chama-se
função de Bessel de primeira espécie de ordem p e representa-se-la
por Jp. A solução yz é representada por / _ p .
Por conseguinte, quando p não é um número inteiro, a solução
geral da equação (1) escreve-se
y = C \Jp -(- C ^ J —p .
1
Assim, se p = - y , a série (5) escreve-se.

1 —
2-3 2 -4 .3 -5 2 .4 .6 - 3 .5 .7
3 5 7
J_ X , X X ,
X ------------------------------------\ - . . .
V xí 3! 5! 7!

(♦) Verifica-se, como se segue, a independência linear destas funções


Consideremos a relação
. 1-
2 (-2 p + 2 ) + 2 - 4 ( - 2 p + 2 ) ( — 2 p + 4)
í/1 1 ----2 (2p + 2) + 2 * 4 (2 p -)- 2) (2p 4)
Esta relação não é constante, dado que tende para infinito quando jc —> 0.
Logo, as funções y-i c são linearmente independentes.
S£RI£S 323

Esta solução multiplicada por y — , chama-se função de Bes-'^


sel ’ notemos que a expressão entre parêntesis recto é o desenvol­
vimento em série do senx. Por conseguinte.

/ (x )= ] / - Í | senx
* ^ JIX
Do mesmo modo se obtém, a partir da fórmula (50:

J ^ (x)= — COS X.
JIX

O integral geral da equação (1) quando p = -^ é

y = C i J j (x) + C 2 J j («x^)
2' "Y
Suponhamos, agora, que p é um número inteiro positivo que
designaremos por «(« > 0). A solução (5) tem, então, um sentido e
representa uma primeira solução particular da equação (1). Mas a
solução (50 nada representa, porque certos donominadores do desenvol­
vimento se anulam.
Para p = n inteiro positivo, tema-se por função de Bessel J,^
i
a série (5) multiplicada pelo factor (quando « = 0, toma-se o
2"re!
factor 1):

/„ ( x ) = — í 1 ■+
2"n! I 2(2n + 2) 2-4(2ra + 2)(2w + 4)

2 •4 • 6 (2rt + 2) (2/1 H- 4) (2n + (i)

ou
oo

(7)
Z J v i(//+ v )i V 2 /

Demonstra-se que é preciso procurar neste caso uma segunda


solução particular sob a forma

K ,^{x ) = J ^ { x ) \ a )\i, x - \ - x ~ ' " ^ h i , x ‘.


/f-- 0
324 CALCULO D IFERENCIA L E INTEGRAL

Substituindo esta expressão na equação (1) determinam-se os coe­


ficientes bk-
A função Kn (^) com os coeficientes assim definidos chama-se,
após multiplicação por um certo factor constante, função de Bessel
de segunda espécie de ordem n.
É uma segunda solução da equação (1), que, oom a primeira,
forma um sistema linearmente independente.
O integral geral escreve-se
y — C ^ J „ ( x ) -j- C 2 K n { x ) . (8)
Notemos que
lim (x) = oo.
3C->0
Por conseguinte, se se quer limitar às soluções finitas para x = 0,
será preciso fazer C2 = 0 na fórmula (8).
Exemplo — Encontrar a solução da equação de Bessel para p = 0

que satisfaça às condições iniciais:


para j =>eO, 'i/ = 2, i/' = 0.
Resolução — De acordo com a fórmula (7) encontra-se a solução particular

v=0

+_L ___
^( 2 \ 2 I (3 !)2 1 2 / ^ '* *
Utilizando esta solução, pode-se escrever a solução que satisfaz às
condições iniciais dadas, a saber:
í/ = 2 /o í^).
No/a — Se se tivesse de procurar o integral geral da equação proposta
procurar-se-ia uma segunda solução particular sob a forma
00
^ o (^) = J q (^) Log a: + 2
ft= 0
Limitamo-nos a indicar que a segunda soluçãov particular, que designamos
por kç^ (jc), se escreve

*•<*> = •'«(*) Loí ' + -S— ( f ) ‘ ( ‘ + 4-) +


+ iã T p ( f r í- + w ) - ...
^ Após multiplicação por um certo factor constante, esta função chama-se
função de Bessel de segunda espécie de ordem zero.
SARIBS 325

Exerdcios
Escrever os primeiros termos das séries de que se conhece o termo geral:
1. Un=^ 1 2 _ d. Ufi —
(n !)2
n (« + !) ’ ‘ * pnf! •
4» «n=(— . 5. «n= y n*+ l—
Estudar a convergência das seguintes séries:

6* “^ + " ^ + " ^ + - - * + 2n + - - - Convergente.


1 . 1 , 1 . . 1 , Resp. Divergente..
Vlõ V 20 1/3Õ *’ Víõ^
n -\-i
«• 2 + 4 + Í + + Resp. Divergente..
1
9• o “!“ 3 —“j* • • • 3 Resp. Divergente.
VT y S "
10 • + C<«ver*cme
11 I ^ Resp. Divergente.

12. 1 1 1 1 j -----11---- '- ’ ** Resp. Convergente.


2 5 ' 10^17^
Estudar a convergência das séries de termos gerais:
1 1
13. Un = - ^ • Resp. Convergente. 14. Resp. Divergente.

15. Un = g ^ -^ ^ . Resp. Divergente. 16. = Resp. Divergente

17. U;i=- Resp. Convergente.


/ i 2 - j- 2iH - |- 3
1
18. “n-i = —
n Log n
R**P- I>ivergwite.
1 1 1
19. Demonstrar a deâgiialdade 1 ---- > Log (n + 1 ) >
2 o n
1 1 1
> 2 + y + -+ í+ i-
20. O teorema de Leibniz é aplicável à série
— ---------- ------- 1---- -------------------- h ... ■
V 2-I V 2 -f-l V 3-I 1 /3 + 1 ~ [/n — 1 "V/n+ 1

Resp. Não é aplicável, dado que os termos da série não decrescem monò-
tonamente em valores absolutos. A série diverge.
Quantos termos é preciso tomar nas séries seguintes para ter a soma a
menos de -1 -
i0«
326 CALCULO D IFERENCIA L E INTEGRAL

1 1 1 1 1
Resp. /I = 10«.
'2- T - 3 + T - 5 + - - - + ( - 1 ) " T + --'
1 1 1 1 1
Resp. n = 10».
1 1
2>5. -r( —ir-^+-«. Resp. n= 10.
2 2-3 ' 2-3-4 2-3-4-Õ

Dizer se as séries seguintes convergem absolutamente:


1
2õ.
32 ■ ■ ■ 52 + 92 + • • • + (
72 (2 n — 1)2 + ...
Resp. Convergência absoluta.
1 1 1 1 1 1 1
26.
T ~ T ■lã" T ■ “ ■■■ ^~ T '2 " ■
Resp. Convergência absoluta.
27.
_J _____ 1__ __ 1_____ -l)n
Log 2 Log 3 ' Log 4 Log 5 Log n
Resp. Semi-convergente.
28. — 1
1 1 . 1
^2
Resp. Semi-convergente.-
Encontrar a soma da série:
29. 1 . 1 . 1
1 .2 -3 ^ 2 -3 -4! + . . . + n(n+l)(« + 2 ) ^****'4
Para que valores de x as séries seguintes convergem?
X x2
30. l-r-y H ^ + -- - + ^ + “ Resp. — 2 < X < 2.
x2 X» X^
31. Resp. — 1 < x < 1.
1
32. 3x + 3*x* + 3»x» + . . . + 3” i"* + . . . Resp. IX | < y .
lOOi 10000x2 , lOOOOOOx*
33. Resp. — c» < X < oo.
1-3 1-3-5 1-3-5-7
34. 8cnx + 2 sen-^ + 4 «en - ^ + ... + 2" ( i—
'3n^*** Resp. — oo < x < oo.

3õ. ...+ .. Resp. — 1 < x < 1 .


1+ l / l 2 + 1^2 " + V»
2^ 3'*
36. ^ + 2J + Resp.—

37. x + ^ x * + - | i x 3 + . . . 4 - ^ x » » + ... Resp. —e < x < e .



38.
39. Achar a soma da série x + 2 x 2 + . . . + n x " + . . . ( | x | < 1 ).
SâRlES 327

Indkaçõo — Escrevei’ a série sob a forma


X ...
x2 + x3 + x 4 + ...
x3_j_x^ _}- . . .
^ ®4 +...
...............................•
Dizer se as séries seguintes são majoráveis«sobre os segmentos indicados:
X x^ x^
(0 < X< 1). Resp. MajoráveL.

• 1H—j—1—^ H— 0—h • • • + — + • • • (0 < •< 1). Resp. N io majorávd.


aenx , aèn 2x , ten 3x
42. 22 . [0, 2ti]. Resp. MajoráveL.
la ^32^***^ na
Desenvolvimento de funções em séries
1
43. Desenvolver em série de potências de x e indicar o intervalo de
10 + x
convergência. Resp. Converge para 10 < x < IOl
44. Desenv<dver cosx segundo as potências de .

45^ Desenvcáver segundo as potências de x.

Resp.

46. Desenvolver eP^ s ^ n d o as potências de (x — 2).

Resp. íS + «* (* _ 2 ) + ^ (x -2 )í ( x - 2 ) s + ;..

47 Desenvolver x» — 2x* -f 5x — 7 segundo as potências de (x — 1). Resp— 3 +


4- 4 (X - 1 ) + (X — 1)2 + (X — 1)3. 168
48. Desenvolver o polinómio xi0-j_2x®—3x’ —dx®+ 3x^ + 6**—^ —2 em série
de Taylor das potências de x — 1; verificar que este polinómio admite 1
como raiz tripla. Resp. / (x) í = 81 (x— 1)3 + 270 (x— 1)^ + 342 (x— 1)®+
+ 330 ( X - 1)3 + 186 ( X - 1)7 + 63 (X - 1)8 + 1 2 (x - 1)» + ( x - l)io.
49. Desenvolver cos<x + a) segundo as potências de x. Respi coso —x seno ~
xa x3 , x^
— 27-cosa + -gTac® « + - ^ c o s f l —...
50* DesenvdvM’ Log x segundo as potências de (x — 1). Kçsp, (x — 1) —

- ^ (X - D*+ ^ (X -1)»- 44- ( * - D* + ■


328 CÁLCULO D IFEREN CIA L E INTEGRAL

51. Desenvolver em série de potências de (jc + 2).


oo

H.p. .- • [ .+
n»l
52. Desenvolver cos^x cm série das potência de . Rép. - ^ +
Jl \2^-l
4“-‘ ( — t )
+ S < - » * ----- ( 2 .- 1 ) .
n= l
1 °°
53. Desenvolver —5- em série de potências de (x + 1). Resp. 5j +
n=0
X ( x + l ) ^ ( - 2 < x < 0). 168

54. Desenvc^ver tg x cm série de potências de Resp. l + 2 x

Escrever os quatro primeiros termos do desenvolvimento em série inteira


das seguintes funções:

55. ♦tg * . DRep.


'
* +, _x 3 + 2x6
_ + _I T x+? ,. . .
x2 4x4 31x6
56. e Rép. e ^
2 ! ^ 4 ! 720
x2 7x4
57. + ...
24
x2 . 7x4
58. Log(l + ^*). Rép. Log2 + f ...
2 8 ^ 384
59. Rép. +

60. (1 + *)*. Rép. l +

61. secx. Rép. 1 .

62. Logcos*. Rép.

63. Desenvolver sen^x em série inteira de x.


, (Ax)3 (/cx)6 (fex)7
Rep. ------- ^ - p + . . .

64. Desenvolver sen^x segundo as potências de x e determinar o intervalo de


2x2 23x4 , 26x« 02n-lx2n
convergência. Resp.
2 ! 4! ^ 6!
A série converge qualquer que seja x.
65. Desenvolver — -— em série inteira Resp. 1 — x* + x* — x« + ...
1+X2
66. Desenvolver arctgx em série inteira.
SfiRIES 329

dx
Indkaçõo — Servir-se da fórmula arc tg x = ^
l + x2 *

Resp. I ---- --------------- ^ + - . . ( - l < x < l ) .

67. Desenvolver
1
em série inteira. Resp. 1 — 2 jc -h 3x2 — 4 jc* + . ..
( 1 4 - x )2

Utilizando as fórmulas do desenvolvimento em série de potências das fun­


ções e*. senx, cosx, L o g (l+ x ), (1 + x)’^ e aplicando diversos processos
desenvolver em séries de potências as funções e determinar os intervalos
de convergência:

68. senhx. Resp.x + - | y + - ^ + . . . (— oo < x < oo).

69, coshx. Resp.l + - | ^ + - ^ + . . . (— o o < x < o o ).

1 (— l)"(2x)*«
7 0 . c o s 2 X,
. Resp.l + - | - 2 ( —o o < i< o o ) .
(2n) !
n =l

71. (1 + x) Log ( 1 t - x ) . R e^. x + ^ (— 1)“


(n— 1) n
n -2

72. (1 - | - x ) e Resp. 1 ^ ^ x^ ( — oo <[ x o o ).

n=2
oo

2 ( I* I<
n=0
74. Í Ü ^ . R e s p . l + ^ + | i + . . . + ^ + ( — oo X o o ).

OO

n=0
-1 /7 ^ n Jl X - ,
76, e* acu x. Resp. x -|- x 2 - |- ------------ g -|— 1_
-TT+-"
( — oo < * < o o ).

1 *8 1.3 i5 1 .3 ... (2 /1— 1),


77. x + y r + x i . R e s p . x - . ^ - j - + ^ ~ + . . . + ( - l ) n + l
2^./i 1
x2n+l

,8. | í 2 S ( ^ 8 ,. 2
330 CALCULO DIFEREN CIA L B INTEGRAL

r j2 ^ í,.R é p . 2
U n=0 CO

sa. f Rép. C + L o g | * l + 2 (2>.H2n)l (“ °° < * < »


n=l

0 < x < oo).

J Í T I ^ ' S 9b — 8 •
ü naal

82. Demonstrar as igualdades


sen <o + x) = sen tf cos x + cos tf sen x
cos(tf-fx) = costf cosx + sentf senx
desenvolvendo os primeiros membros segundo as potências de x.
Calcular, com a ajuda das séries:
83. COS 10° a menos de 0,0001. Resp. 0,9848.
84. sen 1° a menos de 0,0001. Resp. 0,0175.
g5. sen 18° a menos de 0,0001. Resp. 0,3090.
g6. ssn a menos de 0,0001. Resp. 0,7071.
4
87. arctg J . a menos de 0,0001. Resp. 0,1973.
5
88. Log5 a menos de 0,0001. Resp. 1,609.
89. log5 a menos de 0,0001. Resp. 0,609.
90. arcsen 1 a menos de 0,0001. Resp. 1,5708.
91. ^ menos de 0,0001. Resp. 1,6487.
92. loge a menos de 0,00001. Resp. 0,43429.
93^ cos i a menos de 0,00001. Resp. 0,5403.
Utilizando o desenvolvimento em série de Maclaurin da função / (x) =
= calcular a menos de 0,001:
94. y ^ . Resp. 3,107.
95. V to. Resp. 4,121.
96. y^ãÕÕ. Resp. 7,937.
97. {/25Õ. Resp. 3,017.
98.
99. \Í 2 Resp. 1,2598.
Calcular os integrais seguintes, desenvolvendo ein série as funçSes sob os
sinais de integração:
1
s in x dx a menos de 1Q“®. Resp. 0,94608.
100.
S ê RIBS 331

10,. J ,- » • dx a menos de 0,0001. Resp. 0,7468.

102. ^ sen (x2) dx a menos de 0,0001. Resp. 0,1571.


ü
1
2
103. ^ e ^ ^ d x a menos de 0,01. Resp. 0,81.
ü
0,5
I arc^tga; ^
104.
ü
1
^ COS ~ ^xd x a menos de 0,001. Resp. 0,764.

106. ^ Log { í x ) dx a menos de 0,001: Resp. 0,071.

1 *2
107. [ e ^ dx a menos de 0,0001. Resp. 0,9226.
ü
i
5
108. Ç dx a menos de lO.OOOl. Resp. 0,0214.
J V l-x
ü

109
•h
0,5
dx
+ x4
a n>enos> de 0,001. Resp. 0,494. j L o g (l + J ) ^

Indicação — No decorrer da resoluçSo deste exemplo e dois dois seguintes


i preciso ter em vista as igualdades;

V ( —1)"~^ _ 'V 1
n* 6 ’ Zj «2 ~ 12 ’ Z j (2«—
n= l n= l n= l

que serão estabelecidas no § 2 do cap. XVII.


1 1
332 CALCULO D IFERENCIA L E INTEGRAL

Integração de equações diferenciais por séries


113. Encontrar a solução da equação y " = x y que satisfaz às condições iniciais
seguintes: jc = 0, y = \ , y ' = 0.
Indicação — Procurar a solução sob a forma de série.
õ-^-f' 2.3.5.67 + . . . ■ 2.3.5.6 . .. (3/c— 1) 3A:
Resp. 1 + 2.3
114. Encontrar a solução da equação y" + xy' + y = 0 que verifica as condições
x^
iniciais: para jc = 0, y = 0, y' = L Resp. jc---- — |- — ... +
(_l)n+i x2n-l 2 1*0*0
■■■ ... (2n — l) •
115. Encontrar a solução geral da equação
l V + l y ' + ( x 2— j, = 0.
Indicação — Procurar a solução sob a forma
1
(A o+ . h x - í - A 2 3 : - ^ Resp. [ l —^ + ... J +
1
-rC2X ^ ^<^0SX
+C ^ .

116. Encontrar a solução da equação xy" + y' + xy = 0 que satisfaz às condições


x^
iniciais: para jc = 0, y = 1, y' = 0. Resp. 1 ---- ^ +
22 ^ (1.2)22* (1.2-3)22«
r2lí
+ . . . ( - D" +
(A:!)2 22'‘
Nota — As duas últimas equaçSes diferenciais sSo casos particulares da
equação de Bessel
2^V '+ xy' + (x2 —n2) J, = 0
1
quando n = -^ e n = 0.
117. Encontrar a solução geral da equação 4x£/" + 2y' + í/ z=0.
Indkação — Procurar a solução sob a forma de série xP (oq + a^x +
+ OnX^ + ...). Resp. COS "i/^ + ^2 sen ’\/x .
118. Achar a solução da equação (1 — jc^) y" — jcy'= 0 que satisfaz às con­
dições iniciais: y = 0, y ' = 1 para jc = 0. Resp. a;_ |.JL £ !_ _ |_ _ L A ^_ u
1 2 3 2 4 5 '
■*■ 2 4 6 7 + • • •
119. Achar a solução da equação (1 + jc2)y" + 2jcy' = 0 que satisfaça às condi-
m .... X^ X® Xt
ções iniciais: y = 0, y = 1 para jc = 0. Resp. x ----------- 1-------------------- u . . .
, 3 5 7
120. Encontrar a solução da equação y" = xyy' que satisfaz às condições iniciais:
x3 , 2*4 3x5
y * 1, y' * 1 para x * 0. Resp. l -j-j
3! ^ 41 5!
S âR IE S 333

121. Encontrar a solução da equação ( 1 —x ) / = l + x —> que satisfaça às


iniciais: ^ = 0 para = 0 e indicar o intervalo de convergência da série
obüda. Resp. i + ^ +

122. Achar a solução da equação xy'* + y = 0 que satisfaça às condições


iniciais: y = 0, y' = i para jc = 0 e indicar o intervalo de convergência.
x'^
(—o o < x < o o ) .
(1!)2 2I^~l(2 !)3 3 (3!)4 4
2
123. Achar a solução da equação y"+ -^ y'-{-y = 0 que satisfaz às condições

iniciais: y = 1, y' = 1 para x = 0, Resp.

1
1*24. Achar a solução da equação ---- y'-\-y = 0 q u e satisfaz às condições
X
iniciais: y = 1, y' = 0 para x = 0 e indicar o intervalo de convergência da
série obtida. Resp. 1 — ^

2! ^ 22-42 22-42.62
Determinar os três primeiros termos do desenvolvimento em série inteira
das soluções das equações diferenciais seguintes de que se indicam as
condições iniciais:
4x3
125. y' = x^ + y^ ; para x = 0, y ^ ^ í. Resp. 1-f a: f x2 4 - —^ .
é»x2 x3
126. y" = ey-{-x; para x = 0, y = l, í/' = 0. Resp. 1
2 ' 6
x2 x3
127. y '= seny — senx; para a: = 0, y = 0. R esp.---- ------- ----- ...
2 6

Determinar alguns termos do desenvolvimento em série inteira das soluções


das equações diferenciais seguintes de que se indicam as condiçõs iniciai*
indicadas:
x2 2x2
128. = x2; para x = 0, y = 0, y '= 0. Resp. 1+ x + - ^ + — \~— — h
4!
14x6
+ •••
1 . 1
129. y' = y2-i-x3; para x = 0, y = -^- Rcsp- — + T ^+ '« ^^+ Í6

130. y' = x i — y ^\ para z = 0, y = 0. R e s p . 1 * — ^ + — ...

X3
131. í/' = x 2 y 2 _ i ; para x = 0, y = l. Resp. 1— x H — ------ ^ + - 5 ---- •••

x2 2x2 llx^
132. y' = eV-\-xy\ para x = 0, }í = 0. R e^. + — 3“ + "2^37^ + • • •
Ca|>itulo XVH

SÉRIES DE FODRIER

§ 1. Definisão. Posisão do proUema


Chama-se série trigonométrica a uma série da forma

— + o, COS X + 6, sen X uj cos 2x + 62 sen 2x - f ..

OU, sob uma forma mais compacta


oc

+ 2 (a„ COS/JX - f b„ sen nx). ( 1)

As constantes ao, a„ et (n = 1, 2, ...) são os coeficiertíes da


série trigonométrica.
Se a série (1) convergir, a sua soma é uma função periódica f (jc)
de período 2tt, dado que sennjc e cos/uc são funções periódicas de
período 2ir.
De modo que
f (x) = f(x 2n).

Ponhamos o seguinte problema.


Dá-se uma função periódica / ( jc) de período 2tt. Pergunta-se
para que condições impostas a / ( jc) existe uma série trigonométrica
convergente para f (jc).
Este problema será objecto do presente capítulo.
Determinação dos coeficientes da série por meio das fórmulas
de Fourier.
Suponhamos que a função / ( jc), periódica e de período 2tt, pode
ser representada por uma série trigonométrica convergente para / ( jc)
no intervalo (— tt, tt), isto é, que seja a soma desta série:
oo

/(^ ) = -^ + (a„ coswx + bn nx). (2)


n = í
SÉRIES DE FO U RÍER 335

Suponhamos que o integral da função do primeiro membro desta


igualdade é igual à soma dos integrais dos termos da série (2). Isto
terá lugar, por exemplo, se se suposer que a série proposta converge
absolutamente, isto é. que converge a série numérica positiva seguinte

{^)
-f- I flj j -f- j I -f- j «2 I + j ^2 í + • • • + I MI - f I I + •••

A série (1) é, então, majorável e pode ser integrada termo a


termo de — ir a tt. Aproveitemos para calcular o coeficientes a©.
Integremos os dois membros da igualdade (2) de — tt a + ir:

^n

—n
}(x )d x =
n
^ ^ d x
-J t
2
n = l —ji
(ín
a ,i COS n x d x
:i
J
—jx
sen n x dx^ .

Calculonos. separadamente, cada integral do segundo membro:

TÍGq,
—JI
71 n
- a» sen nx
^ cosnardx = | COS n x d x = -------------
— JX — JX

JX JX

cosnx
^ bn s m n x d x = bn ^ sen nx dx = — bn = 0.

Por conseguinte.

J / {x) dx = jtao,

donde
JX

Uq / (:r) dx. (4)

Para calcular os outros coeficientes da série, calcularemos, prèvia-


mente os int^rais auxiliares seguintes.
336 CALCULO D IFEREN CIA L E INTEGRAL

S t n e k forem inteiros t st n = ^k, tem-se

J CO S n x C O S k x d x = 0;
—n
n
J c o s n x s ta k x d x = 0\ (I)
— 71
Ti
í sen rax sen fcr d r = 0 ;

e se n = k.
J cos^ k x d x = n \
- Jt
7i

^ sen kx cos k x d x = 0; (H )
Jti
—T
Jt
Jt

^ stxi^kxdx = n.

Calculemos, por exemplo, o primeiro integral do grupo (I).


Como

COSnx COS k x = [cos (ra + A) x + cos (n — k) x],

tem-se
Jt

Jt
í cosnx cos k x d x =

Jt
~ y f cos (r + Á:) X dx + ^ ^ cos (n — k )x dx = 0?

De igual modo se obtêm as outras fórmulas (!)(*). No que


respeita aos integrais (II) calculam-se directamente (ver cap. X, t. I).
Pode-se calcular, agora, os coeficientes e ó* da série (2),

(*) Com a ajuda das fórmulas


1
cos n x a ta k x = — [stn{n-\-k) x ^ a t n ( n - k ) x],

sennx stn k x = — [— cos (« + A:) x + c o s (n — k ) xj.


S âR IE S DE FO U RIER 337

Para determinar para dado, multipliquemos os dois


membros da igualdade (2) por cos kx:

f ( x ) COS k x = — COS k x +
2

+ (a^i COS n x cos k x + bn sen n x cos k x ) , ( 2' )


71=1

A série do segundo membro da igualdade é majorável, dado


que os seus termos não são superiores em valores absolutos aos
termos da série positiva convergente (3). Pode-se, pois, integrar termo
a termo sob qualquer segmento.
Integremos a igualdade (2') de — tt a tt:

J f {x )c o sk xd x = ^ ^ coskxdx

+ J o o ^ n x c o s k x d x b n ^ sen nx cos/cx
n=l —«

A respeito das fórmulas (II) e (I), vê-se que todos os int^rais


do segundo membro se anulam, excepto o que contém o coeficiente
Por cons^uinte.

J f (x) cos k x d x = J c o ^ k x d x = akn,


donde
n
a /i= — 1 / (x) cos k x ãx. (5)
JX J

Multiplicando os dois membros da igualdade (2) por sen^x e


integrando de novo de — ir a ir. obtém-se:

5 f (x) SCO kx dx = bft J s è a ^ k x d x = b )in ,


—71 —Jl
donde
Jl
1 f
bk = — \ f(x) sên kx dx.
n J (6)
— Jl
338 CALCULO D IFERENCIA L E INTEGRAL

Os coeficientes definidos pelas fórmulas (4), (5) e (6) chamam-se


coeficientes de Fourier da função /(jc) e a série trigonométrica (1)
formada com estes coeficientes é a série de Fourier da função f{x).
Voltemos, agora, ao problema posto no princípio deste parágrafo:
quais as propriedades que deve possuir a função / ( jc) para que a
série de Fourier convirja e que a sua soma
seja igual aos valores da função nos pontos
considerados?
Vamos enunciar um teorema dando as
condições necessárias para que à função / (jc)
seja representável por uma série de Fourier.
Definição — Uma função / ( jc) diz-se
monótona por corte sobre o segmento [a, b\
se se puder decompor esse segmento pelos
pontos jci, JCz. num número finito
de intervalos (a, Xi). {xu JC2) -. (jCn-i.
de modo que a função seja monótona em cada intervalo, isto é, que
seja ou não crescente ou não decrescente.
Resulta da definição que se / ( jc) for monótona por corte e
limitada no segmento [a, 6], apenas pode ter pontos de desconti-
nuidade de primeira espécie. Com efeito, se jc = c é um ponto de
descontinuidade para a função / (jc), tem-se, em virtude da monotonia
da função,
lim / ( x ) = / ( c — 0 ) , lim / (a :) = / (c + 0 ) ,
X->C— 0 «-♦'C+O

isto é, que o ponto c é um ponto de descontinuidade de primeira


espécie (fig. 356).
Temos o teorema seguinte:
Teorema — Se a função periódica / ( jc) de período 2tt é monótona
por corte e limitada no segmento [— tt, tt], a sua série de Fourier
converge em todos os pontos. Á soma da série obtida s(x) é igual
ao valor da função / ( jc) nos pontos de continuidade. Nos pontos de
descontinuidade de / ( jc) a soma da série é igual à média aritmética
dos limites da função à esquerda e à direita; isto é, se jc = c for um
ponto de descontinuidade de / ( jc).
/ ( c - 0 ) + /(c + 0)
S(x)^=c =

Este teorema mostra que a classe das funções representáveis por


séries de Fourier é bastante lacta. Eis porque as séries de Fourier
SÉRIES DE PO URIER 339

têm encontrado uma larga aplicação nos diferentes ramos da mate­


mática. Utiliza-se particularmente com sucesso as séries de Fourier
eni física matemática e nas suas aplicações a problemas concretos
de mecânica e de física (ver cap. XVIII).
Não demonstraremos o teorema que acaba de ser enunciado.
Demonstraremos nos § 8, 9, 10 um outro critério suficiente para que
uma função possa ser desenvolvida em série de Fourier. Abrange,
num certo sentido, uma classe mais restrita de funções.

§ 2. Elxemplos de desenvolvimento de funções


em séries de Fourier
Vamos dar exemplos de desenvolvimento de funções em séries de Fourier.
Exemplo— 1. Dá-se uma função periódica f(x ) de período I tt definida
como se segue:
f(x) = x,

Esta função -é monótona por corte e limitada (fig. 357). Ela admite, pois,
um desenvolvimento em série de Fourier.

Aplicando a fórmula (4) do § 1, obtém-se:

1 f ^ \ x2
ÜQ= — \ X dx = ----- = 0.
Ji J ji 2

Apliquemos a fórmula (5) do § 1 e integremos por partes:


ji jt
I f\
= — X
i r s c nA: xJ x i r
COS kx dx = -*-T- X — ;---- ---------- \ sen kx dx = 0.
J Jt L A: j-ji k i J

De acordo com a fórmula (6) do § 1, determina-se:


Jt
n í !„ + T jc o s fe d z ] = ( - l ) f t + i l .
— Jt
Obtém-se, assim, a série
340 CALCULO D IFEREN CIA L E INTEGRAL

Esta igualdade tem lugar para todos os valores exoepto nos pontos de
descontinuidaae. Nestes pontos a soma da série é igual à média aritmética
aos limites da função à esquerda e à direita, isto é, a zero.
Exemplo — 2. Dá-ss uma função periódica de período 2nr definida como
se segue:
f { x ) = — x para —
f (x ) = x para 0 < x < ji
isto é, que f{ x ) = \x \ (fig. 358). Esta função é monótona por corte e limitada
sobre o segmento — -tt < x < tt.

yn

-5 n -4jr -3 ji -2ti - n 0 n In 5n 5n ^

Fig. 358

Determinemos os coeficientes de Fourier:


Jt 0
flo—— j / W dx = ^ ^ J ( _ x) d x ^ J x d x j = JI,
-J i -J t 0

1 ^ ”
ííft = — J ( —^) COS k x d x - \ - ^ X COS k x d x j =
-J t ü
0 ji
1 r isenfci|o 1 f , is e n fc i |n i
= 1 t L-------í — U + T k
^ _ |J ,e n f c x d * ] =
-Jt ü

=-Lr. T ík L
COS k x

—Jl
COS
^
k x \n

|0 ] =
{ 0 para ^ par.
= — (C O S k n
___ ^
para k ímpar;
Jlfe2
U Jl

bk = - ^ ^ J ( —^) s©n /cx dx-|- ^ X sen A:x d x j = 0 .

Obtém-se, pois, a série


^ . Jl 4 r COS JX , COS 3x , COS 5x COS (2p4-l) X
32 52 (2p+l)2 }■ • ■
Esta série converge para todo o valor, e a sua soma é igual à função
proposta.
SâR IE S DE FO U RIER 341

Exemplo — 3. Define-ss uma função periódica de período 2nr como


se segue:
/ (^) = — 1 para — Ji < x < 0,
f(x) = í Pwa 0 < a : < j i .
Esta função (fig. 359) é monótona por corte e limitada no segmento
— *ir X
Calculemos os coeficientes de Fourier:

n 0 ji

«*0=4- J / W á t = ^ [ J (-i)<te+jáx]=0:
—« Jl 0
0 Jl
^ 1r^ jf /( — -I\ I J . f 1 J 1 sên A
^ Bcn/cx 0 .-----=
sen kx:x Jl
afe = — 1) COS kxdx-\- \ cos kx dx \ = — 1— j-— 0 ;
-Jl kA

0 Jl
COS kx 0 coskx |Ji
= J (— 1) sen A:x d x + ^ sen/cx dx J = ^
-J l k |o ] =
-J l 0

r 0 para
p k par,
^~nk 1=
' ^
para k impar.*.
I nk

A série de Fourier considerada escreve-se, pois,


4 r senx , sen 3x sen 5x sen (2p + l) ^
'w = 4 [ ^ + 2p+l
Esta igualdade é exacta para todos os valores excepto nos pontos de
descontinuidade.

y i

-3 J l -2 n - Jl 0 Jl 2 Jl 3ji X

-1

Fig. 359

Indicamos, na figura 360, como as somas parciais Sn representam com


uma precisão crescente a função /(x) quando n —> oo.
Exemplo — 4. Dá-se uma função periódica de período Iw definida como
se segue:
/(x) = x2, — j i < ; x < ; j i (fig. 361).
342 CALCULO D IFEREN CIA L E INTEGRAL
SÉRIEIS DE FO U RIER 343

Calculemos os coeficieates de Fourier:


jt
1
«0J= -L f x2dx = — - ^
Ji J JI 3
—Jt

1 f o I j 1 r x 2 s e n A:a; |Ji 2 f , , 1
= — \ COS kx dx = — -------- --------- — 7- i ^ s e n kx dx =
n J n \_ k \-n k J ^ J

_______ L f X COS kx ^ 1 f “I 4
^ + \ COS/cx d x j [jx COS A:Ji] =
nk L

■I
4
para k par,

— -p - para k ímpar;
Jt
x2 COS kx
bh= — a;2 s e n kx dx= -]-----
\ -f ^ ^ COS kx dx J =
JX J ^ ji L
Jt
2 rxsen/cxJt ir
-n~T ]
—Jt

Logo, a série de Fourier da função dada, escreve-se


„ jx2 , / cosx COS 2x , COS 3x
n - i - 22 32
Como a função é monótona por corte, limitada e contínua, a igualdade
tem sempre lugar

Fazendo na igualdade obtida x = nr, obtém-se;


oo

6 ~ ^Zl w
«22 •
n= l

Exemplo — 5. Dá-se uma função periódica de período 2*7t definida como


se segue:
/(x) = 0 para — j t < x < 0,
/ ( x ) = x para 0 < x < j i (fig. 362).
344 CALCULO D IFEREN CIA L E INTEGRAL

Definamos os coeficientes de Fourier:


JT (I n

-JT

.n
1 r x sen kx
— — f X COS kx d x — — r — ^ f sen A:x dx I =
^ i
0
ji L 0 A: J ']=
I COS k x r 2
^ para k ímpar,
Jik k
” I 0 para k par;
I = - 1 jr x se n 7/r x7r f^ ^1_ [_
r -------
XCOSA'x -T , 1 f , n
^ + _ J costodxJ =

J_ para k ímpar.
= -----COS Aji =
:ik 1 para k par;

O desenvolvimento em série de Fourier é


... nJi 2 í/COS
COS X . COS 3x ^ cos 5x , \
12 » 32~ ' 5 ^ ‘* j
/ aèn
sen X sén 2x
2 , sen 3x
1 2
Nos pontos de descontinuidade da função f{x) a soma da série é igual
à média aritmética dos limites da função à esquerda e à direita ^ no caso
presente •
Fazendo na igualdade obtida x = 0, obtém-se:
oo
8 ~ Z j (2n —1)2 ■
n=l
§ 8. Úma nota sobre o desenvolvimMito das fdnsões periódicas
em série de Foorier
Indiquemos a propriedade seguinte duma fun^o de período
2it\ tem-se
n A.+2JI
í ■<|)(a:)da:= j Tfi(a:)da-,
- Ji X

qualquer que seja o nómero X.


Com efeito, como
1)3 {£ — 2 j l) = l|3 (g ),
SÊ R IE a DB FO U RIER 345

Fazendo x = ^ — 27t, pode-se escrever, quaisquer que sejam c e d:


d (f+ 2 n d-\-2zi d+2ji
i |: ( | — 2 j t ) d | = J i j ) ( |) d |= J \lp(x)dx
c c+2 ji c+2.t; c+2.t

Em particular, fazendo c = — ir, d = X, obtém-se:


X X+2n
5 ^p(x)dx= J \p{x)dx,
—jt n
por conseguinte,
A.+2JI — Ji n P.+2JI
J ij) (:c) d j: = ^ J i|? (^) d^:* + J ilp (.r) d j: =
A, — Ji Jt

— Jt Jt A. Jt
= ^ '\^{x)dx I {x) dx J ip (a:) da: = ^yp (x) dx.
K — Jt — Jt — Jt

A propriedade mencibnada 'significa: o integral duma função perió­


dica tfi (x) sobre um segmento arbitrário de comprimento igual ao período
tem sempre o mesmo valor. Geomètricamente: as áreas tracejadas sobre
a figura 363 são iguais.

Resulta da propriedade demonstrada que se pode. no cálculo dos


coeficientes de Fourier, substituir o intervalo de integração ( — ir, i r )
pelo intervalo (A, A -f- 2w), isto é, que
A ,+ 2 j t A ,+ 2 j t

1 f f 1
ão = — \ f{x) dx, Un = — \ f (x) COS nx dx,
A, A,
A ,+ 2 j t ( 1)
1 f
= — \ j (x) sen nx dx,
n J

onde A 6 um número arbitrário.


Isto resulta do facto de a função f{x) ser, por hipótçse, uma
fu n ^ o periódica de período hr, por conseguinte, as hinções f (x ) co s n x
346 CALCULO D IFEREN CIA L E INTEGRAL

e / (jc) sen nx são também funções periódicas de período 2ir. Mostremos


com um exemplo como a propriedade demonstrada simplifica, em
certos casos, o cálculo dos coeficientes.

Exemplo — Seja desenvolver em série de Fourier a função f(x) dc


período igual a x sobre o segmento 0 < jc < 2*ir, O gráfico da função / (x)
está representado na figura 364. Esta função é dada no segmento nr]
por duas formulas: fix ) = x + 2'ir sobre o segmento [— nr, 0] e /(x) = x sobre

o segmento [0, nr.] Ora esta função está representada muito simplesmente por
/ (x) = X sobre o segmento [0, 2nr]. Por conseguinte, ter-se-á interesse em
desenvolver esta função em série de Fourier, utilizando a fórmula (1) com X = 0:

1 •
’ 1 r
<7o= — j f (^) dx —— \ xdx = 2n \
0 ' ü
2Jt 2 ji

I f . / v , I f , Ifxscn nx
fln = — \ f ( x) COS n x d x = — \ X COS n x d x = — ----------
J i j J i L n

COS wx"|2Jt
+-^ J c Jo =«•

^ / (x) sên nx dx = ^ X sen nx dx =


0 ü
X COS n x , s c n n x “l2jt
= 4 [- n n2 Jo
Por conseguinte.
2 2 2 2
/ (x) = ji — 2 sen x— ^ sen 2x — — sen 3x—— sen 4x — f 5x— . . .
2 o 4 0

& ta série representa sempre a função dada, exoepto nos pontos de


descontinuidade (os pontos x = 0, 2nr, 47t, ...). Nestes pontos, a soma da
série é igual à semi-soma dos valores limites da função /(x) à direita e à
esquerda (no caso presente a nr).
SâR IE S DE FO U RIER 347

§ 4. Séries de Fourier de funções pares e impares

Resulta da definição das funções pares e ímpares que se ^(;c)


é par se tem
ji ji
J dx = 2 ^ yp (x) dx.
— n 0
Com efeito.

I y\){x)dx= J il) (j:) -f- í ^ (^) d .r = J xj) (—x) dx +


-Jl —Jl 0 0
Jl Jl Jl Jl
+ 5 (^ ) d x = J ij; (a:) da; + ^ \)j (x ) d x = 2 J ij; (x ) d x ,

dado que uma função par goza, por definição, desta propriedade;
x) = ^ (x).
Tem-se, duma maneira análoga, para uma função ímpar y>(x)

Jt Jl Jl

J (p{x)dx= ^ (p {—x) dx + í 9 (^) dx =


— Jl 0 0
Jl Jl

= — ^ (p (x ) d x j (p (x ) d x = 0.
0 0

Se se tiver o desenvolvimento de Fourier duma função f (x) ímpar,


o produto f{ x) c os kx é uma função ímpar e /(x)senà x uma função
par; logo,
Jl

«0 = -^ j f{x)dx = 0,
— Jl
Jl

1 r
«A = — I / (^) COS kx d x = 0 , (1)
3T J
— Jl
Jl Jl

6* = — ^ / (x) sen A;x dx = -^ J / (x) sen kx dx,

isto é, que a série de Fourier duma função ímpar apenas contém senos
(ver exemplo 1, § 2).
348 CALCULO D IFEREN CIA L B INTEGRAL

Se se tiver o desenvolvimento de Fourier duma função par o


produto / (x) sen kx é uma função ímpar e f (x) cos kx é par. pdo que:

/ (x) dx.

-li f (x) COS kx dx, ( 2)

/ (x) sen kx dx = 0,

isto é, a série de Fourier duma função par apenas contém cossenos


(ver exemplo 2, § 2).
As fórmulas obtidas permitem simplificar os cálculos dos coe­
ficientes de Fourier quando a função dada é par ou ímpar. É evidente
que qualquer função periódica não é forçosamente par ou ímpar
(ver exemplo 5, § 2).
Exemplo — Seja desenvolver em série de Fourier a funçSo par / (x) de
período 2*7t definida no segmento [0, w] por
y = x.

Já desenvolvemos esta função cm série de Fourier no exemplo 2, § 2


(ver fig. 358). Calculamos dc novo os coefícientes dc Fourier desta binçâo
utilizando a paridade desta função.
Em virtude da fórmula (2), 6^ = 0, qualquer que seja k\
Jl

_ 2 ["x sen
= — \ X dx=^n, ük =
-í X COS k x dx =

para k par,
~ ~ n L k
^ JiA:2 para k ftnpar.
Voltamos a encontrar os mesmos coeficientes que no exemplo 2, § 2, mas
mais ràpidamente.

§ 5. Séries de F o u rie r d a s fanções de período 21


Seja f{x) uma função periódica de período 2/, em regra diferente
de 2tt. Desenvolvamo-la em série de Fourier.
Façamos a mudança^ de variável

x = ^ t.
n
SÉRIES DE FO U R IER 349

A função / é. então, uma função periódica de t de


período e pode-se desenvolvê-la em série de Fourier no segmento
— 7T < JC < ir :

y + 2 (oftCosAí + ÒA senAO, ( 1)
k = i
ou

í
Jl

—jr
n
í ji

—zi
/(^ í)c o s A íd í,

Voltemos, agora, à antiga variável x:


I n
-í. t= x— dt=^dx.
Ter-se-á, então:
I

^0 — y J ~ T J k^xdx,
-I -I
I ( 2)

bk f(x) s e a k j x d x .
= H -I-
A fórmula (1) transforma-se em:

/W = ~ -f- ^ COS y a: + h/^ sen y x j , (3)


A=1

onde os coeficientes an, bf^ são calculados segundo as fórmulas (2).


Tal é a série de Fourier duma função periódica de período 2/.
Notemos que tudo o que foi dito sobre as séries de Fourier das
funções periódicas de período 2tt mantém-se em vigor para as funções
periódicas de período qualquer 21 O teorema sobre o desenvolvimento
duma função em série de Fourier do § 1, permanece em vigor, bem
como as notas sobre a possibilidade de calcular os coeficientes da série
integrando num segmento arbitrário de comprimento igual ao período
(vei § 3) e de simplificar o cálculo dos coeficientes quando a função
é par ou ímpar (§ 4).
350 CALCULO D IFERENCIA L E INTEGRAL

Exemplo — Desenvolver em série de Fourier a função periódica de período


21 definiüa no segmento [—/; /] pela igualdade / ( jc) = | jc| (fig. 365).

if, —0 ; ãi) = ^ ^ X dx ~ I \
0
^ r 0 para k par.
2 f knx ^ 21 Ç , ^ I
Uf i= — \\
— — x cCO
X oSs —
- yr - d-r
dx = —
—r^ \\ X
x cCOS,
o s k x dd xx = J 4/
para k ímpar.
0 "^0 I
Por conseguinte, o desenvolvimento escreve-se:
( 2 p - 'r 1) Jl
COS — X COS - j - X COS I —^
.. I Al
I 3^ (2p + l)2

§ 6. Desenvtrfvimento em série de F o u rie r


dum a função não periódica
Seja dada no segmento [a, b] uma função monótona por corte / (x)
(fig. 366). Mostremos que esta função pode ser representada nos pontos
de descontinuidade por uma série de Fourier. Consideremos, para esse

efeito, uma fu n ^ o arbitrária periódica monótona por corte ix(x) de


período 2 p . ' ^ \ b — a\ e coincindindo com a função f (jc) no segmento
[a. ó]. (Prolongamos f(x)).
SÉRIES DE PO U RIER 351

Desenvolvamos a função /i (j:) em série de Fourier. A soma desta


série concide em todo o segmento [a, b\ (excepto nos pontos de
descontinuidade) com a função dada / ( jc), isto é, que se desenvol­
veu / (jc) em série de Fourier no segmento [a, b\.
Consideremos em seguida o seguinte caso importante. Seja / ( jc )
uma função dada no intervalo [0. /]. Prolongando arbitràriamente esta

Fig. 367

função no intervalo [— /, 0] (conservando a monotonia por corte)


podemos desenvolver esta função em série de Fourier.
Em especial, se prolongarmos esta função no intervalo — / < x < 0
de modo que / ( jc ) = / ( — jc) , obtém-se, finalmente, uma função
par (fig. 367). (Diz-se, então, que a função / ( jc) foi «prolongada de
maneira par». Esta série desenvolve-se em série de Fourier de cos­
senos. De modo que a função / ( jc ) dada no intervalo [0, /] foi desen­
volvida em série de Fourier de cossenos.
Se se prolonga a função / ( jc) sobre — Z< jc < 0 de modo que
/(jc) = — / ( — jc) obtém-se uma função ímpar, desenvolvendo-se em
série de senos (fig. 368). (Prolongamento ímpar da função / ( jc ) . Por
conseguinte, se for dado no intervalo [0, /] uma função monótona por
corte / ( jc) , pode-se desenvolvê-la ou em série de Fourier de cossenos
ou em série de Fourier de senos.
Exemplo — 1. Seja desenvolver a função f(x) = x no intervalo [0, em
série de senos.
Resolução — Prolonguemos esta função de maneira ímpar (fig. 357).
Obtém-se a série
sen X sen 2x , sen 3x

(ver exemplo 1, § 2).


352 CALCULO D IFEREN CIA L E INTEGRAL

Exemplo — 2. Desenvolver a função f{x) = x no segmento [0, tt] em série


de cosenos.
Resolução — Prolonguemos esta função de maneira par; tem-se:
f(x) = \x\, —

(fig. 358). O desenvolvimento em série de Fourier desta última função é


n i r COS X , COS 3x , cos 5.r
2 Ji i í ' 3 - ' 5'2
(ver exemplo 2, § 2). Por conseguinte, no intervalo [0, -tt], tem-se a igualdade

71 4 r COS X , COS 3x , COS 5x


52

§ 7. Aproximação, em média, duma função dada por meio de


polinómios trigonométricos

A representação duma função em série (de Fourier, Taylor, etc.)


tem este sentido prático que a soma parcial obtida quando se limita
ao termo de ordem n é uma expressão aproximada da função que
se desenvolve. Pode-se levar esta aproximação ao grau desejado
escolhendo convenientemente n. Todavia, o carácter da representação
aproximada pode variar.
Assim a soma dos n primeiros termos da série de Taylor
coincide com a função dada no ponto considerado e não tem neste
ponto derivadas até à ordem n coincidindo com as derivadas da
função considerada. Um polinómio de Lagrange de ordem n (ver § 9,
cap. VII, t. I) coincide com a função considerada em « + 1 pontos.
Vejamos qual é o carácter da aproximação duma função perió­
dica /(jc) por meio dos polinómios trigonométricos da forma

ÜQ
(x) : + X I ah COS kx + bh sen kx.
h=i
em que ao, ai, bu bz, .... são coeficientes de Fourier. isto é
que se aproxima com a soma dos n primeiros termos da série de
Fourier desta função. Façamos algumas notas preliminares.
Suponhamos que se tem y = f (x) no intervalo [a, b] e que se
quer avaliar o erro cometido quando se substitui esta função por
uma outra função <p(.x). Pode-se considerar, por exemplo, que o eri;o
é representado pela expressão max \f(x) - (p (x) | no intervalo [a, b],
o que se chama desvio máximo entre f{x) e <p{x). Mas, por vezes.
SARIES D E FO U RIER 353

é mais natural considerar o desvio quadrático médio 8 cujo quadrado


é, por definição.
b
(x)f dx.

Expliquemos, na figura 369, a diferença entre o desvio quadrático


médio e o desvio máximo.
Suponhamos a função f(x) representada pelo traço a cheio e as
aproximações <pi (x) e <p2 U) pelos ponteados. O desvio máximo da

curva 3^ = (x) é menor que o desvio máximo da curva y = n W .


mas o desvio quadrático médio da primeira curva é maior que o
da segunda, porque a função y — n (x) se distingue notòriamente
de y = f(x) sòmente num pequeno intervalo e, por conseguinte,
caracteriza melhor a função y = f (x) que a primeira.
Voltemos agora ao nosso problema.
Seja dada uma função f(x) periódica de período 2w, Entre os
polinómios trigonométricos do grau n
n

^ + 2 («A COS fcr + Pa sen kx)


k= i

pede-se para calcular, escolhendo convenientemente os coeficientes a*


e ^kl o polinómio cujo desvio quadrático médio, definido pela igualdade

/ (x ) ^ ^ {a k COS kx + sen kx) j dx


- = i i 2 k=i
é mínimo.
23
354 CALCULO DIFBRBNCIAL B INTEGRAL

O problema reside em encontrar o mínimo duma função de


2/1 + 1 variáveis ao, a j, . . On, P i. P2. • • -i Pn-
Desenvolvamos o quadrado da expressão entre parêntesis recto
e integremos t^m o a termo; tem-se:

^ I {/* (^) — 2/ (x) -f 2 (“ A COS Ax 4- Pa sen kx)


—Ji A=1
TC

+ (a* COS -f Pa sen Ax) j| cíx = ^ J f (x) dx —

h=i —ji

n 71 ji

— 2 ^ 1 / (^) cfcc — — J ^ —
-TI k=i -Jt
71 n
- - ^ 7 , Pa f / ( x ) s é n A x d x + — -®
Ji J 2ti 4 J
f dx +
A=1 - ji - n

71 n n ji

A=1 —n h=i —Jt


71 Jt 71 Jt

«A J COS A;x dx + ^ tto Pa ^ sen A:x dx -|-


h=i —n k=í — Jt

n 71 Jt

+í 2
h=i 7 = 1
k=^J
-Jt
COS kx COS jx dx +

71 71 Jt

í COS kx SOT ]x dx 4 “

A= 1 7=1 — Jt

77. 71 Jt

3 2 PaP/ í sen kx sen jx dx.


k=i }=i —Jt
k¥=J
SÉRIES DE FO URIER 355

Notemos que
71 71
>
\ i{x)dx=ao-, / (x) COS kx dx = a h \
V ji
^ •jJ/
“ Jt —Jtji

^ / (x) sen kx dx = bk

são os coeficientes de Fourier da função f{x).


Além disso, em virtude das fórmulas (I) e (II) do § 1, tem-se,
quando k = j,
n 71

J cos^kxdx = n, J sexí^kxdx = n,
—n — 71

n
para k e j quaisquer: í sen kx cos ]x dx = 0.
e, quando, k ^ /,
Jl 71

5 COS kx COS jx d x = 0 , ^ sen kx sèn jx dx = ii.


— 71 — Jl

De modo que
71 n

^ 2n 1 ~ “ 2 ■** ^
+
f+r
Juntando e subtraindo a quantidade
2
k= i
(a^A + Pl).

^ ^
A=1
tem-se

-Jl A=1

■ +■ -^(«0 — «of + y 2
k=i
n

+
(P* ~ *a)^]- (1)
356 CALCULO D IFEREN CIA L E INTEGRAL

Os três primeiros termos desta soma não dependem da escolha


dos coeficientes a i, . .. Pi, •••. P^- Os outros termos
n

(ao — üq)^ + ~2 [(^A — + (Pa — ^a)^]


A=1

são não negativos. A sua soma é mínima (zero) quando


ao = do^ ^ í — d l , . . ., OLn = d jii Pl ~ ^1» • • •» Pn =
Tal é a escolha dos coeficientes ao, a i, .... p^, .... P^ para
o qual o polinómio trigonométrico
n

y + (a^ C O S kx + Pa sèn kx)


A=1

menos se desvia da função / (;c) no sentido de que o desvio quadrático


médio 6^ seja mínimo.
Acabamos de demonstrar o teorema:
Entre todos os polinómios trigonométricos de ordem n, é o
polinómio de Fourier da função f {x) que dá a melhor aproximação
quadrática média desta função.
O desvio, quadrático médio mínimo é
n

( 2)
— 31 A=1

Como 6% ^ 0 , tem-se, qualquer que seja n,


31 n

i J f ( x ) d * > ^ + i 2 ( » í + 6S).
A=1

Segue-se que a série do segundo membro converge quando /i oo


e pode-se escrever

jT Ü^
— 31
^ “I"
A=1
+ ^a)« (3)

Bsta relação é a desigualdade de Bessel.


Limitemo-nos simplesmente a indicar que para qualquer função
limkada monótona por corte, o desvio quadrático médio obtido quando
se substitui esta função por uma soma parcial de Fourier tende para
SÉRIES DE FO U RIER 357

zero quando ap : ô* ^ 0 quando n -* oa. Mas. então, da fór­


mula (2) resulta a igualdade
oo n

h=i -n

chamada igualdade de Liapounov {*). (Indiquemos que A. Liapounov


demonstrou esta igualdade para uma classe de funções mais lacta
que a classe aqui tratada).
Resulta do que acaba de ser demonstrado que, para uma função
que satisfaça à igualdade de Liapounov (especialmente para qualquer
função limitada monótona por corte), a série de Fourier correspondente
dá um desvio quadrático médio nulo.
Nota — Estabeleçamos uma propriedade dos coeficientes de Fou­
rier, que nos servirá no seguimento. Vamos dar, prèviamente, uma
definição.
Uma função diz-se contínua por corte no intervalo [a, ó], se
os seus pontos de descontinuidade de primeira espécie são em número
finito sobre este segmento (ou se é para todo o valor contínua).
Mostremos a seguinte proposição.
Se a função f (x) for contínua por corte sobre o segmento [— tt, ir]
os seus coeficientes de Fourier tendem para zero quando n - » oo, isto é,
lim = 0, lim = 0. (4)
n->00 7l->CO
Demonstração — Se a função f(x) é contínua por corte no inter-
JX
valo [— ir. ir]. O mesmo se passa com /* (jc). Mas. então. | f (x) dx
—Jl
existe e é um número finito (**). Então, da desigualdade de Bessel (3)
oo

resulta que a série 2 («n + &n) converge, o que implica que o seu
th=1
termo geral tende para zero: lim (a%, + 6») = 0, as igualdades (4)
n-H»
estão demonstradas. Assim se tem para uma função limitada contínua
por corte as igualdades
Jl
lim J / (x) C O S n x d x = 0y
n -* -o o — Jl

Jl
lim J / (x) sen n x d x = 0.
n-+>oo — Jl

(♦) Esta igualdade é ainda chamada fórmula de Parseval,


(♦♦) Este integral é a soma dos integrais dos diferentes bocados de funções
contínuas que constituem a função f{x) no intervalo [—‘tt, ‘tt].
358 CALCULO D IFEREN CIA L B INTEGRAL

Se a função f(x) é periódica e de período 2tt, pode-se recopiar


estas últimas igualdades como se segue (com a arbitrário);
a+2n
lim J i {x)Q,osnxdx = 0\ lim J f {x) *stanxdx = 0.
n-»-oo a

Notemos que estas igualdades subsistem se se integrar num inter­


valo [a, b] qualquer, isto é, que os integrais
b b

J / (x) COS nx dx ei J / {x) sén nx dx


a a

tendem para zero quando n tende para infinito, se / (jc) for uma função
limitada contínua por corte.
Com efeito, suponhamos, para fixar ideias, que 6 — a < 2ir e
consideremos a função auxiliar (p{x) de período I tt definida como
se segue:
<P(^) = /(a;) para a ^ x ^ b ,
(p(x) = 0 para 6 < x C a + 2n.
Então; fe a+2jt
l f (x) C O S n x d x = j (f (x) C O S nxdx,
a
a+ 2n
lf{x)sta n xd x= I ^>{x) ^ n n x d x .

Como y>(x) é uma função limitada e continua por corte, os


integrais dos segundosmembros tendem para zeroquando /i-> oo.
Segue-se que os integrais dos primeiros membros tendem também
para zero. A proposição está assim demonstra e tem-se
6 b
lim ] f { x ) e o s n x d x = 0\ lim \ f {x) ^ n n x d x = Q (5)
n->oo a a
quaisquer que sejam a e b e a. função f(x) limitada e contínua por
corte no segmento [a, b\.

§ 8. Integral de Dirichlet
Vamos estabelecer neste parágrafo uma fórmula que exprime as
somas parciais duma série de Fourier por meio de integral. Esta fórmula
ser-nos-á útil nos parágrafos seguintes.
Consideremos uma soma parcial de Fourier para a função perió­
dica f{x) de período 2ir:

«n (^) = Y ^ Sén kx).


SÉRIES DE FO U RIER 359

com
Jl
Oft = — 1 f{t) COS kt dt, bk = — \ f (t) sen kt dt.
II J IX J
—jT —n

Substituamos estas expressões na de (2:), obtém-se:


Jl
(^) = ^ / (t) dt -f-

sen I ,.
+ / (t) C OS A:í dí -)- ---------- \ f(t)ssên
e kt dt
l [ ~ l Jl J
h=l —Jl
ou, introduzindo coskx e senAx sob o sinal soma (o que é legítimo,
porque coskx e sen/:jc não contêm a v^iável de integração).
Jl
/ (í) dt +

2[í
n

k=i
Jl

— JX
f {t) COS kx COS k t d
Jl

t ^ f {t)sènkxsènktdt^,
—Jl
1
Ponhamos agora — como factor e substituamos a soma dos
Jl
integrais pelo i n t ^ a l da soma; tem-se:
Jl n

Sn U (^) kx COS kt + f (t) sèn kx sen /cí]| di.


A =1
ou

dt =
— Jl A==l
n

= ^ j /(O
[■1-+ y > jC o sk (t — x) dt. (1)
A=1
Transformemos a expressão entre parêntesis. Façamos

a„ (z) = y + COS z + COS 2z . -f- c o s n z ;


360 CALCULO D IFEREN CIA L B IN TEGRAL

então.

20n (z) COS Z = COS Z + 2 COS Z COS Z + 2 COS Z COS 2 z + - -

. .. + 2 COS z COS nz = cos z + (1 + cos 2z) + (cos z + cos 3z) +


+ (cos 2z + cos 4z) + • • • + [cos (A^ — 1) Z + cos (/I - f 1) z] =
= 1 + 2cosz + 2cos 2z + ---
. . . + 2 cos (w — 1) z + cos WZ + cos (/I + 1) z
ou
2o;i (z) cos z = 2oyi (z) — cos nz + cos»(w + 1)
cos nz — cos (/z + 1) z
0„ (z) :
2 (1 — cos z)
Ora.
Z Z
cosnz — cos (re + 1) z = 2 sen (2re + 1) sen — ,

1 — cos z = 2 sén^ .

Logo,

sen(2n + l)-2
On (Z) = ■
2 » n |

Pode-se» pois» recopiar a igualdade (1) sob a forma


t ~ ^
, } sên (2n + 1l)-
sên(2n )
Ht) d t.
Jt J í—
2sén-

Como a função sob o sinal soma é periódica (de período 2ir)


o integral conserva o mesmo valor em qualquer segmento de com-
primentó 2w, Segue-se que

ac+ji sén (2n + 1 )
t X

Sn (^) = — f nJ t ) d t.
n J t — X
2 sen
S âR IE S DB FO U RIER 361

Introduzamos a nova variável de integração a fazendo


t — x = a, t = x-\-a.
Então, obtém-se a fórmula

5 sen(2« + l ) ^
«n(^ ) = — \ i{x + ct) -------------------------d a . ( 2)
Jl J a
2 sen

O integral do segundo membro é o integral de Diridúet


Façamos nesta fórmula j{x) = 1; então, Qq = 2, = 0, 6^ = 0
quando /: > 0; logo, U) = 1 qualquer que seja n, e obtém-se a
identidade

^ J sen (2ra-1=* 1)
1= — \ ----------------------^ da, (3)
a
-Jl 2 sen ^
~2
que nos servirá no seguimento.

§ 9. Convergência duma série de Fourier num dado ponto


Suponhamos que a função f{x) é contínua por corte no intervalo
[ - 7T, 7t] .
Multipliquemos os dois membros da identidade (3) do parágrafo
anterior por / (jc) e introduzamos f U) sob o sinal de integração.
Obtém-se a igualdade
Jl a
sen(2« + 1) ^
f{^) í{x) da.
a
2 sen

Subtraiamos, membro a membro, esta igualdade da igualdade (2)


do parágrafo anterior. Obtém-se:

, p sén(2ra + l ) ^
Sn i.x) — / (x) = — \ [/ (x + a) — / (x)]----------------------da.
Jl J O Oí
-Jl 2 sen ~

Vê-se que a convergência da série de Fourier para o valor da


função f (x) no ponto dado depende da convergência para zero do
integral do segundo membro quando oo.
362 CALCULO D IFEREN CIA L E INTEGRAL

Decomponhamos este último integral em dois:


jt a
1 f "2
Sn (^) — f(^ ) = — i [f (so a) — f (x)]------------ sen na da +
jc J ^ . a
2*^2
n
1 1 f
+ - 7 7 —- I [f { x a ) — f (x)]cosnada,
2 Jt J

utilizando a fórmula sen (2n + 1) = sen /i« cos + cos na X sen ^ .


z z ^
Decompondo o primeiro integral do segundo membro desta última
igualdade em três, vem:

a
1 f "2
5/1 ( ^) — /(^) = — \ [/(^ + a) — /(x ) ] ------------ s è n n a d a +
J ^ a
-6 22sen
s e n- -|

-ô a
f COS -7^
ü
j [/(x + a) sén n a da +
— Jl
o a
2”“ 2

71 a
f CO S-y

+ ■ ■J [/ (^ + 0^) sen n a da +
6 o Oí
2sen 2

+ “ \ [/ (^ + a) —- / (x)] y COS na da.

Façamos <Di (a) = f — Í_Í£) . Como / (x) é limitada e con­


tínua por corte, (a) será igualmente uma função periódica de a
limitada e contínua por corte. Segue-se que o último integral do
SÉRIES DE FO U RIER 363

segundo membro tende para zero quando oo. porque é o coefi­


ciente de Fourier desta função. A função
a

( a ) = [ / ( ^ + a ) — / (^ )]
n ^
2 sen 2

é limitada quando — T < a < — 8 e 8 < « < t; tem-se


1
|a ) 2 ( a ) |< [ M 4 - ^ ]
o2 s e n ô-

em que M é o limite superior de | / (jc) | . Além disso, a função ^2 («)


também é contínua por corte. Por conseguinte, em virtude das fór­
mulas (5) do § 7. o segundo e terceiro integrais tendem para zero
quando 00.
Pode-se escrever, por conseguinte.
lim [s„ (x) — / (x)] =

= lim
^-►OO JX
í
J
-6
[/ (x -t- a) — / (x)]
a
c o ^

2sen|
sen wa da. (1)

A integração na expressão do segundo membro é alargada ao


intervalo — 8 < a < 8, logo o integral depende dos valores de f (x)
sòmente no intervalo compreendido entre x — 8 e x + S.
Desta última igualdade deduz-se a importante proposição: a con­
vergência da série de Fourier no ponto considerado x depende sòmente
do comportamento da junção numa vizinhança arbitràriamente pequena
deste ponto.
Tal é o conteúdo do princípio da localização no estudo das séries
de Fourier. Se duas funções /i (x) e Í2 (x) concidem na vizinhança dum
ponto JC, as suas séries de Fourier convergem ou divergem ao mesmo
tempo neste ponto.

§ 10. Algumas condições suficientes para a convergência


duma série de Fourier
Demonstramos no parágrafo anterior que se uma função for
contínua por corte no intervalo [— tt, tt], a convergência da sua série
de Fourier no ponto considerado jco para o valor / (xo) depende
sòmente do comportamento da função numa vizinhança arbitràriamente
pequena [jco — 8, jCo + 8] de centro no ponto jcq.
364 CALCULO D IFEREN CIA L E INTEGRAL

Demonstremos em seguida que se a função é, na vizinhança


de jco, tal que os limites seguintes existam e sejam finitos

l™ / (^0 + oc) — / (Xo)


Jim ----------------------
= 0)
a-^-0 a

l: „ /(^ 0 + OC) — /(X o )


lun ----------------------- = ^2» ( 2)
a-^ + O a

e se a própria função é contínua no ponto Xo (fig. 370), a série de


Fourier converge neste ponto para /(JCo)(*).

Demonstração — Consideremos a função ^2 («) do parágrafo


anterior:

a
COS
(Dj (a) = [/ (aro + a) — / (aro)] •
2O sen —
^

como a função f{x) é contínua por corte no intervalo [— tt, tt] e


contínua no ponto Xo. é. por conseguinte, continua numa certa vizi­
nhança [xo — S, Xo + 5] do ponto Xo. Logc a função (a) é contínua
em todos os pontos em que e | a [ < 8 . A função ^2 («) não é
definida para a = 0.

(*) Se as condiç5es (1) c (2) forem verificadas, diz-se que f ( x ) no


ponto X tem uma derivada à direita e uma derivada à esquerda. Representou-se
na fif?. 370' uma função tal que = tg (pi, kz = tg (pa, /cj ^ kz. Se k^ = kz,
isto é. se as derivadas à direita e à esquerda forem iguais, a função é derivável
no ponto dado.
SÉRIES DE FO U RIER 365

Procuremos os limites lim O 2 (a) e lim O 2 (oc) utilizando as


Q—
►O—0 Q—
►0~j~0
condições (1) e (2):
a

lim d)2( a ) = lim [ / (xq + a) — / (xo)]


a-^0—0 a-^0—0 o a
2 sen

a -►0—0 CC OC 2
sen-

= lim /(^o + oc ) - / ( ^ o) ^
a -►0—0 a
a
~2
X lim ---------- lim C O S — = A :i* l-l = /p i.
a -►0—0 OC a -^ 0—0 2
sen

Por conseguinte, se se definir a função $2 (<*) fazendo ^2 (0) = ku


ela será contínua no intervalo [— 8, 0] e, por conseguinte, limitada.
Demonstra-se, duma maneira análoga, que
lim d)2 (<^) = ^2-
a->0+0
Logo a função $2 (a) é limitada e contínua no intervalo [0. 8].
Assim, a função $ 2 (a) é limitada e contínua por corte no inter­
valo [— 8, 8]. Voltemos à igualdade (1) do § 9 (designando x por Xo):

lim [s„ (xo) — / (xo)] =


n->oo
c a
t r “2
= lim ^ \ [/(x o + a ) - / ( x o ) ] -------------- s é n r a a d a
-ô 2sén^

OU
õ
Um [s (xo) - / (xo)] = lim ^ \ O 2 (a) sen na da.
«-►oo n-^°o ^ J
366 CAIXULO D IFEREN CIA L E INTEGRAL

Tendo em atenção as fórmulas (5) do § 7. conclui-se que o


limite do segundo membro é nulo. e portanto.
lim [s„ (xo) — / (xo)] = 0
7l->00
OU
lim 5;i(:to )= ( tq).

O teorema está demonstrado.


A diferença entre o teorema demonstrado no § 1 e o teorema
acima consiste no que se segue; pedia-se no teorema do § 1 para a
convergência da série de Fourier no ponto Xo para o valor / ( jco),
que JCo fosse um ponto de continuidade sobre o segmento [— tt, tt]
e que a função fosse monótona por corte, enquanto que aqui se pede
que a função seja contínua no ponto jCo que tenham lugar as condi­
ções (1) e (2) e que a função seja contínua por corte e limitada no
intervalo [— tt, tt]. É evidente que estas condições são diferentes.
N ota— 1. Se uma função contínua por corte for derivável no
ponto Xo, é evidente que as condições (1) e (2) tenham lugar e que
se tenha ki = )fc2. Por conseguinte, num ponto de derivabilidade da
função f{x), a série de Fourier converge para o valor da função nesse
ponto.
Nota — 2. I — A função considerada no exemplo 2 do § 2
(fig. 358) verifica as condições (1) e (2) nos pontos 0, dz 27t, ifc 47t, ...
Ela é sempre derivável. Logo a série de Fourier desta função converge
em cada ponto para o valor desta função.
II — A função do exemplo 4, § 2 (fig. 361) verifica as condi­
ções (1) e (2) nos pontos dt tt, ± Stt, ± Stt. Ela é derivável para
todo o valor, logo representável por uma série de Fourier em cada
ponto.
III — A função do exemplo 1, § 3 (fig. 357) é descontínua nos
pontos di 7T, dz 37T, dz Stt. É Sem pre derivável. logo a série de Fourier
converge para o valor desta função em todos os pontos, excepto nos
pontos de descontinuidade. Nos pontos de descontinuidade, a soma
da série de Fourier é igual à média aritmética dos valores limites
da função à esquerda e à direita: é nula no caso considerado.

§ 11. Analise harmónica numérica


A teoria da decomposição das funções em séries de Fourier
chama-se análise harmónica. Vamos fazer, agora, algumas observações
sobre o cálculo aproximado dos coeficientes de Fourier, isto é, sobre
a análise harmónica numérica.
SÉRIES DE FO U RIER 367

Como se sabe, os coeficientes de Fouríer da função f(x) de


período 2tt são definidos pelas fórmulas
jt n
1 f 1 r
Oo— — \ f(x)dx-, a* = — \ / (^) COS kx d x ;
n J Jt J
—Jl —Jl
Jt

1 f
bk = — \ f(x) sên kx dx.
Jt J .
— Jt

Em muitos casos encontrados em prática, a função /(jc) é dada


quer sob a forma de quadro (quando a dependência funcional é
obtida experímentalmente) quer por uma curva traçada por um aparelho.
O cálculo dos coeficientes de Fourier faz-se, então, por meio de méto­
dos de integração aproximada (ver § 8, cap. XI, t. I).
Consideremos o segmento — tt < jc < tt de comprimento lir.
Pode-se sempre reduzir a este caso, escolhendo convenientemente a
unidade sobre o eixo Ox,
Dividamos o segmento [— ir, tt] em n partes iguais pelos pontos
Xq jc , Xj, ^ 2, • • Xj^ — J l.

O comprimento dum segmento parcial é, então,


A 2 ji
í^x = — .
n
Designemos os valores da função f (x) nos pontos Xq, Xu X2 .......
por
yoy Uu U2i .-M yn-

Tomamos estes valores quer no quadro, quer sobre a curva da


função.
Utilizando, por exemplo, a fórmula dos rectângulos (ver fórmula (1),
§ 8, cap. XI, t. I). determinam-se os coeficientes de Fourier:
n n

2 V 2 V
COS k X i ,

i=l i=l

sâ ik x i.
368 CALCULO D IFERENCIA L E INTEGRAL

Têm sido elaborados esquemas, que simplificam o cálculo dos


coeficientes de Fourier. Não podemos aqui perder-nos em detalhes, mas
indiquemos que existem aparelhos (chamados analizadores harmónicos)
que, segundo o gráfico da função dada, permitem calcular aproxima­
damente os coeficientes de Fourier.

§ 12. O integral de Fourier


Seja f(x) uma função d^inida em qualquer intervalo (— 00, 00)
e absolutamente integrável nestç intervalo, isto é, que o integral

5 \f{^)\dx= Q (1 )

existe. Suponhamos, além disso, que f (x) admite um desenvolvimento


em série de Fourier em qualquer intervalo (— /, + /):
00

f{x) = - ^ + a ACo s y o: + bu s e n y j :, ( 2)
A=1
onde
I I
ah = ~Y ^ f{t) COS ^tdty bk = ~Y ^ f (0 ~ ^ (3)
-I -I

Substituindo na série (2) os valores dos coeficientes e bf^ tirados


das fórmulas (3), pode-se escrever:
00 I

/(^■) =

j
-I
/(í)ííí + -y 2
I
h= i
(f
-I
f(t)c o s ^ td t^ c o s Y ^

h=i -I -I
00 I
kn , kji ^ kn
+ y 2 j j ^ c o s y í c o s — :r + sen - t sen — x
I I - I
k= í -I
ou
I I

-^J dí Í
00

/ (^) = / (0 + y 2 ^ COS dt. (4)


I
-I h= i - I
SÉRIES DE FO U RIER 369

Estudemos o problema da forma do desenvolvimento (4) quando


se passa a limite para / oo.
Introduzamos as notações seguintes:
Tí 2n kn . A
«1 = — , a2 = — , •1 — » ••• — • (5)
/ I I /
Substituindo-as em (4) teremos:
I oo I

/(^ ) = - ^ + ( í / ( O COS «A (í — àak (0)


A=1 -I
Quando / -^ oo o primeiro termo do segundo membro tende para
zero. Com efeito.
I «>
<
-I
Para cada valor fixo de Z a expressão entre parêntesis é uma
função de (ver fórmulas (5)) tomando os seus valores de a oo.
Notemos, sem o demonstrar, que se a função f(x) for monótona por
corte em cada intervalo infinito, limitada no intervalo infinito e que
satisfaz à condição (1) a fórmula (6) tomará, se /-> + oo, a forma
oo oo

^ ~ ^ f f a { t — x) da. (7)
0 —oo

A expressão da direita chama-se integral de Fourier da função / (x).


A igualdade (7) tem lugar para todos os pontos em que a função
é contínua. Nos pontos de descontinuidade é a igualdade
oo oo

J f(t)cosa{t-x)dt] = (?’)
0 —oo
que'é verificada.
Transformemos o integral do segundo membro da igualdade (7)
desenvolvendo cos a (/ — x):

cos a {t — x) = COS a í COS a x + sen a t scnax.


24
370 CALCULO D IFEREN CIA L E INTEGRAL

Substituindo esta expressão na fórmula (7) e fazendo sair cosax


e senajc de debaixo do sinal de integração nos integrais em que a
integração é realizada na variável í, obtemos
oo oo

/(^ ) = ( ! / (t) C O S a t COS a x da +


0 —oo

M(í
oo oo

+ / (t) sen a t sen a x da. (8)

Cada um dos integrais em í, situados entre parêntesis, existe,


porque a função / (0 é absolutamente integrável no intervalo (— oo, + oo)
de modo que as funções / (0 cos a t q f ( t ) sen a t são também absoluta­
mente integráveis.
Consideremos os casos particulares da fórmula (8).
1. Suponhamos que f (x) é par. Neste caso f ( t ) cos a t é uma
função par e f ( t ) sen a t uma função ímpar de modo que temos:

J f (t) cos a t d t = 2 ^ f{t) cos a t d t ,


— oo 0
00
1 f ( t ) s c a a t d t = 0.
— oo

Neste caso a fórmula (8) põe-se sob a forma


oo oo

K í / (t) cos a t cos a x da. (9)

2. Suponhamos que f(x) é ímpar. Analizando a natureza dos


integrais da fórmula (8), teremos neste caso;
oo oo

f(^) ~ ^ í ^ ^ sen sèn a x da. (10)

Se a função / (x) não é definida senão no intervalo (0, oo), pode-se


representá-la para x > 0 tanto pela fórmula (9) como pela fórmula 10.
No primeiro caso definimo-la complementarmente para o ihtei*valo
( — 00, 0) sob a condição de a função ser par e no segundo ser ímpar.
SÉRIES DE FO URIBR 371

Sublinhamos, uma vez mais. que nos pontos que apresentem


descontinuidades convém substituir / U) nos primeiros membros das
igualdades (9) e (10) pela expressão

/ ( ^ + 0) + / ( ^ - 0 )
2
Voltemos à fórmula (8). Os integrais entre parêntesis são funções
de a. Introduzamos as notações:
oo

1 r
A { a ) = — \ f {t) COS at dtj
— oo

oo

B {á)= ^ ^ / (í) sen a t dt.


— oo

Então, pode-se escrever a fórmula (8) sob a forma:


oo

/ (^) = í (^) COS ax -j- B (a) sen ax] da. (11)


0

Diz-se que a fórmula (11) dá o desenvolvimento da função j{x)


em harmónicas de frequência a que variam duma maneira contínua
de 0 a 00. A lei de distribuição das amplitudes e das fases iniciais
em função da frequência a é expressa pelas funções A{a) t B (a).
Voltemos à fórmula (9). Façamos

F (a) = ^ ^ ( 12)

A fórmula (9) toma, então, a forma

/ — °°
/ (:r) = y —\ (a) cos ax da. (13)

A função F (a) chama-se transformada-cosseno de Fourier da


função / (x).
Se na igualdade (12) F («) for a função dada e / (í) a função pro­
curada. será, então, uma equação integral para a função f(t). A fór­
mula (13) dá a solução desta equação.
372 CALCULO D IFERENCIA L E INTEGRAL

Com base na fórmula (10), pode-se escrever as seguintes igualdades:


__ oo

d) (a) = 1 / — \ f{t) sèn a t dt. (14)


r JT J ^
ü
/—
/ (x) = \ / — 1 O (a)sên a x d a. (15)

A função ^ (a) chama-se íransformada-seno de Fourier da


função f(x).
Exemplo — Seja ^ (p > q, a; > 0).

Determina-se, segundo a fórmula (12), a transformada-cosseno de Fo\irier:

( 0 ,) = l / Ã J , - P í COS « < á í = | / - | - ^ 5 ^ .

0
Segundo a fórmula (14) determina-se a transformada-seno de Fourier:

0
Com a ajuda das fórmulas (13) e (15), obtém-se as relações recíprocas:

oo
f COS a x
(i > 0)
n J p2 + a2
0
oo
2 Ç a^_ax
da = e-P* ( x > 0 ).
Jt J P « + a2

§ 13. Forma complexa do integral de Fourier


No integral de Fourier (fórmula (7), § 12) a função de a, que
se encontra entre parêntesis, é par e, por conseguinte, é igualmehte
determinada para os valores negativos de a. Em virtude do que acabámos
de dizer pode-se recopiar a fórmula (7) sob a forma:
oo oo

H í f (t)cosa { t — x ) d t ^ da. (1)


— oo — oo
SÉRIES DE FO U RIER 373

Consideremos, agora, a expressão seguinte identicamente nula


M oo

í ( í / (^) — x)dt)da = 0 .
—M — oo

Q primeiro membro é idênticamente igual a zero, porque a


função de a entre parêntesis é uma função ímpar e o integral duma
função ímpar tomada nos Tunites de — Af a + Af é igual a zero.
É evidente que
M oo

lim J (J f (t) séa o í { t x ) dt)da = 0


M -^ o o — M — oo

OU
oO oo

I (5/(0 — •^) ^0 í í a = 0 - ( 2)
— oo — oo

Nota — Notar-se-á aqui o facto seguinte: o integral convergente


nos limites infinitos é definido como se segue:

J (p ( a ) d a = J 9 (a ) d a + J q) ( a ) da =
-o o — oo c

C M
= lim 5 (}) ( a ) da + lim 5 <P (« ) (*)
M-^oo —M

com a condição de cada um dos limites do segundo membro existir


(ver § 7, cap. XI t. I). Ora. escrevemos na igualdade (2):
M
J q )(a )d a = lim JI < p (a )d a . (•*)
M -> o o — M

Pode suceder, pois, que o limite C*"*) exista enbora os lim ita do
segundo membro da igualdade (*) não existam. A expressão do segundo
membro da igualdade (**) chama-se valor principal do integral. Assim,
consideremos na igualdade (2) o valor principal do integral impróprio
(exterior). É neste sentido que se deve compreender os integrais que
encontramos neste parágrafo.
Multipliquemos os membros da igualdade (2) por ^ e acres-
ceiltemo-los às partes correspondentes da igualdade (1); obtém-se, então;
oo oo

/(x>
-àll!
— oo — oc
/ (í) (c o s a { t — x) i sên a ( t — x)) dt da
374 CALCULO D IFEREN CIA L E INTEGRAL

OU
oo oo

iait—x) dt da.
/(^) H t\e (3)
— oo — oo

É precisamente a forma complexa do integral de Fourier, Pode-se


pô-la sob a forma:
oo oo

/W = V9.n. Jf (-L,
1/2 ji
(
\V2n J^ V 2 jc /

Esta última igualdade permite-nos escrever:

F*(a) = - ^
V2k j L
f
oo

(4)

oo

j {x) = \ F* (a) e ~ d a . ( 5)
y 2n J

A função F* (a) definida pela fónnula (4) chama-se transformada


de Fourier da fu n ^ o /(/). A função / ( jc) definida pela fórmula (5)
chama-se transformada inversa de Fourier para a função F* (o) (as
transformadas diferem pelo sinal de 0-

Exercidos

1 Desenvolver em série de Fourier no intervalo (— ir, ir) a função)

/(x) = i
„, 1 22 /! COSX
a , COS 3x , cos 5x , ^,
32 + 52 + • •• j -r
T ” n I 1*
' sen X sen 2 i , sen 3x

3. Utilizar a desenvolvimento cm seno da função f { x ) = 1 no intervalo (0, tt)


1 1 1
para calcular a soma da série 1 — õ + -?-----n -ir ••• Rcsp. — .
o 5 7 4
2. Utilizar o desenvolvimento cm série de Fourier da função / (x) = x* para
1 i 1
calcular a soma da s é rie --------- i- -l —____-— 1- . . . R esp.----- .
1
12 2* ^ 3-! 42 ^ 12
SÉRIES DE FO U RIER 375

4. Desenvolver em série de Fourier no intervalo ( — nr, n r ) a função


,, , ji2 x2 COS 2a: , cos3a: cos 4x ,
~~Í2 ^ ”^2 I 32 42 ^ *• *
5. Desenvolver em série de Fourier no intervalo ( — nr, nr) a função
... (JI + a:)
/(^)=— ò para -3X X 0|

/ (®) = y (Ji—a:) para 0 < x < j t .

1 1
Resp. sen a :+ — scn2a: + -^ sèn3x-|-. . .
6. Desenvolver em série de Fourier no intervalo ( — nr, nr) a função
/ ( a : ) = —x p a r a — J i < a ; < 0 ,
f ( x ) = 0 para 0 < a: < ji.

Resp ____ ?- "V COS ( 2 n + i ) x y sen nx


- 4 n Z j (2n+l)2 Z j ^ n '
m—0 n = l

7. Desenvolver em série de Fourier no intervalo ( — nr, nr) a função


f(x) = í para — J i < a : < 0 ,
/ ( a : ) = —2 para 0 < a : < j t .

Resp. - l - ± y s e n( 2r e+l ) x
^ 2 71 ZJ 2n + l
n = 0

S. Desenvolver a função / (jc) = no intervalo (0, nr) em série de senos.


00

n = 1

Desenvolver a função y = cos 2x no intervalo (0, nr) em série de senos.


_ 4 r sen X , 3 sen ;3a: , 5 sen 5x
*^*®**' ~~n L22— l " ^ 2 2 ^.32 ' 22—52 ■]
10. Desenvolver a função y = sen x no intervalo (0, nr) em série de cosseno*^.
n 4 r 1 , COS 2x COS 4x “I
*^*®*’' i r L 2 ' ^ ' 1—2 2 1 —4 2 ■*■■■’]■
11. Desenvolver em série de Fourier a função y — e^ no intervalo (— l, /).
y j \ «n 7ZTLX
.-t . . ^ ( —l ) " c o s —^
Resp. + l{e‘ - e - ‘) 2
21 /2-|-n2ji2
n = 1

(—l) " -i n sen -


+ 7t{et-e-l) 2 i2 + n2jt2
n= 1
376 CALCULO D IFEREN CIA L E INTEGRAL

12. Desenvolver a função / (x) = 2x no intervalo (0, 1) cm série de senos.


oo
Resp. i - A2 VI
y COS 2njix
n ^ n
n = 1

13, Desenvolver a função f (x) = x no intervalo (0, /) cm série dc senos.


sen nnx
I- S
n = 1
< -!)" ■ —
I

14. Desenvolver a função


/ a;para0< x< l,
2— X para l < x < 2
no intervalo (0, 2)*. a) em série de senos; b) em série de cossenos.
(2n-{-í) nx
sen i———-—
■) ^ S (-O "' i2 .+ h - '
ii - bO

,1 ^ V
Y 2j (^+ 1)2
n = * 0
OapiMo XVIII

EQUAÇÕES DA FÍSICA MATEMAMCA

§ 1. Principais tipos de equações da física matemática

Chamam-se equações principais da física matemática (no caso


duma função de duas variáveis independentes) às equações diferenciais
seguintes às derivadas parciais dà segunda ^ordem.
I — Equação da onda:
d^u d^u
(1)
dt^ do?
Somos levados a considerar esta equação quando do estudo dos
processos das vibrações transversais duma corda, das vibrações longi­
tudinais dum tronco, das oscilações da corrente eléctrica num fio, das
vibrações de torção da árvore, das oscilações dos gases. etc. Esta
equação é a mais simples do tipo hiperbólico.
II — Equação do calor ou equação de Fourier:
d^u
— = a^ ( 2)
dt do?
Somos levados ao estudo desta equação quando em presença de
problemas apresentados pelos processos de difusão do calor, da filtração
de líquidos ou de gases num meio poroso (por exemplo, a filtração
do petróleo e dos gases nos grés sob cobertura), de certos problemas
da teoria das probabilidades, etc. É a equação mais simples do tipo
parabólico.
III — Equação de Laplace:

d^u . d^u
:0. (3)
da:^ Oy^
Somos levados ao estudo desta equação uma vez postos em
presença de problemas apresentados pelos campos eléctricos e magné­
ticos, o estado estacionário de calor, a hidrodinâmica, a difusão, etc.
É a equação mais simples do tipo elíptico.
378 CALCULO D IFEREN CIA L E INTEGRAL

Nas equações (1), (2) e (3) a função procurada u depende de duas


variáveis. Pode-se igualmente estudar as equações correspondentes ao
caso em que a função procurada comporta um maior número de
variáveis. Por exemplo, a equação das ondas no caso de três variáveis
independentes é da forma:

d^u _ 2 ( d^u
+ ( 1')
dy^
a equação do calor no caso de três variáveis independentes é da
forma:

( 2 ')
dt V 03? dy
dy^ /
a equação de Laplace a três variáveis independentes é da forma:

d^u d^u d^u


+ = 0. (3')
dx^ ‘ dy^

§ 2. Ei^tabelecimento da equação para cordas vibrantes.


Formulação do problema aos lim ites. Estabelecimento da
equação para oscilações eléctricas nos fios
Em física matemática entende-se por corda um fio flexível e
elástico. As tensões que aparecem na corda num momento arbitrário
do tempo são dirigidas segundo a tangente ao seu perfil.
Seja / o comprimento da corda que no instante inicial é dirigido
segundo o segmento do eixo Ojc de O a /. Suponhamos que as extre­
midades da corda estão fixadas nos pontos x = 0 c x = L Se se desvia
a corda da sua posição inicial depois solta-se-la ou se, sem desviar
a corda, se imprime aos seus pontos uma certa velocidade ou melhor
se se afasta a corda imprimindo ao mesmo tempo uma certa velocidade
aos seus pontos, os pontos da corda serão, então, animados dum certo
movimento e dir-se-á que a corda vibra. O problema consiste em
determinar á forma da corda para todo o instante arbitrário do tempo
e em determinar a lei do movimento de cada um dos pontos da corda
em função do tempo.
Apenas consideraremos os pequenos desvios dos pontos da corda
da sua posição inicial. Pode-se, portanto, admitir que o movimento
dos pontos da corda se efectua perpendicularmente ao eixo Ox e
num mesmo plano. Nesta hipótese o movimento ondulatório da corda
é descrito por uma única função u (x, t) que dá a deslocação do
ponto da corda de abcissa x no instante t (fig. 371).
EQUAÇÕES DA FÍSICA MATEMATICA 379

Como apenas consideraremos os pequenos desvios da corda no


plano (x, u) podemos supôr que o comprimento do elemento
da corda é igual à sua projecção sobre o eixo Ox, isto é(*) que
M iM ^ = X2 — Xi, Suporemos do mesmo modo que a tensão é idêntica
para todos os pontos da corda; designemo-la por T.
Consideremos o elemento M M ' da corda (fig. 372). Nas extre­
midades deste elemento actuam as forças T segundo a tangente à

corda. Suponhamos que as tangentes formam com o eixo Ox os


ângulos cp e q) + Aq). A projecção sobre o eixo Ou das forças que
actuam sobre o elemento M M ' será igual a T sen (q) + Aq)) — T sen q».
Como o ângulo q é pequeno, pode-se pôr tg q sen q ,e teremos:

r sen (q + Aq) — T sen q


1 du{x A.r, t) du {x, t)
7’ tg(q) + A(p) — T t g ( ( =
1 dx dx
„d^u(x -\-d Ax, t) .
= T ---- T---------------- Ax : O<0<1
dx^
(aplicámos aqui o teorema de Lagrange à expressão entre parêntesis).
Para obter a equação do movimento, é preciso igualar à força
de inércia as forças exteriores aplicadas ao elemento. Seja p a densidade
linear da corda. A massa do elemento da corda será pAx, A aceleração
do elemento é —
dt^ *

(*) Esta hipótese equivale a desprezar a grandeza cm relação a 1.


Com efeito,
ac2 :V2 X2
Vl+ dl = j ^ 1 + -^ ... j I dx = X2— x^.
.rj
380 CALCULO D IFEREN CIA L E INTEGRAL

Por conseguinte, teremos em virtude do princípio de d’Alembert:

pAx — Y = ^ ^
df dx^
T
Simplificando por Ajc e fazendo — = obtemos a equação do
movimento
É } L — 2 d^u (1 )
dt^ dx'.2 •

Obtivemos a equação dita equação de onda que é a equa^o das


vibrações da corda. Para determinar completamente o movimento da
corda não basta só a equação (1). A função procurada w (jc, t) deve
ainda satisfazer às condições dos limites, indicando o que se produz
nas extremidades da corda (jc = 0 e j c = / ) e ò s condições iniciais,
descrevendo o estado da corda no instante inicial (/ = 0). Por con­
dições iniciais entende-se do mesmo modo o conjunto das condições
dos limites e das condições no instante inicial.
Suponhamos, por exemplo, que, como o admitimos, as extremi­
dades da corda para jc = 0 e x = I são imóveis. Então, para todo
o t devem ser verificadas *as igualdades:
u (0, t) = 0, (2')
u (l, t) = 0. (2'>
Estas igualdades constjtuem as condições dos limites para o
nosso problema.
No momento inicial t = 0 a corda possui a forma que lhe demos.
Suponhamos que esta forma é definida pela função f ( jc ) . Assim
deve-se ter
u(x, 0) = u\t=o = f(x). (3'>

Deve-se, além disso, fixar a velocidade no momento inicial em


cada ponto da corda, que é determinada pela função ç>(jc). Assim
deve-se ter
du
= cp(x). (3')
dt t=0

As condições (30 e (3'0 são as condições iniciais.


Nota — Em particular, pode-se ter f(x) = 0 e 9 (jc) = 0. Se estas
condições forem verificadas, a corda está em repouso e, por conse­
guinte, u(x, t) = 0.
EQUAÇÕES DA FÍSICA MATEMATICA 381

Como o indicámos mais acima somos conduzidos à equação (1)


pelos problemas apresentados no caso das oscilações eléctricas nos
condutores. Examinemos este caso. A corrente eléctrica num condutor
é caracterizada pela grandeza i (x, 0 e a tensão v (jc, /). que dependem
da coordenada x do ponto do condutor e do tempo t. Considerando
um elemento de condutor Ajc, podemos escrever que a queda de tensão
õv
no elemento A jc é igual a v (jc, 0 “ v (jc + A jc, t ) ^ — — A jc. Esta
queda de tensão forma-se da tensão ohmiana (ohmienne) igual a iR A jc

e da tensão induzida igual 3 , ^ L ^ x . Logo


ot

— í!ix = iR Ax L Ax, (4 )
dx dt

em que R e L isão, respectivamente, a resistência e o coeficiente de


auto-indução, calculados para uma unidade de comprimento do con­
dutor. O sinal menos indica que o sentido da corrente é oposto ao
crescimento de v. Dividindo por A jc, obtemos a equação

^ + iíí + i Ü _ o . (5)
dx dt
A diferença das intensidades da corrente que entra e sai do
elemento Ajc no decorrer do tempo Aí será

i {x, í) — i (x + Ax, t) ^ Ax Aí.


dx
dv
Ela é dispendida pela carga do elemento igual a C Ax — At e
a fuga pela superfície lateral do condutor, em consequência da imper­
feição do isolamento, igual a AvAxAt (A designa aqui o coeficiente
de fuga). Igualando estas expressões e dividindo por AxAt obtemos a
equação
dv
ÍÍ-+C A v = Q. (tí)
dx dt

Costuma-se chamar às equações (5) e (6) equações do telégrafo.


Pode-se obter do sistema de e q u a ç ^ (5) e (6) uma equação
contendo apenas a função desconhecida / (x, í) e uma equação que
contenha apenas a função desconhecida v (x, í). Derivemos os termos
382 CALCULO D IFEREN CIA L E INTEGRAL

da equação (6) em relação a jc e os termos da equação (S) em relação


a / e multipliquemo-las por C. Subtraindo uma da outra obtemos:

d^i dv di dh
+ A --C R — - C L ^ = 0 ,
do? dx dx d?

Substituindo nesta última equação — pela sua expressão tirada


dx
da equação (5). obtemos:

- ^ + a ( - ) -C R — - C L ^ = Q
dx^ V dt ) dx dt^

OU

d^i
= C L - ^ - \ - { C R + AL) — + A R i. (7)
do? dt^ dt

Duma maneira análoga, obtém-se uma equação determinante


v(x, 0:
dv
= C L ^ + (CR AL) ARv. (8 )
dx'^ de dt

Se se puder desprezar a fuga de corrente pelo isolamento (A = 0)


e a resistência (R = 0) as equações (7) e (8) reduzem-se a equações
de onda:

2 d^i dPv d^v


dx^ de dx^ de

em que se designou ^ . As condições iniciais e os limites são


formulados para o problema tendo em conta as condições físicas.

§ 3. R esolução d a equação d a s co rd as v ib ran tes pelo m étodo


d e sep aração d a s variáveis {método de Fourier)
O método de separação das variáveis (ou método de Fourier)
que vamos considerar é típico para a resolução de numerosos problemas
de física matemática. Seja determinar a solução da equação

d^u d^u
(1)
de do?
EQUAÇÕES DA FÍSICA MATEMÁTICA 383

satisfazendo às condires iniciais:

u (0 , t) = 0, (2)
u(l, t) = 0, (3)
u{x, 0) = f(x), (4)
du
= 9 (5)
dt
Procuraremos uma solução particular (não idênticamente nula)
da equação (1) que satisfaz ^ condições dos limites (2) e (3) sob a
forma de produto de duas funções X (x) e T (í) de que a primeira
apenas depende de a: e a segunda de /:

u (z, t) = X (x) T (t). ( 6)

Efectuando esta substituição na equação (1) obtemos: X (jt) T"(í) =


= a* X " (x) T (í) e dividindo os termos da igualdade por a^XT

.£ 1 (7)
ê T ' ■X
O primeiro membro desta igualdade contém uma função que
não depende de a:, e o segundo uma função que não depende de t.
A igualdade (7) não pode ter lugar a não ser no caso em que o
o primeiro e o segundo membros não dependam nem de x nem de t,
por outras palavras, sejam iguais a um número constante. Designemo-lo
por — A em que X > 0 (consideraremos mais adiante o caso X < 0).
Então,

— = —
a^T X

Obtemos destas igualdades duas equações:


X" + % X = 0, (8)
T" + a \ T = 0. (9)
As soluções gerais destas equações são (ver, cap. XIU, § 21, t. I):

X{x) = A c o s V l x + B s c a V k x , ( 10)

T {t) — C COS a V à í + 77 sen a V x í , (11)


em que A, B, C, D são constantes arbitrárias.
384 CALCULO DIFERENCIAL E INTEGRAL

Substituindo as expressões de X {x) t T (í) na igualdade (6),


obtemos:
u (x, t) = {A COS V à x -f- B sen {C c o s a V X t sen a V X t).
Escolhamos agora as constantes A q B úq maneira que sejam
verificadas as condições (2) e (3). Como T (/) ^ 0 (no caso contrário
teríamos u (jc, 0 = 0. o que contradiz a nosSa hipótese), a função X (jc)
deve verificar as condições (2) e (3), isto é. que se deve ter X (0) = 0,
X{1 = 0). Substituindo os valores jc = 0 e x = / na igualdade (10).
obtemos, em virtude de (2) e (3):
0= ^ 1 + B0,
0 = A COSV k l + B sen V I i.
A primeira equação dá-nos A = 0 & resulta da segunda:

B s e n V k l — O.
B ^ O . porque no caso contrário teríamos X = 0 e u = 0. o que
contradiz a hipótese. Por conseguinte, deve-se ter

sen V k I = 0,
donde

Vk = - (n = l . 2, . . . ) ( 12 )
I
(não tomamos o valor n = 0, porque neste caso teríamos = 0e m= 0).
Obtemos assim:

X = B sen — x. (13)
I

Os valores obtidos de k chamam-se valores próprios para o


problema dos limites dado. As funções X (x) correspondrates chamam-se
funções próprias.
Nota — Se tivéssemos tornado em vez de — A, a expressão -1- X = it*.
a equação (8) seria da forma
X " — l ê X = 0.
A solução geral desta equação é:
X = Ae** -f-
A solução além de zero no caso duma equação desta forma
não pode verificar as condições dos limites (2) e (3).
EQUAÇÕES DA PtSICA MATEMATICA 385

Conhecendo |/X podemos, utilizando a igualdade (11), escrever:

1 (t) = C C O S ---- í + z; sen------í (//: 1, 2, ...) . (1^.)


^ I I
Para cada valor de n, e. por conseguinte, por cada X, substi­
tuindo as expressões (13) e (14) na igualdade (6), obtemos a solução
da equação (1) que verifica as condições dos limites (2) e (3).
Designemos esta solução por //„ {.r, t) :
. tízi ( ann ^ . .. ann \
u„ (x, t) = sen — X I C „ c o s-----1 D sen —— M . (1 •v
I \ I I /
Para cada valor de n podemos escolher as constantes C & D c
eis porque escrevemos e (a constante B está inclusa em ® ^n)-
Como a equação (1) é linear e homogénea, a coma das soluções é
também uma solução e eis porque a função representada pela série
cx
u {x, <) = S (X. t)
7?=1
OU
cv

u (x, t) = COS- j - 1 + D„ sen----- í j sen — x (IH)

é também uma solução da equação diferencial (1) que verifica as


condições dos limites (2) e (3). É evidente que a série (16) não será
a solução da equação (1) a não ser no caso em que os coeficientes r„
0 Dn sejam tais que esta série convirja e que convirjam as séries
obtidas depois da derivação termo a termo em relação a x e a r.
A solução (16) deve ainda satisfazer às condições iniciais (4)
e (5). Obtê-la-emos por uma escolha adequada das constantes C„ eD„ .
Fazendo na igualdade (16) 7 = 0, obtemos (comp. a condição (4));

f { x ) = ' ^ j C n sèn (17)


71 = 1
Se a função f(x) é tal que se pode desenvolvê-la em série de
Fourier (ver § 1, cap. XVII) no intervalo (0, /) a condição (17) será
verificada se se fizer

X. 2 r ,, , nji ,
Cn = — y { ^ ) s e a — x dx. (18)

25
386 CALCULO DIFERENCIAL B INTEGRAL

Depois, derivando os termos da igualdade (18) em relação a í


e fazendo / = 0, obtemos em virtude da condição (S) a igualdade
oo
ann nn
(p(x)
n=i
Determinemos os coeficientes de Fourier desta série:
I
^ arm 2 f , , nn ,
D n -----= — I (P (a:) sen — x a x
I IJ I
0
OU
I
2 f , . nn ,
Dn = ----- I cp w sen — X dx. (19)
ann J I

Demonstramos, assim, que a série (16) cujos coeficientes e D^,


são determinados pelas fórmulas (18) e (19) representa, se ela for
duplamente derivávd termo a termo, a função u {x, t) que é a solução
da equação (1) e verifica as condições iniciais e os limites (2)-(S).
Nota — Resolvendo o problema considerado para a equação da
onda por um outro método pode-se demonstrar que a série (16) é
igualmente a solução no caso em que não for derivável termo a termo.
A função f(x) deve, então, ser duas vezes derivável e ^(jc) uma vez
derivável.

§ 4. E quação d a propagação do calo r num a b a rra .


E nunciado do problem a ao s lim ites
Consideremos uma barra homogénea de comprimento /. Supore­
mos que as perdas são eliminadas por isolamento térmico da superfície

iz r
0 <
Fig. 373

lateral da barra e que em cada ponto da sua secção transversal a


temperatura é idêntica. Estudemos o processo da p re^gação do calor
na barra.
Disponhamos o eixo Ox de maneira que uma das extremidades
da barra coincida com o ponto .r = 0 e o outro com o ponto x — l
(fig. 373).
EQUAÇÕES DA FÍSICA MATEMÁTICA 387

Seja u (jc. 0 a temperatura na secção da barra de abcissa x- no


instante t. Estabeleceu-se. experimentalmente, que a velocidade de pro­
pagação do calor, isto é. a quantidade de calor que penetra pela
secção de abcissa x no decorrer dum intervalo de tempo unitário, é
determinada pela fórmula
,du
( 1)

em que S designa a superfície da secção da barra considerada, k o


coeficiente de condução térmica (*).
Consideremos o elemento da barra, compreendido entre as secções
de abcissas Xi e jcs (jc2 — Xi = Ajc). A quantidade de calor que passa
pela secção de abcissa Xi no decorrer do tempo A/ será

4 9 , ------ SM, ( 2)

de igual modo para a secção de abcissa x^:

SM . (3)
X=X2

A quota de calor Agi — Agz no elemento da barra no decurso


de tempo M será igual a

4 9 , - 4 (?.= [ - i g SM - k p
dx X=X2
SM
X=Xi

k^A xSM (4)


dr
du
( aplicamos o teorema de Lagrange à diferençá ^dx
3C=3Cl
du
dx X = X 2
Esta quota de calor no decurso de tempo Aí é consumida com a
)•
elevação da temperatura do elemento da barra duma grandeza

AÇi — AÇ2 = cpAxiS Au

(*) A velcKidade dc propagação do calor, ou a velocidade do fluxo de


calor, é determinada por:
A<?
g = lim
At
em que AQ designa a quantidade de calor que passa pela secção S no decurso
do tempo A/.
388 CALCULO DIFERENCIAL E INTEGRAL

\Q^ — \Q i ^ roA./'ò---- Aí,


dt

em que c designa a capacidade calorífica da substância da barra,


c a densidade da substância da barra (pAor»? é a massa do elemento
da barra).
Igualando as expressões (4) e (5) da mesma quantidade de calor
Afíi — aG z. obtemos:

k^ A.nV Aí = c{) A.rA Aí


dt
ou
üu k d^u
dt r(> d.i^

Designando — = a*, obtemos, íinalmente:


cp
du 2
( 6)
dt dx^

É a equação da propagação do calor {equação do calor) numa


barra homogénea.
Para que a solução da equação (6) seja inteiramente determinada,
a função u (x, t) deve verificar as condições iniciais, correspondentes
às condições físicas do problema. As condições iniciais para a solução
da equação (6) podem ser diversas. As condições correspondentes ao
primeiro problema dos limites para 0 < í < T são as seguintes:

(7)

U(0, t) = y\h{t). ( 8)

U(l, í) = Vl)2(í). (9)

Do ponto de vista físico a condição (7) (condição inicial) cor­


responde a como se para r = 0 a temperatura nas diferentes secções
da barra fosse dada igual a <p(x). As condições (8) e (9) (condições
dos limites) correspondem a como se as extremidades da barra para
jc = 0 e X = I mantivessem uma temperatura igual, respectivamente,
a (0 e ^2 (0-
Demonstra-se que a equação (6) tem uma solução única no
domínio 0 < i: < Z, 0 7" que verifica as condições (7), (8) e (9).
EQUAÇÕES DA FÍSICA MATBMATICA 389

§ 5. Propagação do calor no espaço


Consideremos o processo de propagação do calor no espaço a três
dimensões. Seja u (jc, y. z> t) a temperatura no ponto de coordenadas
( a , y, z) no instante t. Estabeleceu-se, empiricamente, que a velocidade
de passagem do calor pela superfície A5, isto é, a quantidade de calor
fornecida durante a unidade de tempo, é determinada pela fórmula
(análoga à fórmula (1) do parágrafo anterior)

, àti . (I)

en. que k designa o coeficiente de condução térmica do meio consi­


derado que supomos homogéneo e isotrópico, n o sector unitário orien­
tado segundo a normal à superfície As no sentido de propagação do
calor. Em virtude do § 14, cap. VIII, tomo I, podemos escrever
âu du du , du
^ — COS a + — COS p + ^ Y'
ân dx dy dz
em que cosa, cosjS, cosy são os cossenos directores do vector //. ou
du
= ti grad u .
dn
du
Usando a expressão ^ na fórmula (1), obtemos:

AÇ = — k n grad u As.
A quantidade de calor que passa no decurso do tempo At pela
superfície As, será igual a
àQ Aí = — kn grad u Aí As.
Voltemos ao problema que apresentamos no começo deste pará­
grafo. No meio considerado isolemos um pequeno volume V limitado
pela superfície 5.
A quantidade de calor que se propaga pela superfície S será
Ç = — Aí J J k n grad u ds. ( 2)

em que n é o vector unitário orientado segundo a normal exterior à


supe^cie S,
É evidente que a fórmula (2) dá a quantidade de calor que
penetra no volume V (oú que deixa o volume V) no decurso do
tempo Aí. A quantidade de calor que penetra no volume V conduz
ao aquecimento da substância desse volume.
390 CALCULO DIFERENCIAL E INTEGRAL

Consideremos um volume elementar Av. Suponhamos que no


decurso de lapso de tempo A/ a sua temperatura é elevada de àu.
É evidente que a quantidade de calor consumida para elevar a
temperatura do elemento Av será igual a

c A í;o A u ^ cAup Aí,


dt

em que c é a capacidade calorífica da matéria ê p a densidade.


A quantidade global de calor consumido no aquecimento no volume V
no decurso do tempo Aí será

Aí du.

Mas é a quantidade de calor que ao penetrar no volume V no


decurso do tempo Aí, é determinada pela fórmula (2). Temos, assim,
a igualdade

Aí íí k n grad ~ J ^^

Dividindo por Ar, obtemos:

I ^ An grad w d s = | ^ ^ cp dv. (3)

O i n t ^ a l de superfície, que forma o primeiro m onbro desta


equação, pode ser transformado segundo a fórmula de Ostrograddcy
(ver § 8, cap. XV) fazendo F = k grad u :

n grad u ) n d s = div {k grad u) dv.


s V
Substituindo o integral duplo do primeiro membro da igualdade (3)
por um integral triplo, obtemos:

m div (/c grad u) dy = ^ j | cp dv

ou

( “ 111
du
div {k grad u) — c p -----| di; =±= 0.
dt
(4)
EQUAÇÕES DA PISICA MATEMATICA 391

Aplicando o tèorenla da média ao integral triplo do primeiro


men^bro (ver § 12, cap. XIV), obtemos:

di v (/c grad — cp 0,
. ^=^1, y=Vi, z=zi = (5)
u )

L d t

em que P(x, z) é um ponto do volume V,


Como podemos considerar um volume arbitrário V no espaço
a três dimensões, em que se efectua a propagação do calor e como
supomos que a função sob o sinal de integração na igualdade (4) é
contínua, a igualdade (5) será verificada em cada ponto do espaço.
Assim

cp = di v grad { k u ) . (6)
d t

Mas

(ver § 14, cap. VIII. t I) c

div ( i grad «) = . | - ( t I ) + I ; ( i I ) + i ( t Q

(ver § 9. cap. XV). Substituindo na equação (6) teremos:

c ,- ^ = ± h ^ ] + ± ( k ^ ) + lU È í) „)
d t d x V d x ) d yy V
\ d yy ) d z \ d z )

Se k é uma constante, então.

d i v ( k gradu) = A:div (gradu) = k ( ^ ^


\ d ã f d i T d z ^ l

e a equação (6) dá neste caso


d u ( , ^ u \

ou fazendo ^ ~ ®

d u _ ^2 / I ( f ‘u \
(8)
d t d y ^ d z ^ J
392 CALCULO DIFERENCIAL E INTEGRAL

Sob uma forma resumida a equação (8) escreve-se:

du
=
dt
. d^u d^u , j j ¥ 1 A
em '.que Aw = ------- \ - ------ h — é o operador de Laplace. A eqita-
dx^ dy^ dz^
ção (8) é a equação da propagação do calor no espaço a três dimensões.
Para obter a sua solução única que satisfaz ao problema apresentado
é preciso dar-se as condições iniciais.
Suponhamos que temos um corpo íl cuja superfície é a. Con­
sidere-se neste corpo o processo de propagação do calor. No momento
inicial a temperatura do corpo é dada. Isso corresponde a como se
conhecessem os valores iniciais para ^ = 0, por outras palavras às
condiões iniciais:
u(x, y, z, 0) = q)(.r, y, z)

Além disso deve-se conhecer a temperatura em qualquer ponto Af


da superfície a do corpo em qualquer momento t de tempo, as con­
dições dos limites:
u(M, = t) (10)

(Outras condições de limites são possíveis).


Se a função procurada u {x, y, z, 0 não dependesse de z, isso
corresponderia a cómo se a temperatura não dependesse de z, obte­
ríamos a equação
du _ ‘2.(
(I I)
dt

dita equação da propagação do calor sobre o plano. Se se considerar


a propagação do calor num domínio plano D de fronteira C, as con­
dições dos limites do mesmo modo que (9) e (10) são, então, assim
formuladas:
u\x, y, 0) = i [ >{ x, y),

u{M , t) = yi>{M. t).

em que e ^ são funções dadas, M um ponto da fronteira C.


Se a função u não depender nem de z nem de y, obtemos a
equação ^ ^ equação da propagação do calor numa barra*
ot ox^
EQUAÇÕES DA FÍSICA MATEMÁTICA 393

§ 6. Resolução do primeiro problema dos lim ites para


a equação do calor pelo método das diferraças finitas
Do mesmo modo que para o caso das equações diferenciais
ordinárias, por ocasião da resolução das equações das derivadas par­
ciais pelo método das diferenças finitas as derivadas são substituídas
pelas diferenças correspondentes (ver fig. 374):

du (x, t) _ u ( X + /^, i) — u (x, t)


(1 )
âx h

d^u(x, t) I u (x + h, t) — u (x, t) u (x, t) — u(x — h, l)


------ ^ ^ —
dx^ h
(III
du{x,t) u { x h , t ) — 2u(x, t ) -\-u(x — h, t)
~ d ? ~ ^ S ----------------------- ■
duma maneira análoga
du{x, í) u{x, t 1 ) — u(x, t)
(3)
dt I

O primeiro problema dos limites para a equação do calor (ver § 4)


enuncia-se da maneira seguinte. Pede-se para determinar a solução da
equação
du _^2 ^ “ (4)
dt dx^

que verifica as condições iniciais


u(x, 0) = {p(x), (3)
w(0. í) = i|5, (<), 0 - ^ t - ^ T . («)
u{l, í) = 'l’2 (<). o < í< r, (7)

isto é. determinar a solução u (x, t) no rectângulo delimitado pelas rectas


t = 0, X = 0, X = L, t = T se se conhecer os valores da função pro­
curada sobre três dos seus dados: t = 0, x = 0, x = L (fig. 375).
Cerquemos este rectângulo duma grade formada pelas rectas
x= ih, 1= 1 , 2 , . . . ,
t — kl, k = l , 2, . . . .
394 CALCULO DIFERENCIAL E INTEGRAL

e determinemos os valores aproximados das soluções nos nós desta grade,


isto é, nos pontos de intersecção destas rectas. Introduzamos as notações:
u { i h , k l ) = Ui^ fe. Escrevamos em vez da equação (4) as equações c o t -

T
j
1
m i) í
í
H k ) M \m A
(x-h,t) (x,t) (x-hh,t) 1
1 í
LX
Fig. 374 Fig. 375

respondentes em diferenças finitas para o ponto {ih, kl). Em confor­


midade com as fórmulas (3) e (2) obtemos:

A+1 — k_ k— Ut-1, h ( 8)
I
Definamos u

A+l ------^ h + ^í-1, ft)‘ (9)

Resulta da fórmula (9) que se se conhecer os três valores na série


de ordem k: ut, h, í^í + i . ft» k, pode-se determinar o valor ^+1
na série de ordem (/: + !). Conhecemos todos os valores sobre a
recta r = 0 (ver fórmula (5)). Segundo a fórmula (9) determinamos
os valores sobre todos os pontos interiores do segmento t = 1. Os
valores nas extremidades deste segmento são-nos conhecidos em virtude
das fórmulas (6) e (7) Assim, determinamos fila por fila, os valores
da solução procurada para todos os nós da grade.
Está demonstrado que se pode obter, s ^ n d o a fórmula (9), um
valor aproximado da solução não para um valor arbitrário do quo-
/j2
ciente dos passos h e i mas, sòmente no caso em que A fór-
2a^
mula (9) simplifica-se particularmente se o passo / segundo o eixo t
for escolhido de maneira que

i - ^ = 0
EQUAÇÕES DA FÍSICA MATEMATICA 395

ou

1 = ^

Neste caso a equação <9) toma a fonna:

“ í, ft+l = Y (“ i+l, h + “ í-l, *)• ( 10)

Esta fórmula é particularmente cómoda para os cálculos (fig. 376).


Determina-se, pelo método indicado; a solução entre os nós da grade.
O valor da solução entre os nós da grade pode
ser obtido, por exemplo, por extrapolação, tra­
çando um plano para todos os três pontos do (íM V
espaço (x, t, u). Designemos por (x, t) a solução
assim obtida com o auxilio da fórmula (lU)
depois da extrapolação. (i+ w
Demonstra-se que
Fig. 376
limw/,(x, t) = u{x, t),
h->-0
em que u (x, r) é a solução do nosso problema. Está, finalmente, demons­
trado (*) que
t ) - u { x , t)\< :M h \
em que M é uma constante independente de A.

§ 7. Propagação do calor muna barra infinita


Suponhamos que no instante inicial é fixada a temperatura
de diversas secções duma barra infinita. Pede-se para determinar a
distribuição da temperatura da barra nos instantes seguintes. (£-se
conduzido ao problema da propagação do calor numa barra infinita
no caso do estudo dos problemas físicos, sendo o comprimento da
barra tão grande que a temperatura dos seus pontos interiores nos
momentos considerados apenas depende de muito pouco das condições
nas extremidades da barra)
Se a barra coincide com o eixo Ox, o problema matemático
enuncia-se da maneira seguinte. Determinar a solução da equação

du __ 2 d^u
( 1)
d t~ ^

(*) Um enunciado mais detalhado da questão é dado na obra de L. Collatz


cNumerische Behandlung von Differentiagleichungen». Bri. Sprínger. 1951.
396 CALCULO DIFERENCIAL E INTEGRAL

no domínio — o o < j c < o o , /> 0 , verificando á condição inicia)


„(./•, 0) = ( | (./•) (1^)
Apliquemos, para determinar a solução, o método de separação
das variáveis (ver § 3), isto é, vamos procurar uma solução particular
da equação (1) sob a forma de produto de duas funções:
U(:r, t ) = X ( . i ) T(t)
Substituindo na equação (1) teremos:
X (x) T (/) = T (/)
ou
iV

Cada um destes quocientes não pode depender nem de jc, nem de í


e eis porque os igualamos a uma constante (♦) — X}. Obtemos de (4)
duas equações:

(b)
E, resolvendo-as, obtemos:

X = A COS k.r + B sen Ã./


Substituindo em (3), obtemos:
//;, (j\ t) = çQg _j_ ^ sçjj y (7)

(a constante C está inclusa em /í (X) e B (X)).


Para cada valor de X obtemos uma solução da forma (7). As
constantes arbitrárias A e B têm para cada valor de X valores definidos.
Razão porque se pode considerar que A c B são funções de X.
O somatório das soluções da forma (7) é também uma solução em
consequência da linearidade da equação (1)):

[/I (X) COS Xj ' + B (X) sen X.r]

(♦) Como, segundo o sentido do problema 7 ( 0 deve ser limitado qualquer


que seja t, se q>(x) for Itmiado, deve ser negativo. Eis porque escrevemos — X*,
EQUAÇÕES DA FÍSICA MATEMATICA 397

Integrando a expressão (7) em relação ao parâmetro X nos limites


de 0 a 00 obtemos igualmente uma solução

u{x, t) = ] e [A (X) COS \ x -\- B (k) sen \x] dk, ( 8)

se A { k ) c B (A) forem tais que este integral, a sua derivada em relação


a / e a sua derivada segunda em relação a x existam e se obtenham
derivando o integral em ordem sl t e sl x. Escolhamos A (k) e B (X)
de modo que a solução u (jc, t) satisfaça à condição (2). Pondo na
igualdade (8) t = 0, obtemos em virtude da condição (2):

u (x, 0) = fp (x) = ( [A fX) COS kx B (X) sen kx] dk. (9)

Suponhamos que a função ^ (a:) é tal que pode ser representada


por um integral de Fourier (ver § 12, cap. XVII):

<1 (./•) — - ^^^ ^


0 —^
011

(( (x)
=U[(Í (( ( a ) C O S Ka da ) cos kx

+ ( í cp (a) sen ka da^ sen X.r dk. (10)


•X'

Comparando os segundos membros de (9) e (10), obtemos:

1 f
^ (X) = — \ (T (a) COS ka da,
^ J — «X
oc (II)
1 f
ti ( X ) = — 1 q: (a) sen k a da.
n J
398 CALCULO DIFERENCIAL E INTEGRAL

Substituindo os valores encontrados de (X) e B (X) na fórmula (8),


obtemos:

(f (a) COS Ka d a^ cos Xx -f-


0 —oo
+ 00

+ (í fp (a) sen Xa da^ sen Xx^ dX =


— oo
oo oo

= ^ I ^ <P(a) (cos Xa cos Xx-j- sèn Xa ,sèn Xx) d a j dX =


0 —oo
oo oc

= ^ ^ ^ q) (a) cos X(a — x) d a ^ dX


0 — oo

OU invertendo a ordem de integração, temos, finalmente:


oo oo

u{x, Í ) = ^ I |cp(a) ^j e“ ““^ '‘ cosX (a — x)dX j da.j (12>


— oo 0

É a solução do problema que havíamos posto. Transformemos


a fórmula (12) Calculemos o integral que figura entre parêntesis:
oo oo

í e “*^’' 'c o s X (a — x)dX= —


a V t J
cosP zdz. (13>

Esta transformação do in t^ ral foi efectuada com o auxilio das


substituições:

a X ^ t = z, ^ ---- ^ = P (1^)
aV í
Introduzamos a notação
oo
^ (P) = J cos Pz dz. (15)
0
Derivando (*) obtemos:
OO

K ' (p) = — J e“ *’ z sèn Pz dz.


0

(*) Demonstra-se fòdlmente que se pode derivar.


EQUAÇÕES DA FÍSICA MATEMATICA 399

Integrando por parte, vem:

K' (P) ^'sen fiz]o “ y j ^


0
ou

p
^ ' ( p ) = - f ^ (p )-
In t^ a n d o esta equação dtferencial, obtemos:

K (p) = Ce (16)
Detenninemos a constante C. Resulta de (15):

K{0)

(ver § S, cap. XIV). Por c o n s ^ in te , na igualdade (16) deve-se tei

Assim

(17)

^Substituamos o valor (17) do integral (15) em (13):

1 V jt 4
COS %{a — x)d%- ■e
aVt 2
Substituindo pela sua expressão (14). obtemos finalmente o
valor do in t^ ral (13):
oo . (g—x)»
1 1/ n 4a*í
C O S l { a — x)dX = ^ V T (18)

Substituindo esta expressão do integral na solução (12) teremos,


finalm ante:

t \ J
u{x, t) = -----L _ \ <ip(a) e da. (19)
2 a V n í -o
400 CALCULO DIFERENCIAL E INTEGRAL

Esta fórmula, chamada integral de Poisson, é a solução do pro­


blema posto sobre a propagação do calor numa barra infinita.
Nota — Pode-se demonstrar que a função u(x, í) definida pelo
integral (19) é a solução da equação (1) e satisfaz à condição (2) se
a função ^(jc) for limitada no intervalo infinito ( — oo. oo).
Estabeleçamos o sentido físico da fórmula (19). Consideremos a
função
0 para — oo < x < Xq,
(p (x) =

Então, a função
{■ (x) para x„ <
para Xq
< a:o + Ax,
+ Ax < x < oo
(20)

(a-x)'-

—s
oc
kn^l
n (X. t) = «P (oc) e da (2 1)
2a V jtí

é a solução da equação (1) adoptando para r = 0 o valor y>* (x).


Em virtude de (20) pode-se escrever;

^ . 4a2t
u (x, t) ■ ( (p (a) e
1 = da.
laV^nt/
J
-0
Aplicando o teorema da média a este último integral, obtemos:
(g—A')^
* (p (c )A :r ka^t
u (z, t) = —— ■e ^0 < ê < ^0 + A:r. ( 22)
2 a l/ ji í

A fórmula (22) dá o valor da temperatura num ponto da barra


em cada instante de tempo se para / = 0 a temperatura da barra for
em toda a extensão u* = 0. com excepção do segmento [jCo, Xo + Ajc]
onde é igual a 9 (x), É precisamente a soma das temperaturas da
forma (22) que dá a solução (19). Notemos que sc p é a, densidade
linear da barra, e a capacidade calorífica da substância, a quantidade
de calor no elemento [jCo, Xo + Ax] para / = 0 será
AÇ ^ (p(^) Az pc (23)
C onsider^os em seguida a função
(l-xV
1 tka^i
-= - e (24)
2a^nt
EQUAÇÕES DA FÍSICA MATEMATICA 401

Comparando-a ao segundo membro da fórmula (22) e tendo


em conta (23) diz-se que ela dá a temperatura em qualquer ponto
da barra num momento arbitrário de tempo t, se para r = 0 na
secção í (caso limite quando Ax -> 0) se encontrasse uma fonte instan­
tânea de calor que dispensasse uma quantidade de calor Q = cp.

§ 8. Problemas que conduzem ao estudo das soluções das


equações de Laplace. Enunciado dos problemas de lim ites
Neste parágrafo consideraremos certos problemas que conduzem
à resolução da equação de Laplace

d^u d^u ^ p
(1)
dx^ dy^ dz^

Como já mencionamos, o primeiro membro da equação (1)

d'^u , d^u , d^u .


— H—
dx'^ dy^ d^
é chamado operador de Laplace. As funções u que verificam a equação
de Laplace são chamadas funções harmónicas.
I — Distribuição estacionária da temperatura num corpo homo­
géneo. Seja um corpo homogéneo Q limitado por uma superfície a.
Mostramos no § 5 que a temperatura em diversos pontos do corpo
verifica a equação (8):
du
dt \dx^ dy^ dz^J

Se o processo for estacionário, isto é, se a temperatura não


depender do tempo, mas ünicamente das coordenadas dos pontos do
corpo, então, — = 0 e, por conseguinte, a temperatura verifica a
d t

equação de Laplace
d^u dru d^u_^
( 1)

Para que a temperatura do corpo seja determinada univocamente


a partir desta equação é preciso conhecer a temperatura sobre a
superfície a. Formula-se, então, da maneira seguinte o problema de
lii^tes para a equação (1).
26
402 CALCULO DIFERENCIAL E INTEGRAL

Determinar uma função u(x, y, z) que verifica a equação (1)


no interior do volume ü e tomando em cada ponto M da superfície a
os valores dados:
u\o = yÍ){M). ( 2)
Este problema é chamado problema de Dirichlet ou primeiro pro~
blema de limites para a equação (1).
Se sobre a superfície do corpo a temperatura não for conhecida,
mas se se conhecer o fluxo de calor em cada ponto da superfície que
é proporcional a ^ (ver § 5) ter-se-á sobre a superfície a. em vez
on
da condição inicial (2). a condição
du
= ij)* (M). (3)
dn
O problema da procura da solução da equação (1) que verifica
a condição inicial (3) é chamado problema de Neumann ou segundo
problema de limites.
Se se considerar a distribuição da temperatura sobre o domínio
plano D, limitado pelo contorno C, a função u dependerá de duas
variáveis x q y e verificará a equação

^ + - = 0
dx^ ^ V
que se chama equação da Laplace para o plano. As condições iniciais
(2) ou (3) devem ser verificadas sobre o contorno C.
II — Fluxo potencial dum líquido ou de um gás. Equação de con
tinuidade. Suponhamos que no interior do volume O. limitado pela
superfície a (particularmente O pode ser ilimitado) se produz o escoa­
mento dum líquido. Seja p a densidade do líquido. Designemos a
velocidade do líquido por
v = V x i - { - V y j - \ - v^k, (5)
em que Uy, são as projecções do vector v sobre os eixos
de coordenadas. Isolemos no corpo íl um pequeno volume o>. limitado
pela superfície S. Para cada elemento As da superfície S no decurso
do tempo At passa uma quantidade de líquido
AQ = p r n As At,
em que n é o vector unidade orientado segundo a normal exterior
à superfície 5. A quantidade total do liquido ô que penetra no volume o>
(ou que se escoa do volume ü>) exprime-se pelo integral
Ç = Aí ^ J p v n ds (6)
8
EQUAÇÕES DA FÍSICA MATEMATICA 403

(ver §§ 5 e 6, cap. XV). A quantidade de líquido no volume a>


no instante t era

(D

No decurso do tempo Al a quantidade de líquido variará, em


consequência da variação da densidade, duma grandeza

(7,
Ü)
Supondo que o volume o> não está ligado a fontes, concluímos
que esta variação é devida a um afluxo de liquido cupa quantidade
é determinada pela igualdade (6). Igualando os segundos membros das
igualdades (6) e (7) e simplificando por Ar, obtemos:

d(ú. (8 )
S (0
Transformemos o integral duplo do primeiro niembro segundo a
fórmula de Ostrogradsky (§ 8, cap. XV). A igualdade (8) torna-se, então:

d(ú

ou

Sendo o volume tomado arbitràriamente e sendo a função sob


o sinal de integração contínua, temos:

+ div (pi^) = 0 (9>


dt
ou

^ (Pí^«) + Y (pVy) + f (PV,) = 0. (9')


dt dx dy dz
Ê a equação de continuidade de escoamento dum fluído compresslveL
Nota — Em certos problemas, por exemplo, quando se faz o estudo
do escoamento do petróleo ou dos gases através dum terreno poroso
para o poço, pode-se adoptig:
k
v = — — gradp,
404 CALCULO DIFERENCIAL E INTEGRAL

em que p é a pressão, k o coeficiente de permeabilidade e

dt dt

\ = const. Substituindo na equação de continuidade (9), teremos:

— div (/cgradp) = 0
dt
011

(10)
dt d x \ dx) d y \ dyJ
du/ dz\ dz)
Se Â; é constante, esta equação toma a forma:

(1 1 )
dt A. \dx^ dy^ dz^) ’

e encontramos a equação do calor.


Voltemos à equação (9). Se o fluido é incompressível, p = const,
dp
^ = 0 e a equação (9) escreve-se:

div (v) = 0. ( 12)

Se o movimento é potencial, isto é, se o vector v é o gradiente


duma certa função

V = grad (p,
a equação (12) toma a forma:
div (grad cp) = 0
ou
d^<p d \ d \
+ H + ^ = 0, (13)
dx^ dy dz^

por outras palavras, a função potencial da velocidade <p deve verificar


a equação da Laplace.
Em numerosos problemas, como por exemplo, nos problemas de
filtração, pode-se adoptar
v = — *1 gradp,
EQUAÇÕES DA FÍSICA MATEMATICA 405

em que p é a pressão. ki uma constante; obtemos, então, a equação


de Laplace para determinar a pressão

0. (13’)
dx^ dy^ dz^
As condições iniciais para a equação (13) ou (130 podem ser
formadas da maneira seguinte:
1. Dá-se sobre a superfície a os valores da função procurada p
que é a pressão (condição (2)). É o problema de Dirichlet.
2. Dá-se sobre a superfície a os valores da derivada normal
dn'
por outras palavras, o fluxo que atravessa a superfície (condição (3)).
É o problema de Neumann.
3. Dá-se sobre uma porção da superfície a os valores da função
procurada p (a pressão) e sobre uma porção da superfície os valores
da derivada normal (o fluxo através da superfície). É o problema de
Dirichlet-Neumann.
Se o movimento é plano-paralelo, isto é, se a função (p (ou p)
não depender de z, obtém-se a equação de Laplace no domínio a
duas dimensões D de fronteira C:

d \ d \
0. (14)
dx^ dy^
As condições iniciais do tipo (2), problema de Dirichet. ou do
tipo (3), problema de Neumann. são dadas sobre o contorno C.
III — Potencial duma corrente eléctrica estacionária. Suponhamos
que num meio homogéneo que preenche um certo volume V passa
uma corrente eléctrica cuja densidade em cada ponto é dada pelo
vector J (x, y, z) = J:,i + J y j + Jzk. Suponhamos que a densidade
da corrente não depende • do tempo t. Suponhamos, ainda, que o
volume V considerado não contém fonte de corrente. Por conseguinte,
o fluxo do vector J através da superfície fechada S situada no
interior do volume V é igual a zero:
JJ Jnds= 0,
s
em que n é o vector unidade dirigido, sçgundo a normal exterior,
à superfície.
Com base na fórmula de Ostrogradsky podemos concluir que
d iv J " = 0. (15)
406 CALCULO DIFERENCIAL E INTEGRAL

A lei de Ohm generalizada permite determinar no mèio condutor


considerado, a força eléctrica E :

E = — (16)
X
ou
J=kE,
em que A é a permeabilidade do meio que consideramos constante.
Resulta das equações gerais do campo que se o processo for esta­
cionário, o campo vectorial E é irrotacional, isto é, que rot JEJ = 0.
Então, do mesmo que no caso do estudo do campo das velocidades
dum líquido, o campo vectorial é um campo potencial (ver § 9. cap. XV).
Existe uma função <p tal que
E = grad q). (17)
Em virtude de (16). obtemos:
J = X grad q). (18)
Resulta de (15) e (18):
Adiv (grad q)) = 0
ou
d‘ (p 5"(p d^(p
■ 0 . (19)
dz^

Obtivemos a equação de Laplace. Resolvendo esta equação para


as condições iniciais correspondentes, obtemos a função y> e. segundo
as fórmulas (18) e (17), obtemos a corrente J e a força eléctrica E .

§ 9. Equação de Laplace « n coordenadas ciKndricas.


Resolução do problema de Díríchlet para um arco com
valores conslantes da função procurada sobre os
círculos Interior e exterior
Seja u (x. y, z) uma função harmónica de três variáveis. Entíio,
por definição.
d^u d^u
0. (1)
dx^ dy^ dz^
Introduzamos as coordenadas cilíndricas (r, f, z):
ar = rco&(p, i/ = rs€ntp, z = z
EQUAÇÕES DA FÍSICA MATEMÁTICA 407

donde.
r = Vx^ + y \ (p = arc tg — , z = z. (2)
X

Substituindo as variáveis independentes x, y, z por r, <p e z


obtemos uma função u*:
u{x, y, z) = u*{r, (ç, z).
Encontremos a equação que deve satisfazer u* {r, (p, z) como
função das variáveis, r, <p c z- Temos:

du du* dr ^ du* diç


dx dr dx d(ç> dx

cFu
dx^ df \ dx) dr dx^ dr á(p dx dx

di^u* ^ Y I
(3)
dw^ \ dx ) d(p dx
duma maneira análoga

d^u*
dr^ \ dy /) dr dy
dy^ dr d^
^(p dy dy

+ (4)
d^^ \ dy ) d{ç dy,2 ’

alem disso.
d^u cPu*
(5)
dz^ dz^
Encontramos as expressões para
dr dr d^r d^r d^> ^cp d \ d \
dx ' dy ' dx^ ’ dy"^ ’ dx dy dx'
a partir da ioualdade (2). Fazendo a soma dos segundos membros das
igualdades (3), (4) e (5) e igualando o resultado a zero (visto que a
soma dos primeiros membros destas igualdades é nulo em virtude de (1)),
obtemos:
d^^u* , 1 di^ , 1 , d^ú
( 6)
dr^ r d r ' ^ r ^ 5(p^ + ^ = “-
408 CALCULO DIFERENCIAL E INTEGRAL

É a equação de Laplace em coordenadas cilíndricas.


Se a função u não depender de z e depender de jc e 3^, a fun­
ção u* que não depende a não ser de r e ^ verifica a equação

d u* 1 dV
■ 0, (7)
dr^ r dr
em que r e y> são as coordenadas polares para o plano.
Achemos agora a solução da equação de Laplace no domínio D
(arco) limitado pelos círculos C i: — R \ e C^: x* + t/* = i?*
que tomara os valores limites seguintes:
W|Cl = Wl, (8)
U I C2 = «2. (9)
em que Ui e são constantes.
Resolveremos o problema em coordenadas polares. É evidente que
é lógico procurar uma solução que não dependa de <f.
A equação (7) toma, então, a forma:
____Q

dr^ r dr

Integrando esta equação, obtemos:


u = C1Log r -(- Cz- (10)
Determinemos Ci e C2 das condições (8) e (9):

Uj = Cx Log i?i 4* Cz,


Uz = Cl Log /? 2 “I" C2.
Daí tiramos
Uz — Uj Log Ri
Ci = Cz = Ui— (Uz — Ui)
L o g —^ 1 T?2
Ri
Substituindo os valores achados de Ci e C2 na fórmüla (10),
obtemos, finalmente:
r
Log —
u = Ui H-------------- - {Uz — Ui). (11)
Log E l
Ri
EQUAÇÕES DA FÍSICA MATEMATICA 409

Nota — De facto, resolvemos o problema seguinte: determinar uma


função u que satisfaça à equação de Laplace no domínio limitado pelas
superfícies (em coordenadas cilíndricas):
r=R^, r = y ? 2, 2 = 0, z = H,
e que verificam as condições dos limites seguintes:
U \ r ^ R ^ = U U \r= R .^ =

du
= 0, fí = 0
dz z=n
(problema de Dirichlet-Neumann). É evidente que a solução procurada
não depende nem de z, nem de e é dada pela fórmula (11).

§ 10. Resolução do problem a de D irichlet p a ra o circulo


Seja no plano Oxy um círculo de raio R, de centro na origem
das coordenadas e uma função /(^) dada sobre esse círculo (ç> é o
ângulo polar). Pede-se para determinar uma função u (r, (p) contínua
no círculo (inclusivé sobre a fronteira), que verifica no interior do
círculo a equação de Laplace
d“u , d^u
— = (1>
dx^ dy^
e adoptando sobre a circunferência do círculo os vectores dados
u |,= fí = /((p). (2)

Resolveremos o problema em coordenadas polares. Escrevamos a


equação (1) nestas coordenadas:
d^u \ du 1 d^u
0

ou

d^u du . d^^u
:0. ( l ’>
dr^ dr '
Procuraremos a solução pelo método de separação das variáveis
fazendo
u = O (cp) R(r). (3 )
Substituindo na equação (10 .obtemos;
r^d) (fp) R" (r) + rd ) (tp) R ' (r) + d>" (cp) (r) = 0
410 CALCULO DIFERENCIAL E INTEGRAL

OU
<l>" (((;)____ r R " (r) + r]{’ (r) = - k ^
i^)
Cl) (if.) “ R (r)

Como o primeiro membro desta equação não depende de r,


e o segundo membro de <p, eles são. por conseguinte, iguais a um
número 'constante que designaremos por — k^. Assim a igualdade (4)
dá duas equações:
d)” ((()) + /^l) ((p) = 0, (5)
r^R" (r) + rR ' (r) - k~R (r) = 0. (')')
A solução geral da equação (5) será
(1) = A COS ^’(p + sen (6)

Procuraremos a solução da equação (5') sob a forma R (r) =


Substituindo R (/*) = cm (5'). obtemos:

r m {m - 1) + rmr^~ ~ = 0
ou
— lê = 0.
Podemos escrever duas soluções particulares linearmente indepen­
dentes e A solução geral da equação (5') será
R = Cr^ + Dr~^ (7)
Substituindo as expressões (6) e (7) em (3):
Uk = AC O S k (ç + Bh sen Arcp) (8 )
A função (8) será a solução da equação (1') para todo o valor
de k, diferente de zero. Se /: = 0 as equações (5) e (5') escrevem-se

a ) " = 0, rR"{r) + / ? '( r ) = 0,


e. por conseguinte,
Uo = {Ao + (Co + D qLog r) (8')
A solução deve ser uma função periódica de <p, visto que para
um mesmo valor r para <p c (p + 27t devemos chegar à mesma solução;
trata-se. com efeito, dum mesmo ponto do círculo. Eis porque é
evidente que na fórmula (8') seja preciso que Bo = 0. Procuraremos
a solução contínua e finita no círculo Por conseguinte no centro
do círculo para r = 0 a solução deve ser finita, e por consequência
é preciso que na fórmula (8') Do = 0 e na fórmula (8) 0 ^ = 0
EQUAÇÕES DA FÍSICA MATEMATICA 411

Assim, o segundo membro de (8') reduz-se ao produto A qC q que


designaremos por AÍ/2, Assim

Un=- (8 )

Procuraremos a solução do nosso problema sob a forma de soma


das soluções do tipo (8), visto-que a soma das soluções é uma
solução. A soma deve ser uma função periódica de (p. Se-lo-á também
se cada termo da soma for uma função periódica de (p, Para isso k
deve tomar valores inteiros. (Notemos que se tivéssemos igualado
os membros da igualdade (4) ao número + não teríamos obtido
uma solução periódica). Podemos limitar-nos aos valores positivos

A:= l, 2,
visto que, as constantes A, B, C, D sendo arbitrárias, os valores nega­
tivos de k não dão novas soluções particulares.
Assim

u(r, ( p ) = y + 2 ( ^ 1 „ COSncp + 5 „ sen ra(p)r" (9)

(a constante Cn está inclusa em Aj,, e 5^). Escolhamos, agora, as


constantes arbitrárias Aj^ e de maneira que sejam verificadas as
condições iniciais (2).
Substituindo na igualdade (9) r = R, obtemos em virtude da con­
dição (2):
oo

/(9 ) = ^ + (A n COS n(p + B j, s è a ncp) ( 10)


n= l

Para que tenha lugar a igualdade (10) é preciso que a função


admita um desenvolvimento em série de Fournier no intervalo ( — ir, ir)
e que A nR ^ e sejam os seus coeficientes de Fourier. Por con­
seguinte, An Q Bn são determinados, segundo as fórmulas:
71

n = ~7;r \ f{ t) c o s n t d t,
nB j
31 (11)
1 f
n = —— \ f (t) sên nt dt.
ici?" J
412 CALCULO DIFERENCIAL E INTEGRAL

Assim, a série (9) com os coeficientes determinados, segundo as


fórmulas (11), será a solução do nosso problema se ela puder ser
duas vezes derivada termo a termo em ordem a r e a (mas isso
não foi demonstrado). Transformemos a fórmula (9). Substituindo
e Bn pelas suas expressões (11) e efectuando certas transformações
trigonométricas, obtemos:
n oo .T

u ( r , < f ) = ^ j f ( t ) d t + ^ ^ j / ( t ) c o s n ( t — ( f : ) d t ^ ^ ' j =

—.1 /<= 1 —.l

- Ji n= i

Transformemos a expressão que figura entre parêntesis (♦):

1+2 2 - . 9 ) = 1 + 2 (i)"
71 = 1 71 = 1

_ZL í(í-(f) _ÍL -i(í-<f)


= 1 + — --------— + — -------------- :
1 _ JLg-Kí-t)
R

-(ir 2 íd2
R ^-r
— 2 i ? r c o s ( í — (p) +
1 — 2 —
R
COS (t — (p ) +
(i)
C*) Durante a demonstração determinamos a soma duma progressão
geométrica infinita,* cuja razão é um número complexo de módulo inferior
à unidade. Esta fórmula da soma duma progressão geométrica pode ser
estabelecida da mesma maneira que para os números reais. É necessário, no
entanto, ter em conta a definição de limite duma função complexa da
variável real. A variável independente é aqui n (ver § 4, cap. VII, t. I).
EQUAÇÕES DA FÍSICA MATEMATICA 413

Substituindo a expressão que figura entre parêntesis na fórmula (12)


pela expressão (13), obtemos:
.T
Rr>2 — r2
U{l\ (|): /(<)• dt. (14)
2ji J J{~ — 2rJ{ COS (t — (f) +

A fórmula (14) chama-se integral de Poisson. Demonstra-se,


analisando esta fórmula, que se a função / (y>) for contínua, a função
w(r, (f>) definida pelo integral (14) verifica a equação (!') e quando
r R teremos u (/*, (9), por outras palavras u (r, ^) é a solução
do problema de Dirichlet que apresentamos para o círculo.

§ 11. Solução do problema de Dirichlet pelo método


das diferenças fínitas
Seja um domínio D no plano Oxy limitado pelo contorno C.
Seja dada sobre 0 contorno C uma função contínua /. Pede-se para
determinar a solução aproximada da equação de Laplace

^ ^ay2 — » ( 1)
ox
que verifica a condição dos limites

u\c = f. (2)
Tracemos duas famílias de curvas
x = i h et y = kh, (3)

em que h é um número dado. tomando i q k sucessivamente valores


inteiros. Diremos que o domínio D está recoberto por uma grade
(quadrícula). Os pontos de intersecção das diferentes rectas serão cha­
mados nós da grade.
Designemos o valor aproximado da função procurada no ponto
X = ih, y = kh por isto é. u{iK kh) = h- Assemelhamos
o domínio D ao domínio da grade D* constituído pelo conjunto dos
quadrados contidos inteiramente no domínio D, assim como alguns
quadrados cortando a fronteira C (pode-se não ter isso em conta).
Assemelha-se o contorno C ao contorno C* constituído por segmentos
de recta do tipo (3). Em cada nó situado sobre o contorno C* demos
o valor /* igual ao valor da função / correspondente ao ponto mais
próximo do contorno C (fig. 377).
Não consideramos os valores da função procurada a não ser
para os nós da grade. Como já indicámos no § 6, por ocasião da
414 CALCULO DIFERENCIAL E INTEGRAL

resolução pelo método aproximado, as derivadas são substituídas pelas


diferenças finitas:

,k — h 4~ h
dd? ~ih, y—hh

d \í í/. k+1 — fi 4~ U}^


01/ x= ihy y ^ h l i

A equação diferencial (1) é substituída pela equação das dife­


renças finitas (depois da simplificação por h^):
k— /j -f- Ui-x^ k + /i+i — + Ui^ = 0
ou (fig. 378)

~ + h+ /í + 1 + ^i-1, k+ A-l)« (4)

Para cada nó da grade situando no interior do domínio D* (e


não situado sobre a fronteira de C*) componhamos a equação (4).

Fig. 377 Fig. 378

Se o ponto (jc = /A, y = kh) for vizinho do ponto do contorno C*,


no segundo membro da igualdade (4) teremos os valores de /*.
Obtemos, assim, um sistema não homogéneo de N equações a N incó­
gnitas {N i o número de nós da grelha situados no interior do
domínio D*).
Demonstremos que o sistema(4) possui uma solução que é
única. É o sistema de N equações lineares a N incógnitas. Ele possui
uma solução única no caso em que o determinante do sistema for
diferente de zero. O determinante do sistema é diferente de zero se
o sistema homogéneoapenas tiver uma solução trivial (nula).
EQUAÇÕES DA FÍSICA MATEMATICA 415

O sistema será homogénéo se /* = 0 para os nós da grade situados


sobre o contorno C*. Demonstraremos que neste caso todos os valores
Ui^ para todos os nós interiores da grade são nulos. Suponhamos que
no interior do domínio existe ^ diferentes de zero. Para fixar
ideias, suponhamos que o maior valor desses valores é positivo.
Designemo-lo por Wj, ^ > 0.
Em virtude da fórmula (4) 'escrevemos:
1
h = + h + h+ i + ^i-i, h + A-l)- (4')

Esta igualdade apenas é verificada para o caso em que todos


os valores u do segundo membro são iguais ao maior valor íÍ í ,
Temos assim cinco pontos para os quais o valor da função procurada
é üi^ Se nenhum destes pontos estiver sobre a fronteira, demons­
traremos tomando um de entre eles e escrevendo para ele a igual­
dade (4) que noutros pontos o valor da função desconhecida será
também igual a ãj, ft. Prosseguindo assim atingiremos a fronteira e
demonstraremos que para um ponto da fronteira o valor da função
é igual a fe. Mas isso contradiz o facto de que nos pontos da
fronteira f* = 0.
Supondo agora que no interior do domínio temos um valor
negativo mínimo demonstraremos da mesma maneira que sobre a
fronteira o valor da função é negativo, o que contradiz também a
condição apresentada.
Assim o sistema (4) possui uma solução que é única.
Os vedores de Ui^ determinados a partir do sistema (4) cons^
tituem os valores aproximados da solução do problema de Dirichlet
formulado anteriormente. Estabeleceu-se que se a solução do problema
de Dirichlet para um dado domínio D e uma dada função f existe
(designemo-la por u{x, y) e se ui^ for a solução do sistema (4),
teremos, então, a relação
Iu(^, y) —Ui, hI< (5)

em que A é uma constante independente de h.


Nota — É por vezes possível, embora isso não esteja demons­
trado rigorosamente, utilizar o processo seguinte para avaliar o erro
da solução aproximada. Seja a solução aproximada para um
passo igual sl 2 h , u\^k ^ solução aproximada para um passo igual a A,
Eh(x, y) o erro da solução u\% Então, temos a igualdade aproximada
1 ,.(h) ■
{x, y) ^
416 CALCULO DIFERENCIAL E INTEGRAL

nos nós comuns das grades. Assim, para determinar o erro da solução
aproximada para um passo h, é preciso determinar a solução para um
passo 2A. A terça parte da diferença destas soluções aproximadas é
precisamente a avaliação do erro da solução para o passo h. Esta
nota pode respeitar igualmente à resolução da equação do calor pelo
método das diferenças finitas.

Exerdcios

I. Estabelecer a equação das vibrações de torção duma barra homogénea


cilíndrica.
Indicação — O momento giratório na secção da barra de abeissa x é
d0
determinada pela fórmula M=^GI , em que 0 (oc, r) é o ângulo de tór­
ax
ção da secção da abeissa x no momento t, G o módulo de deslize, / o
momento de inércia polar da secção transversal da barra.
^20 ^ a20 ^ GI
Resp. -T— = -----
dí2 ax2 ’ em que
H a2 = ----
^ k á o momento de inércia da
unidade de comprimento da barra.
d20 d20
Determinar a solução da equação -r:— =
aí 2 ax2 , Que verifica as condições
0(0, í) = 0, 0(Z, í) = 0, 0(x, 0) = (p(x), ^ ^ = Q,em ^ue

q,(x) = ^ p a r a 0<x<A ,

«PW = — - ^ ^ + 2 0 0 para y < x < / .

Dar uma interpretação mecânica do problema. Resp. 0 {x, t) =


CD

_ 80q i2k-\-Í)nx (2k-\-\)nat


------------ sen 2----- 1— -— COS ----- .
(2A: + 1)2 I I
k= Q

Estabelecer a equação das oscilações longitudinais duma barra cilíndrica


homogénea.
Indicação — Se u{x, /) designar a deslocação da secção do cilindro de
abeissa x no momento /, a contraeção de traeção T da secção x é determinada
pela fórmula T — ES , em que E é o módulo de elasticidade do material,
ox
S a superfície' da secção transverçal da barra.
Resp. —fl2 ,em que a^— — . p a densidade do material da barra.
dí2 dx2 p ’
Uma barra homogénea de comprimento 21 sob a acção das forças aplicadas
às suas extremidades curvou-se duma grandeza 2X. No instante í = 0
liberta-se-la da acção das forças exteriores que lhe foram aplicadas. Deter-
EQUAÇÕES DA FÍSICA MATEMATICA 417

minar o afastamento u {x, t) da secção da barra de abcissa x no instante /


(o meio do eixo da barra está situado no ponto de abcissa x = 0).

(— {2k-\-\)nx {2k-\-\)nat
Resp. a(*, t) = ^ 2 7Í
(2/r-t-l)2 21 21
/í=0
5, Uma barra de comprimento / está fixa por uma das suas extremidades
e sobre a outra age uma força de extensão P. Determinar as oscilações
longitudinais da barra se para / = 0 a força P não agir.

P«p y ( - l ) " ;en ( 2 n + l ) n i (2n+ l)ji(í/


2 j (2« + 1)2 2Í ------- 21-------
71=0
(ver o problema 3 para o sentido de E c S).
6. Determinar a solução da equação que satisfaz às condições

àu (x, 0)
w (0, 0 = 0, u(I, i) = A sen co/, u (x, U) = 0, = 0.

Dar uma interpretação mecânica do problema.

A sen — X sèn coí


a
Resp. íi (x, t)z
sen — I
a
2A(úa njiat
sen — ^— sen_ . tuix
~ 2j „ / nna I I
"= ‘ “ (— )
Indicação — Procurar a solução sob a forma de soma de duas soluções:

A sen— x sènco/
a
U -V ~ {-IL \ OÚ W -

sen — I
a
é a solução que verifica as condições:
dv (x, 0) dw (x, 0)
i;(0, 0 = 0, V (l, 0 = 0 , V (x^ 0) = —w{x, 0),
Ft dt
^ Supõe-se que sen ^ ^ q j

7. Determinar a solução da equação - ^ = a2 ^ que verifica as condições:


dt dx^

27
u(0, 0 = 0, u ( l, 0 = 0, í > 0 ,u ( x , 0) =
I X

l — x para
para 0 < x ^

< x < /,
,
418 CALCULO DIFERENCIAL E INTEGRAL

_ (2 1 1 1 1 ^ (2 .+ 1)n,z
Resp (., 0 = - ^ 2 - f c V ^ sen -
I
71 = 0

Indicado — Resolver o problema pelo método de separação das variáveis.

8. Determinar a solução da equação -^ ^ = r/2 JlHL que verifica


vei as condições:
Üt OX'i- ’
X (I — x)
u (0, i) — u (0, l) --- 0, u (.r, 0) = -
Z2
(2n+l)2Ji2g2f
1 /2 (2/1 1) nx
Resp. u(x, í) 2
.^3 ZJ (2/i-hl)^ I
n=0

9. Determinar a solução da equação — , que verifica as condições:


dt dx^

=0, u (/, t) = “o. u ( i, 0) = fp(x).


dx la=0

Indicar o sentido físico do problema.


oo
(2n-\-\) Ji
Resp. u{x, /) = u o + 2 j COS------
n=0
. 2 r (2n-\-i)nx. (—
cm que ^n = - ) <P(x) C O S - ^ - j------
(2/1 +1)

Indicação — Procurar a solução sob a forma u = + v (x , /).

10. Determinar a solução da equação = g2 que verifica as condições:


dt dx2 *

«(0, 0 = 0 , = u ( i, 0)=(p(x).

Indicar o sentido físico do problema.

Resp. „ (X, í) = 2 p (p + l) + p2 ' sén


/ ’
n=l
I
que /1„ = — j qp(i' sen dx, p = H l , pi, p2. •••. PnS*o »»

positivas da equa^o tg p = — tL .
EQUAÇÕES DA FÍSICA MATEMATICA 419

Indicoção — Na extremidade jc = / da barra produz-se uma troca de calor


com o meio ambiente, em que a temperatura é igual a zero.
11. Determinar (segundo a fórmula ( 10 ) do § 6 fazendo /t = 0,2), a solução
j j - du « d^u que verifica as condições:
aproximada da equaçao ---- —2
dt ux^

u(x, 0 ) = o :^ -|— x j , u(0, 0 = 0 , u { [ , 0 = “^ i 0 < í < 4 / .

12. Determinar a solução da equação de Laplace d'^u , d'^u = 0 na zona


dx^ dy^
0 <C x<;a, 0 < < y < o o , que verifica as condições:
u(0, y) = Q, u ( a , y) = 0, u { x , 0) = A ( l — <») =0.

2 ^ 1 — z- y nnx
Resp. u ( i , í) = _ 2 — ^ s e n ------
n = l

Indicação — Determinar a solução pelo método de separação das variáveis.


13. Achar a solução da equação de Laplace d “U _ rectângulo
dx^
0 < ! x < ; í7, que verifica as condições:
u(x, 0) = 0 ,'u (x , ò) = 0, u(0, y) = A y { b — y),

^ (fl» !/) = 0.
(2 n-j-l) n (g —i) (2 n ^ l ) n y
Resp. u ( x , t)-
SAb^ b ò
Jl3 S (2n + l)3 g jj (^^ + 1)
71=0 b

14. Determinar a solução da equação


d^u . d^u
dx^ ' dy^ = 0 no interior do and
limitado pelos círculos x 2 - \ - y 2 = Rl^ x2 + y2 = B l , que verifica as condições:
du
dr r=fíl X2jii?i ’ “ l'-=«2 “2-
Dar uma interpretação hidrodinâmica do problema.
Indicação — Resolver o problema em coordenadas polares.

Resp. u = U 2 ~ Q - • Ro
Log —^ .
2A.JI
^2^ d^u
15. A função u(x, y) = e - y senx é a solução da equação ^ ^ -x- . „ = 0
dx2 oy^
no quadrado 0 <x<l, que verifica as condições:
(0 , y) = 0 , u ( l , y) = e~í'senl, u(x, 0 ) = scnx, u (x, l) = e~iscna:.
16. Nos problemas 12-15 resolver a equação de Laplace para condições dos
limites dados pelo método das diferenças finitas no caso áe h = 0,25.
Comparar a solução aproximada com a solução exacta.
Copíhilo XIX

CALCULO OPERACIONAL E APLICAÇÕES

Na hora actual, o cálculo operacional (ou simbólico), é um dos


domínios importantes da análise matemática. Em física, em mecânica,
em electrotécnica e noutros ramos da ciência utiliza-se os métodos do
cálculo operacional para a resolução de diferentes problemas. O cál­
culo operacional encontrou uma aplicação particularmente larga na
tecnologia moderna da automação e das telecomunicações. Neste capítulo
(com base na matéria dos capítulos precedentes) serão precisamente
expostas as noções fundamentais do cálculo operacional bem como os
métodos da sua aplicação à resolução das equações diferenciais ordi­
nárias.

§ 1. Original e imagem

Seja dada uma função da variável real t definida para t > 0


(por vezes consideraremos que a função / (/) está definida num intervalo
infinito — 00 < r < 00 . mas / (0 = 0 quando t < 0). Suporemos que
a função /(/) é contínua por corte, isto é, tal que, em cada intervalo
finito, ela possui um número finito de descontinuidades de primeira
espécie (ver § 9, cap. II, t. I). Para assegurar a existência de certos
integrais no intervalo infinito 0 < / < oo imporemos à função / (t)
restrições complementares. Suporemos precisamente que existem números
positivos constantes A i e 5o tais que
I/ (í |< M e ^ o ‘ ( 1)

para todo o valor arbitrário de t tomado no intervalo 0 < í < oo .


Consideremos o produto da função f(t) pela função complexa
da variável real (*) /, em que p = a + iò (a > 0) é um número
complexo:
/w . ( 2)

(♦) A respeito das funções complexas da variável real, ver § 4, cap. VII, t I.
CALCULO OPERACIONAL E APLICAÇÕES 421

A função (2) é também uma função complexa da variável real /:


ib)t - i a + - ihl
e - ‘’'í(t) = e
= e~"'f (t) COS bt — ie "'/ (/) sen bt.
/
Considereremos em seguida o integral impróprio
oo cv-

J e ~ ( t ) d t = ^ e~ “'/ (t) COS bt d t — i \ e ~'" / (/) sen bt dt. (.H)


0 0 0

Mostremos que se a função /(/) verifica a condição (1) e a > So,


então, os integrais do segundo membro da igualdade (3) existem e a
convergência desses integrais é absoluta. Consideremos, primeiramente,
o primeiro destes integrais:
oo oc

0
í
/ (t) COS b t d t l ^ \ \e ^ f (t) COS bt dt *
0
oo oo

-(a—So)t dt : M
Cl — Sq
0 0

Estima-se da mesma maneira o segundo integral. Assim o integral


oo

\ e-^^f (t) dt existe. Ele define uma certa função de p, que designa-
ü
remos (*) por F (p):
F {p )= ^ dt. (4)

A função F ip) chama-se transformada de Laplace ou imagem L


ou simplesmente imagem de f (t). A função / (0 chama-se original ou
função objecto, O facto de F (p) ser a imagem da função f(t) é assim
notada:
(5)
OU

(6)
ou

L{f {t ) } = F{p). (7)**)


(♦) A função F(p) para p ^ O é uma função da variável complexa (ver
entre outros o livro traduzido do russo de V. Smionov «Curso de Matemáticas
Superiores», vol. III, segunda parte).
(**) Utiliza-se também outros símbolos de correspondências. É assim
que em vez da notação -í- se emprega também o símbolo J e escreye-sc
no caso da fórmula (6) / (t) □ F (p) (N.d.T.).
422 CALCULO DIFERENCIAL E INTEGRAL

Como veremos em seguida o sentido da introdução das imagens


reside no facto de que elas permitem simplificar a resolução de nume­
rosos problemas, em particular de reduzir a resolução das equações
diferenciais ordinárias a certas operações algébricas simples que per­
mitem determinar a função imagem. Conhecendo a imagem pode-se
determinar o original quer por meio das tábuas prèviamente compostas
«original-imagem» (dicionário de imagens) quer pelos métodos que
exporemos mais à frente. Perguntas se põem. então, naturalmente.
Seja dada uma certa função F (p). Existe uma função / (/) em
que F ip) é 3, imagem? Se ela existe, é única? As duas perguntas
recebem uma resposta positiva se F (p) e f (í) satisfizerem certas con­
dições. Em particular a unicidade da imagem é estabelecida pelo
teorema seguinte que enunciaremos sem demonstração.
Teorema da unicidade — Se duas junções contínuas (p{t) e ^(/)
possuem uma mesma imagem L F (p) essas junções são idênticamente
iguais.
Este teorema será duma grande utilidade para tudo o que se
seguirá. Com efeito se na resolução dum problema prático pudermos
determinar a imagem da função procurada, e em seguida obtivermos
o original segundo a sua imagem, podemos concluir em virtude do
teorema formulado que a função obtida é a solução do problema posto,
e que não existem outras soluções.

§ 2. Im agens d a s funções Gq (f), sen f, cos t


I — A função / (O assim determinada
j{t) = \ para
f{t) = 0 para t < 0
chama-se junção unidade de Heaviside e representa-se por <to (0- O grá-

õ o (t)
/

0 t
Fig. 379

fico desta função está representado na figura 379. Obtemos a imagem L


da função de Heaviside:
oo

-Vt d t = —
^ K {t)} = \ <?
- í p
0
CALCULO OPERACIONAL E APLICAÇÕES 423

Assim (♦)
(8)

OU, mais exactamente.

Em certos tratados de cálculo operacional chama-se imagem da


função /(O à expressão

í^*(p)=ple-^*fit)dt.

Neste caso tem-se : Oq (t) le, por conseguinte, C C, mais


exactamente Coq (t) C.
II — Seja f (0 = sen t; então.
oo
e ^ (—p sen í — cosí) 1
L { s e n t} = \ e sea t d t =
- í 7 + 1 0 p^+1

Assim,
sen í (9)
p== + i

III — Seja / (0 = COS t\ então.


oo

L {C O S í } = í e -^ + o s í d í = ^ 2 > ! 1 - L z Z £ 2 ÍÍ)

J p" + 1 0 p +1
Assim,

COS í
p ( 10 )
■p " + l

(*) Na altura do cálculo do integral I dt poder-se-ia tê-lo represen-


0
tado como a soma dos integrais de funções reais; teríamos obtido o mesmo
resultado. Esta nota diz respeito, igualmente aos dois integrais seguintes.
424 CALCULO DIFERENCIAL E INTEGRAL

§ 3. Imagens das funções com escala modificada da variável


independente. Inmgens das funções sen cos at

Consideremos a imagem da função / {at), em que a > 0:

Efectuemos uma mudança de variável no segundo integral, fazendo


z = at\ por conseguinte, dz = a dt\ obtemos, então;

1 f "
ou L{f {at)} = - y “ í {z)dz
0

Assim, se

então.
(11)
Cl a / ->/(«<)•

Exemplo — 1. Obtemos imediatamente da fórmula (9), em virtude de (11):

sen a t
. 1 1
1
' ‘ (t ) +
ou

sen at ( 12)
• p2 4_a2 •

Exemplo — 2. Obtemos da fórmula (10), em virtude de (11):

1 a
COS a t

ou

COS a t
• p2 + a2 • (13)
CALrCULO OPEHACIONAX. E APLICAÇÕES 425

§ 4. Propriedade de linearidade da im a g ^

Teorema — A imagem da soma de várias funções, multiplicadas


por constantes, é igual à soma das imagens destas funções multiplicadas
pelas constantes correspondentes, por outras palavras, se

n t) = yc,í,(t) (i4>
i=“l
(Tf são constantes) e
F{p)-^í(l), /•'>(!>) ^ li
então
F ( p ) = S ('iFi(p)- (14'>

Demonstração — Multipliquemos todos os termos da igualdade (14)


por e integremos em / nos limites de 0 a oo (pondo fora os
factores Ci de debaixo do sinal de integração); obtemos a igual­
dade (140.
Exemplo — 1. Determinar a imagem da função

/(/) = 3sen 4/ —2 COS õ/.


Resolução — Em virtude das fórmulas (12), (13) e (14'), obtemos:
12 2p
/.{/(/)} = 3-
p 2 + ifi
-2-
“ p2 + 2r> /> 2 -fl6 /^ 2 _ ^ 2 5 •

Exemplo — 2. Determinar o original cuja imagem é dada pela expressão

A’/ \ i
p2_^4 /,2_|_9

Resolução — Representemos F(p) da maneira seguinte:

-2ü- P
2 p2 + (2)í /-^ + (3)i
Por conseguinte, em virtude das fórmulas (12), (13) e (140, obtemos:

f(t)=^— sen 2 /+ 20 COS 3L

Resulta do teorema da unicidade do § 1 que é o único original que cor­


responde à função dada F(p).
426 CALCULO DIFERENCIAL E INTEGRAL

§ 5. Teorema do deslocamento
Teorema — Se F (p) é a imagem da função f (0. então» F (p + a)
é a imagem da função f (/). por outras palavras

se
então, (15)
ão. / ’(> + a ) - > e - “ VW- /
(Supomos aqui que Re (p + o) > Jo)-
Demonstração — E>eterminemos a imagem da função f-""* / {t ) :

Assim,

O teorema demonstrado alarga, notàvelmente/ a classe das imagens


para as quais o original pode fàcilmente ser encontrado.

§ 6. Imagem das funções senh a t, cosh af,


sen í/f, COS af
Resulta imediatamente da fórmula (8) em virtude da fór­
mula (15) que

(16)
p + a
De uma maneira análoga

- 1 — .V ea t . (16')
p —a

Subtraindo dos termos da relação (160 os termos correspondentes


da relação (16) e dividindo as diferenças obtidas por 2, obtemos:

1 ( — í---------- L _ ' ) ^ l ( e “ ‘ _ e - “ ')


2 \p — a p + a/ 2

ou

a
-V s h a í. (17)
p~~2— a T
CALrCULO OPERACIONAL E APLICAÇÕES 427

De igual modo fazendo a soma de (16) e de (160:

— ch at . (18)

Resulta da fórmula (12), em virtude das fórmulas (15):

-> e “ ‘ senffl/ (19)


(/. af +

Da fórmula (13). em virtude das fórmulas (15), resulta:

P+ « - r e “ ‘cos at ( 20 )
(p + a )- +
Exemplo— 1. Determinar o original cuja imagem é dada pela fórmula
7

Resolução — Transformemos F(p) de maneira a dar-lhe a forma da


expressão do primeiro msmbro da relação (19):

p24-i0p + 41 (p + 5)2-hl6 4 +
Assim,
f
r (/ pX
)=- 7 \
4 (P + 5 )i + 4-;
Por conseguinte, em virtude da fórmula (19), teremos:

f (p) ^ sen A t
* 4
Exemplo — 2. Determinar o original cuja imagem é dada pela fórmula

p 2 -( -2 p + 1 0

Resolução — Transformemos a função F (p):

P~f~3 _ (P~l~ i)~f~2 _ p-r t


p2 + 2 p + 1 0 ( p + l) 2 + 9 (p + l)2 + 3 2 + ( p + |) 2 ^ . 3 2
P+1 I 2 3
( p + l ) 2 + 32-t- 3 ( p + l ) 2 + 32

em virtude das fórmulas (19) c (20), obtemos o original

F (p) COS 3/ + - ^ e~^ sèn 3/


o
428 CALCULO DIFERENCIAL E INTEGRAL

§ 7. Derivasão da imagem
Teorema — Se F ( p ) f (0> então,

(2 1 )
dp
Demonstração — Demonstremos, primeiramente, que se f{t) veri­
fica a condição (1), então, o integral

\e {— t f í {t) dt ( 22)
existe.
Por hipótese | / ( 0 I < p = a ib, a > Sq; além disso
temos a > 0 e 5o > 0. É evidente que existe um número e > 0 que
verifica a desigualdade a > 5o + e. Do mesmo modo que no § 1
se demonstra a existência do integral

Calculemos, seguidamente, o integral (22):

] I e - ^ " í 7 (0 I d í = 1 1 ( í ) I dt.
0 0

Sendo a função limitada, e menor em valor absoluto do


que um certo número N para todo o valor í > 0, pode-se escrever:

] 1e " ^ 'í 7 ( t ) \ d t < N ] \ (0 I dí = iV I \f{t)\dt<oo


0 0 0

Demonstramos, assim, a existência do integral (22). Ora, este


integral pode ser considerado como a derivada de ordem n em ordem
ao parâmetro ("■) p do integral

Assim da fórmula

0(*)

(*) Estabelecemos no preâmbulo a fórmula de derivação do integral


definido em relação a um parâmetro real (ver § 10, cap. XI, t. I). Aqui o
parâmetro p é um número complexo, mas a fórmula de derivação permanece
válida.
CALCULO OPERACIONAL E APLICAÇÕES 429

obtemos a fórmula
(X

0 ^ 0
Estas duas igualdades dão-nos
oc

^ 0
isto é, a fórmula (21).
Utilizemos a fórmula (22) para obter a imagem da função
potência. Escrevamos a fórmula (8):

P
Obtemos desta fórmula em virtude da fórmula (21):

t
OU

7 - '
De uma maneira análoga

Para n qualquer, obtemos:


n\ . f 71

,71+ 1—►t . (23)

Exemplo — 1. Obtemos da fórmula (ver (12)


oo
---- = \ sen at dt,
4-
P^-\- J
0

derivando o primeiro e o segundo membro em relação ao parâmetro p:

, T*í sen “ í • (24)


Exemplo — 2. Obtemos da fórmula (13), em virtude da fórmula (21):

t COS a t . (25)
(p2 4-a2)2
430 CALCULO DIFERENCIAL E INTEGRAL

Exemplo — 3. Obtemos da fórmula (16), em virtude da fórmula (21):

----í----(26)
(p + a)'^ ■

§ 8. Imagem das derivadas


Teorema — Se F (p) f (t), então.

(27)

Demonstração — Em virtude da definição de imagem duma função


podemos escrever:

(28)

Suporemos que todas as derivadas /'(f). /"(/)/ . ..


encontraremos, satisfazem à condição (1), e, por conseguinte, que o
integral (28) e os integrais análogos para as derivadas sucessivas existem.
Efectuando a integração por partes do integral do segundo membro da
igualdade (28), obtemos:

0 0
Orá, segundo a condição (1)
<ao

\ \mè~^^j{t) = 0 et ] ( t ) d t = F(p)
t-^OO 0
Eis. porque
L{rit)} = -fiO )+pF{p).

o teorema está demonstrado. Consideremos, seguidamente, a ima­


gem das derivadas de qualquer ordem. Substituindo na fórmula (27) a
expressão pF (p) — f(0) em vez de F (p) e substituindo / (/) por f (i),
obtemos:
p[p^(p)-f(0)]-r(0)^r(t)

ou, tirando os parêntesis,

p V ( p ) - p / ( 0 ) - / '( 0 ) ^ . r ( < ) . (29)


CALCULO OPERACIONAL E APLICAÇÕES 431

A imagem da derivada de ordem n será

p " F { p ) - [ p ^ - ' f { 0 ) + p " - ^ r (0) + . . .


(30)

Nota — As fórmulas (27), (29) e (30) simplificam-se se / (0) =


/'(O) = ... = /(" -D (0) = 0. Neste caso obtemos:

F{p)^f{t),
p F Í p ) - > f (t),

§ 9. Dicionário de imagens
Para facilitar a utilização das imagens obtidas agrupamo-las num
quadro.
Nota — Se tomarmos para imagem da função /(/)

F*{p)=p]e-^^f{t)dt,
0

convém nas fórmulas 1 a 13 do quadro, multiplicar as expressões da


primeira coluna por p. Quanto às fórmulas 14 e 15 elas serâo da
forma: como F* (p) = pF (p), substituindo na parte esquerda da fór­
mula 14 F(p) pela expressão ^LSE) e multiplicando por p, obtemos:
P

14'. (
dp" \ p
Substituindo na parte esquerda da fórmula 15

Fi(p) = ^ , FAp) = ^
P P
e multiidicando por p, obtemos:
í
15' - F: ip) F t (p) ^ \ fi (t) Í2 {t - T) dl.
P J
432 CALCULO DIFERENCIAL E INTEGRAL

QUADRO X
CALCULO OPERACIONAL E APLICAÇÕES 433

§ 10. Equação auxiliar duma equação diferencial dada


Seja dada uma equação diferencial linear da ordem n de coe­
ficientes constantes ai, 02. . •/ «n :
ã^x , ^ ' cfcr ,
+ + > . • + dn-i — + dn^it) = f {t)- (31)
d/t cít dt

Pede-se para determinar a solução desta equação x = x(t) para


r > 0, que verifica as condições iniciais:

(0) = Xo, X (0) = xi, . . x<"-‘’ (0) = xV"! (32)

O problema já tinha sido resolvido da seguinte maneira: procura­


vamos a solução geral da equação (31) contendo n constantes arbi­
trárias; seguidamente determinavamos as constantes de maneira que as
condições iniciais (32) fossem verificadas.
Exporemos agora um método mais simples de resolução deste
problema, o método do cálculo operacional. Procuraremos a imagem
L da solução x{f) da equação (31) que verifica as condições (32).
Designemos esta imagem L por x (p); assim x {p)~T ^ (0-
Suponhamos que a imagem da solução da equação (31) existe,
bem como as suas derivadas até à ordem n inclusivé (uma vez achada
a solução podemos verificar a validade desta suposição). Multipliquemos
os dois membros da igualdade (31) por em que p = a + ib e
integremos em t nos limites de 0 a 00:

oo

dí - f Cl ( e ■dt +
J dí" J dt
0 0
oo oo

... + f e~^^x (i) d t = 1 (33)


J %/
0 0

Na parte esquerda da igualdade temos a imagem L"da função x{t)


e das suas derivadas, na parte direita a imagem L da função f(t) que
designaremos por F (p). Por conseguinte, a igualdade (33) pode ser
posta sob a forma:

{ -^ } + } + - - + a n L { x (t)) = L (/ (í)}.

28
434 CALCULO DIFERENCIAL E INTEGRAL

Substituindo nesta igualdade as imagens da função e as suas


derivadas pelas expressões (27), (29) e (30), obtemos:

aQ{p^x{p) — "Xq +P^' ^^0


•^0 +
"t" • • • +
“t" ^0^ +
3..^ . , Jn-2h
+ (P^^ (P) ~ [p'^ ^^0 -f-P^ ^^0 + • • • + +

+ an-i{P^(P) — M } + flni(p) = ^(P)- (3^*


A equação (34) chama-se equação auxiliar ou equação imagem.
Nesta equação a incógnita é a imagem ~x {p), que determinamos a
partir desta equação. Transformemos esta equação deixando no pri­
meiro membro os termos que contêm ~x ( p ) ’

Í(P) [^OP'' + + . . . + «n-lP + ^n] =


= + •••+ +
+ + • • • + 4'" +

+ «n-2[p^0 + ^ó] + ^n--l [^o] + ^(P)- (34')

O coeficiente de x (p) no primeiro membro da igualdade (340


é um polinómio em p de ordem n que se pode obter se no primeiro
membro da equação (31) se substituir as derivadas pelas potências
correspondentes de p. Designemo-lo por q)^ ( p ) :

(P) = ^oP^ + ^ + • • • + ^n-lP + 05)


O segundo membro da equação (340 está assim composto:
o coeficiente ^ multiplicado por Xq,
o coeficiente a^-2 é multiplicado por pxç^ + x ’^

o coeficiente Ui é multiplicado por + ... +


o coeficiente Oo é multiplicado por P'^~^Xq + p^"^Xy + . . . +
Façamos a soma de todos estes produtos. Juntemos ainda a
imagem do segundo membro da equação diferencial F (p). Todos os
termos do segundo membro da igualdade (340, excepto F (p), cons­
tituem após agrupamento dos termos semelhantes, um polinómio de
grau n — 1 cujos coeficientes são conhecidos. Designemo-lo por (p)«
Assim, a equação (340 pode ser escrita:

X(P) <f>7i (P) = i^n-iiP) + f ( p) -


CALCULO OPERACIONAL E APLICAÇÕES 435

Determinemos x (p) desta equação:


^ n -l(p ) , F{p) (36)
x{p) =
(P) , tPn (P)
Resulta que x (p) assim determinado, é a imagem da solução
da equação (31), que verifica as condições iniciais (32). Se agora
determinarmos a função jc* (0 cuja imagem é a função x (p), definida
pela igualdade (36), resultará, então, do teorema da unicidade for­
mulado no § 1 que jc* (0 é a solução da equação (31) que verifica
as condições (32), isto é,
X * (t) = X {t).

Se a solução da equação (31) for obtida pelas condições iniciais


nulas: Xq = x'^^ = xl = ... = = 0, então, na igualdade (36). tere­
mos (p) = 0 e ela será da forma

x(p) = F(P)
<Ph (p )
ou

x{/?) = - F{p)
(36')
flop" + <*iP" * + ••• + fl/i '
Exemplo — 1. Determinar a solução da equação
dx _

que verifica a condição x = 0 para í = 0.


Resolução — Formemos a equação auxiliar

^(p) ( p + l ) = 0 + y ou

Decompondo a fracção do segundo membro em fracções elementares,


obtemos:

• '« “ T - T + r -
Utilizando as fórmulas 1 e 4 do quadro 1, encontramos a solução:
X (/) = 1—
Exemplo — 2. Determinar a solução da equação
d^x
-9 x = 1,.
dt^
que verifica as condições inidais: xq = Xq= 0 para r = 0.
436 CALCULO DIFERENCIAL E INTEGRAL

Resolução — Escrevamos a equaçáo auxiliar (340

í( p ) ( p 2 + 9) = ± ou í(p ) = - ^ - ^ .

Decompondo esta fracção em fracções elementares, obtemos:


1 1
*(p)= . 9 9
p2 + 9 ■ p
Em virtude das fórmulas 1 e 3 do quadro 1, obtemos a soluç&o:

* :(< )= — ^ c o s 3 t + - | - .
Exemplo — 3. Determinar a solução da equação
á^x dx
*2 dt
que verifica as condições inidais: xq = Xq= 0 para / = 0.
Resolução — Escrevamos a equação auxiliar (340

®(P) (p2 + 3p + 2) = - ^
ou
X{p)z
1
p2 (p2 + 3 p + 2 ) p 2 (p + l)(p + 2 )-
Decompondo esta fracção em fracções elementares pelo método dos
coeficientes indeterminados, obtemos:
, 1 1 3 J _ , _ 1 ________ 1
2 p2 4 p "T" p + i 4 (p + 2)
Segundo as fórmulas 9, 1 e 4 do quadro 1, obtemos a solução:

Exemplo — 4. Determinar a solução da equação


d^x dx
- d ír + 2 - 5 r + 5 ® = * " * ’
que verifica as condições: Xq = \, Xq= 2 para f = 0.
Resolução — Escrevamos a equação auxiliar (34')

^(p)(p2 + 2p + 5 ) = p . l + 2 + 2 .1 + L{sení}
ou

*(P) (P* + 2j» + 5 ) = p + 4 +


p2+l •
donde obtemos x(p):
P+4
*(p)=
p 2if-2p + 5 ^ (p 2 + i)(jt,a + 2 p + 5 ) •
CALCULO OPERACIONAL B APLICAÇÕES 437

Decompondo esta ültima fracção do segundo membro em fracções ele­


mentares, podemos escreveV:
1 1 r / 1 I 1
10 10 5
x(p)-.
p2 + 2pH-5 + ’
ou
p+1 ,2 9 2
*(P)=-77T
10 (p + l)2 + 2 * ^ 10-2 (p + l)2 + 22
___ 1_ P I 1 1
10 ’ p 2 + l 5 'p 2 + l ■
Em virtude das fórmulas 8, 7, 3, e 2 do quadro 1, obtemos a soluçfto:
11 29 1 1
* ( í) = - jõ - e“‘ c o s 2 < + - ^ e - 's e n 2 < — cosí + -g-sent
OU, finalmente:
11 29 \ 1 1
( c o s 2 í + - ^ sen co st + - ^ s e n í.

§ 11. Teorana da decom poslj^


Resulta, da fórmula (36) do parágrafo precedrate, que a imagem
da solução duma equação diferencial linear se compõe de dois termos:
o primeiro é uma fracção racional regular de p, o segundo termo
uma fracção em que o numerador é a imagem do segundo membro
da equação F (p) e o denominador do polinómio q>n (p)-
Se F(p) é uma fracção racional, o segundo termo será também
uma fracção racional. É preciso também, saber encontrar o original
cuja imagem é uma fracção racional r ^ l a r . A bordarm os esta ques­
tão no presente parágrafo. Suponhamos que a im agm L duma certa
função é uma fracção racional r ^ l a r de p:

^ n -l(P )
<Pn(P)
Pede-se para achar o original. No § 7 do cap. X, t I, mostramos
que cada fracção racional r ^ l a r pode ser rqiresentada sob a forma
de fracções elementares de 4 tipos:

I.
p~ — a

II.
iP — a f
438 CALCULO DIFERENCIAL E INTEGRAL

I I I ----- onde as raízes do denominador são com­


«■2
plexas, isto é. < 0,

IV. — , „ — í: , onde > 2. as raizes do denominador


(p2 + a ,p -f 02)'* ’
são complexas.
Encontremos o original para cada uma das quatro fracções ele­
mentares. Para as fracções do tipo I, obtemos, em virtude da fórmula 4
do quadro 1:
— -----
p —a
Para a fracção do tipo II» em virtude das fórmulas 9 e 4 do
quadro 1, obtemos:
^ , - ^ A — ----- . (37)
i p - a f { k - i ) l

Consideremos agora as fracções do tipo III. Efectuemos as se­


guintes transformações*
Ap + B _________ Ap + B____________
/ + «,p + ^ ^ ~

, Ü
2
:= A

Designando aqui o primeiro e o s ^ n d o termo, respectivamente


por M e N, obtemos, em virtude das fórmulas 8 e 7 do- quadro 1:
í / ^
M-*Ae ^ co sí y -----
' 4

(n AaÁ 1 , i / a\
N
V 2 / , / aj ^ 4
y a , - -
CALX:ULO OPEPwACIONAL e a p l ic a ç õ e s 439

Assim, finalmente:

Ap + B
X
/)“ -|- a^p + (I2

Áa^
B -
X .1 COS í sen t (38)

V/ flo —
/ » - 4

Não abordaremos aqui as fracções elementares do tipo IV para


não nos lançarmos em cálculos bastante fastidiosos. Para alguns casos
particulares esta questão será analisada mais adiante.

§ 12. Eixemplos de resolução das equações diferenciais e dos


sistemas de equações diferenciais pelo método do cálculo
operacional
Exemplo — 1. Determinar a solução da equação
d'^x . , ^
— —-[- kx = sen õx,
(11“
que verifica as condições iniciais = 0, = 0 para / = 0.
Resolução — Formemos a equação auxiliar (340
3 .
a:(p) = (p-2 +
3
X (p) (P-+''i) = p2 + 9 ’ .j) ( p 2 ,4 )

1
x{p) = - p 2 j - 9 ^ p 2 -H 4 5 -f- 9 ‘ 19
donde obtemos a solução
3 1
X(t)= — scn2t— -^ sen3í.
^ ^ 10 5
Exemplo — 2. Determinar a soluçãoda equação
d^x
+ X — 0,
dt^
que verifica as condições iniciais: xq = 1, Xq= S, = 8 para / = 0.
Resolução — Formemos a equação auxiliar (340
(p ) (P ^ + 1 ) = • 1 + P •3 + 8,
obtemos
— . p2-]-3p4-8_ p2 -|- 3/> -|- 8
p3_|_l - ( p + l ) ( p 2 _ p + l) •
440 CALCULO DIFERENCIAL E INTEGRAL

Dscomponhamos a fracção racional obtida em fracções elementares:

8 -P + ^ ^ 2 . 1
(P + 1)(P*—/’ + !) P+1 P*—P + 1 “ P+ 1
1 Vã
P~2 11

Utilizando o quadro 1, escrevemos a soluçSo:

x(t^=
!/=2«-'-Fe2' ^ *) •

Exemplo— 3. Determinar a soluçáo da equaçto


d^x
dt^ - x = í cos2t,
que verifica as condições iniciais: x = 0, xó = 0 t = 0.
Resolução — Escrevamos a equação auxiliar (34')
1 8
x(p) ( p * + l) = -
P* + 4 (p2+4)a
donde
1
I (p)= -
9 p2 + l +
^ 9o- —
~ « 9 í—
p2_^4 I / + 3--j (p2 + 4)2 •
“^

Por conseguinte,
5
X ( 0 = — 9 “ s c o í + - j g » n 2 í + y ^ - is e n 2 t— / cos2t j .

Ê evidente que o método do cálculo operacional permite igualmente


resolver os sistemas dc equações diferenciais lineares. Mostremo-lo no exemplo.
Exemplo — 4. Determinar a sedução da equação

^ + 4f + 3 ,-0 ,

que verifica as condições iniciais: x = 0, y ~ 0 para r = 0.


Resolução — Designemos x (í)-í-x (p ), y ( t ) ^ ~ y í p ) e escrevamos o sistema
das equações auxiliares:

3p + 2)x<p) + py ( p ) = l
p
í»® (p) + (4 p + 3 ) y (p) = 0.
CALCULO OPERACIONAL B APLICAÇÕES 441

Resolvendo este sistema, obtemos:


4p-|-3_________ 1
x (p )= -
p ( p + l ) ( l l p + 6) 2p 5(P + 1) 1 0 (llp + 6 )’
1 / _ 1 ______ 11 \
y(p) = (llp + 6 ) ( p + l) ~ 5 V p + 1 llp + 6 ) ■

Segundo as imagens obtemos de cada vez o original, isto é, as soluções


procuradas do sistema:

1 1 3 -jit
* W = T - T " ''- 10

1
í'W=-5-(«”' —e “ ).

Resolve-se, duma maneira análoga, os sistemas lineares de ordem superior.

§ 13. Teorem a do enrolam ento (Conyolutiorí)


Na altura da resolução das equações diferenciais pelo método
do cálculo operacional servimo-nos repetidas vezes do
Teorema do enrolamento — Se Fi (p) e F 2 (p) são as imagens das
junções /i (0 e /2 (í). wto é, se

PÁP)-^h{t) e
então. Fx (p) F^ip) é a imagem da junção

j / l W / 2 ( < — x)dT,

por outras pãlavras

Fi ip) F 2 (p) ^ j fi (t) fz (t - X) dx. (39)

Demonstração — Determinemos a imagem da fu n ^ o

UiÍT)fz(t-x)dx,
0

partindo da definição de imagem

U í (t:) f z i t — x:)dx} = °l [ U i { t ) f z i t — x)dx]dt.


0 0 0
442 CALCULO DIFERENCIAL E INTEGRAL

O integral do segundo membro é um integral duplo, que se


toma no domínio limitado pelas rectas t = 0, t = / (fig. 380).
Mudando a ordem de integração neste inte­
gral. obtemos, então:

(x) h (t — x) dxj =
0

= I l/i (x) ] {t — x) dt ]dx.

Efectuando a mudança de variável t — r = z no integral interior,


obtemos:

í e~^'Í 2 ( t - r ) d t = J (z) dz =
T 0
= J e-^^h (2) dz = e-^^F^{p).
0
Por conseguinte,

L \ U i W h {t — t) dx] = °lfi (x) e~^'^F2 (p) dx =

= Fo_(p) J e ^ Vi (x) dx = Fj (p) Fi (p).


Assim,
í / i (x) A (í — x) dx ^ F j (p) F zÍp ).

É a fórmula 15 do quadro 1.

N ota— 1. A expressão I /i (x) f ^ { t — t ) dx chama-se enrola-


0
mento (ou produto de composição) das duas funções fi (t) e /2 (0*
A operação do cálculo correspondente chama-se transformação de
enrolamento de duas funções e tem-se, então.
t t
i’ /i (x) A (< — x) dx = J /, (í — t) a (x) dx.
0 0

A validade desta última igualdade pode ser estabelecida efectuando


a mudança de variável t — r = z no integral do segundo membro.
Exemplo — Determinar a solução da equação
d^x

que verifica as condições iniciais: xq = = 0 para / = 0.


CALCULO OPERACIONAL E APLICAÇÕES 443

Resolução — Escrevamos a equação auxiliar (34")

^(P) (p2+l) = ^(p),


1
em que F {p) é a imagem da função / (f). Por conseguinte, x (p) ■
-
p2 + l fiP),
— — V sen t € F (p) f (t). Aplicandb o teorema do enrolamento (39)
e designando =^2ÍP). ^(P) = ^i(p), obtemos
p2 + l
I
: (r) z= ^ / ( t ) sén (t — x) dx. (40)

Nota — 2. Com a ajuda do teorema de enrolamento pode-se


determinar fàcilmente a imagem do integral duma dada função, se
se conhecer a imagem dessa função; precisamente se /r (p) -V / (í)
então.

/ (x) dx. (^il)

Com efeito, se introduzirmos as notações


1
/i(0 = /(0, f 2 Ít) = U então. F,Íp) = F(p), F^{p) =

Substituindo estas funções na fórmula (39), obtemos a fórmula (41).

§ 14. Equações diferenciais das oscilações mecânicas.


Equações diferenciais da teoria dos circuitos eléctricos
Sabe-se da mecânica que as oscilações dum ponto material de
massa m são descritos pela equação (*)
\ dx k 1 ,
- ^ + — — + — ^ = — /i(0: (42)
dt m dt m m

designando aqui x o vértice do ponto duma certa posição, k a rigidez


do sistema elástico, por exemplo da mola, a força de resistência ao
movimento é proporcional (com um coeficiente de proporcionalidade A)
no primeiro grau da velocidade, /i (t) é a força exterior ou de per­
turbação.

(*) Ver, por ‘exemplo, cap. XIH, § 26, t. II, em que se estabeleceu uma
equação deste género na altura do estudo das oscilações dum peso fixado
a uma mola.
444 CALCULO DIFERENCIAL E INTEGRAL

A solução duma equação do tipo (42) descreve igualmente as


pequenas oscilações de outros sistemas mecânicos com um grau de
liberdade, por exemplo, as vibrações de torção do volante sobre um
tronco flexível, se x for o ângulo de rotação^
do volante, m o momento de inércia do
volante, k a rigidez à torção do tronco e
m/i (0 o momento das forças exteriores em
relação ao eixo de rotação. As equações do
tipo (41) descrevem não sòmente as oscilações
mecânicas mas também os fenómenos que se
desenrolam nos circuitos eléctricos.
Seja um círculo eléctrico, composto
duma inductância L, duma rçsistência R e
duma capacidade C, ao qual é aplicado uma
força electro-motriz E (fig. 381). Designemos
por i a corrente no circuito e por Q a carga do condensador, então,
como se sabe da electrotécnica I e Q verificam as equações seguintes:

L — + R i + ^ = E, (43)
dt C

dQ
(44)
dt

Obtemos da eq ua^o (44):

âQ di
(44')
dt

Substituindo (44) e (44') na equação (43), obtemos para Q uma


uma equação do tipo (42):

L ^ + R ^ + — Q = E. (45)
dt^ dt C

Derivando os dois membros da equação (43) e utilizando a


equação (44), obtemos a equação que determina a corrente i:

dh di , i , dE
--- -ti--------------1 t -- -----• (46)
dt^ dt C dt

As equações (45) e (46) são equações do tipo (42).


CALCULO OPERACIONAL B APLICAÇÕES 445

§ 15. Resolução da equação diferencial das oscilações

Escrevamos a equação das oscilações sob a forma

d^x dx
Ui a2X = f(t), (47)
de dt

onde o sentido mecânico ou físico da função procurada x, dos coefi­


cientes üi, Ü2 , e da função / (/) é fàcilmente estabelecida comparando
esta equação com as equações (42), (45) e (46). Determinemos a solução
da equação (47). que verifica as condições iniciais: x = Xq, x ' = x^
para r = 0.
Formemos a equação auxiliar para a equação (47):

^(P) (P^ + ^iP + ^2) = ^OP + ^0 + « 1^0 + 7 ^(P)y ("^8 )

em que F (p ) é a imagem da função /(/). Obtemos da igualdade (48):

^o(p) + ^o + (íi^o , f^ip) (49)


^ÍP)=^' 2 , , ' 2 ,
P + « !/? + «2 P + d \ P + a2

Assim, para a solução Q (/) da equação (45), que verifica as


condições iniciais: Q = ç ' = Ç' para r = 0, a imagem será da
forma

Q (p ) — ^ (^ o P Qo) RQo _j_________ E {p)


1
L p ^ -\- R p + — Lp^ + Rp -)-

O carácter da solução depende essencialmente das raízes do binó­


mio p* -f ãip + Ü2 \ elas serão complexas, reais e distintas ou reais
e iguais. Consideremos em detalhe o caso em que as raízes do binómio

sao complexas, isto é, quando < 0. Analisar-se-á duma


maneira análoga os outros casos.
Como a imagem da soma de duas funções é igual à soma das
suas imagens, em virtude da fórmula (38) o original para a primeira
446 CALCULO DIFERENCIAL E INTEGRAL

fracção situado na parte direita de (49) será da forma

- ^ — — — — ~>e . i / .
X q COS t y Ü2--------- f-
II a:ocosf
P + ^ iP + ^2 L ^
^0 +
+ sent (50)

Determinemos, em seguida, o original correspondente à fracção


^(P )
+ dip + Ü2

Utilizaremos aqui o teorema do enrolamento atendendo a que

1 e. ^
P + ^ iP + ^2 -I / cq 4
y a , - -

Por consequência, obtemos, segundo a fórmula (39)

F{P)
P + a i P + <h

-T
sen {t — x) Ü2 — — dx. (51)

Assim, obtemos de (49) tendo em conta (50) e (51):


XqCL\^
^0 +
u(i)=e 2^ sent +
XqCOSt +

f(x)e ^ sen (í — x) Ü2 — — dx. (52)


fl? J ^ 4
CALCULO OPERACIONAL E APLICAÇÕES 447

Se a força exterior / (r) = 0. isto é, se estivermos em presença de


oscilações mecânicas ou eléctricas livres, a solução é dada pelo primeiro
termo do segunda membro da expressão (52). Se os valores iniciais
são iguais a zero: xq = = 0, então, a solução é dada pelo segundo
termo do segundo membro da igualdade (52). Consideremos estes casos
mais em detalhe.

§ 16. Estudo das oscilações livres


Suponhamos que a equação (47) descreve oscilações livres, isto é,
que / (/) = 0. Introduzamos, para maior comodidade na escrita das
fórmulas, as notações: = 2n, k\ = Então, a equa­
ção (47) tomará a forma
d^x , ^ dx
+ 2n k^x = 0. (53)
dt dt
A solução desta equação X] que verifica as condições iniciais
(x = Xo, X = x ' para / = 0) é dada pela fórmula (50) ou pelo pri­
meiro termo da fórmula (52):

xi(t) = e I arocos/ci^ - f senk l í ] . (54)


ki

Façamos Xq = a, ^ = 6. É evidente que para todo o a e b


/Cl
se pode escolher Af e 8 de maneira que se tenha a = M sen 8,
6 = Af COS 8, então, tg 8 = ajb. Escrevamos a fór­
mula (54) sob a forma:

x\ = [ M COS k(t senô + M sen/c^ícos ô],

pode-se, então, finalmente escrever a solução assim:

x\ = 'Vc? + b^e sen ( k i t 6), (55)

A solução (55) corresponde às oscilações amortecidas.


Se 2n = fli = 0, isto é, se se desprezar a fricção interna, então,
a solução será de forma

x\ = V d ^ + 6^ sen (k^t + ô).


Teremos neste caso oscilações harmónicas. (No tomo II, cap. XIII,
§ 27, são dadas, nas figuras 270 e 271, os gráficos das oscilações
harmónicas e amortecidas).
448 CALCULO D IFERENCIA L E INTEGRAL

§ 17. E^studo das oscflações harmónicas amortecidas


no caso duma força exterior periódica

Na altura do estudo das oscilações elásticas dos sistemas mecâ­


nicos e particularmente na ocasião do estudo das oscilações eléctricas,
teve-se de considerar diversos tipos de forças exteriores / (0- Estudemos
em detalhe o caso de uma força exterior periódica. Suponhamos que
a equação (47) é da forma

4- 2n + k^ x = A sen coí. (56)


dt^ ^ dt ^
Para elucidar a natureza do movimento basta considerar o caso
em que Xq = x' = 0. Poder-se-ia obter a solução da equação segundo
a fórmula (52). mas é mais cómodo aqui do ponto de vista metódico
obter a solução efectuando todos os cálculos intermediários.
Escrevamos a equação da imagem

x{p)(p +2 n p + k) = A-^ 2’
P +©
donde obtemos:
A 0) (57)
x{p) =
{p^ + 2np + k)ip + (D )

Consideremos o caso em que 2 n ^ 0 (n^ < k^). Decomponhamos


a fracção da parte direita em fracções elementares:

________ A(ú Np + B I
______ __ ______________ ^ (58)
(p^ + 2np-{-k’‘){p’‘ + <ú^) p^ + 2np + k^ + (o* '

Determinemos as constantes B, C, D pelo método dos coeficientes


indeterminados. Utilizando a fórmula (38) obtemos de (57) o original

x{t) = co^) sen (út — 2n(ú cos coí +


{k^ — (0^* + 4n^(ú‘

— nt
(2«“^ + w^) —— sen kit + 2n© cos * i í ] | ; (59)
kl

aqui de novo ki = — ra*. É precisamente a solução da equação (56),


que verifica as condições iniciais = 0 para t = 0.
CALCULO OPERACIONAL E APLICAÇÕES 449

Consideremos o caso particular para o qual 2n = 0. No sistema


mecânico, por exemplo, isso corresponde ao caso em que não há
résistência interna, nem amortecedores. Pará
um contorno eléctrico isso corresponde ao
caso em que i í = 0, por outras palavras,
que a resistência interior do circuito é nula.
A equação (56) toma. então, a forma

. k^x = A sen o)í, (60)


dt^
e obtemos a solução desta equação que
verifica as condições Xq = = 0 para
/ = 0, se pusermos na fórmula (59) n = 0:

x ( t ) = — ^ ---- \ — (osenAí+ A:senü)/j.

Temos aqui a soma de duas oscilações harmónicas: as oscilações


próprias de frequência k:
A ___ 0^
tpr sen Aí
co^ k
e as oscilações forçadas de frequência a>:

^forc (0 sen (úí.

No caso em que A > (d, o carácter das oscilações está rq>re-


sentado na figura 382.
Voltemos, uma vez mais, à fórmula (59). Se 2n > 0, o que tem
lugar para os sistemas mecânicos e eléctricos considerados, o termo,
que comporta o factor e que representa as oscilações próprias
amortecidas, decresce ràpidamente quando t cresce. Para t suficiente­
mente grandes o carácter das oscilações é determinado pelo termo que
não contém o factor isto é, o termo

x{f) {(A^ — (ú^) sen o)í — 2/zo) cos (oí}. (62)


(A^ — co^)^ + 4n^(ú^
Introduzamos as notações:
A (A^ - (o")
71/cos ô; — /r.2 2\2 I / 2 2 T l/ S C n ô ,
(A^ — + 4/1^03“ {k — (0 ) 4n (0
(63)
450 CÁLCULO D IFEREN CIA L E INTEGRAL

onde
M=
2..2
V(k^ — (ú^f + Artoí
Pode-se escrever assim a solução (62):

x(t) = sen (íoí + ô). (64)

Resulta da fórmula (64) que a frequência k das oscilações for­


çadas corresponde à frequência a> da força exterior. Se a resistência
interna caracterizada pelo número n for fraca e se a frequência «>
for próxima da frequência k, a amplitude das oscilações pode tomar-se
arbitràriamente grande, pois x) denominador pode tornar-se também
tão pequeno quanto se queira. Quando n = 0, = k} a solução não
é expressa pela fórmula (64).

§ 18. ^ Solução da equação das oscilações no caso da ressonância'


Consideremos o caso particular em que = 2n = 0, isto é, quando
a resistência é nula, e a frequência da força exterior coincide com a
frequência das oscilações próprias A: = <i>. Neste caso, a equação toma
a forma
-\-k^x = A sen kt. (65)
dt^ ^

Procuraremos a solução desta equação que verifica as condições


iniciais: :to = 0, = 6 para r = 0. A equação auxiliar será
k
p^+ 1^
donde
Ak
í(p ) = :
(p^ + k Y
Obtivemos uma fracção racional regular do tipo IV que não
estudamos sob a sua forma geral. Para determinar o original pela
imagem (66) recorreremos ao artifício seguinte. Escrevamos a iden­
tidade (fórmula 2, quadrol):

,- p t sen kt dt. (67)


- A
CALCULO OPERACIONAL E APLICAÇÕES 451

Derivemos (*) os dois membros desta igualdade em relação a k:

1 2k^
e COS ktdt.
= í
Utilizando a igualdade (67). escrevemos, assim, esta igualdade:
oo
2k^
t COS k t ------sen k t dt.
(p^ + k ^ f k

Daí resulta imediatamente:

Ak
sen k t — t COS k t
VA:

(obtemos, desta fórmula, a fórmula 13 do quadro 1). Assim, a solução


db equação (65) será

A í\
x{t) sen k t — t COSk t (()S)
2k\k
Estudemos o segundo termo desta solução

X n ( t ) - - - ---------- 1 C O S k t :
2k

quando t cresce, esta grandeza não é limitada. A amplitude das osci­


lações correspondente à fórmula (680 aumenta indefinidamente quando t
cresce indefinidamente. Por conseguinte, a amplitude das oscilações cor­
respondente à fórmula (68) aumenta também indefinidamente. Este
fenómeno que tem lugar quando a frequência das oscilações próprias
coincide com a frequência da força exterior chama-se ressonância (ver
igualmente, tomo II, cap. XIII, § 29, fig. 273).

§ 19. Teorema do retardamento

Suponhamos que a função f (t) é idênticamente nula para / < 0


(fig. 383, a). Então, a função / (/ — to) será idênticamente nula quando
t < to (fig. 383, 6). Demonstremos o teorema do retardamento seguinte.

(*) O integral do segundo membro pode ser representado sob a forma


de soma de dois integrais da variável real em que cada um depende do
parâmetro k.
452 CALCULO D IFERENCIA L E INTEGRAL

T e o r e m a — Se F (p) é a imagem da junção f (í), então 6“^^" F(p)


é a imagem da função f (t — to), isto é, se f(t)-»-F(p), então.

f ( t - Q ^ e ~ ^ * ' ‘F {p ).

-toL

I
^0
(b)

Fig. 383

Demonstração — Por d e fin iç ã o de im a g e m , te m e s :

= í e ' ' ’7 it - Q dt + S e - P '/ ( í - to) dt.


0 ^0

O p rim e iro in te g ra l d o s e g u n d o m e m b ro d a ig u a ld a d e é ig u al
a z e ro , v is to q u e f ( t — t o ) = 0 q u a n d o t < t o . E fe c tu e m o s , n o se g u n d o
in te g ra l, u m a m u d a n ç a d e v a riá v e l f a z e n d o t — t o = z :

L(/ (f _ í„)} = ]e~ (z) dz = e - ] e~^^f (z) dz = e~^<^F{p).


0 0
f ( t - to)
A s s im , e-P'« F (p).
Exemplo — Estabelecemos no § 2 para a função unidade de Heaviside:
j
^0 W ^ -----• Resulta, em virtude do teorema demonstrado, que para a função
• P

6o(t-M

0 h t
Fig. 384
Oq (/—h) representada na figura 384, a imagem L será isto é,
1 ^ ’
ao Ae-P'».
• P
CALCULO OPERACIONAL E APLICAÇÕES 453

Exercícios

Determinar a solução das seguintes equações para os valores iniciais dados:

1. 2x = 0, x' = 2 para / = 0 . Rcsp. x = Ae~^—


dt
rf3.r d^x
2 — x = 2y x ' ---- 0 , x ' = [ para 0. Rcsj5. x = \ — t + e^,
dt^ dV^
4»at
3. 2 n - ^ -!-(<»*-!-6*) X- 0, x = in, i ' - xi para t = 0 . Resp. * = - ^ X
dt- dt ^
X [J^o^ <*os bt 1 (x;,—xoa) sén bt],
/,. 4 ^ —3 - ^ + 2 x = í-®', x -=l. x' = 2 para < = 0. Resp. x = -j^e®'+4^<>' +
dt- dt 1 ^ 1
2 ,

5. -|- m ^x = a COS /?/, x = jtoi - ^ó- ^ R^SP- ^ -• ^2 _y^s. ^

X (COS nt—cosmt)-\-XQ cos m í + - ^ scn mí.

6. ^d/2 — ^ = í 2, o: = 0. x' = 0 para í = 0. Rcsp. x = 2^* — o


7 . + ^= x = x’ = x ' = 0 para t = 0.Resp. i = - ^ ( < * - 3 t + - | j e ' -

1 I 1 f ^ t /5 nm * ^

,3 1 2 "
8. - r 4 - ^ - * = l, xo = *ó’=®õ = 0 para <= 0. Resp. ®= 1— y ' ' " ‘“ "J
d(3
t V3
X COS

9. 2 - ^ + * = **“ í. xo=®i = ®í = ^lT= 0 para í = 0. Res^ ^ = y X


X [ e '( t - 2 ) + e -'(t + 2) + 2scnt]
10. Determinar a soiuçSo do sistema das equações diferenciais
d*x , _4 L,_n

que verifica as condições iniciais zo = yo = ^i = yó = 0 ^= 0.


1 1 1
Resp. .r (0 = ”- y cosí + - ^ e ^ + - ^ e - ^

! / ( <) =—y c o s í— ^ « -' + 1.


índice alfabético

Amplitude, 106 Determinante


Análise harmónica, 366 — funcional, 187
Area, 176 — de Wronski, 77, 79
— de superfícies, 190 Divergência, 252
Astróide, 53 Domínio
Cálculo operacional, 420-453 — de convergência, 287
Campo de direcções, 21 — fechado, 160
— irrotacional, 255 — de integração, 161
— potencial, 254 — regular, 163, 177, 203
— solenoidal, 255
— tubular, 255 Elipse, 231
Catenária, 13, 15 — de inércia, 199
Centro de gravidade, 200, 212, 240 Elipsoidal, 207
Circulação, 226, 248 Envoltório, 43-49
Coeficientes de Fourier, 338 Equação(ões)
Comparação com um integral, 278 — de Bernoulli, 35-37
Condição(ões) — de Bessel, 319
— de convergência das séries, 267, — característica, 82, 120
270, 272, 276, 278, 281, 283. * — do calor, 377, 388, 393-401
— iniciais, 18, 61, 380, 388, 392 — de Clairaut, 51-53
— dos limites, 380, 388, 392 — de continuidade, 402
Coordenadas — das cordas vibrantes, 378, 379
— curvilíneas, 185
— das diferenças finitas, 414
— cilíndricas, 208
— diferencial, 13, 16
— esféricas, 210, 211
— de derivadas parciais, 16
Coesenos, 308
— de diferenciais totais, 37-40
— imagem L, 424
— homogénea, 26-29, 75
Curva integral, 19, 62
— integral, 19, 62
Densidade, 194 — geral 19, 62
— superficial, 194 — particular, 19
Desigualdade de Bessel, 356 —lineares, 32-35, 75
— de Bouniakovsky, 199 — homogéneas, 75-89
Desvio máximo, 352 — não homogéneas, 75, 89-103.
— quadrático, 352, 353 — ordinárias, 16
456 CALCULO DIFERENCIA L E INTEGRAL

Equação(ões) Grade, 413


— ordem, 17 Gradiente, 253
— solução 17
— geral, 18, 61 Igualdade de Liapounov, 357
— particular, 19 Imagem da função, 420
— de variáveis separáveis, 23-26 Integração gráfica, 73-75
— de Lagrange, 53-55 Integral curvilíneo, 223-232, 235-240
— de Uplace, 257, 377, 401-409 — dependente de um parâmetro. 215
— em coordenadas cilíndricas, 408 — de Dirichlet, 361
— de onda, 377, 380 — duplo, 160; 168
— do telégrafo, 381 — em coordenadas polares, 177-185,
— do tipo elíptico, 377 189
— hiperbólico, 377 — de Fourier, 368-374
— parabólico, 377 — geral, 19, 62
Esfera, 192, 208 — múltiplo, 160
Espiral logarítmica, 60 — particular, 19
— de Poisson, 182, 400, 413
Factor integrante, 40 Integral
Fase inicial, 106 — singular, 50
Fluxo do campo vectorial, 242, 248, 253 — de superfície, 240-245
Formulas — triplo, 2C2-212
— de Adams, 138 — em coordenadas cilíndricas, 208
— de Euler, 308 — espéricas, 210
— geral do binómio, 309-311 Intervalo de convergência, 298, 300
— de Green, 234, 260
— de Liouville, 78 Jacobiano, 187, 212
— de Ostrogradsky, 250, 252
Linha quebrada de Euler, 132
— de Stokes, 248 — de corrente, 57
Frequência das oscilações, 106 — equipptencial, 57
Funçãoís) — de nível, 57
— de Bessel, 322-324
— contínua por corte, 357 Método das diferenças finitas, 393-395,
— harmónica, 257, 261, 401 -413-416
— homogénea, 26, 27 — de Euler, 131-133
— ínnpar, desenvolvimento em série de — de Fourier, 382, 396, 409
Fourier, 347 — de separação das variáveis, 382,
— linearmente dependentes e linear­ 396, 409
mente independentes, 87, 88 Momento
— logarítmica, 312 — de inércia, 195, 212
— monótona por corte, 338 — estático, 201
— objecto, 421 Mudança de variáveis
— par, desenvolvimento em série de — num integral duplo, 185-190
Fourier, 348 — num integral triplo, 208-212
— próprias, 384
— unidade de Heaviside, 422, 452 Nós da grade, 413 i
ÍNDICE ALFABÉTICO 457

Opsrador — de Fourier, 334-352


— hamiltaníano, 253-257 — harmónica, 268
— de Laplace, 256, 260, 401 — integração e derivaçõo, 293, 296,
— nabla, 253 296, 301-303
Original, 420 — de Maclaurin, 306-308
Oscilações, 103, 443 * — majoráveis, 288 -
— amortecidas, 107, 447 — númerica, 264
— forçadas, 105, 107-112 — semi-convergente, 2 ^ , 286 , ,
— harmónicas, 106, 440 —— ' • -- ^ ^ e ' T a ^ r , T6? ^v
— livres, 105-107, 44^ ~ ttig<>hoíniitricí 33^4‘^ ^ ^
*
- tmiformeménte* çòpyêrgçnte^^ 290
Parábola de segurança,f*47"’"’"^ Sistema de eqüàções diferenciais, 1J3,
Pêndulo matemático, 69Í71 ..118................. . - V
Período das oscilações, |l06 — normal de equações diferenciais,
Ponto } 113
— interior do domíni^), 162 •Soluçãofões)
— singular, 45 j — geral,.J^, 61
Potencial, 238, 250 \ — linearmente depçndentes-. e " Hnear-
Primeiro problema dos lijnites, 388, 402 niente indcpendenjtes, <7fr81». 88
Problema de Dirichlet, 402, 405-416
— dos limites, 388, 4o|"'^'^ — ^ngplar, >58 ‘Ti' C ' ^ '
— de Neumann, 402, l405
Produto de composição, 442^ * .... ....... — dupla, 163, 168
Progressão geométrica, 265 — intgral, 160, 202
Propagação do calor, 386-392 — parcial duma série, 264
Raio, 177 — duma série, 264
— de convergência duma série, 298 — continuidade, 290
Regra Teorema de Abel, 297
— de Cauchy para a convergência
— de enrolamento, 441
duma série, 276
—de Leibniz, 281
— de Alembert, 272 — do retardamento, 451
Ressonância, 112,451 Teoria da estabilidade, 125-131
Resto duma série, 288
Termos da série, 264
Rotacional, 248
Trabalho, 223, 224, 232
Segundo problema dos limites, 402 Trajectórias
— velocidade cósmica, 71-73 — isogonais, 55, 59, 60
Senos, 306 — ortogonais, 55-58
— Imagem L, 422 Transformada-cosseno de Fourier, 371
Série(s) 264 Transformada de Fourier, 374
— absolutamente convergente, 285, 286 — inversa de Fourier, 374
— alternada, 281 — de Laplace, 421
— contiuidade da soma, 290, 301 Transformada-seno de Fourier, 372
— convergente, 264-267
— divergente, 264, 268, 271 Variação das constantes, 90, 100
— inteira (de potências) 296, 303 Vector turbilhão, 248
— de funções, 287 Volume, 174, 207

Você também pode gostar